Главная Юзердоски Каталог Трекер NSFW Настройки

Наука

Ответить в тред Ответить в тред
Check this out!
<<
Назад | Вниз | Каталог | Обновить | Автообновление | 1536 174 459
Тред вопросов и ответов Аноним 28/10/23 Суб 17:48:26 584346 1
наукич.jpg 239Кб, 1299x870
1299x870
Перед тем как перейти к тредам, убедитесь, что вы ознакомились с правилами: https://2ch.hk/static/rules.html#sci

Это прикрепленный тред, где интересующийся анон может задать свои вопросы (и, возможно, даже получить ответ). Здесь можно общаться, а также обратиться к модератору.

Однако, помните, что /sci/ - собирательный раздел для всех наук, но если вы глубоко заинтересованы обсуждением политики - вам в /po/, математики - /math/, философии - /ph/, психологии и психиатрии - /psy/, медицины - /me/, космоса и астрономии - /spc/.
Аноним 28/10/23 Суб 22:48:12 584349 2
https://mir24.tv/news/16467844/detenyshi-pterozavrov-okazalis-gotovy-k-poletu-s-rozhdeniya
Как птерозавры могли летать сразу после вылупления? У птиц и рукокрылых полёт — приобретаемый навык, а не врождённый + требует регулярной тренировки. Птиц, родившийся в неволе, выдохнется через пару минут полёта, тупо умения и стамины не хватит. А хладнокровный (мезотермический?) рептилоид рождался уже с достаточной мышечной массой, выносливостью и умением использовать воздушные потоки / маневрировать / приземляться, не сломав себе лапы?
Аноним 29/10/23 Вск 04:25:57 584352 3
>>584346 (OP)
Вот как сейчас можно читать WOS или Science? На сайхабе новых статей нет. Готов умеренно потратиться
Аноним 29/10/23 Вск 10:20:51 584356 4
Верните прошлый закреп, и да, если я заинтересован в физике, куда мне идти? Нахуй?
Аноним 29/10/23 Вск 11:23:13 584358 5
>>584356
Оставаться здесь. В закрепленном посте указаны доски для тех наук, которые вышли за пределы /sci/ в свете своей наибольшей востребованности. В любом случае, их можно обсуждать и здесь, но будьте готовы к тому, что вам мало кто ответит по поводу математики или философии, ибо для этого есть свои разделы. Физику же можно обсуждать в закрепленном треде или создать свой собственный, - ограничений на это нет.
Аноним 29/10/23 Вск 16:52:34 584361 6
Двач
Аноним 29/10/23 Вск 21:07:55 584363 7
ogog14660846732[...].jpg 689Кб, 1200x628
1200x628
Сколько теоретически может существовать изолированная экосистема в ограниченном объёме? Проще говоря, в банке.
Мох, водоросли, какие-то растения, мелкие почвенные насекомые. На практике это всё может расти и зеленеть в закупоренной банке год, два. Дольше просто не проверял. А тысячу лет сможет? Миллион? Что там будет через 50 миллионов лет, при условии, что банка не разрушится и не вмёрзнет в лёд?
Аноним 30/10/23 Пнд 04:45:27 584364 8
>>584363
Очень мало, так как в такой экосистеме все висит на балансе освещенности и количестве веществ в круговороте. Обычно это прокисает на 10-20 год, если экосистема на 500-1000 литров. И причина, обычно, в выводе веществ в говно и невозможности его быстрого расщепления.
Экосистема планетарного биома имеет показатель вещества в сотни раз выше к жизни, и именно этим и успевает обеспечивать и поддержание жизни и прирост ее.
В замкнутом пространстве такое не реализовать. Это, кстати, одно из доказательств системного архитектора, лол.
Аноним 30/10/23 Пнд 09:34:05 584370 9
Термодинамика говорит, что все добро переработается в говно.
Но это не точно.
Может быть там смогут выжить какие-нибудь экстремофилы, которые просто впадут в анабиоз до момента разрушения банки.
Аноним 31/10/23 Втр 11:58:54 584397 10
Можно ли использовать обратный пьезоэффект тобишь электрострикцию для передачи механического усилия?
Я имею ввиду, вот есть кристалл кварца, который при давлении меняет потанцевалы.
Но если сделать нить из SiO2, сделать жгут из сотни таких и пропускать заряды противоположных значений по каждой из них, получится ли добиться значительного момента силы? Скажем, сдвинуть многотонный кусок железа.
Аноним 31/10/23 Втр 17:15:55 584404 11
>>584397
Понадобится уйма энергии, но да, теоретически это возможно.
Аноним 31/10/23 Втр 18:47:45 584409 12
>>584346 (OP)
Мочухан, докажи-ка антуану истинность точного определения термина "наука" - или тупой пиздабол.
Аноним 31/10/23 Втр 21:34:05 584414 13
>>584409
Не хочу.
Можете закрывать тред.
Аноним 31/10/23 Втр 22:07:45 584416 14
А электрослабое состояние вещества всё ещё прозрачно для нейтрино или они там активно переизлучаются подобно фотонам во внутренностях солнца? И соответственно при таких энергиях и концентрациях вещества нейтрино становятся полноценными реагентами, а не просто инертным квантовым пеплом? А есть ещё более инертные частицы, которые начинают жечь только при суперобъединении?
Аноним 01/11/23 Срд 02:09:49 584422 15
>>584416
>электрослабое состояние вещества
Это как?

Сечение взаимодействия нейтрино с электронами или кварками можно повысить не таких уж многими способами.
-Первое это очень сильно нагрев вещество, такие температуры возможно разве что в пике взрыва сверхновых или аккреции вещества на релятивистские объекты.
-Другой способ это повышение количество частиц вещества в объеме - короче повысить плотность. Плотность тут должна быть на порядок выше ядерной. Такое даже у нейтронных звезд редко бывает.
-Запихнуть вещество в очень глубокую потенциальную яму, глубже его энергии покоя. Только у нейтронных звезд, которые близкие в коллапсу в черные дыры.
-Или наоборот сильно охладить вещество, чтоб полезли квантовые эффекты, которые подавляются декогеренции. На отдельных частицах или системе частиц работает, но на макро телах не очень.

>А есть ещё более инертные частицы, которые начинают жечь только при суперобъединении?
Полно! На любой вкус найдутся расширения стандартной модели. Суперсимметрия дает целый зоопарк неуловимых джо.
Аноним 01/11/23 Срд 12:24:30 584430 16
>>584409
определения придумывают люди, они не даны свыше
об определениях договариваются
следовательно истинное определение — то, о котором есть договорённость в рамках дискуссии
айда договариваться

наука — это область человеческой деятельности, знания в которой приобретаются научным методом

соглашайся с моим определением или предлагай своё
Аноним 01/11/23 Срд 21:02:15 584447 17
А где Артемий, что случилось?
Неужели раздел может перестать быть говном?
Аноним 01/11/23 Срд 21:35:44 584449 18
>>584447
А почему он "говно"? И что (в твоем понимании) нужно сделать для того, чтобы он перестал "им" быть?
Аноним 02/11/23 Чтв 00:18:04 584450 19
>>584449
Хуй знает может ли что либо помочь этому гадюшнику. Но закрепленный Татьяныч это такой пиздец кринжовый был.
Аноним 02/11/23 Чтв 02:07:25 584452 20
>>584346 (OP)
Наконец эта ублюдская рожа исчезла, спасибо.
Аноним 02/11/23 Чтв 03:11:41 584453 21
>>584346 (OP)
Каким образом в электрической цепи при увеличенном напряжении, но при той же силе тока, можно передавать больше электроэнергии, если сила тока по определению это количество электричества проходящее через проводник за единицу времени?
Аноним 02/11/23 Чтв 03:48:01 584454 22
>>584453
Напряжение распространяется со скоростью света и заставляет электроны в конце цепи двигаться. Успешность их борьбы с сопротивлением в средине цепи не так важна.
Аноним 02/11/23 Чтв 03:49:52 584455 23
В аналогии с водой, напрежение это давление, а сила тока это сколько воды протекает. Вот и думай
Аноним 02/11/23 Чтв 03:52:30 584456 24
>>584454
скорость света в материале
конец цепи там где питаемое устройство, средина всё что между ним и источником энергии
Аноним 02/11/23 Чтв 08:22:00 584459 25
Как происходит редактирование генов с CRISPR/Cas на практике? Ну вот я знаю, что есть такие-то нуклеазы с наноножницами, они по рофлу режут гены и все такое. Но как от этой теории прийти к практическому применению? Как сделать светящуюся гигамышь убийцу?
Аноним 02/11/23 Чтв 12:01:11 584464 26
>>584459
Берешь гены гигантизма, светимости и неукротимой жажды человекоубийства и вставляешь их в ДНК мышки.
Аноним 02/11/23 Чтв 12:22:41 584468 27
>>584464
Как их брать и как вставлять? Общую теорию на уровне научпопа то я знаю.
Аноним 02/11/23 Чтв 13:48:08 584470 28
>>584459
Берешь клетку, растворяешь у нее мембрану так, чтобы ДНК не пострадало.
Берешь бактерию, вытаскиваешь у нее специальные белки, которые аккуратно режет и превращает ДНК в РНК без каких либо превращений.
Выискиваешь из вырезанных РНК нужную.
Берешь бактерию с кассетами и впрыскиваешь в нее фрагмент нужной РНК. Вуаля фрагмент РНК запихнут в ДНК.
Разбираешь бактерию на части, получаешь кассеты.
Выбираешь нужную.
Эти кассеты пускаешь в клетку, а они уже сами себя врежут в ДНК.
Аноним 02/11/23 Чтв 13:53:16 584471 29
>>584470
Да меня не идея интересует, а как там это все в лаборатории устроено. Что в пробирках и каким образом получается генная модификация. Как пришли от идеи, к инъекции чудо жидкости с переносчиками ГМО.
Аноним 02/11/23 Чтв 16:58:00 584475 30
>>584471
А хули тут не понятого?
Выращивают нужные клетки или бактерии, потом их поливают всякой химий, получившийся рассольчик запихивают в центрифугу, где все разделяется.
ДНК и РНК это кислоты и они легко обматываются вокруг ионов. Поэтому есть целая куча реактивов для их связывания.
С белками сложнее, но те что нужны, можно вытащить вместе с ДНК и РНК, ибо они связанны с друг другом.
Получившиеся вещества уже дополнительно фильтруют, поливают другой химией и эту гадость вкалывают клетках или бактериям под микроскопом.
Аноним 02/11/23 Чтв 19:17:09 584480 31
На нашу жизнь влияют как-то квантовые случайности, которые противоречили бы принципу причинности? Я имею ввиду не кот Шрёдингера, а допустим, согласно закону причинности человек должен выбрать а, но квантовая неопределённость говорит б. Или это работает только на оооочень больших дистанциях?
Аноним 02/11/23 Чтв 19:20:56 584482 32
Рандомайзеры действуют по какому-то алгоритму, или там в основе тоже квантовая неопределенность?
Аноним 02/11/23 Чтв 21:50:59 584485 33
Уверен, что ответа не получу, поскольку здесь отвечают только на стандартные вопросы по теориям относительности. Но попробовать стоит.

Есть жиробас 100кг, который прыгает с метрового стола на весы. Какой вес покажет дисплей? Упругость ног, весов и пола, а также сопротивление воздуха и прочий кринж игнорируем.
Аноним 03/11/23 Птн 00:33:29 584488 34
>>584454
>>584455
Ну так каким именно образом большее напряжение позволяет передать больше ватт энергии, при прочих равных, если за скорость и объем передачи электричества отвечает сила тока? Откуда берётся эта энергия? Как это выглядит на атомном уровне?
Аноним 03/11/23 Птн 04:26:34 584495 35
>>584488
Напряжение передают фотоны, от электронов требуется только не мешать.
Аноним 03/11/23 Птн 06:40:20 584498 36
Аноним 03/11/23 Птн 07:23:55 584499 37
Аноним 03/11/23 Птн 15:22:52 584516 38
>>584453
ток — количество зарядов
напряжение — их скорость
при сохранении количества зарядов можно передавать больше электроэнергии, увеличив их скорость
Аноним 03/11/23 Птн 17:20:31 584527 39
>>584516
а еще есть такая штука как сопротивление
Аноним 03/11/23 Птн 18:21:28 584529 40
>>584527
есть, да
двигаясь внутри проводника заряды иногда сталкиваются с атомами, из которых состоит проводник, отдавая им часть своей кинетической энергии
атомы проводника от этого начинают дёргаться сильнее обычного (=повышение температуры)

чем больше зарядов — тем чаще происходят столкновения
чем выше скорость зарядов — тем больше энергии они отдают
Аноним 03/11/23 Птн 20:09:25 584538 41
Аноним 03/11/23 Птн 20:32:30 584542 42
>>584485
>>584538
не найдёшь ты весов, которые в описанных условиях покажут эффекты теории относительности

ускорение свободного падения 9.78м/с/с
то есть пролетев метр, он будет двигаться со скоростью 9.78м/с
c=1, тогда скорость в момент приземления в единицах скорости:
9.78 / 299792458 = 3.262256851037927e-8
релятивистская масса = масса / лоренц-фактор
лоренц-фактор = 1 / (1 - v^2)
масса чела = 100 / ((1 - ((3.262256851037927e-8)^2))^0.5) = 100.00000000000006

твои выдуманные весы покажут 100.00000000000006кг
Аноним 03/11/23 Птн 20:33:45 584543 43
>>584542
Шизик, таблетки прими.
Аноним 03/11/23 Птн 20:33:47 584544 44
>>584542
быстрофикс
лоренц-фактор = 1 / ((1 - v^2)^0.5)
Аноним 03/11/23 Птн 20:34:25 584545 45
>>584542
опечатался, быстрофикс:
лоренц-фактор = 1 / ((1 - v^2)^0.5)
остальное правильно
Аноним 03/11/23 Птн 20:36:59 584546 46
>>584545
Любые весы покажут 150+, иди нахуй шизоид.
Аноним 03/11/23 Птн 20:40:11 584547 47
>>584546
вопрос был про теорию относительности
и про несуществующие весы, для которых можно пренебречь упругостью
Аноним 03/11/23 Птн 20:44:47 584548 48
>>584547
Нет, иди перечитывай вопрос пока не прозреешь.
Аноним 03/11/23 Птн 20:48:50 584549 49
Аноним 03/11/23 Птн 23:18:29 584558 50
>>584480
Да. Пример локализация электрона в атоме, с классической точке зрения он там движется выше скорости света.
Квантово вырожденное вещество - всякие сверхтекучести и сверхпроводимости, в них образуется вихри, которые ебут классическую казулаьность.
Ну общий пример это квантовые вычисления как таковые.
Аноним 04/11/23 Суб 02:08:28 584563 51
>>584529
Если напряжение равно произведению тока и сопротивления, то. Получается что на напряжение ни ток, ни сопротивление не влияет.
...что странно. Зачем тогда провода нужны?
Аноним 04/11/23 Суб 07:32:10 584567 52
>>584563
>Зачем тогда провода нужны?
R = p * l / S, где
p - удельное сопротивления материала
l - длина отрезка проводника
S - площадь поперечного сечения (при условии, что по всей длине оно одинаковое)
Аноним 04/11/23 Суб 16:12:14 584587 53
>>584563
>Зачем тогда провода нужны?
Провод нужен для проведения поля. Обычные материалы вокруг нас не являются металлами и вообще плохо проводят ток, или проводят не так, как нам надо (ток связанных зарядов, смещения, etc).
Аноним 04/11/23 Суб 18:00:49 584599 54
Тупой, наверное, вопрос ответ на который я видимо сам и напишу.
Короче, в рабочей машине адово ебашит статическим электричеством.
Вопрос, если купить антистатическую обувь то вопрос ведь не решится? Ибо бьет током по голым рукам ведь.
Правильно думаю?
Аноним 04/11/23 Суб 22:50:47 584603 55
>>584599
У тебя спецовка с мехом? Просто элекризуешься от одежды, как эбонитовая палочка на уроке физики.
Аноним 05/11/23 Вск 01:56:30 584607 56
>>584349
Ну можно то же самое по хождение и млекопитающих сказать. Многие могут стоять и ходить с рождения, но человек не умеет даже голову держать, начинает ползать только в полгода, а ходить в год-полтора.
Видимо, у птиц есть какие-то важные плюсы, которые исключают полёт с рождения, а у птерозавров их не было.

Можно потеоретизировать.
У птиц кости полые, более хрупкие, зато проще летать во взрослом возрасте, например.
Аноним 05/11/23 Вск 06:09:40 584612 57
>>584599
Поставь цепь если у тебя грузовик и антистатический хлястик (или как он там называется) если легковушка.
Аноним 05/11/23 Вск 07:44:54 584616 58
>>584599
Я просто чехлы заземлил. Взял ленту-скотч для кондейщиков и по бокам чехлов нанес тонкую линию к самому тоннелю, прямо за ручником. И все. Плюшевые чехлы тоже генерят много статики из-за синтетики и меха.
Аноним 05/11/23 Вск 15:39:22 584621 59
Безымянный.jpg 63Кб, 800x600
800x600
Привет аноны.
Будет ли в теории работать схема пикрил? Вопрос не в экономической целесообразности, потому-что она очевидна никакая, а именно в технической возможности.
Есть Озеро на стометровом холме. Если подвести от него трубу к дому у подножья, то из трубы по закону сообщающихся сосудов будет постоянно литься вода. А если эту воду будут греть земные недра, то она будет двигаться в том же направлении и обогревать дом?
Аноним 05/11/23 Вск 15:49:57 584622 60
>>584621
Мне кажется, она не будет у тебя успевать нагреваться.
Аноним 05/11/23 Вск 16:56:32 584625 61
Почему селекционеры не могут вывести более плодоносный шафран? Типа сильно упадёт качество специи, потому что природа тупо не сможет её столько синтезировать?
Аноним 05/11/23 Вск 17:30:08 584626 62
>>584625
>шафран
При селекции ирисовых у них возникают проблемы с количеством и качеством эндосперма или клеток зародыша. Гибридизация ирисовых требует десятилетий. А сейчас - тяп-ляп и в продакшн. Деньги делай или сделает Какой-нить Глоценко\Дрочун\Писюнчук.
Аноним 05/11/23 Вск 18:16:25 584627 63
Почему официальная новука отрицает существование гавваха?
Ацтеки видимо в своих пирамидах просто по приколу устраивали массовые жертвоприношения?
Аноним 05/11/23 Вск 19:07:50 584628 64
>>584627
Почему отрицает? Наука ничего не отрицает, она лишь говорит "у нас нет пруфов существования"
Аноним 05/11/23 Вск 19:29:37 584629 65
1-1-1.jpg 51Кб, 555x370
555x370
А что если использовать перегретый пар в ледоколах, но топить льды не сверху, чтобы пар не рассеивался в атмосфере, а снизу, чтобы струя пара разделяла воду и лед прослойкой, смещала центр тяжести льда выше точки опоры и тот бы сам ломался под своим весом. Взлетит?
Аноним 05/11/23 Вск 20:40:39 584631 66
Аноним 05/11/23 Вск 22:42:13 584634 67
>>584629
Ледокол давит лед ВЕСОМ. Он не греет лед.
Аноним 05/11/23 Вск 22:44:33 584635 68
>>584629
А еще - центр тяжести льдин, как и у айсбергов - в нижней части льдин. И его не сместить.
Аноним 05/11/23 Вск 22:47:55 584636 69
>>584627
Ну хорошо. Вот я ученый, я начал вникать в твое сверхъестественное. С чего начнем исследование?
Аноним 06/11/23 Пнд 02:05:06 584642 70
>>584636
Исследовать принцип выделения гавваха, на примере той же СВО и зиккурата на Красной площади. Понять, как перестать кормить тварей.
Аноним 06/11/23 Пнд 02:08:51 584644 71
>>584628
Вот есть такой пруф, как НЛО, есть документально подтвержденные похищения и садистские опыты над людьми гуманоидами. Но официальные ученые это не признают, уфология у них проходит по разделу креационизма и плоской Земли.
Аноним 06/11/23 Пнд 03:34:14 584645 72
>>584644
>есть документально подтвержденные похищения
Да, я в документалке "4 вид" видел видосы с камер. Там даже видосы с видеорегистраторов копов есть.
Аноним 06/11/23 Пнд 03:54:58 584646 73
>>584645
Хуй с ними – с копами. Чего скажешь про три известных видео: go fast, gimbal и flir?
Аноним 06/11/23 Пнд 03:56:37 584647 74
>>584642
Выдели предмет исследований. С научной точки зрения ты предлагаешь найти душу, выделить ее эманации, изучить ее спектр частот? Какой научный инвентарь необходим?
Аноним 06/11/23 Пнд 07:41:31 584648 75
>>584647
>Какой научный инвентарь необходим?
Котел - 1шт
Шар хрустальный - 1шт
Палочка волшебная - 1шт
Очаг с адским пламенем - 1шт

Расходники: Глаза жаб, крылья летучих мышей, кровь девственницы, змеиные языки - по необходимости.
Аноним 06/11/23 Пнд 09:34:18 584649 76
>>584648
Не, я начну с электрографа Кирлиан. Вышеуказанные препы и оборудование - это прямо высшая магия.
Аноним 06/11/23 Пнд 11:22:11 584651 77
>>584647
Компьютер, мозги, что-то чем можно их сканировать.
Аноним 06/11/23 Пнд 13:03:08 584654 78
>>584651
Я вижу, ты еще повидлодаван и не можешь в джедайские речи. Зачем ты тут, отрок? Тебе стоит сесть да диету воздержания и повысить уровень мидихлорианов в себе, чтобы не задавать червеимперские вопросы славному республиканцу...
Аноним 08/11/23 Срд 04:58:01 584699 79
NSRWAirPump1.png 131Кб, 480x528
480x528
Как работают эти клапаны ебаные? Обясните
Аноним 08/11/23 Срд 06:16:58 584700 80
>>584699
Как дверь которая может отрываться только в одну сторону.
Аноним 08/11/23 Срд 14:00:16 584704 81
>>584700
Как жопа, внутрь огурцы впускает, обратно не выпускает
Аноним 08/11/23 Срд 14:33:45 584705 82
>>584704
За этим наверняка стоит интересная история.
Бильярд-кун, ты ли это?
Аноним 09/11/23 Чтв 13:12:31 584719 83
imgstouch (6).png 86Кб, 720x720
720x720
Допустим я хочу знать всю ныне существующую математику, абсолютно все разделы до нынешнего времени, сколько примерно часов у меня уйдет на это при среднестатистической обучаемости?
Аноним 09/11/23 Чтв 13:42:03 584720 84
>>584719
Есть брать чистую математику, только только абстракции, только общие случаи и обобщения, только хардкор, то это примерно 20+ лет по академическим программам.
Если подключать сюда прикладные вещи и теорию алгоритмов, то это растянется на тыщи лет.
Аноним 09/11/23 Чтв 14:09:03 584722 85
>>584719
Если у тебя полный начальный математический базис, и есть алгоритмы обучения, то 400 часов проанализированного материала. И примерно 200 часов на закрепление с проверкой.
Только зачем тебе это?
Аноним 09/11/23 Чтв 23:23:32 584732 86
Физики, когда квантовая гравитация? Накидаю за щеку если не завтра.
Аноним 10/11/23 Птн 15:17:52 584749 87
Почему в природе 3 поколения частиц? Так боженька захотел?
Аноним 10/11/23 Птн 16:43:25 584751 88
9-fmafEXtGs.jpg 40Кб, 361x480
361x480
Почему если дрочить или ебаться с похмелья, то оргазм в разы круче, чем в обычном состоянии и если в обычном состоянии меня хватает на 2-3 раза, то после похмелья нормой будет 7-10 раз за сутки? Спрашивал на медаче, там не ответили, но сказали, что подобная ситуация при отходняках с амфы.
Аноним 10/11/23 Птн 18:20:57 584754 89
солнечный полюс, земной полюс, 2 реки, 3 облака нижнего уровня, 0 облаков верхнего уровня, 1 ледяная гора, одна лавовая гора и 3 фонтана и на луне тоже самое, а солнце, меркурий и венеру нахуй.
Аноним 10/11/23 Птн 19:06:14 584755 90
>>584749
>3 поколения частиц
Поколения есть у лептонов, а не у частиц. Частицы лишь делят на три группы.
Аноним 10/11/23 Птн 19:11:37 584756 91
с какой вероятностью помрёшь, выпив литр напополам масла и водки с противорвотным?
Аноним 10/11/23 Птн 19:17:20 584757 92
Аноним 10/11/23 Птн 20:17:18 584759 93
>>584756
У тебя от поллитра масла придёт пизда поджелудочной, особенно если очень холодной водой запить. Тут даже водка не нужна.
Аноним 10/11/23 Птн 20:19:31 584762 94
всем привет, всю жизнь интересовался гуманитарными дисциплинами, но испытывал небольшие комплексы по поводу того, что в точных науках я всегда был опущ, иногда появляется интерес влезть в математику\физику\химию, но я не знаю что именно выбрать. С другой стороны, не хочется решать кучу абстрактных задач и формул из разряда "рассчитайте в каком очке встретятся два атома, выпущенные из анальной пушки". Ну так вот, реквест такой: что можно изучить, чтоб была практическая польза\можно было бы проводить всякие эксперименты в реал лифе или на компутере (если возможно)? ну вот химики реально сидят дома со своими пробирками и кайфуют. Но это дорого и опасно, что ещё можно придумать?
Аноним 10/11/23 Птн 21:48:02 584764 95
>>584755
Поколения есть как у лептонов, так и у кварков. Причем есть прямое соответствие по квантовым числам. В природе прям три копии одного говна.
Всякие бозоны порождаются калибровочными симметриями над ними.
Аноним 11/11/23 Суб 01:51:43 584772 96
>>584645
Мишаня, ты?

21 октября 2022 г. NASA объявило состав группы по изучению неопознанных аномальных явлений. Это исследование будет сосредоточено на несекретных данных.

23 декабря 2022 г. президент Байден подписал закон Джеймса М. Инхофа о полномочиях в области национальной обороны на 2023 финансовый год. В этот закон были включены специальные меры защиты для разоблачителей UAP (спасибо u/joeyisnotmyname за подборку этих страниц). Я повторю это еще раз: в NDAA на 2023 год были включены меры защиты информаторов КОНКРЕТНО в отношении UAP.

1 июня 2023 г. НАСА раскрывает информацию о наблюдениях НЛО на Ближнем Востоке на первой в истории публичной встрече, посвященной наблюдениям НЛО.

6 июня 2023 г., всего через полгода после подписания Байденом закона NDAA 2023 г., офицер ВВС США и бывший сотрудник разведки Дэвид Груш становится разоблачителем НЛО, утверждая, что в США существует сверхсекретная программа по поиску разбившихся НЛО.

9 июня 2023 г. Томас А. Монхейм, генеральный инспектор разведывательного сообщества, считает заявления Груша "срочными и заслуживающими доверия", что открывает путь к проведению слушаний в конгрессе.

14 июля 2023 г. лидер сенатского большинства Чак Шумер включил в NDAA в качестве поправки свой новый закон о раскрытии информации об UAP от 2023 г. В тот же день NDAA был принят. Я настоятельно рекомендую прочитать как можно больше текста Закона о раскрытии информации об UAP, если у вас есть на это время, - формулировки очень точные и убедительные.

18 июля 2023 г. координатор и пресс-секретарь Совета национальной безопасности Джон Кирби признал, что НЛО и UAP начинают создавать проблемы для ВВС США.

26 июля 2023 г., всего через полтора месяца после того, как Груш сообщил об этом, состоялись двухпартийные слушания в Конгрессе по UAP. Конгрессу предоставляется 177-страничная хронология, которая начинается с 1947 года. На этих слушаниях дают показания разоблачитель Дэвид Груш, Дэвид Фравор и Райан Грейвс. В течение почти двух часов их допрашивали о личном опыте, имеющихся у них доказательствах, а также о том, как следует поступать в дальнейшем. В связи с тем, что Груш пользуется официальными каналами защиты информаторов, он не может разглашать сведения, которые остаются конфиденциальными, но предлагает имена как враждебных, так и сотрудничающих свидетелей, а также конкретные места нахождения найденных материалов, если это будет делаться по соответствующим каналам на закрытом заседании, т.е. на S.C.I.F. К сожалению, на слушаниях не было времени для заключительных слов, но вы можете прочитать заключительное слово Груша здесь (а также его вступительное слово). Из его заключительного выступления мне особенно запомнились следующие цитаты: "Действительно, будущее нашей цивилизации и наше понимание места человечества на Земле и в космосе зависит от успеха именно этого процесса" и "Продвигаясь по этому пути, мы сможем обеспечить необычайный технологический прогресс в будущем, когда наша цивилизация превзойдет нынешние достижения в области двигателей, материаловедения, производства и хранения энергии".

27 июля 2023 г. Сенат принял МНОГОЧИСЛЕННЫЕ меры UAP, включая презумпцию немедленного раскрытия информации, дополнительные меры защиты информаторов и др.

13 Сентября 2023 В Мексиканском конгрессе выставили на обозрение трупы "инопланетян"

14 сентября 2023 НАСА выкатило отчет главный вывод которого «Главный вывод из исследования заключается в том, что еще многое предстоит узнать»

18 октября 2023 года Годовой отчет о неопознанных аномальных явлениях за 2023 год был представлен Конгрессу Министерством обороны и Управлением директора национальной разведки (ODNI) в среду, согласно пресс-релизу Пентагона, выпущенному в тот же вечер.
Аноним 11/11/23 Суб 10:15:29 584776 97
>>584762
Физика электромагнитных волн. Можешь всякие приёмники-передатчики конструировать, гаусс-пушку собрать, вкатиться в радиолюбительство или клепать на ардуино какие-нибудь тестеры. Всё это относительно дёшево и безопасно, плюс находится на стыке физики, математики, инженерии, а в случае ардуино — ещё и программирования. Научишься работать руками и писать скрипты.
Аноним 11/11/23 Суб 18:09:14 584786 98
Пространство не гнется. У него нечему гнутся. Тот, кто выдает изменение кривую траекторию за кривое пространство - мошенник или дурак.
Пространство реального мира 3х-мерно. Мерность пространства - это выбор. Люди для удобства выбрали угол 90, три линии под углом 90 перекрывают все пространство. Нет возможности провести четвертую. Кто пытается рассказать про 4х-мерное, 5-ти,23, 65 мерное пространство является или мошенником, или дураком.
Пространство не может быть двумерным и одномерным. Это подмена терминов. Кто называет площадь и прямую пространством является мошенником или дураком.
Мерности пространства нельзя сложить с временем и получить пространственно-временной континуум. Как нельзя сложить пространственные мерности с ветром и получить пространственно-временной континуум. Нет, ну сложить то конечно можно, но это мир гарри поттера или дурки, а не науки. Любой второклашка знает, что складывая 3 апельсина с 1 тапкой не получить ни 4 апельсина, ни 4 тапки, потому что у этих предметов разные свойства. Любой, кто так делает в науке - либо мошенник, либо дурак.
У пространства нет конца. У пространства нет начала. У пространства нет середины. У пространства нет свойств, кроме возможности вмещать и давать свободно перемещаться в любом направлении материи. Из этого следует строгий факт, что пространство бесконечно в любом из направлений. Кто рассказывает про конец пространства, его раздутие, или зависимость пространства от времени является или мошеником или дураком.
Я могу долго продолжать, но не вижу смысла.

Любая теория, акисоматика которой начинается с подобных подтасовок, является или мошеннической, или тупой. Нет смысла обсуждать что-то в этой теории, если ее аксоматика построена на подлоге. Это не наука, это фентези, бред, мошенничество, обман, развод.

Если кто-то хочет что возразить - включаете камеру на вашем мобильном телефоне и покажите треду как вы гнете пространство или засовываете руку в 4е измерение. Ждем простейший эксперимент, а не фентезийное словоблудие.
Аноним 11/11/23 Суб 18:47:59 584787 99
>>584786
Откуда дурная паста? С ютуба воруешь смешных чуваков?
Аноним 11/11/23 Суб 19:06:08 584791 100
>>584786
>Нет возможности провести четвертую.
Время же.
>5-ти
Тут сложнее.
Аноним 11/11/23 Суб 19:25:30 584793 101
>>584787
написал за 3 минуты.
дурная? данный тезис является бредом потому что " твой аргумент".

>>584791
>Время
не имеет отношения к пространству, не является его мерностью.
описывает скорость протекания процессов уже внутри пространства.
Аноним 11/11/23 Суб 20:27:34 584795 102
>>584793
Каждый раз когда мы добавляем новую ось. Мы видим, что каждой координате новой оси, соответствует определенное состояние
описываемого предыдущими осями пространства. Время работает точно так же как и еще одна добавленная к пространству ось.
Аноним 11/11/23 Суб 20:38:08 584797 103
>>584795
когда мы к осям 1,2,3 добавляем ось 4 под углом в 90 градсуом, мы получаем ось, которая соответствует оси 1, когда добавляем 5 ось, то она соответствует оси 2, когда добавляем 6 ось, то она = оси 3, когда седьмую ось, то она соответствует оси 1. мы может проделать экспериментально 2354656843453454234534413123123124345656867342342322334345465676787 итераций и каждая новая ось будет равна 1,2 или 3.

если это утверждение неверно, пожауйста возьмите мобилу, уголок под 90 и покажите миру феномен несоответсвия.

и причем тут время?
Аноним 11/11/23 Суб 22:07:16 584798 104
23657689-4.jpg 71Кб, 640x426
640x426
Есть тут тред про чудеса и тайны?

Нашел сегодня случайно такое, пещера/шахта из мегалитов с прямыми углами
https://ru.m.wikipedia.org/wiki/Харахора
Учёные, ответьте, что это? Вход в шамбалу, город атлантов или база рептилоидов?

Единственное гуглится что какой-то чел сказал что это якобы естественное образование. Но если так должны же быть аналоги, а они есть? Как например базальт трескается на бревна, это дивно, но это в тысячах мест такое, и миллион фоток гуглится. Кто нибудь знает про прямоугольные дырки в горах?

Ещё там странная небольшая пещерка есть, но явно рукотворная https://viktorkotl.livejournal.com/115863.html

И свастика видимо сделанная диверсантами или тайными нацистскими исследователями во время войны дидов 1942. Хотя пещера обнаружена только в 2011 после обвала.
Аноним 11/11/23 Суб 22:11:00 584799 105
>>584797
Ты хорошо знаешь линейную алгебру? Десятимерное пространство как-нибудь представляешь?
Аноним 11/11/23 Суб 22:18:44 584800 106
>>584799
если примат что-то может представить или не может, это не говорит о том существует это или нет.

10 мерное пространство не существует, но шиз из больнички степанова-скворцова или из больнички кащенко может его представить. а шиз-математик даже описать наукообразными циферками. но то, и то являются формами глупости.
Аноним 12/11/23 Вск 00:48:38 584801 107
>>584800
Математические модели не описывают реальность, они лишь дают инструмент для предсказания. Трехмерное пространство настолько же нереально, насколько и четерехмерное
Т.е. ты дурачок для нас не потому что, внезапно обнаружил, что четырехмерное пространство-время это выдумка, а потому что считаешь, что трехмерное пространство существует не как абстракция математическая
Аноним 12/11/23 Вск 07:40:08 584804 108
>>584800
Это уровень Лысенко какой-то. Стыдно должно быть такую хуйню нести.
Аноним 12/11/23 Вск 08:59:07 584806 109
>>584797
>когда добавляем 5 ось, то она соответствует оси 2
Нет, не соответствует. Она соответствует всей предыдущей размерности разом.
Аноним 12/11/23 Вск 09:08:38 584807 110
>>584798
Вполне возможно что что-то какую-то пещеру естественного происхождения допилили фашисты и местные коллаборационисты, чтобы прятаться в этой хуйне. 100-200 человек имея примитивную технику не только такую хуйню могут сделать и выпилить.
Аноним 12/11/23 Вск 09:26:54 584809 111
Науч... научач... наукач .. в общем объясни почему можно до дыма нагреть влажные листья в микроволновке, но сухим будет абсолютно похуй. Типа вода так разогревается что ... поджигает листья?...........
Аноним 12/11/23 Вск 12:35:40 584810 112
>>584809
Микроволновка разогревает воду которыми пропитана влажные листья.
Аноним 12/11/23 Вск 12:38:41 584811 113
>>584453
Это невозможно. Изменив напряжение изменится и сила тока
Аноним 12/11/23 Вск 12:55:01 584812 114
Какой кинетической энергией обладает электрон на первой орбите в атоме водорода?
Аноним 12/11/23 Вск 12:56:39 584813 115
>>584809
В микроволновке стоит лампа-генератор (магнетрон), которая создает волны такой частоты, что атомы водорода начинают резонировать и нагреваться. Именно потому можно разогревать вещества, в которых есть вода, сахар или свободный водород.
А также сушить СВЧ всякие вещи, от белья и до древесины и камня.
Отсюда, кстати, все мифы про разваливание еды на атомы. Это правда.
Аноним 12/11/23 Вск 13:00:59 584814 116
изображение.png 4Кб, 164x56
164x56
Аноним 12/11/23 Вск 13:03:10 584815 117
>>584813
Не водород, а колебательные уровни связей -OH
Молекулярный водород ты так хуй нагреешь

И чтобы разломать молекулу возбуждая колебательные уровни, это надо чтобы у тебя молекула разъебывалась даже от простого нагревания на плите
Аноним 12/11/23 Вск 13:08:40 584816 118
>>584815
Нет, гидроксо-ион имеет другую частоту резонанса. Именно атомарный водород. Тупо, как в МРТ, только с другим физическим входом.
Попробуй нагреть метан, ради интереса.
>>584815
>И чтобы разломать молекулу возбуждая колебательные уровни, это надо чтобы у тебя молекула разъебывалась даже от простого нагревания на плите
Тут без комм. Мне не интересно спорить, успехов вам, здоровья, пенсий и т. д.
Аноним 12/11/23 Вск 13:56:29 584818 119
Почему человек (да и большинство животных в принципе) не может есть (полноценно усваивать) траву, листья, кору, корешки, древесину? Разве это бы не добавляло ахуенных эволюционных преимуществ и живучести?
Аноним 12/11/23 Вск 14:13:20 584819 120
>>584818
Выживание просчитавшихся и не сумевших достать нормальной еды лузеров такое себе эволюционное преимущество.
Ну и травку придется жевать круглосуточно чтоб наесться никакой тебе науки и ремесел. И аппарат для жевания постоянно поддерживать даже когда не пользуешься.
Аноним 12/11/23 Вск 15:12:56 584820 121
>>584804
математика появилась когда примат начал описывать количественные отношения и геометрические формы реального мира.
>инструмент для предсказания
форм и количественных отношений реального мира
>четерехмерное
если примат начинает описывать математикой мир нереальный, то он либо шиз, либо мошенник, либо писатель. 16 лапые кошки с 4 хвостами в 10 мерном пространстве существуют только у шизов, мошенников, писателей. это не имеет отношения к реальности и математике.

>трехмерное пространство существует не как абстракция математическая
т.е. ты живешь в абстракции? и сейчас из нее капчуешь?

>>584804
фееричная критика. в детском саду даже дети могут выдавить конструкцию это "глупость потому что ... ". ты не можешь.

>>584806
т.е. ты когда последовательно расскладываешь яблоки по 3м корзинам. 1 яблоко в 1, 2 яблоко во вторую, 3е в 3, 4 яблоко в 1, то 5 яблоко у тебя попадает сразу во все три? лол...
Аноним 12/11/23 Вск 15:15:43 584821 122
>>584810
>>584813
он спросил про поджигание. вы ему рассказали про кипячение воды. НОУКА
Аноним 12/11/23 Вск 15:16:58 584822 123
>>584816
В МРТ используется ядерный магнитный резонанс, он вообще не при чём тут
>Тут без комм. Мне не интересно спорить, успехов вам, здоровья, пенсий и т. д.
О... слился... ну после твоего примера неполярной молекулы водорода, понятно почему слился, думал тут школьники сидят только, отгадай почему пример с H2 нихуя не работает, ответ начинается на дип... и заканчивается ...ент
Аноним 12/11/23 Вск 15:23:42 584823 124
Аноним 12/11/23 Вск 16:09:10 584824 125
>>584820
Я живу в реальном мире, но осей x y и z нигде тут не нарисовано

Если такой умный, запиши мне пожалуйста видео на телефон, чтобы я увидел три декартовые оси и где ты к ним подходишь и руками трогаешь эти оси
Аноним 12/11/23 Вск 16:41:05 584825 126
>>584824
а я где то утверждал, что оси существуют как материальные объекты?
или ты создал соломенное чучело и стал с ним воевать?
Аноним 12/11/23 Вск 16:41:59 584826 127
>>584821
Вода поглощает электромагнитные волны излучаемые микроволновой антенной. Поглощая эти электромагнитные волны вода нагревается, повышается его температура это энергия переходит листьям и они загораются.
Аноним 12/11/23 Вск 16:43:30 584827 128
>>584814
Это я понимаю. Но где доказательство, что электрон имеет какую-то кинетическую энергию?
Аноним 12/11/23 Вск 16:44:16 584828 129
>>584820
>т.е. ты когда последовательно расскладываешь яблоки по 3м корзинам
Яблоки и корзины это не размерности пространства.
Аноним 12/11/23 Вск 17:04:48 584829 130
>>584828
это пример последовательности счета.
ты сказал что >Нет, не соответствует. Она соответствует всей предыдущей размерности разом.

рандомно поставленный треугольник в пространстве образует первую плоскость
поставленный к нему второй треугольник под углом в 90 образует вторую плоскость
третий треугольник поставленный углом в 90 к первому и второму треугольнику образует третью плоскость

поставь пожалуйста 4й треугольник под углом 90 к 3 предыдущим так, чтобы он >соответствовал всей предыдущей размерности разом, а не одной из трех
поставь пожалуйста 5й треугольник под углом 90 к 3 предыдущим так, чтобы он >соответствовал всей предыдущей размерности разом, а не одной из трех
поставь пожалуйста 6й треугольник под углом 90 к 3 предыдущим так, чтобы он >соответствовал всей предыдущей размерности разом, а не одной из трех

сними это на видео.
Аноним 12/11/23 Вск 17:07:59 584830 131
>>584826
от чая в микроволновке можно прикурить сигарету?
Аноним 12/11/23 Вск 17:12:58 584831 132
>>584830
Просто возьми кусочек фольги и положи на сигу в СВЧ.
Аноним 12/11/23 Вск 17:20:21 584832 133
>>584827
Еще раз на формулу посмотри, скудоумец.
Аноним 12/11/23 Вск 17:29:28 584834 134
>>584831
но листья не являются металлом
Аноним 12/11/23 Вск 17:34:33 584835 135
>>584829
>это пример последовательности счета.
Да, но пример последовательности счёта это не размерность пространства.
Аноним 12/11/23 Вск 17:40:29 584836 136
>>584834
В случай листьев вода в них испаряется (переход из жидкого состояния в газ) и выходит, но передаёт часть энергии сухим листьям вот они и загораются.
Аноним 12/11/23 Вск 17:47:33 584837 137
>>584835
пример счета количества не пример счета количества. понял
Аноним 12/11/23 Вск 18:41:18 584845 138
>>584837
Нет, не правильно. Пример пространства это не пример счёта.
Аноним 12/11/23 Вск 19:59:29 584847 139
>>584845
пример счета пространственных мерностей - это не пример счета. ясн
Аноним 12/11/23 Вск 20:11:03 584848 140
>>584825
Ну если что "нематериальное" то это манямирочек и обман
Аноним 12/11/23 Вск 20:20:39 584849 141
>>584848
пространство материально?
Аноним 12/11/23 Вск 20:56:06 584850 142
>>584849
В нашем ограниченном восприятии - не материально.
А вообще - материально.
Аноним 12/11/23 Вск 20:59:13 584851 143
>>584836
Чо у тебя за сиги такие, что мокрый кончик загорается?
Аноним 12/11/23 Вск 21:05:46 584852 144
>>584850
что у пространства является материальным? где это найти? покажешь?
Аноним 12/11/23 Вск 21:52:00 584857 145
>>584852
>что у пространства является материальным?
Гомеоморфность его топологии. И вообще, это не моя тематика, обратись к псиъху номер 2, уж он точно все знает, раз уж метрику углами меряет.
Аноним 12/11/23 Вск 21:56:54 584858 146
>>584857
эта гомеоморфность с тобой в комнате? можешь показать ее?

> раз уж метрику углами меряет.
а чем ее надо мерить? хвостами котов?
Аноним 12/11/23 Вск 22:11:03 584860 147
>>584858
>эта гомеоморфность с тобой в комнате? можешь показать ее?
Я же не Перельман. Это он ее находит везде и доказывает. Мне достаточно и пятимерной конструкции. А уж ее наличие, как подтверждение теории метрики, я вижу в материальном проявлении всего вокруг себя и во всем перцепционном многообразии.
Аноним 12/11/23 Вск 22:11:37 584861 148
>>584858
>а чем ее надо мерить? хвостами котов?
Языками лахтовиков.
Аноним 12/11/23 Вск 22:28:02 584863 149
>>584860
>Это он ее находит
>Мне достаточно и пятимерной конструкции. я вижу в материальном проявлении всего вокруг себя

т.е. шиза одного чела тебя не устроила, поэтому ты придумал свою.

>во всем перцепционном многообразии.

я тебя научил как видеть 3. это высота, ширина, длина. берешь прямой угол и проверяешь строгость данного утверждения. научишь как видеть 4 и 5 измерение без помощи санитаров?
Аноним 12/11/23 Вск 23:14:49 584867 150
>>584863
>т.е. шиза одного чела тебя не устроила, поэтому ты придумал свою.
Аму-Няму! Покарай его!
>>584863
>научишь как видеть 4 и 5 измерение без помощи санитаров?
Для этого нужно собрать пять мужиков и диван, чел. Ты готов пройти в наш сороковой рад-град-градус? РГГ ждет тебя, сахарилло!
Аноним 12/11/23 Вск 23:15:01 584868 151
>>584807
Фашисты вроде туда не дошли. Кроме может диверсантов.
Аноним 12/11/23 Вск 23:18:40 584869 152
>>584826
Если вода нагреется до температуры возгорания она ебанет так примерно как тротил.
Аноним 12/11/23 Вск 23:19:58 584870 153
>>584836
Блэт, при испарении листья охлаждаются.
Аноним 12/11/23 Вск 23:22:09 584871 154
>>584759
так на видео мужик 3 литра масла выпил
Аноним 13/11/23 Пнд 01:43:08 584873 155
>>584871
Наеб. Мужик мочу пил.
Аноним 13/11/23 Пнд 02:33:52 584874 156
>>584836
>>584869
Водяной пар в незакрытой системе может поджечь целлюлозу?
Аноним 13/11/23 Пнд 06:10:01 584876 157
Если бы состав воздуха был не как сейчас (кислород 21, азот 78, остальное 1%), а резко изменился, например, кислород 30, азот 69, или кислород 40, азот 59, смогли бы люди дышать при таком составе атмосферы? Животные и растения были бы такими же или другими?
Аноним 13/11/23 Пнд 08:45:13 584879 158
>>584876
А тебе зачем, пидорас? Атмосферу засрать хочешь?
Аноним 13/11/23 Пнд 10:53:30 584895 159
>>584820
>фееричная критика. в детском саду даже дети могут выдавить конструкцию это "глупость потому что ... ". ты не можешь
Ты свои посты читал? У тебя куча сильных утверждений без доказательств, причём видно, что знаний по теме у тебя нет.
Например, мерность пространства это не выбор, это его свойство. Например, это количество векторов в базисе. Они не обязательно должны быть ортогональны друг другу, как ты писал, достаточно, чтобы они были линейно независимы. Так, как бы ты не выбирал базис для нашего пространства, в нём всегда будет три вектора, поэтому пространство трёхмерное.
Время прибавляют как четвёртое измерение, потому что оно хорошо вписывается в уравнения протекания процессов как четвёртый базисный вектор с множителем c (скорость света в вакууме).
Искривление пространства - это интерпретация тех же уравнений протекающих процессов. Причём, никакой другой интерпретации не существует. Например, LIGO пару лет назад засёк гравитационные волны - а это ни что иное как сжатие/разжатие пространства.
И так далее.
Я поэтому и назвал твои посты уровнем Лысенко - он так же нихуя не знал, подгонял действительное под желаемое, использовал ненаучные аргументы в спорах.
Аноним 13/11/23 Пнд 12:46:56 584900 160
Поцаны, а как же быть с тем, что пространство-время это вторичная структура, которая порождается алгебраическими свойствами комплексных пространств?
Аноним 13/11/23 Пнд 13:06:00 584901 161
>>584874
Нет, точно. Если его сжечь и успеть получить плазмоид в свч потоке - да. Но это нестандартные условия и свч обычно ломается при обратной связи.
Аноним 13/11/23 Пнд 13:09:16 584902 162
>>584876
Дышать смогли бы. Но выжили бы не все, точно.
Дело в том, что человек имеет зависимость - парциальное давление газов/давление жидкости внутри организма. И пока она изменится и подстроится под внешку - помрет 9/10. В принципе, я тебе описал происходящее в предыдущем вымирании.
Аноним 13/11/23 Пнд 13:10:43 584903 163
>>584900
В абстракции? Да пох.
Аноним 13/11/23 Пнд 23:15:04 584932 164
>>584901
откуда там дым тогда?
Аноним 14/11/23 Втр 00:26:46 584934 165
>>584895

> сильных утверждений без доказательств,
приведи любое

>видно, что знаний по теме у тебя нет.
а мне видно, что у тебя лапша на ушах

> мерность пространства это не выбор, это его свойство
фееричное заявление, я выбрал угол 60 вместо 90. у нас изменилось количество мерностей. их стало 5.

покажи в каком месте это свойство в рельности изменилось с 3 до 5. на видео запиши пожалуйста трасформацию "свойства".

>как бы ты не выбирал базис для нашего пространства, в нём всегда будет три вектора,
я только что показал дал тебе иной выбор в 5 векторов. хочешь заниматься бесмысленным математическим дрочиловом - выбирай рандомный угол, дели 360 на него, вычитай 1 и вуаля.

пространство девятимерно, потому что мы выбрали 36.
пространство восьмимерно, потому что мы выбрали 40.
пространство семимерно, потому что мы выбрали 45.
пространство шестимерно, потому что мы выбрали иррациональное 51,428....
пространство пятимерно, потому что мы выбрали 60.
пространство четырехмерно, потому что мы выбрали 72
пространство трехмерно потому что мы выбрали 90

опровергай. любая выбранная мерность конвертируется в другую без нарушений и равна целому.
неважно на сколько кусков ты выберешь порезать апельсин, хоть на 3, хоть на 45, хоть 456, он соберется обратно в целый объект.
хочешь страдать - намути 456 мерное пространство и считай в нем.

>Время прибавляют как четвёртое измерение,
ты не можешь прибавить секунды к метрам. у тебя не может быть один вектор в секундах, а второй в метрах. ты не можешь работать в этой системе. это является клиническим бредом, либо полнейшим непониманием скаляров, либо мошенничеством.

единственное как ты можешь получить четырехмерное пространство - это если ты выберешь другой базис. но времени там не будет.

если ты не согласен - пожалуйста включи телефон и продемонстрируй как ты двигаешь своей рукой. сначала вверх-вниз, потом влево вправо, потом назад-вперед, а потом вперед во времени - назад во времени.

если ты это не продемонстрируешь, то ты либо ошибался всю жизнь, и тебя обманули, либо ты все понимаешь, но пытаешься обмануть меня, приравнивая свойства скалярной величины измеряющей расстояния к величине, измеряющей скорость протекания процессов. ты не можешь приравнивать одно к другому и называть это все мерностями пространства..

ты не можешь приравнять улитку к носорогу и назвать их улитками.
ты не можешь приравнять пчелу к гравитации и назвать их пчелами.
в пятый раз для самых глупых и очень медленно - ты не можешь приравнять выбранную лично тобой мерность пространства с базисом в 90' в метрах и приравнять ее к времени в секундах, назвав тоже мерностью пространства.
это действие является нарушающим закон тождества, а следовательно наука тут заканчивается и начинается гарри-поттер.

>посты уровнем Лысенко
лысенко тут ты. над кантором все смеялись. над минковским тоже в 19 веке. все нормальные посоны делали химию, физику, радио, лампочку, аккумы, движки, паровые, двс, телеграф, телефон.

что миру дал твой минковский, эйнштейн и кантор? где результаты вашей шизы?

>ненаучные аргументы
сказал фанат ото и сто, построенных на постулатах. мило.
Аноним 14/11/23 Втр 01:23:39 584935 166
>>584934
Так легко же, просто держишь руку некоторое время неподвижно, вот она и подвинулась на c*t, и даже размерность в метрах
Аноним 14/11/23 Втр 01:52:09 584937 167
>>584935
>потом вперед во времени - назад во времени.
>держишь руку некоторое время неподвижно, вот она и подвинулась

обратное движение во времени продемонстрируй.

> на c*t, и даже размерность в метрах
этот ментальный пиздец я пока комментировать не буду. жду пруфа обратного движения во времени, там и поговорим
Аноним 14/11/23 Втр 02:30:57 584938 168
>>584937
>обратное движение во времени продемонстрируй.
Для времени наше субъективное восприятие ограничено только в одну сторону, как храповик не дает проворачивать что-то обратно, так и наше физическое тело устроено так, чтобы мы не могли двигаться в обратную сторону
Аноним 14/11/23 Втр 02:41:02 584939 169
>>584938
ты сейчас занимаешься демагогией.
ты либо возвращаешь свою руку во времени и меняешь свой пост, на который я сейчас отвечаю, крепя к нему фото например кабана, либо ты не возвращаешься во времени, потому что не можешь.
все очень просто.

если ты не возвращаешься назад во времени, но продолжаешь утверждать что время является пространственной мерностью, то ты либо безумен, либо глуп, либо лгунишка.

> мы не могли двигаться в обратную сторону
т.е. оснований считать что это пространственная мерность никаких, кроме постулатов из мира фентези и нарушений тождества?
Аноним 14/11/23 Втр 02:59:27 584940 170
>>584939
У тебя время входит в физические законы буквально симметрично пространственным координатам
Аноним 14/11/23 Втр 03:20:05 584941 171
>>584940
плевать, куда оно входит или не входит.

мы сейчас занимаемся подсчетом пространственных мерностей под углом в 90 друг к другу.
три насчитали. свойства определили. можно взад-вперед гонять по любой из трех, ничто не мешает.
ты говоришь что четвертая - это время. я прошу пруфнуть, как ты гоняешь по ней взад-вперед. ты говоришь, что тебе уже что то мешает. так является время пространственной мерностью, или ты пытаешься натянуть сову на глобус?

дальше будет смешной разговор, где ты будешь мне доказывать, как ты определил угол в 90' между высотой и временем, между шириной и временем, между глубиной и временем.

если ты не понимаешь что такое время, то это твоя проблема, а не проблема времени.
Аноним 14/11/23 Втр 03:45:11 584942 172
>>584932
>откуда там дым тогда?
Пар перегретый?
Аноним 14/11/23 Втр 03:56:08 584943 173
>>584942
они загораются буквально. а не парят.
Аноним 14/11/23 Втр 10:34:29 584949 174
драсте

а может ли так оказаться что за всей физикой всевозможной нашей вселенной стоит пустота?
ну что на самом деле ничего нет ничего было
а то что якобы есть это какое-то мнгновение флуктуации которо тут же исчезает бесследно как будто и не было

и что результат один: все если и существует то мгновенно и бесследно и без всякой причины и следствия исчезает в абсолютное небытие

слышал чето подобное про сумму всей энергии или как там что она равна нулю

короче может ли такое быть что мы в пустоте и она схлопнется и мы исчезнем как будто и никогда и не было
Аноним 14/11/23 Втр 11:27:07 584951 175
>>584949
Ты сначала определись что такое пустота.

Нулевой баланс энергии это про то, что в эпоху инфляции отрицательная энергия инфлатонного поля, которое породило инфляцию, равна энергии всей нарожденной материи(излучение + вещество).
Тут не никакой пустоты, наоборот есть стартовые параметры.
Уважаемые моды Аноним 14/11/23 Втр 13:30:04 584958 176
Хочу, чтобы по этому треду можно было ракету построить, а не узнать как относится к трансгендерам или что есть сознание, с философской позиции.
Сойбои увлечены психологиями и философиями, а также другими науками связанные с обществом.
Задал здесь, однажды, биологический вопрос, но получил опять, ебучий и связанный с обществом.
Доску можно переименовывать в "Человек и мир"
Аноним 14/11/23 Втр 16:13:52 584961 177
>>584941
>мы сейчас занимаемся подсчетом пространственных мерностей под углом в 90 друг к другу.
А зачем мы этим занимаемся? А какое определение 90 градусов ты используешь? Мерность пространства не связана с углами, держу в курсе
>так является время пространственной мерностью, или ты пытаешься натянуть сову на глобус?
Так обычные x y z это тоже натягивание совы на глобус, их нигде нет, это абстракция удобная

Люди раньше думали что земля плоская, а потом обнаружили, что нет. Так и с пространством-временем, ты же плоскоземельщик почти буквально "иди посмотри на горизонт, он же плоский..."
Аноним 14/11/23 Втр 16:34:23 584962 178
>>584961
> какое определение 90 градусов
половина развернутого угла

> Мерность пространства не связана с углами
правда? а с чем?
тогда мерности пространства - это ЦыWd, петух, 4 макаронины, оля, запятая, запятая. согласен? готов работать в таких терминах? ноука!!

> их нигде нет,
круто. я гле то говорил, что есть?
я по-моему 10 раз повторил, что мерности и их количество - это выбор. мы выбрали мерности которые лежат друг к другу под углом в 90, получилось 3. они заполнили все пространство без остатка.

я тебе уже предлагал другие варианты, ты проигнорил.

пространство девятимерно, потому что мы выбрали 36.
пространство восьмимерно, потому что мы выбрали 40.
пространство семимерно, потому что мы выбрали 45.
пространство шестимерно, потому что мы выбрали иррациональное 51,428....
пространство пятимерно, потому что мы выбрали 60.
пространство четырехмерно, потому что мы выбрали 72
пространство трехмерно, потому что мы выбрали 90

в любом из этих вариантов мы заполняем все пространство без остатка. это экспериментально доказуемо.

но ты говоришь что тут еще будет где то время. я все жду где ты пруфнешь угол. и свободу перемещения. чтобы твоя абстракция была корректной.

>Люди раньше думали что земля плоская
не ебет что там раньше приматы думали, дорогой демагог. воюй с ветряными мельницами в другом месте. можешь доту скачать.

> ты же плоскоземельщик почти буквально "иди посмотри на горизонт, он же плоский..."
уводишь тему хуй пойми на что, имея ноль предпосылок. как блядь в здравом рассудке можно назвать человека плоскоземельщиком, когда тот ни говорил
а) что он плоскоземельщик
б) не аппелировал к плоской земле или чему-то подобному
в) считал мерности пространства

у тебя логика поломалась? из этого не следует дедуктивный вывод, что я плоскоземельщик. просто твоя позиция слаба, у тебя закончились аргументы? и поэтому вместо чтобы со сной согласиться, тебя начало трясти.

но раз тебе можно сделать неебацца логичный вывод из данного диалога, что я якобы топлю за плоскую землю, то почему мне нельзя? такой же логичный вывод - ты сегодня ебал немецкую овчарку. это следует из количества букв Н в твоем сообщении.
Аноним 14/11/23 Втр 17:56:08 584968 179
>>584962
Ну ладно, давай сначала начнем рассуждать о действии в механике и интегралах движения
Аноним 14/11/23 Втр 18:12:52 584970 180
>>584962
>пространство девятимерно, потому что мы выбрали 36.
Нормальные люди чертят оси так, чтобы точек с невозможными координатами не было. Для этого нужно чтобы движение вдоль одной оси никак не влияло на координаты точки на других осях.
Аноним 14/11/23 Втр 18:35:03 584971 181
>>584970
и что немозможного в осях под 36 градусов?

>>584968
давай. а потом обсудим усталость тормозного диска при растяжении на разрыв. какое это имеет отношение к подсчету координатных прямых?
Аноним 14/11/23 Втр 18:42:51 584972 182
>>584971
>и что немозможного в осях под 36 градусов?
Точку (1,0,0,0,0,0,0,0,0) в своем девятимерном пространстве найди.
Аноним 14/11/23 Втр 18:54:45 584973 183
>>584972
в смысле найди?

ты не понимаешь что если разрезать торт из центра не 3мя линиями под углом в 90, а 9ю под углом в 36, то ничего не поменяется? вообще ничего? кроме сложности рассчета.

она будет в (1,0,0), если перевести в трехмерное. нашел. что дальше? причем тут время?
Аноним 14/11/23 Втр 18:59:29 584974 184
>>584973
>она будет в (1,0,0)
Не будет. Точка (1,0,0,0,0,0,0,0,0) невозможна.
У точки (1,0,0), при переходе в твою систему координат, вместо нулей обязаны быть разные значения.
Аноним 14/11/23 Втр 19:04:17 584975 185
>>584974
> Точка (1,0,0,0,0,0,0,0,0) невозможна.
лол.
таааак. давай по порядку. ты можешь поставить точку отсчета и прикрепить к ней 10 палок так, что угол межу каждой палкой в точке отсчета будет 36?
Аноним 14/11/23 Втр 19:10:00 584976 186
>>584975
>>584974
я переформлирую. а то доебешься до слов.

ты можешь поставить точку отсчета и прикрепить к ней 10 палок так, что угол межу двумя любыми ближайшими палками в точке отсчета был 36?
да или нет?
Аноним 14/11/23 Втр 19:20:26 584977 187
>>584976
Сделай это сам. А потом попробуй поставить точку (1,0,0,0,0,0,0,0,0). И осознай что сотворил.
Аноним 14/11/23 Втр 19:26:48 584979 188
2023-11-1419.25[...].mp4 6504Кб, 1364x740, 00:00:29
1364x740
Аноним 14/11/23 Втр 19:50:02 584980 189
>>584979
...а теперь спроецируй ее на все остальные оси
Аноним 14/11/23 Втр 20:18:47 584982 190
>>584980
хорошо. уболтал, я конечно могу полезть в залупу, как лобачевский или эйнштейн, с постулатами запрета такой проекции, но мне лень.

хорошо. вот опытным от обратного путем мы нашли количество палок координат. их 3.
не больше.
и что дальше?
Аноним 14/11/23 Втр 20:37:10 584984 191
Не могу кое что понять, гравитационное поле как известно искревление пространства-времени которое массой создается, распространяется сферически во все стороны по закону обратных квадратов, на землю действует сила гравитации со стороны солнца и перпендекулярно ей сила движения, почему земля не падает на солнце когда перед ним проходит, солнце ведь тоже движется и не улетает от него когда за ним, пространство то искривленное вместе с солнцем движется?
Аноним 14/11/23 Втр 21:09:03 584986 192
Аноним 14/11/23 Втр 21:23:54 584987 193
>>584986
у пространства нечему гнуться. модель некорректна. автор даун или мошенник.
но если ему можно нарушать тождество, то почему мне нельзя? я могу покатать шары по столу и сказать что пространство не гнется.
а если на столе сделать рельсы и покатать шарики, то можно сказать что у пространства есть невидимые рельсы

этот видос рассчитан либо на зомбирование пубертатных детей, либо на совсем патау с капающей слюной.
Аноним 14/11/23 Втр 21:40:35 584988 194
>>584984
>солнце ведь тоже движется
в смысле вокруг ядра галактики?
двигаясь вокруг ядра галактики Солнце тащит Землю вместе с собой
Земля вокруг ядра галактики движется с той же скоростью, что и Солнце
Аноним 14/11/23 Втр 21:49:49 584989 195
>>584988
Да я вроде сам понял, вообщем похоже когда земля перед солнцем движется если за перед солнца считать направление в котором оно движется земля увеличивает скорость и центробежная сила увеличивается, она компенсирует то что земля в более напряженном поле находится, а когда ща ним наоборот уменьшается.
Аноним 14/11/23 Втр 22:29:13 584992 196
>>584987
> у пространства нечему гнуться. модель некорректна. автор даун или мошенник.
Он вначале говорит, что это модель. Модель работает, имеет предсказательную силу, значит моделью пользуются. Но можно быть долбаебом как ты, кричать что это неправда и поэтому моделью пользоваться нельзя, и вместо того чтобы делать по этой модели расчеты движения планет и звёзд и летать к Луне и на Марс надо строить суперкомпьютер для расчёта гравитационного взаимодействия и остановить все развитие на сотни лет. Какой компьютер нужно построить для полёта на Луну по правильной модели? Его на земле хотябы построили?
Аноним 14/11/23 Втр 23:10:32 584993 197
>>584992
это неверная модель. мне повторить в 35 раз? у пространства нечему гнуться.
гнутая траектория не значит пространство погнулось.
гнутая траектория - это просто гнутая траектория.
футбольный мяч летящий по баллистической траектории не гнет пространство. пространству похуй как мяч летит. прямолиненйно или криволинейно.
пространству абсолютно похуй.

> вместо того чтобы делать по этой модели расчеты движения планет и звёзд и летать к Луне и на Марс надо строить суперкомпьютер для расчёта гравитационного взаимодействия и остановить все развитие на сотни лет. Какой компьютер нужно построить для полёта на Луну по правильной модели? Его на земле хотябы построили?

сформулируй мысль. я не понял что тут было сказано. идея такова: "без релятивистских поправок не долететь до луны?" я все верно понял?

так я спешу тебя расстроить что матан наших полетов, что американских есть публичном доступе. там нет никаких релятивистских поправок, многомерного пространства, лямбда сидиэм моделей и прочей срани. ее даже в жпс нет, вся документация в паблик домене.

к чему ты это тогда написал?
Аноним 14/11/23 Втр 23:25:37 584996 198
Стикер 575Кб, 160x150
160x150
>>584992
1. Своими глазами видит, что все шарики падают на большой шар
2. Знает, что Земля не падает на Солнце
3. Говорит с ебалом эксперта: >Модель работает

Что ты такое?
Аноним 14/11/23 Втр 23:43:27 584998 199
А разве частицы движутся не по градиенту напряженности гравитационного поля, которое есть искривление пространства-времени?
Аноним 15/11/23 Срд 00:11:28 585001 200
3456757798.png 229Кб, 798x766
798x766
Так шо, мы стоим на закате (очередного) золотого века навуки?
В общем то и без всяких навучных изысканий это очевидная истина. За 30+ лет своей жизни я не наблюдаю никаких прорывов в технологиях. Ну кроме разве что гаджетов, хотя и они основаны на довольно старых технологиях, которые довели до ума. В остальном все теже машины, самолеты, ракеты, дома и т.д, что и 50-100 лет назад.
Компухтеры вроде резво начали, но в итоге тоже сдулись. Еще лет 5-10 и технология дойдет до своего пердела. При этом даже на горизонте ничего нового не видно, даже в очень отдаленной перспективе.
Аноним 15/11/23 Срд 00:28:47 585002 201
>>585001
навука кончилась в 1920е-30е. с тех пор просто шла инженераи имплементация в железо того, что "успели" открыть до релятивистов и квантователей. ничего существенного в фундаментальной науке с тех пор более не происходило. никаких существенных феноменов не было открыто или нормальных моделей построено, потому что теоретические построения исходят из ложной аксиоматики с гнутыми пространствами и сказками про вакуум из ничего рождающий что-то.
вообще чудо, что инженеры рожают иногда до сих пор вопреки моченым, которые занимаются откровенным бредом. вся натурфилософия превратилась в инфоцыганщину с шарлатанами-мошенниками, имеющими академические степени, на полном серьезе рассказывающими, что гнутая прямая - это тоже прямая, а не кривая. а то и вообще вводящие неопределяемые понятия. им место в тюрьме или в дурке.
перспективы грустные. пиздец короче.
Аноним 15/11/23 Срд 00:44:58 585003 202
>>585001
Ты просто не замечаешь, как и люди в 50-x не замечали, ибо между разработкой в лаборатории и попаданием тебе на глаза в магазине огромный лаг в 20-30 лет
Биологи много всякого говна наоткрывали за последние годы, генная терапия для медицины это же буквально как холодный термоядерный синтез для физиков.

А по поводу твоей картинки, объемы научных статей просто стали больше (поэтому на одну статью меньше влияния) так еще и сама наука стала тяжеловесной, даже открыв невероятные феномены, ты лишь создашь еще одну ветку, а не полностью перевернешь всю науку.

А вообще жди третью мировую, вот там попрет так попрет
Аноним 15/11/23 Срд 03:29:32 585004 203
>>584934
>> мерность пространства это не выбор, это его свойство
>фееричное заявление, я выбрал угол 60 вместо 90. у нас изменилось количество мерностей. их стало 5.
Я не просто так сказал, что ты нихуя не знаешь.
Базис по определению линейно независимый, а твои векторы будут линейно зависимые. Можно выбрать три вектора в пространстве, такие, что углы между ними попарно будут 60 градусов, и они составят базис трёхмерного пространства, но как только ты добавишь любой вектор к этим трём, это перестанет быть базисом, потому что этот четвёртый вектор можно будет выразить через первые три.
Так вот, это первый семестр физ и мат факультетов, а ты дальше рассуждаешь о вещах, которые требуют старших курсов. Ты не просто не обладаешь нужными знаниями, ты даже не знаешь, что такие знания существуют.
Твоё требование про телефон это опять Лысенковщина. Типа, если электрон не щупал руками, значит его нет. Я не виноват, что твой мозг воспринимает время именно последовательно. Можешь видео поперематывать туда-сюда, вот тебе и движение во времени вперёд-назад.
ОТО подтверждается опытами. Не существует ни одного эксперимента, который бы опровергал ОТО.
Аноним 15/11/23 Срд 03:43:15 585005 204
>>585001
Я сосачую вот этого >>585003, но хочу добавить, что наука реально замедлилась.
Дело в том, что всё простое уже открыто. Понимание СТО доступно многим, ОТО уже сложнее. Квантмех в разных его частях может быть понятен от мало кому до долей процентов от всех людей.
Индустриализация высвободила много человеческих ресурсов для брейншторма реальности, научный метод позволил сделать поиск более качественным, но это просто дало линейное ускорение логарифмическому или вовсе асимптотическому процессу познания.
Это примерно как взять историю мировых рекордов по, например, прыжкам в высоту. Мы сначала увидим быстрый рост, который будет всё замедляться и замедляться. Так и в науке. У нас были ебейшие открытия в первой половине XX века, неплохой прогресс во второй половине, но теперь уже ничего крышесносного не происходит. Да, что-то открывают потихоньку, но гораздо реже и менее значительное.
Аноним 15/11/23 Срд 03:59:03 585007 205
>>584996
>Знает, что Земля не падает на Солнце
Но ведь Земля падает на Солнце...
Аноним 15/11/23 Срд 04:38:54 585009 206
>>585002
Ну все же до 60-70ых что то да придумывали, а потом уже начался явный спад.
>>585003
Ой да иди ты нахуй. Все я замечаю, ибо прожил больше четверти века. Ты знаешь сколько всего менялось за этот срок в 19 веке или в начале 20 века? От паруса до парохода, от семафора до телеграфа, от дирижабля до самолета и т.д. Армиям приходилось каждые 15-20 лет проводить полное перевооружение, ибо появлялась куча нового оружия (а сейчас из нового только игрушечные дроны). Так что пиздуй нахуй со своими потешными робопылесосами и гейфонами.
Какая нахуй биология мать твою? Корону победили средневековыми методами, карантином и прививками. Спидораки как были так и есть, вон Джобса даже его миллионы не спасли. Термояд это пустышка для прогрева гоев, уже как 50 лет там никаких сдвигов. Коколайдер вообще ничего нового не открыл, хотя это детище всего мира и ничего подобного ближайшие лет 50 уже не создадут. Про кокосмос я вообще молчу, он сдох и превратился в мавзолей куда залетают только по великим праздникам. ИИ вообще смехатура для фотожопа
Аноним 15/11/23 Срд 04:43:07 585010 207
>>584996
> 1. Своими глазами видит, что все шарики падают на большой шар

Еблан, в примере на видео шарики падают на большой шар из-за потерь на трение-качение, а в космосе их нет.
Аноним 15/11/23 Срд 04:52:54 585011 208
>>585005
Да нихуя индустриализация не дала. Людей стало в 8 раз больше, а открытий в 10 раз меньше. А крупных открытий вообще по нулям. Тупо перевели людей из одной сферы труда в другую, все равно что поменяли шило на мыло.
Принципиальная проблема в том любые технологии имеют конечную точку развития, и бесконечно паразитировать на них не получится. А чтобы открыть что то новое, как показала практика, бабла и кучи краснодипломных долбоебов не достаточно. Для этого нужно буквально божественное озарение.
Кто то даже предсказывал технологическую пустыню в ближайшие десятилетия или даже столетия. К чему в общем то все и идет. Так что футуристы коколонизаторы соснули хуйца
Аноним 15/11/23 Срд 09:47:43 585013 209
>>585001
>мы стоим на закате (очередного) золотого века навуки?
Не очередного, а последнего.

>Еще лет 5-10 и технология дойдет до своего пердела
Не, не так скоро.

>на горизонте ничего нового не видно
Ну как сказать...
- Темная материя/энергия толсто так намекает, что есть куда расти. Руки опускать пока рановато.
- Гравитационно-волновая астрономия началась как наблюдательная наука только в 2015 году.
- Ни много ни мало, а в 2025 году вполне может случиться самая настоящая долгожданная новая физика:
9 июля 2023 года коллаборация Muon g-2 завершила наработку экспериментальных данных, продолжавшуюся шесть лет. Первые результаты, полученные после обработки данных первого года работы, были опубликованы 7 апреля 2021 года[12]. Учёные сообщили, что результаты исследований мюонов бросают вызов Стандартной модели и, соответственно, могут потребовать пересмотра существующей модели элементарных частиц. Результаты первых трёх лет сбора данных коллаборация опубликовала в августе 2023 года. Ожидается, что окончательные результаты, основанные на статистике за полные шесть лет измерений, будут представлены в 2025 году.

https://ru.wikipedia.org/wiki/Muon_g-2

Тем не менее, никакую гипотетическую новую физику в около/бытовые технологии превратить не удастся. Это даже не фантастика, это сказки.
https://www.youtube.com/watch?v=navIPCYhSss
Аноним 15/11/23 Срд 11:47:21 585018 210
>>585013
>Не очередного, а последнего.
С чего бы? Ты плохо знаешь историю. Таких прорывов была целая куча. И еще будут, но видимо не сейчас
>Не, не так скоро.
Уже почти дошли до предела. Если 20 лет назад каждое поколение процов выдавало в десятки раз большую производительность, то сейчас прибавка лишь в 2-3 раза, да и то не факт. Ну и дальнейшее уменьшение транзисторов физически невозможно.
>Темная материя/энергия толсто так намекает, что есть куда расти. Руки опускать пока рановато.
А причем тут технологии?
>Гравитационно-волновая астрономия началась как наблюдательная наука только в 2015 году
Очередная хуйня без задач
>Ни много ни мало, а в 2025 году вполне может случиться самая настоящая долгожданная новая физика
Как уже заебали эти громкие заголовки... А вот как обстоят дела в реальности https://www.youtube.com/watch?v=BW7xaKua71o
Аноним 15/11/23 Срд 12:28:56 585019 211
>>585018
>Уже почти дошли до предела
5-10 лет - это роадмап какого-нибудь Интела или Н-Видии. И через 10 лет они закрываться не собираются.
Как минимум будут пилить аппаратную поддержку нейронок.

>Темная материя/энергия
>А причем тут технологии?
Технологии тут не удел. Но доска все-таки посвящена науке и для науки это имеет фундаментальное значение.

>Гравитационно-волновая астрономия
>Очередная хуйня без задач
Нонешними средствами мы не можем заглянуть в прошлое Вселенной раньше, чем 300 000 лет от начала расширения, т. к. до этого времени Вселенная была непрозрачна.
Гравитационным волнам на прозрачность пофигу, поэтому с их помощью можно заглянуть вплоть до самого начала. В частности, это позволит экспериментально доказать (или опровергнуть) живем ли в Мультивселенной или наша Вселенная единственная.

>эти громкие заголовки
Это не громкие заголовки, это многолетний эксперимент, движущийся к завершению. Обоснованно ожидается, что будет открыто пятое взаимодействие.

> в реальности
Как же заебали эти наивные реалисты
https://ru.wikipedia.org/wiki/Наивный_реализм
Аноним 15/11/23 Срд 12:33:06 585020 212
>>585001
> Еще лет 5-10 и технология дойдет до своего пердела. При этом даже на горизонте ничего нового не видно, даже в очень отдаленной перспективе.
Ебать дурачок ты малой.
На горизонте еще столько всего, что на твои 10 000 жизней не хватит работать.
Аноним 15/11/23 Срд 12:33:36 585021 213
>>585018
>Очередная хуйня без задач
У гравитации до хрена задач.
Аноним 15/11/23 Срд 13:37:29 585025 214
>>585004
налил водички, красиво.

>только ты добавишь любой вектор к этим трём, это перестанет быть базисом, потому что этот четвёртый вектор можно будет выразить через первые три.

а знаешь почему так? потому что пространство трехмерно.
теперь сманяврируй как ты добавляешь сюда четвертое измерение под названием время и не видишь противоречий.

И Я ВСЕ ЖДУ ВИДЕО, ГДЕ ТЫ ДОСТАНЕШЬ КАМЕРУ И ПОКАЖЕШЬ МНЕ СВОЙСТВО ОБРАТНОГО ПЕРЕМЕЩЕНИЯ ВО ВРЕМЕНИ ЭКСПЕРИМЕНТАЛЬНО.

НЕ ЗАБУДЬ ЗАОДНО ПОГНУТЬ ПРОСТРАНСТВО.

>Твоё требование про телефон это опять Лысенковщина.
мое требование называется эксперимент. ты отрицаешь это действие, поздравляю ты шарлатан и крутишь жопой.

>Можешь видео поперематывать туда-сюда
больше демагогии. больше, мошенник.

>ОТО подтверждается опытами.
ой, а что такое. т.е. эксперимент, чтобы пруфнуть примитивную аксиоматику ты провести не можешь и крутишь жопой. а вот якобы эксперимент где в 100000 больше матана и составляющих ты проводишь и все подтверждается. так ничего не подтверждается, вы просто мошенники, которые выдаете желаемое за действительное и обманываете всех вокруг.
gps не работает на ОТО или поправках СТО. документация в доступе - приноси пруф.

> твой мозг воспринимает время именно последовательно.
твой мозг воспринимает, что ты сейчас не сосешь у немецкого шепарда. на деле ты сосешь. НОУКА постановила, что лично ты зоофил!11

>>585007
на твоей модели все падает за 15 секунд. нам осталось жить 15 секунд. проведем эксперимент. ждем.

ты наебал. твоя модель не сработала

>>585010
а еще знаешь чего нету в космосе? ГНУТОГО ПРОСТРАНСТВА АЗАЗА
Аноним 15/11/23 Срд 14:04:16 585027 215
>>585004
>Можешь видео поперематывать туда-сюда, вот тебе и движение во времени вперёд-назад.
Итак эксперимент, где мы будем пруфать одинаковые свойства у пространственных мерностей и времени.

Запускаем эксперимент.
Все это должно двигаться единомоментно. Я понимаю что у вас не 10 рук, но вы справитесь. Записываем на пленку:
1) шарик летит вниз,
2) другой шарик летит вверх.
3) шарик катится влево
4) другой шарик катится вправо
5) шарик катится от нас
6) другой шарик катится к нам
7) песочные часы ссыпают песок вниз
8) другие песочные часы ссыпают тоже вниз. ой, а что такое. время работает только в одном направлении?

но нам мошенник посоветовал мошенническое действие.
>Можешь видео поперематывать туда-сюда,
воспользуемся. воспроизводим видео в обратном направлении:

1) шарик летит вверх
2) другой шарик летит вниз.
3) шарик катится вправо
4) другой шарик катится влево
5) шарик катится к нам
6) другой шарик катится от нас
7) песочные часы ссыпают песок вверх
8) другие песочные часы ссыпают тоже вверх. ой, ДА ЧИВО ТАКОЕ ТО....НЕПРИЯТНА

экспериментально доказано - время не является мерностью пространства, не дает свободу перемещения, эйнштейн мошенник, ты - манька-религиозный верун в авторитеты.

не плакай.
Аноним 15/11/23 Срд 17:31:24 585030 216
>>585019
Короче будет как с коколайдером, много шума, мало толку
Но ты главное ВЕРЬ
>>585020
Всем насрать на эти теории, речь о практическом применении.
И на горизонте нет никакой альтернативы транзисторам
Уже лет так 30 визжат про кококвантовые компухтеры, а на деле они уступают даже не самому новому айфону
>>585021
Ну подтвердят одну из доминирующих теорий и че? Впрочем не факт что подтвердят.
Такое ощущение у местных шизов какая то форма дислексии. Им говоришь о замедлении развития, они в ответ начинают кидать какие то громкие заголовки, что вот вот ухх!! Надо только подождать!! Только раньше не ждали, а делали
Аноним 15/11/23 Срд 18:58:37 585033 217
>>585027
Ты предлагаешь измерять время с помощью органов чувств, но наше чувство времени это просто сломанная рулетка, которая умеет только разматываться, но умеет сматываться, наше тело просто ебануто устроено, а ты пытаешься реальность натянуть вокруг восприятия, ну это немного тупо. Пока люди не начинают вмешиваться в процессы, все частицы не различают разницы между туда и обратно (ну.. там с оговорками).
Аноним 15/11/23 Срд 20:37:54 585034 218
>>585033
>предлагаешь измерять время с помощью органов чувств,
это ты предлагаешь, я такого нигде не говорил

>наше чувство времени это просто сломанная рулетка
наше чувство времени идеально. иначе бы я бы не смог играть ритмично на гитаре.

> но умеет сматываться,
доказательства будут? я пока слышу только голословные утверждения

> наше тело просто ебануто устроено,
в данном случае банут твой мозг/
который не понимает чем метр отличается от секунды.
что это разные сущности.
первая измеряет расстояние. вторая измеряет скорость протекания конкретного физического процесса в конкретных физических условиях. метры не зависят от секунд. секунды не зависят от времени.

дегенерат эйнштейн привязал метр к секунде через свет. основание для этого действия было только Я СКОЗАЛ. или по другому постулат. постулаты не являются наукой. потому что постулировать можно любой бред. это работает в фентези и рпг, но не работает в реальности. ей наплевать что там напостулировал дегенерат эйнштейн.

метры все также не зависят от секунд. а моя фраза что ты сосал у собаки, также врядли имеет отношение к реальности.

> не начинают вмешиваться в процессы,
парадокс наблюдателя появляется только у дегенератов, которые напридумывали себе постулатов. а потом реальность взяла и нассала на ролеплей данжнмастера. потому что реальности похуй на то какой ее видит данжн мастер. виновата не реальность, а дегенерат данжн мастер, которые не может ее засунуть в корректную модель.

в реальности никаких парадоксов не существует. там все примитивно тупо и скучно. но вы своими фентезийными постулатами ломаете мозг и себе и окружающим.
Аноним 15/11/23 Срд 20:41:06 585035 219
>>585034
> метры не зависят от секунд. секунды не зависят от времени.
секунды не зависят от метров.* фикс
Аноним 15/11/23 Срд 20:52:56 585036 220
>>585034
>скорость протекания конкретного физического процесса в конкретных физических условиях
Что такое скорость?
Аноним 15/11/23 Срд 22:15:02 585037 221
>>585034
>чем метр отличается от секунды
метр отличается от секунды размером
1 секунда = 299792458 метров

>скорость протекания конкретного физического процесса в конкретных физических условиях
расстояние, на которое конкретный физический процесс в конкретных физических условиях успевает переместиться во времени за время своего протекания

мимодругой
Аноним 15/11/23 Срд 22:37:04 585038 222
Аноним 15/11/23 Срд 23:11:55 585039 223
>>585038
температура самовоспламенения
Аноним 15/11/23 Срд 23:13:35 585040 224
>>585036
пройденное расстояние в пространстве за эталон времени, который определен, как скорость протекания конкретного физического процесса в конкретных физических условиях

>>585037
красиво. >секунды равны метрам.
но идиот почему то я, а не ваша манятеория с вашим гуру-шарлатаном

ананасы равны карликовым слонам.
сигареты равны стальным трубам
а нефть равна кофе

> процесс в конкретных физических условиях успевает переместиться во времени за время своего протекания
конкретный физический процесс, который мы выбрали эталоном времени, мой будильник перемещается вокруг центра земли, вокруг солнца, в рукаве галактике, а вся эта срань куда то двигается.
где релятивистские поправки? ах, да, это другое.
Аноним 15/11/23 Срд 23:30:51 585041 225
>>585040
Быстро летящие частицы дольше живут.
Аноним 15/11/23 Срд 23:44:58 585042 226
image.png 25Кб, 300x100
300x100
Почему эта машинка не поедет? Или поедет?
Аноним 15/11/23 Срд 23:53:56 585043 227
>>585040
Т.е. у тебя скорость определяется через время, а время через скорость? Окей, хули
Аноним 16/11/23 Чтв 00:06:24 585044 228
>>585039
что её достигает? пар?
Аноним 16/11/23 Чтв 00:07:44 585045 229
>>585042
когда магнит тянет машину, машина тянет магнит
Аноним 16/11/23 Чтв 00:09:33 585046 230
>>585044
>пар?
что-то что с ним контактирует
Аноним 16/11/23 Чтв 00:12:04 585047 231
>>585040
>ананасы равны карликовым слонам
не, не равны
если ананас заменить на кролика (или наоборот), результат поменяется

от замены секунд на соответствующее количество метров (или наоборот) не поменяется ничего
Аноним 16/11/23 Чтв 00:15:45 585048 232
>>585046
ну лист контактирует. он загорается. как?
Аноним 16/11/23 Чтв 00:24:00 585049 233
>>585048
Вода греется и греет лист. У листа температура самовозгорания ниже чем у воду. Вот он и самовозгорает.
Аноним 16/11/23 Чтв 00:33:03 585051 234
>>585043
господи. ты можешь различить скорость перемещения в пространстве и скорость закипания кастрюли воды?
скорость перемещения в пространстве и скорость изменения цвета хамелеона?
скорость перемещения в пространстве и скорость сгорания листа бумаги?

ни кастрюля с водой, ни хамелеон, ни лист бумаги никуда не перемещаются. но скорость процесса есть.

у тебя ментальный ступор после эйнштейна?

>>585047
ананас = метру
секунда = карлику
метр=секунде
ананас=карлику

опровергай.
Аноним 16/11/23 Чтв 00:45:14 585052 235
>>585051
>ни хамелеон, ни лист бумаги никуда не перемещаются
их части перемещаются
Аноним 16/11/23 Чтв 00:46:40 585053 236
>>585049
то есть пар зажигает лист?
Аноним 16/11/23 Чтв 00:46:41 585054 237
>>585052
куда у тебя перемещается вскипающая кастрюля? ты в палате номер 6?
Аноним 16/11/23 Чтв 01:55:16 585056 238
>>585051
Что такое скорость?
Аноним 16/11/23 Чтв 02:29:26 585057 239
Аноним 16/11/23 Чтв 12:29:37 585062 240
>>585057
Скорость перемещения
Аноним 16/11/23 Чтв 13:12:15 585064 241
Аноним 16/11/23 Чтв 14:15:32 585068 242
>>585064
Что такое
>скорость протекания конкретного физического процесса в конкретных физических условиях
???
Аноним 16/11/23 Чтв 16:32:55 585069 243
>>585068
напишу сказку. так уж и быть. а то вы настолько промыты что не понимаете, чотакое время...

представим ты разумное теплокровное существо. у тебя нет часов. ни стрелочных, ни песочных, ни атомных, никаких. даже солнечных, потому что ты живешь допустим в пещере подо льдом антарктиды в 5 км от поверхности, где темно. снизу камень и мерзлая почва, по бокам тонны льда, сотнекилометровая сеть пещер

итак ты разумное теплокровное существо из подо льда антарктиды. ты не слышал ни про эйнштена, ни в курсе про аристотеля, то что ты летаешь вокруг звезды, ты не знаешь про космос, ты нихуя не знаешь.

ты просто разумное теплокровное существо из подземелья. и вот ты решил заняться наукой. как ты заимеешь концепт времени?

предлагаю рандомный вариант.

ты знаешь что лед тает, если ты долго будешь греть его свои пузом. откалываешь кусок льда объемом примерно в свою залупу и кладешь себе на пузо, он естественно тает.
ты называешь это 1 залупа.
2 таких куска последовательно тают за 2 залупы.
3 куска последовательно тают за 3 залупы.
полкуска тает за ползалупы.
треть куска тает примерно за треть залупы.

поздравляю ты освоил время и примитивные часы (скорость протекания конкретного физического процесса в конкретных физических условиях) .

далее ты измерил длину своего хвоста. и назвал это 1 хвост. поздравляю, ты изобрел расстояние.

ты измерил свою пещеру от стены до стены, получилось 160 хвостов.

ты пригласил кореша из соседней пещеры помощь тебе, и попросил пройти твою пещеру от стены до стены. кореш пошел, а ты последовательно клал себе куски льда на пузо.

кореш прошел расстояние в 160 хвостов за 4 залупы. скорость движения кореша = 40 хвостов в залупу.

ты все еще ничего не знаешь ни про вселенную, ни про солнце, ни про скорость света, ни про эйнштейна, ни про вращение планеты, но у тебя уже есть величины для измерения времени и расстояния. причем довольно точные.

через 20000 залуп развития этой цивилизации появится местный эйнштейн-дегенерат, будет рассказывать, что расстояние относительно таяния льда, что 1 залупа на самом деле = 1 хвосту. и что время неотделимо ото льда и зависит от скорости его таяния. появится местный дегенеративный релятивизм. будут рассказывать что если использовать 3 пуза, чтобы погреть лед, то расстояние блядь сокращается. а когда проверят, что нихуя не сокращается, то будут вилять жопой и рассказывать про эффект наблюдателя. и тд тд.

сказка-аналогия закончена.
Аноним 16/11/23 Чтв 16:52:53 585070 244
>>584980
>>584974
Я тут подумал 5 минут, а ведь у этой хуйни есть простое элегантное решение, без прибегания к постулатам.

Ты говоришь, что такая точка невозможна, и держишь в голове угол проекции на другую ось естественно в 90 градусов. Ну так в модели пространства, где у тебя угол между осями 36 градусов - тебе внезапно надо проецировать под углом 36, а не под 90. Там где 40, проецировать под углом 40, а не под 90 и тд. Вот и вся аксиоматика.
(1,0,0,0,0,0....) возможна без нарушения чего либо. Можешь взять линейку и потратить полминуты и понять, что все проецируется.

Там кстати полезут интересные строгие математические формулы при проекции на дальние оси, назовем их осями второго, третьего и далее порядков, типа фрактальных прогрессий, не знаю как это корректно обозвать. Думаю это дерьмо уже где-то описано у любителей дрочить шизомногомерия просто в других терминах. Но пока лень про эт думать и писать.
Аноним 16/11/23 Чтв 17:17:01 585071 245
>>585070
Не надо думать, Анон. Надо проецировать.
Аноним 16/11/23 Чтв 17:18:43 585072 246
>>585054
Частицы кастрюли двигаются быстрее - кастрюля греется.
Аноним 16/11/23 Чтв 17:47:53 585074 247
>>585069
А есть какое-то строгое определение, а не графомания?
Аноним 16/11/23 Чтв 18:09:51 585075 248
>>585074
ладно, ты потроллить решил.

Время – сравнительная количественная характеристика физических процессов, происходящих в пространстве, привязанная к
длительности физического процесса, взятого за эталон.

скорость = расстояние в пространстве / время.

>>585072
я рад за частицы. вскипевшая кастрюля = 1 единица времени. 2 вскипевших кастрюли= 2 единицы времени. и тд.
в эталонной кастрле может быть литр воды и все, хоть политра, хоть поллитра и курица, хоть кокакола и свиная голова. какое мне дело что там происходит с частицами свиной головы и частицами курицы и воды, когда мне важен факт вскипания, чтобы взять ео за эталон?

>>585071
я спроецировал. все сошлось.
количество мерностей пространства - это выбор. единицы времени - это выбор. единицы расстояния - это выбор.
Аноним 16/11/23 Чтв 18:29:43 585076 249
>>585075
>я спроецировал. все сошлось.
у тебя не должно сходиться. ни под каким углом
Аноним 16/11/23 Чтв 18:49:54 585077 250
>>585075
>происходящих в пространстве
Т.е. процесс происходит, если что-то движется в пространстве? Не чувствуешь какой-то наеб?
Аноним 16/11/23 Чтв 21:09:50 585086 251
>>585076
возьми нарисуй и спроецируй. все сходится

>>585077
а ты можешь чем то двигать вне пространства? жду видео.
Аноним 16/11/23 Чтв 21:22:52 585087 252
>>585086
>возьми нарисуй и спроецируй. все сходится
...а теперь черти линии от точки до каждой оси под одним и тем же углом.

>а ты можешь чем то двигать вне пространства? жду видео.
Мораль в том что количество движения в пространстве влияет на количество движения во времени. Отсюда и связь между метрами и секундами.
Аноним 16/11/23 Чтв 22:12:49 585088 253
>>585087
>.а теперь черти линии от точки до каждой оси под одним и тем же углом.
они идут в ноль, в начало координат.

>Отсюда и связь между метрами и секундами.
нет. просто кринж и какой то паралич мозга.

если ты берешь эталоном метра длину банана и эталоном часа за сколько вода вытекает из дырявого ведра, то ты считаешь скорость в длинах банана/опустошенные ведра.

из этого не следует что длина банана фундаментально связана с опустошением ведра.

как мне еще изъебнуться и показать бредовость всех потуг найти связь между рандомно взятыми эталонами?
Аноним 16/11/23 Чтв 22:52:41 585089 254
>>585088
>они идут в ноль
нет не идут, по такой системе каждая точка вне оси будет нулем на всех осях
Аноним 16/11/23 Чтв 22:55:33 585090 255
>>585088
идея в том что пройденное любым объектом за единицу времени расстояние всегда одинаково
и проходится оно частично в пространстве, частично во времени
Аноним 16/11/23 Чтв 23:06:02 585091 256
>>585088
метр и секунда — это абстракции
их объективно не существует
объективно существуют расстояния между объектами в пространстве и во времени
метр и секунда — это два рандомных эталона, которые придумали люди
эталон можно "приложить" вдоль любого направления, включая временнóе
Аноним 17/11/23 Птн 01:51:43 585098 257
>>585089
нет, с чего вдруг? оно будет работать по паттернам
(1,1,1,0,0,0,0,0,0)
(0,0,0,1,1,1,0,0,0)
(0,0,0,0,0,0,1,1,1)
если примитивно.

а если дрочить обмазываясь калом, как любят математики, то появится доп вариант оперировать не точками, отрезками и площадями, а любыми фигурами, например четырехугольниками, или пятиугольниками. надо что бы количество мерностей было больше или равно, чем кол-во углов основания фигуры.
кому нехуй делать и любит дрочить в позе раком левой пяткой - может поинтегрировать какое-то дерьмо в такой системе, находя площади многоугольных фигур.

есть путь дальше - вывести аксиоматику и матаппарат интегрирования площади внутри объема в доп координатах внутри допкоординат.

только нахуя это все надо. там можно голову сломать. я вот нарисовал в блендере пространство деленное под 60, покрутил и понял, что этот монстр работает, все там проецируется. но там на каждой итерации можно сдохнуть запутаться и обмякнуть.

и даже если я пиздец как неправ, то я все еще не вижу причем тут время.

>>585090
оно не проходится ни в каком времени. потому что времени не существует объективно. а вот расстояние объективно и существует.

>>585091
>метр и секунда — это два рандомных эталона, которые придумали люди
спасибо что повторил ровно то что я тут талдычу.
>объективно существуют расстояния между объектами в пространстве и во времени
нет батенька, тут вы занимаетесь подменой терминов. расстояние между пиком ленина и эльбрусом не имеет в себе никакого времени.

>эталон можно "приложить" вдоль любого направления, включая временнóе
у времени нет направления. у пространства есть расстояние и есть направление движение по нему материального объекта. у времени нет направления. если ты считаешь, что есть, то я просто в 24 раз требую, если ты не согласен - продемонстрировать движение во времени назад.
движение вбок по времени.
если ты не можешь, то это объективно. а раз это объективно, то время и расстояние - разные сущности.
одно является свойством самого пространства, второе является свойством материи.
Аноним 17/11/23 Птн 02:15:18 585099 258
>>585098
>оно не проходится ни в каком времени. потому что времени не существует объективно. а вот расстояние объективно и существует.
Угу... расстояние то реально существует, не выдуманная концепция а РЕАЛЬНОСТЬ
Аноним 17/11/23 Птн 02:22:31 585100 259
>>585099
расстояние между пиком ленина и эльбрусом реально?
Аноним 17/11/23 Птн 08:21:30 585105 260
>>585100
>расстояние между пиком ленина и эльбрусом реально?
Только для рожденного ползать.
Рожденный ползать сворачивать пространство своё не может от того и ползает. Но далеко не уползёт. У него вилка. Эволюция или вымирание.
Аноним 17/11/23 Птн 11:30:32 585109 261
>>585098
>нет батенька, тут вы занимаетесь подменой терминов. расстояние между пиком ленина и эльбрусом не имеет в себе никакого времени.
нет батенька, тут вы ошибаетесь
между пиком ленина и эльбрусом есть расстояние в пространстве и есть расстояние во времени
с точки зрения пика ленина эльбрус находится немножко в прошлом
с точки зрения эльбруса пик ленина находится немножко в прошлом
нельзя отдалиться от объекта в пространстве, не отдалившись при этом и во времени
Аноним 17/11/23 Птн 12:20:12 585111 262
Никто сможет ответить, что произойдет с водой, если ее сжимать при огромных давлениях как в центре планеты, например. Учоные еще не определились как работают жидкости))))
Аноним 17/11/23 Птн 13:55:52 585113 263
>>585100
Время за которое свет летит от пика ленина до эльбруса реально?
Аноним 17/11/23 Птн 13:56:43 585114 264
>>585111
Ты пробовал задавать вопрос в гугл?
Аноним 17/11/23 Птн 13:57:23 585115 265
>>585111
Никто не сможет ответить сколько у твой мамки было половых партнеров )))))
Аноним 17/11/23 Птн 15:33:56 585116 266
>>585113
вы еврей? ответьте на мой вопрос.
на ваш отвечаю - нет. есть только расстояние.

>>585109
>расстояние во времени
нет, его нет. есть только расстояние в пространстве. время за которое кто-то проходит это расстояние - это действие. что-то может совершать действие, а может и не совершать.
между эльбрусом и пиком ленина можно преодолевать расстояние, воспроизводя репчик. значит есть расстояние в репчике?

>с точки зрения пика ленина эльбрус находится немножко в прошлом
>с точки зрения эльбруса пик ленина находится немножко в прошлом
нигде они не находятся. это шиза относительности. проистекающая из того что мы все делаем относительно скорости света, запостулировав свет как инвариант.

если мы запостулируем репчик по воздуху, как инвариант, то что, получится, что горы относительно друг друга находятся побольше в прошлом?

>нельзя отдалиться от объекта в пространстве, не отдалившись при этом и во времени
постулирую почту как инвариант. я живу в квартире 44, и шлю письмо в квартиру 45. кидаю его в почтовый ящик на стене. приходит почтальон, вынимает письма из ящика, везет в на почту, там сортируют, почтальон выезжает раскидывает, приходит житель 45 квартиры и вынимает письмо. прошло 3 дня.

выводы - выходит между квартирой 44 и 45 расстояние во времени 3 дня? и пространство погнуто?
только не крути носом. это буквально так вы применяете ОТО
Аноним 17/11/23 Птн 16:15:45 585117 267
Чому гейство это плохо?
Аноним 17/11/23 Птн 17:08:19 585118 268
>>585116
>на ваш отвечаю - нет. есть только расстояние
А как вы это поняли? Мы можем измерить и одно и другое... в чём разница?
Аноним 17/11/23 Птн 17:22:16 585119 269
>>585118
между вершинами 2х горе есть расстояние? есть
между вершинами 2х гор есть время? нет

из-за того что примат начал измерять время полета чайки, звука, света, брошенного бычка время в пространстве не появилось. а вот расстояние мужду ними как было так и останется, хоть ты его замеряй, или не замеряй. хоть замеряй неправильно. расстояние в пространстве- объективно существует.
за сколько бычок, чайка или шмеля пролетят это расстояние - пространству поебать. могут вообще никуда не лететь. ему похуй.
Аноним 17/11/23 Птн 17:26:51 585120 270
>>585116
>нет, его нет
а вот говорят, между Землёй и Солнцем есть расстояние во времени
дескать мы видим Солнце каким оно было ~8 минут назад
врут?

и ещё про какое-то аномальное смещение перигелия Меркурия
нет аномального смещения?
Аноним 17/11/23 Птн 17:30:49 585121 271
>>585119
>между вершинами 2х горе есть расстояние? есть
Т.е. просто "зуб даю есть"? Что считаешь критерием существования?
>за сколько бычок, чайка или шмеля пролетят это расстояние - пространству поебать. могут вообще никуда не лететь. ему похуй
В том дело что не похуй, в разных системах отсчета расстояние разное будет
Аноним 17/11/23 Птн 17:39:45 585122 272
>>585120
>, между Землёй и Солнцем есть расстояние во времени
т.е. ты мне доказываешь существование времени тезисами, выходящими из парадигмы ОТО?
что дальше? будешь доказывать непорочное зачатие парадигмой из библии? а гиперлуп парадигмой из звездных войн лукаса?

>дескать мы видим Солнце каким оно было ~8 минут назад
и что дальше? мне почтальон из 45 в мою 44 квартиру привез письмо спустя 3 дня после его отправки. это значит что между квартирами в полуметре расстояние в 3 дня?

хватит блядь называть задержку сигнала расстоянием.

> аномальное смещение перигелия Меркурия
представляешь размер половины листика а4?
Аноним 17/11/23 Птн 17:45:28 585123 273
>>585121
> разных системах отсчета расстояние разное будет
не будет. оно будет везде одинаковым. у тебя мозг окончательно сломался?
из-за того что ты запускаешь чайку, цессну, су27, калибр, свет между двумя вершинами - расстоянию между вершинами абсолютно похуй.

> Что считаешь критерием существования?
между твоим носом и пяткоой есть расстояние? если оно есть, то это объективно. если ты между носом и пяткой пустишь муравья или бросишь камень, то пройдет время. ты сделаешь субъективный выбор, считать свою задержку сигнала по скорости муравья или по полету камня. это блядь субъективный выбор, потому что времени нет. ты черрипикаешь процесс как эталон
Аноним 17/11/23 Птн 18:33:51 585124 274
>>585123
>не будет. оно будет везде одинаковым. у тебя мозг окончательно сломался?
Ты споришь с результатами экспериментов? Ладно
>если оно есть, то это объективно
Мои глаза его измеряют с помощью фотонов, прикинь?

>ты черрипикаешь процесс как эталон
Предложи другой процесс, но такой, который не меняет расстояние в разных системах отсчета. Есть какой-то магический способ измерять расстояние без действия?
Аноним 17/11/23 Птн 18:38:42 585125 275
Заметил, что многие ученые довольно поверхностные, например, недавно прочитал неплохую книгу одного американского доктора биологии и его жены 2020 года, в целом мне очень понравилось, но автор на мой взгляд сделал ошибку в одном суждении: они подумали, что все люди одомашнены в примерно равной степени и поэтому когда он попытался объяснить наличие жестокости у людей, то делал это уже имея большое заблуждение насчет людей. А так-то понятно, что какие-то люди более одомашнены, чем другие, которые могут не сильно отличатся от наших диких, вымерших родственников, таких как гомо эректусы и гомо сапиенс неандертальцы и вот от таких людей и происходит жестокость, часть из них - психопаты.

В общем на мой взгляд это ошибка авторов книги в следствии недостаточно глубокого изучения вопроса.
Аноним 17/11/23 Птн 18:45:24 585126 276
>т.е. ты мне доказываешь
нет, я ж вопрос задал, а ты не ответил
врут ли все, кто говорят, что Солнце с земли видно с задержкой в 8 мннут?

>это значит что между квартирами в полуметре расстояние в 3 дня?
почтальон из 45 в мою 44 квартиру привез письмо спустя 3 дня после его отправки. это значит что между квартирами в полуметре расстояние в 3 дня?
между квартирами 0.5 метра
между событиями начала и конца доставки 77706205113600 метров

>представляешь размер половины листика а4?
представляю
а ты опять не ответил
есть аномальное или нет?
Аноним 17/11/23 Птн 19:14:39 585128 277
>>585124
>Ты споришь с результатами экспериментов? Ладно
пример эксперимента.

>Мои глаза его измеряют с помощью фотонов, прикинь?
т.е. когда ты закрываешь глаза, расстояние между пяткой и носом нет?

> который не меняет расстояние в разных системах отсчета.
он меняет его только в вашей болезной теории, где вы постулируете постоянство скорости света и невозможность ее превысить. без экспеиментом, без нихуя, без доказательств, просто на основании я скозал.

>Есть какой-то магический способ измерять расстояние без действия?
время без попытки его измерить не существует.
расстояние без попытки его измерить существует.

тигр ничего не знает про метры, ни про секунды. но до жопы лани есть объективное расстояние.
Аноним 17/11/23 Птн 19:31:17 585129 278
>>585126
>врут ли все, кто говорят, что Солнце с земли видно с задержкой в 8 мннут?
не, врут. и что из этого следует?
ты вызвал два такси на двух телефонах, стоишь на шоссе.
ты видишь эти такси на карте, они на равном расстоянии от тебя.
приложение на 1м телефоне говорит что такси приедет через 1 минуту со скоростью 100кмч.
приложение на 1м телефоне говорит что другое такси приедет через 2 минуты со скоростью 50 кмч.
какой фундаментальный вывод из этого следует? (никакого)

>между событиями начала и конца доставки 77706205113600 метров
круто, пространство погнуто. шуе ппш

>есть аномальное или нет?
объясняю. хорошо что представил.
аномалия есть. но объяснение ОТО притянуто за уши.

рассказываю как был сделан "ноучный" эксперимент якобы подтвердивший ОТО с перигелием меркурия.

1) вот у тебя половинка листа А4. только век назад это была не бумажка, а фотопластинка такого же размера. ебнули эмульсию, серебра. сфоткали. проявили

2) прошел год - взяли ДРУГУЮ фотопластинку с ДРУГОЙ эмульсией сфоткали в ДРУГОЕ ВРЕМЯ отличающееся на несколько секунд с ДРУГИМ состоянием атмофсеры с ДРУГОЙ окружающей тепературой с ДРУГИМ атмофсерным давлением, сфоткали, проявили ДРУГИМ составом, залили ДРУГИМ закрепителем, получили ДРУГОЕ расположение зерен серебра и тд

сравнили 1) и 2) и нашли что одно зерно серебра уехало как якобы предсказывает ОТО = ОТО доказано.
это буквально сука так было сделано. буквально блядь. я не шучу.


это пиздец, а не наука.
Аноним 17/11/23 Птн 19:53:23 585130 279
>>585129
>какой фундаментальный вывод из этого следует?
если мы доверяем приложениям, то можно сделать вывод, что от таксопарка до тебя выбирай (разные названия одного и того же):
1.66 километра
1.03 сухопутной мили
0.92 морской мили
0.0000055 секунды

>пиздец, а не наука
я правильно понимаю, что человечество до сих пор не знает, из-за чего прецессирует орбита Меркурия?
Аноним 17/11/23 Птн 19:56:14 585131 280
>>585130
опять 25.

метры = секундам.
негры = меду
лабутены = кочерге

>из-за чего прецессирует орбита Меркурия?
да
Аноним 17/11/23 Птн 23:05:57 585132 281
edMezon.mp4 14056Кб, 640x360, 00:05:56
640x360
>>585131
это ж и почему мезоны долетают — тоже не ясно, получается
Аноним 17/11/23 Птн 23:18:16 585133 282
>>585132
все что говорится под портретом эйшнтейна не является наукой, потому что у людей там сломан мозг. саентологическая секта.
я комментировать это не буду.

потому что пространство не гнется, время не является пространственной мерностью, постулат про инвариант скорости света неверен, постулат что нет нечего быстрее скорости света нарушается самим же светом который в плазме хуярит быстрее, а в плазме бод углом еще быстрее, и прочая и прочая. это не наука. это марксизм-ленинизм с циферками.
Аноним 17/11/23 Птн 23:24:49 585134 283
Time-dilation-0[...].png 55Кб, 1920x678
1920x678
>>585133
чем объясняется увеличение длины траектории фотона в движущейся системе зеркал по сравнению со стационарной?
Аноним 17/11/23 Птн 23:30:35 585135 284
>>585134
а, скорость света не инвариант, тупанул, сорри
Аноним 17/11/23 Птн 23:38:07 585136 285
>>585134
ничем не объясняется, потому что это вялый эксперимент, а если прямо - нерелевантное криво поставленное говно, где все цифры притянуты за уши релятивистской аджендой, чтобы получить грантик.

чтобы просто хоть как-то измерить скорость света, хоть сколько нибудь точно - тебе понадобятся километровые установки, потому что она очень большая.
чтобы измерить с минимальными знаками до запятой - тебе понадобится стерильная многокилометровая установка
а чтобы померить то, что у тебя на пике тебе понадобится едущая со значительной скоростью многокилометровая стерильная установка , где все внешние воздействия устранены.

как ты понимаешь ничего подобного нигде не делалось. это все маняфантазии.

как говорится если эксперимент не поставлен в реальности, то тем хуже для реальности (c) ото и группа шизов математиков
Аноним 18/11/23 Суб 00:34:42 585137 286
>>585136
> чтобы просто хоть как-то измерить скорость света, хоть сколько нибудь точно - тебе понадобятся километровые установки, потому что она очень большая.
Эм?

The Laser Interferometer Gravitational-Wave Observatory (LIGO) типа просто макет?
Аноним 18/11/23 Суб 00:38:40 585138 287
>>585137
на ligo проводят другой маняэксперимент, а не тот, который у тебя на картинке. я хуй знает к чему ты это запостил.
Аноним 18/11/23 Суб 06:59:25 585148 288
Вот щас щас очко человечеству ии порвет.
Как можно в это верить
Аноним 18/11/23 Суб 10:58:10 585153 289
>>585148
Человечество уже в рабстве ИИ, лучшие умы человечества пущены для того, чтоб делать ИИ лучше. Считай принесли самый дорогой ресурс человечества в жертву кремневому богу.
Аноним 18/11/23 Суб 15:53:16 585161 290
8c4134909ce756e[...].gif 6373Кб, 800x450
800x450
Все машины времени в массовой культуре, что я видел, перемещают человека не только во времени, но и в пространстве (хотя этот аспект никогда не подчеркивается и не осмысливается).

Я правильно понимаю, что в силу движения Солнечной системы в пространстве необходимо еще и с огромной точностью телепортировать человека куда-то на поверхность Земли, иначе он в 99,9999% случае отправится в другое время в космос? Где-то разбирались подобные вопросы?
Аноним 18/11/23 Суб 17:07:01 585165 291
>>585161
>Где-то разбирались подобные вопросы?
да, в этом треде.
перемещения во времени невозможны, потому что время не является мерностью, в нем невозможно перемещаться. конец сказке.
Аноним 18/11/23 Суб 17:07:45 585166 292
>>585165
мерность простраснтсва*
Аноним 18/11/23 Суб 17:50:09 585167 293
Я патлач. Постоянно вычёсываю у себя дохуя волос, за время их накапливается целая куча просто.
Почему в таком случае у меня 99.9% волос всё ещё одной длины, и мне не лезут в глаза волосы которые выросли на месте вычесанных?
Аноним 18/11/23 Суб 18:45:42 585168 294
Почему радиосвязь при своем появлении не вытеснила полностью проводные телефоны (как беспроводной телеграф полностью вытеснил обычный)?
Почему даже к моменту появления компьютеров люди продолжали пользоваться проводными телефонами?
Почему на основе радио не создали интернет в 1950-х годах и на первых персоналках (конец 1970-х) не было радиомодема, а для интернета приходилось подключаться по dial-up даже в начале нулевых?
Только сейчас обычные телефоны начинают уходить в историю, вытесняясь мобильной связью с ее высокими скоростями и нетребовательной инфраструктурой (достаточно лишь построить вышку, в жилых домах никакого оборудования проводить не нужно).
В чем секрет долгоживучести проводных телефонов и в чем их преимущество над радиоволнами?
Аноним 18/11/23 Суб 18:55:33 585169 295
>>585165
Время это вообще какая-то НЕХ, наука не может сказать что это такое. Поэтому машина времени находится за пределами понимания науки, мы можем лишь сказать, что в природе нет ни единого факта передачи энергии или материальных объектов из будущего в прошлое. Плюс это нарушило бы закон сохранения энергии и позволило бы создать вечный двигатель. В современной научной картине мире вечные двигатели невозможны.

На бытовом уровне можно описать перемещение во времени как процесс изменения объектов. Разные объекты могут изменяться с разной скоростью, как в парадоксе близнецов. Также время как-то связано с гравитацией, еще одной плохо понятной областью для современной науки (см. темную материю и энергию).

В математических моделях есть шкала времени и четырехмерное пространство, но это именно математическая модель, а не реальность.

Ну и в черных дырах со временем и массой происходят сплошные аномалии, которые и представить нельзя.
Аноним 18/11/23 Суб 21:25:39 585175 296
>>585168
Стабильность. Радио сигнал постоянно помехи для стабильности нужны хорошо экранированные провода.
Удобство. Мобильные телефоны настолько удобней что стабильность уже не тянет.
Аноним 18/11/23 Суб 21:30:35 585176 297
>>585169
черных дыр не существует. это тоже математическая маняфантазия.
в теории существуют черные звезды, где вторая космическая меньше гравитационной постоянной, поэтому свет не может просто улететь. и то врядли такие объекты возможны. нужна очень большая масса.
но ко всем маняфантазиям релятивистов о черных дырах, гнутых пространствах и замедленных временах эти объекты не имеют отношения.

>парадоксе близнецов.
нет никакого парадокса близнецов
Аноним 18/11/23 Суб 22:11:16 585178 298
>>584809
чё всё? лучшие умы доски ничего не смогли ответить внятного?
Аноним 19/11/23 Вск 00:01:03 585179 299
Аноним 19/11/23 Вск 00:23:39 585181 300
>>585179
помаши листиком бумаги над кастрюлей с пельменями
Аноним 19/11/23 Вск 01:36:05 585185 301
изображение.png 945Кб, 1024x730
1024x730
>>585181
Во первых, махать нужно не листиком бумаги, а сушёными листьями. Во вторых, влаги над кастрюлей с пельменями несравненно больше, чем в сушёных листьях. В третьих, эта влага идёт извне, а не находится внутри, как в случае с сушёнными листьями которые зажигает микроволновка. Так что пикрил.
Аноним 19/11/23 Вск 06:11:45 585186 302
image.png 8348Кб, 3000x2000
3000x2000
>>585185
давай так. какой химический процесс в пробирке мне загуглить, где из вещества каким то образом абсорбируется вода, и вещество из-за этого само загорается?
Аноним 19/11/23 Вск 07:07:36 585187 303
>>585125
>ошибка авторов книги в следствии недостаточно глубокого изучения вопроса.
Ты не прав, ошибка тут вот в этом:
>американского доктора биологии
Американский биолог с докторской степенью не может принять простую вещь - на планете обитает 1 вид людей с разным генетическим разнообразием, которое делит людей на типы. Признав такое нетолерантное свойство, этот биолог отправится кормить голубей на улицу и потеряет степень.
Аноним 19/11/23 Вск 07:09:40 585188 304
Аноним 19/11/23 Вск 10:31:47 585190 305
https://www.youtube.com/watch?v=RFh3akPPwBE

Как вкатиться в кибернетику как на видео?
Чтобы изучать эффективные менеджерские схемы с точки зрения кибернетики?
Аноним 19/11/23 Вск 21:58:10 585210 306
>>585148
Это ебнутая секта футуристов
>>585153
Ага, а еще в рабстве у солнца, воздуха, воды, нефти, газа, огня, зерна, коровы и т.д. Таблетки прими, раб
Аноним 20/11/23 Пнд 14:16:54 585226 307
есть ли люди, решающие задачу 3+тел в уме?
Аноним 21/11/23 Втр 08:19:47 585246 308
>>585226
Да, если у них развитый звуковой вектор.
Аноним 21/11/23 Втр 08:20:51 585248 309
Посоветуйте легких учебников, чтобы хотя бы чуть-чуть понимать разные науки.
Аноним 21/11/23 Втр 12:27:01 585256 310
>>585248
Легкие учебники слишком скучные, сложные учебники слишком сложные. Всем не угодишь.
Аноним 21/11/23 Втр 13:08:46 585259 311
>>585248
Поварнин. Как читать книги
Сасскинд, Грабовски. Теоретический минимум
Семихатов. Всё, что движется
Никитин. Происхождение жизни
Аноним 22/11/23 Срд 00:18:12 585286 312
>>584346 (OP)
Можно ли изобрести "чудо печку", чтобы бытовой мусор "превращать в чистый кислород"?
Аноним 22/11/23 Срд 00:20:07 585287 313
>>585259
Благодарю.
>>585256
Ага. Вот бы кто-нибудь взял и написал легкий учебник, который интересно изучать.
Или сложный учебник, который легко понимать.
Аноним 22/11/23 Срд 01:23:15 585291 314
Аноним 22/11/23 Срд 12:01:57 585296 315
>>585286
Можно, но она будет жрать дохуя энергии, причем больше чем выделиться при сжигание топлива для получения оной.
Аноним 22/11/23 Срд 20:33:45 585300 316
к абортосрачу Верно ли, что вытащить из бабы живую зиготу и поддерживать её жизнедеятельность за копейки нет никаких трудностей?
Аноним 22/11/23 Срд 20:35:18 585301 317
>>585300
Зигота, зигота перейди на Федота.
Аноним 22/11/23 Срд 20:49:53 585302 318
>>585301
зигота стримит доту
Аноним 22/11/23 Срд 23:13:21 585306 319
>>585300
Трудности начинаются, когда зигота начала дифференциацию клеток и стала вырабатывать сигнальные вещества, которые включают целую гормональную программы в тушке матери, замыкания тем самым петлю обратной связи. Разрыв этой связи несет очень серьезные последствия как для зиготы, так и для матери.
Аноним 23/11/23 Чтв 08:52:51 585309 320
Есть тут жизнь какая-нибудь?

Посоветуйте годных учебников по ядерной физике/химии, с упором на теоретическую базу, где будет рассказываться, какие бывают изотопы и с чем их едят, какие продукты распада у разных веществ, и всё с этим связанное. Какие (Приблизительно) процессы происходят в атоме, базовые формулы, термины необходимые для вката во всё это.

Учебники по физике и химии для школы - сразу нахуй. Во-первых, там всё очень поверхностно, с кучей условностей и упрощений, которые только сбивают с толку, а во вторых, это я уже читал
Аноним 23/11/23 Чтв 17:44:20 585314 321
>>585309
Матан, линал, КЭД знаешь?
Аноним 23/11/23 Чтв 18:12:17 585315 322
>>585306
А заморозить зиготу в одноклеточном состоянии (чтоб либо она была в анабиозе и в любой момент можно оживить, либо просто жила, но не делилась) возможно? Речи о дальнейшем делении и формировании личинуса не идёт. Типа чтоб сказать: вот, мы не убили человека, живёт себе. Ну да, немножко недоразвитый, но не убили же, да?
Аноним 23/11/23 Чтв 23:52:40 585317 323
Я понимаю что это фильм, что там все натянуто.
Но он позиционирует себя как псевдонаучный.
Но даже тут у меня вопрос.
Это для тех кто фильм смотрел.

Понятно что раздел про науку, но все же.

Посмотрел фильм Прибытие 2016 года, я его и раньше смотрел.
Там инопланетяне изменили разум Луизы подобно своему, а они находятся сразу во всех точках временной линии одновременно. То есть для них понятие времени, оно не такое как у людей.
И всегда возникает вопрос, как Луиза узнала номер генерала?
Если её сознание находиться сразу во всех точках, но во всех точках определенной временной линии, линии где человеки напали на инопланетян. И в этой временной линии нет того банкета, за все хорошее где генерал лично прилетел к Луизе и сообщил свой номер.
Это сюжетная дыра, или я что то не понимаю?
Ну то есть она видит будущее, грубо говоря. Будущее где человеки напали на инопланетян. Но берет информацию из будущего где генерал дал её номер в будущем, где в прошлом она позвонила ему и остановила нападение.
Аноним 24/11/23 Птн 05:26:44 585326 324
Интеллект во взрослом возрасте можно повысить? Не в смысле общую эрудированность или профессиональные навыки, а конкретно когнитивные способности головного мозга. Или если генетически тебе суждено быть 98iq дуралеем, то ты им и останешься?
Аноним 24/11/23 Птн 06:13:06 585327 325
>>585314
Частично. Мои знания ограничиваются первым курсом вузика
Аноним 24/11/23 Птн 07:21:44 585328 326
>>585326
Можно. Но в молодости это сделать проще.
https://habr.com/ru/articles/395965/
Мозг настолько нейропластичен, что может относительно нормально функционировать даже без 90% своих тканей, по крайней мере в единичных случаях и при постепенной потере этих тканей, чтобы было время на адаптацию.
Что уж говорить о возможности развития интеллекта. Но тебя на харкаче могут начать убеждать в обратном, вера в полный генетический детерменизм и/или в то что развиваться можно только в молодости здесь очень распространена из-за того что тут каждый второй самооправдывающийся лентяй и/или нытик.

>>585186
>где из вещества каким то образом абсорбируется вода
Она не абсорбируется.
Она нагревается до такой степени, что начинает нагревать другие составляющие сушёного листа, от чего лист возгорается. Не вижу ничего непонятного в этом. Лист же может загореться от зажигалки, независимо от того что в нём есть вода? Может. Почему он не может загореться от очень-очень горячей воды, которой настолько мало, чтобы перекрыть листу доступ к кислороду, и она настолько горяча, что не может забрать у листа энергию, которая нужна ему для возгорания, а может только наоборот, отдавать?
Аноним 24/11/23 Птн 07:47:26 585331 327
>>585328
>может относительно нормально функционировать даже без 90% своих тканей
>Что уж говорить о возможности развития интеллекта.
Так ты ничего и не сказал этой ссылкой. Возможность адаптации мозга к деградации никак не доказывает возможности мозга к прогрессиированию во взрослом возрасте.
Аноним 24/11/23 Птн 08:22:47 585332 328
>>585331
Отчасти это, конечно, справедливо, но как деградировавший мозг мог бы функционировать на описанном в статье уровне без его развития? Адаптация к уменьшению размера подразумевает, что среднее "качество" мозга на кубический сантиметр должно быть увеличено.
В любом случае, этой статьёй я просто подкрепил мои утверждения, которые основываются на личном опыте, это не железобетонный пруф, да.
Аноним 24/11/23 Птн 14:33:45 585339 329
Как на практике осуществима генная модификация при помощи crispr-cas? В интернете одна хуйня научпопная, не нашёл ни как получить белки криспр и кас9 из бактерий, ни как заложить в кас9 нужную последовательность аминокислот. Если первое, могу предположить, получают, используя хроматограф, то со 2 вообще не понятно нихуя.
Что конкретно надо с чем смешивать, начиная с сырья в виде культуры бактерий и химических реактивов с небольшой молекулярной массой, чтобы геном редактировать у, например, червей?
Аноним 24/11/23 Птн 16:43:09 585340 330
>>584346 (OP)

Вопрос по физике. Начитался всякой херни о квантовой запутанности, и словил несварение мозга. Анон поясни - есть две запутанные частицы. Измение состояния одной меняет состояние другой. Причём мгновенно. Что будет, если одну из них подвергнуть релятивистким эффектам? Они как-то согласуют системы отчета, или у них, у каждой будет своё мгновенно?
Аноним 24/11/23 Птн 17:14:11 585341 331
>>585317
Там нет временных линий есть один сценарий детермизма, или даже фатализма.
Как герою фильма сказать что ониумрет в конце сюжет останется тот же
Аноним 24/11/23 Птн 18:26:21 585342 332
>>585340
Две или более запутанные частицы на самом деле одна квантовая система и происходит одно измерение, но двумя детекторами.
Вся запутанность между детекторами, а не частицами, которые измеряют. И чтоб запутанность пронаблюдать, нужно приготовить нужное состояние для частицы и синхронизировать детекторы. Вот тут и причиность ебет всю мгновенность.
Аноним 24/11/23 Птн 19:51:01 585343 333
>>585340
>Измение состояния одной меняет состояние другой. Причём мгновенно.
Нет.
У тебя есть красный и синий шарик. Ты, не глядя, кладешь их в две коробки. Потом одну из них открываешь и волшебным образом узнаешь что лежит во второй.
Аноним 25/11/23 Суб 09:19:01 585352 334
>>585343
Иди читай матчасть. Твои шарики образуют статистическую смесь, это не выпуклая комбинация элементов гильбертова пространства. Тогда как для суперпозиции внедиагональные элементы матрицы плотности ненулевые, откуда и возникает интерференция и запутанность.
Аноним 25/11/23 Суб 16:31:56 585381 335
>>585352
Метод окраски шариков сути процесса не меняет.
Факт в том, что если один из них перекрасить, то со вторым не случиться вообще ничего. Связь между ними существует только в твоем воображении.
Аноним 25/11/23 Суб 17:24:52 585384 336
>>585381
Где я хоть что-то сказал про покраску?
Ещё раз, открой учебник, хуже не будет.
Аноним 25/11/23 Суб 20:11:15 585385 337
>>585384
>открой учебник, хуже не будет.
Еблан, если ты не можешь простыми словами донести мысль до другого, то не факт, что ты эту мысль сам понимаешь
Аноним 25/11/23 Суб 21:53:23 585386 338
вот этот >>585352 правильно говорит
суперпозиция состояний это линейная комбинация состояний как векторов, то есть есть одно единое состояние, выражаемое как сумма
а то о чем пишет >>585343 это вероятностная смесь, то есть когда веса (вероятности) просто отражают степень неопределенности
эти понятия различали почти столетие назад, в нормальных учебниках или даже статьях википедии будет все написано
советую книжку чанга про квантовые вычисления для вката
это >>585381 вообще какой-то нерелевантный бред
Аноним 25/11/23 Суб 23:03:17 585387 339
>>585386
Суть в том, что если бы частицы действительно были связаны, то можно было бы с легкостью организовать мгновенную связь через любые расстояния. А ее всё нет и нет.
И все эксперименты состоят из тупого открывания одной коробки с последующими попытками представить знание о содержимом другой коробки результатом телепортации.
Но это всё просто болтовня.
Аноним 26/11/23 Вск 04:30:05 585389 340
>>585387
Ты просто не шаришь за теорию измерений, поэтому не понимаешь в чём не прав. А тратить время на пояснения всем лень, т.к. ты наверное даже математику не знаешь
Аноним 26/11/23 Вск 06:29:35 585391 341
>>585315
>А заморозить зиготу в одноклеточном состоянии (чтоб либо она была в анабиозе и в любой момент можно оживить, либо просто жила, но не делилась) возможно?
Конечно и это давно распространенная практика. И эмбрионы замораживают (на стадии нескольких клеток), рекорд с родами 20 лет вроде.
Аноним 26/11/23 Вск 14:48:38 585396 342
>>585389
> Я сам не понимаю, потому обьяснить не могу
Понятно
Аноним 27/11/23 Пнд 04:39:58 585405 343
Trinity Test La[...].mp4 167Кб, 840x412, 00:00:04
840x412
Наукач поясни что происходит в центре на шебм. На шебм первые секунды первого в мире испытания технологии ядерного оружия. Я вообще не шарю, я так понимаю это плазма, но почему плазма делает "пуньк" сама в себя, ну как будто какой то портал открывается и туда плазма вбирается, как в кине епта научно фантастическом. Эта херь обьяснена уже? Че за эффект тогда)
Аноним 27/11/23 Пнд 11:08:04 585406 344
>>585405
В плазме скорость звука значительно выше чем в окружающей среде, отчего часть ударных волн на границе раздела фаз сами себя отражают и сжимают.
Так же при расширении плазменный шар охлаждается, превращается в газ или пыль.
В сочетании этих факторов плазма сжимается собственной ударной волной и одновременно худеет из-за охлаждения.
Аноним 27/11/23 Пнд 14:53:51 585408 345
>>585406
Нихуя себе, спасибо анонче
Аноним 29/11/23 Срд 11:52:31 585439 346
Вселенная расширяется потому что из большего числа мест до нас начинает долетать свет или потому что тёмная энергия? Со временем мы будем видеть звёзд больше или меньше?
Аноним 29/11/23 Срд 13:36:08 585441 347
Можно ли скрестив гены псины и хряка получить свинособаку!!?
Аноним 29/11/23 Срд 13:45:56 585442 348
>>585439
>Со временем мы будем видеть звёзд больше или меньше?
Меньше. В какой-то момент вообще вся Вселенная станет тёмной. Галактики-то ещё будут, но где-то там настолько быстро удаляющиеся, что свет их никогда нас не достигнет. Будущие обитатели галактик будут думать что их галактика и есть вся Вселенная, как мы до Хаббла считали. А может и мы тоже чего-то важного для понимания уже не видим
Аноним 29/11/23 Срд 14:13:24 585445 349
>>585439
Потому что темная энергия.
И наоборот, нового места становится больше, а наполнение этого места тоже самое.
Аноним 30/11/23 Чтв 18:59:50 585479 350
Есть какие-то современные виды птиц, уходящие эволюционными корнями в меловой период? Или все современные птицы происходят от одного предка, жившего уже после мелового вымирания?
Аноним 30/11/23 Чтв 19:42:37 585480 351
>>585479
> Или все современные птицы происходят от одного предка, жившего уже после мелового вымирания?
Предка которого инопланетяне завезли? Птицы прямые потомки динозавров.
Аноним 30/11/23 Чтв 20:02:13 585481 352
>>585480
Общий предок-птица современных птиц появился до или после мел-палеогенового вымирания?
Аноним 30/11/23 Чтв 20:02:43 585482 353
>>585480
Общий предок-птица современных птиц появился до или после мел-палеогенового вымирания? >>585480
Аноним 30/11/23 Чтв 21:20:26 585484 354
1. Почему в русскоязычной среде принято отрицать/приуменьшать значение IQ? В англоязычных странах это делают только по политическим мотивам. У нас же IQ отрицает и народ, и публика с претензией. Это так в вузах преподают? Откуда такое отношение?
2. Почему в РФ настолько распространено отрицание потепления?
Аноним 30/11/23 Чтв 21:35:52 585485 355
>>585484
>1. Почему в русскоязычной среде принято отрицать/приуменьшать значение IQ? В англоязычных странах это делают только по политическим мотивам. У нас же IQ отрицает и народ, и публика с претензией. Это так в вузах преподают? Откуда такое отношение?
IQ показывает исключительно надроченность на логических задачках. Решал задачки — показываешь высокие результаты. Не решал — не показываешь. С таким же успехом "уровень интеллекта в вакууме" можно было бы измерять способностью распознавать цвета (точнее, знать названия всяких серобуромалиновых оттенков) или скоростью решения кубика-рубика. Тупо приобретаемый навык.

>2. Почему в РФ настолько распространено отрицание потепления?
Потому что ещё в лохматые годы учёные говорили о климатических циклах и о том, что через десятилетия в Москве будут бананы расти. Т.е, никакого антропогенного потепления от того, что коровы пукают, нет. Периоды потеплений-похолоданий были всегда, на протяжении всей истории человечества.
Аноним 30/11/23 Чтв 21:47:22 585486 356
>>585485
>IQ показывает
Твое неверное мнение это очень интересно, конечно, но я спрашивал о другом.
>Потому что ещё в лохматые годы учёные говорили о климатических циклах
У тебя не больше 90, я думаю. Понимание текста даётся с трудом.
Аноним 30/11/23 Чтв 21:50:14 585487 357
>>585486
> но я спрашивал о другом.
На западе давно признано, что важность IQ равна 20%, а на 80% рулит эмоциональный интеллект.
Аноним 30/11/23 Чтв 21:52:34 585488 358
>>585487
80? Ты как, шнурки легко себе завязываешь, проблем нет? Держись там.
Аноним 30/11/23 Чтв 22:33:09 585489 359
>>585484
>1. Почему в русскоязычной среде принято отрицать/приуменьшать значение IQ? В англоязычных странах это делают только по политическим мотивам.
Так у нас тоже политические. Вон как сейчас топят за IQ, мол, господин Барин выиграл в генетическую лотерею и получил приз - высокое айсикью и надел рабов с тянками, ему положено, генетические детерминизм, блекпилл, всё такое. Какая-то правая либерал-шиза прямиком с лахтоцентра.

> У нас же IQ отрицает и народ, и публика с претензией. Это так в вузах преподают? Откуда такое отношение?
Так всё правильно же. В России сохранилось ещё материалистическое мышление, путём нехитрых рассуждений всегда можно прийти к выводу что айкью нихуя не влияет, а идеология и обучение влияют в сотни и тысячи раз больше. Логично, хуле.

>2. Почему в РФ настолько распространено отрицание потепления?
Уже нет. Два лета выдались жаркими шопиздец, дождей нет, зимы нет. Теперь каждый говорит "а про потепление-то есть, кажись они правы были".
Аноним 30/11/23 Чтв 22:46:09 585491 360
>>585489
>Вон как сейчас топят за IQ
Кто?
>Какая-то правая либерал-шиза прямиком с лахтоцентра.
што
>путём нехитрых рассуждений всегда можно прийти к выводу что айкью нихуя не влияет
Аахахахахах. И ты даже сам не понял, что сказал, это прекрасно.
>Два лета выдались жаркими шопиздец, дождей нет, зимы нет.
Ноучная доска, ясно.
Аноним 30/11/23 Чтв 22:52:16 585492 361
>>585491
> Кто?
> што
> Аахахахахах. И ты даже сам не понял, что сказал, это прекрасно.
> Ноучная доска, ясно.
Ты это, хотя бы предложение пиши чуть более развёрнуто, не знаю, какие-то утверждения в них делай, аргументы, какие-то опровержения. Например "я опровергаю тебя и привожу это как доказательства". Понимаешь что я?
Аноним 30/11/23 Чтв 23:00:47 585493 362
>>585492
Я тебя услышал: оказывается, всё наоборот, и все топят за IQ, потому что государство проплатило IQ-ботов с целью продвижения правого либерализма. Понял, принял.
Спорить с тобой про валидность IQ я не собираюсь.
Аноним 01/12/23 Птн 07:48:15 585503 363
>>585493
>потому что государство проплатило
Только не государство, а ужратые жиды, и наверняка проплатили пижженными бабками. Потому что, никаких государств - не существует в принципе, без государственного суверенитета, на глобальном-то рынке, лол.
Аноним 01/12/23 Птн 08:00:32 585504 364
>>585484
Потому что результаты айкю зависят от тренировки и мотивации, то есть не являются объективным показателем интеллекта. А на западе люди туповатые и склонные к депрессии, поэтому им нужно чем то компенсировать свои недостатки
Мне 38 годиков, помню 20 лет назад зимы были -25 по всей центральной РФ. Сейчас дай Бог -10, иногда -15 бывает. Так что потепление отрицают только тупые зумеры, которые жизни не видели
>>585485
Верно. Это как умение играть в шахматы или покер. Никто же не говорит что калькулятор умнее человека, только по тому что он задрочен на быстрое вычисление определённых задачек
Потепление уже как лет 20 заметно невооруженным взглядом
>>585488
Еще 100 лет назад средний айкю в США был 70 баллов. И что? За 100 лет надрочились выдавать на 30 баллов больше, только и всего
Я понимаю что айкю-дрочерам неприятно это осознавать, но такова реальность
Аноним 01/12/23 Птн 08:21:42 585506 365
>>585489
Самое важное это социальные навыки
Аноним 01/12/23 Птн 08:58:31 585509 366
image.png 190Кб, 597x593
597x593
>>585493
>Я тебя услышал: оказывается, всё наоборот, и все топят за IQ, потому что государство проплатило IQ-ботов с целью продвижения правого либерализма. Понял, принял.
Ну а что не так? Это не совсем правый либерализм, это просто правая шизофрения ради оправдания правления барина. Там как мартышек тренируют: Почему барин? Потому что айкью и гены! Что делать? Принимать блекпилл и не качать лодку!

Ты же не думал что все эти треды c всякими редпилами и iq, которые регулярно появляются, это всё просто так шизики срут годами? Не думал же что проправительственные блогеры, форсящие эту хуйню, форсят бесплатно?

>Спорить с тобой про валидность IQ я не собираюсь.
Тут и спорить невозможно, с чем ты собрался спорить-то, с тем что IQ может заменить хорошее образование? Или с тем что IQ постоянно росло? Что там, генетика поменялась у людей за 50 лет? Это же звучит как шизоидная хуета, бред просто, шизофрения порвачков, таблетки где, виу-виу-виу дурка едет нахуй блядь

Вся эти хуйня с IQ это просто форс ради барина. Если раньше барина оправдывала церковь, то сейчас форсят IQ и превосходство в генах. Это изначально форсилось как политическая хуета, ею остается и поныне.

>>585506
Неа. Идеология. Сначала идеология потом всё остальное, включая образование.
Аноним 01/12/23 Птн 09:08:12 585510 367
>>585509
>с тем что IQ не может заменить хорошее образование
слоуфикс
Аноним 01/12/23 Птн 09:13:05 585511 368
>>585509
>Что там, генетика поменялась у людей за 50 лет?
стандартное оправдание детям стало хватать еды
Аноним 01/12/23 Птн 09:15:12 585512 369
>>585511
Тогда получается в США и Европе все голодными были целых сто лет, негоже такое говорить. Это уже нестандартное оправдание.
Аноним 01/12/23 Птн 11:00:09 585514 370
>>585512
>Это уже нестандартное оправдание
Нестандартное для кого? Это буквально мэйнстрим исторической науки, принимаемый всеми. Ты всех своим скудным необразованным умишкой не наделяй
Аноним 01/12/23 Птн 12:02:52 585515 371
>>585514
Ты бредишь. По твоему 100 лет назад люди были даунами? Но историческая наука сей тезис не подтверждает.
Дело банально в тренировке. Людей поколение за поколением прогоняли через шкалы, где они тренировали способности решать различные тесты. Это тупо приспособленность к определенной среде
Тащемто, уже давно доказано, что даже пару дней тренировок перед тестами повышают результаты на 10 баллов. А мотивация в виде деняг, еще на 20 баллов
Аноним 01/12/23 Птн 12:27:24 585516 372
>>585504
>зависят от тренировки и мотивации, то есть не являются объективным показателем
охуенно
Аноним 01/12/23 Птн 12:58:38 585517 373
>>585516
Да охуенно. Это показатель мотивации и тренировки тестов, а не интеллекта
Аноним 01/12/23 Птн 13:14:43 585518 374
почему при механическом взаимодействии двух электрически нейтральных тел в процессе трения заряды переходят от одного тела к другому?
Аноним 01/12/23 Птн 14:45:28 585519 375
>>585515
>По твоему 100 лет назад люди были даунами
Даунами они, конечно, не были, но по-моему очевидно, что 100 лет назад средний европейский рабочий питался хуже (особенно в детстве), чем сегодняшний зумер, но даже в этот период уровень жизни уже был лучше уровня городских рабочих в 19 веке
>Людей поколение за поколением прогоняли через шкалы, где они тренировали способности решать различные тесты. Это тупо приспособленность к определенной среде
Это правда, действительно, если хорошо кормить ребёнка, а также заставлять его заниматься умственным трудом, он, вероятно, будет умнее. Проблема заключается в том, что на разные человеческие популяции эффект Флинна тоже работает и голодные суб-сахарские негры отстают от своих упитанных афро-американских братьев, но последние всё равно катастрофически уступают белым, восточным азиатам, евреям, высшим индийским кастам, несмотря на то, что все эти люди живут в США уже несколько поколений и существуют в одной пиндосской системе образования, но почему то натренированные поколениями решать тесты негры всё равно отстают от таких же натренированных решать тесты белых и азиатов. И даже если сказать, что жесткое расовое угнетение насрало чёрным в статистику, то почему расовые ограничения не насрали в статистику азиатскому населению США?
Аноним 01/12/23 Птн 16:13:17 585521 376
>>585518
Гугли трибоэлектрический эффект.
Это комплексное явление, которое в одном посте не объяснишь.
Аноним 01/12/23 Птн 16:35:52 585522 377
>>585514
Речь про IQ, а не про историческую науку, о чем ты? Связь между доедание и айсикью вообще нихуя не доказана, кстати, тупо корреляция ради корреляции.

Нестандартная она потому что все эти тесты имеют политическую основу - каждый шиз который рассказывает про тесты будет рассказывать про негров, про то какие они тупые, про ещё какую-то порвачью хуйню, но никто в своей пропаганде не будет рассказывать что Европа и США недоедали последние 100 лет, или имеют огромные проблемы с образованием, это очень неприятно.

>>585519
> Это правда, действительно, если хорошо кормить ребёнка, а также заставлять его заниматься умственным трудом, он, вероятно, будет умнее.
@
> Проблема заключается в том, что на разные человеческие популяции эффект Флинна тоже работает и голодные суб-сахарские негры отстают от своих упитанных афро-американских братьев, но последние всё равно катастрофически уступают белым, восточным азиатам, евреям, высшим индийским кастам, несмотря на то, что все эти люди живут в США уже несколько поколений и существуют в одной пиндосской системе образования, но почему то натренированные поколениями решать тесты негры всё равно отстают от таких же натренированных решать тесты белых и азиатов.

Ну и что это? Ты так ничего и не понял?

Тебе сначала придётся доказать что они действительно доедают и действительно обучаются, ведь оказывается что "эффект Флинна" может по-разному на разные человеческие группы влиять, что вполне очевидно.

Но доказать ты это не можешь потому что таких исследований не проводится, а значит можешь смело свои порвачьи шизосказки выкидывать нахуй - они просто не имеют доказательств, зато имеют огромнейшую проблему в виде роста IQ на 30 балов за 100 лет.
Аноним 01/12/23 Птн 16:42:52 585523 378
Аноним 01/12/23 Птн 16:53:44 585524 379
image.png 190Кб, 597x593
597x593
>>585523
А это что тогда? Врёти?
Аноним 01/12/23 Птн 19:45:42 585528 380
>>585522
>придётся доказать что они действительно доедают
Доказываю, чёрные - группа населения с самым большим ожирением, а значит они доедают
>и действительно обучаются
Среднее общее в США обязательно, значит, обучаются. В школе. Да-да и тесты там тоже есть.
Есть возражения?
Аноним 01/12/23 Птн 20:45:33 585529 381
>>585528
>Есть возражения?
У тебя вместо доказательств пуки в лужу, о чём ты? Нужны научные исследования по большим когортам.
Аноним 01/12/23 Птн 20:55:56 585530 382
>>585521
>Всеобъемлющая теория электризации пока не построена
ну ясно
Аноним 01/12/23 Птн 21:06:27 585531 383
>>585529
>Ты не можешь доказать, что они действительно доедают
>Доказываешь
>Ваши доказательства - не доказательства
Аноним 01/12/23 Птн 21:12:30 585532 384
>>585531
Порвак, плиз. То что ты пукнул какой-то хуйней является доказательством только в среде порвачков. Доказательства так не рабоатют.
Аноним 01/12/23 Птн 21:15:51 585533 385
>>585532
Маня, ты написал >Тебе сначала придётся доказать что они действительно доедают и действительно обучаются
И оказалось, что они и доедают и обучаются
Аноним 01/12/23 Птн 22:05:52 585534 386
>>585519
И че с того? Количество неравно качество
>но почему то натренированные поколениями решать тесты негры всё равно отстают от таких же натренированных решать тесты белых и азиатов.
Кто сказал? Майк Тайсон например жил в гетто, как и большая часть нигеров. Так что ни о каких равных условиях тут речи не идет.
Аноним 01/12/23 Птн 22:15:40 585535 387
Heritabilité-in[...].jpg 118Кб, 727x549
727x549
corrélation-int[...].jpg 103Кб, 710x374
710x374
crime-1.jpg 42Кб, 690x529
690x529
2023-12-0202-06[...].png 298Кб, 597x769
597x769
>>585485
Обоссываем тупую скотину за два шага:
1. Если iq негенетический, а механически надрачиваемый, как так получается, что у близнецов выросших в разных семья он коррелирует, несмотря на разные условия жизни (пик 1), а у людей разного происхождения выросших в одной семье во взрослом возрасте никакой корреляции нет ? (пик 2)
2. В случае, если iq генетический, но отражает ТОЛЬКО способность решать тесты в школе, каким образом получается, что iq коррелирует с агрессивностью (пик3), или уровнем дохода(пик 4)? Как способность решать тесты в школе может быть связана с шансом сесть в тюрьму и вообще с любой интеллектуальной деятельностью, которая у этих людей получается лучше, хотя они всего лишь "поколение за поколением прогоняли через шкалы, где они тренировали способности решать различные тесты"?
Аноним 01/12/23 Птн 22:19:28 585536 388
>>585190
Кибернетика это лженаука, придуманная фашистами.
Аноним 01/12/23 Птн 22:21:13 585537 389
>>585534
>Кто сказал? Майк Тайсон например жил в гетто, как и большая часть нигеров. Так что ни о каких равных условиях тут речи не идет.
Так это же пример против твоего же мнения, если черные доступа к образованию не имеют, но iq у них всё равно растёт, значит образование и iq не связаны, а значит растёт он у них по другим причинам
Аноним 01/12/23 Птн 22:30:55 585539 390
>>585535
>iq коррелирует с агрессивностью (пик3)
А что имеется в виду? Хитрый социопат с высоким IQ будет умело манипулировать окружающими и строить на публике доброго няшу, будучи на самом деле тираном. А вот тупой дебил, нажравшись, устроит пьяный дебош. Это отнюдь не значит, что у второго уровень агрессии выше, чем у первого.

Покажите исследования, где IQ будет коррелировать со склонностью к психопатии, социопатии, маккиавелизмом, нарциссизмом, абьюзом и т.д. Нет таковых, конечно. Можно иметь сверхвысокий IQ и полностью нулевую эмпатию. У чиновников и депутатов, хитрых шифрующихся педофилов и многих других психопатов довольно высокий IQ, что и позволяет им умело маскироваться под нормальных людей и управлять последними в своих целях. Только некоторые шизики видят их истинную сущность и создают теории о рептилоидах.

>или уровнем дохода(пик 4)
Здесь, думаю, очевидно. Успешно сдаешь тесты вроде ЕГЭ - получаешь лучшее образование. Хорошее образование определяет уровень дохода.
Аноним 01/12/23 Птн 22:48:29 585540 391
>>585539
>что имеется в виду
Блядь, ну там же подпись есть под картинкой даже, какой вопрос задали опрашиваемым
>Это отнюдь не значит, что у второго уровень агрессии выше, чем у первого
Буквально это и значит, ты просто слово "агрессивность" странно как то понимаешь, что в манипуляциях окружающими агрессивного? (Но на самом деле это просто дело определения, конечно)
Аноним 01/12/23 Птн 23:29:12 585541 392
>>585533
Ты тупой, блядь, что ли, порвак? Доказательство это не когда ты пукаешь какой-то хуйней.

Доказательства это несколько научных независимых исследований.
Аноним 01/12/23 Птн 23:32:23 585542 393
image.png 190Кб, 597x593
597x593
>>585535
Давай я тебя обоссу за 1 шаг.

1. IQ за 100 лет повысился на 40 баллов.
Аноним 02/12/23 Суб 00:14:45 585543 394
>>585509
Зачем ты пытаешься критиковать тему, в которой не разбираешься?
Ну если тебе не нравится IQ мы можем напрямую пойти в генетику и изучать как наследуются гены, которые ассоциируются с интеллектом или с обучаемостью, или с заработком, кстати гены "богатства" и гены интеллекта не полностью совпадают (по очевидным причинам). И хули? Будешь сейчас рассказывать, что люди от рождения одинаковые рождаются? Или что у всех одинаковый потенциал?
Аноним 02/12/23 Суб 00:17:50 585544 395
>>585542
А как то, что его можно надрочить противоречит тому, что успешность его надрачивания зависит от генов? Втф?
Аноним 02/12/23 Суб 09:41:55 585561 396
>>585542
>В случае, если iq генетический, но отражает ТОЛЬКО способность решать тесты в школе, каким образом получается, что iq коррелирует с агрессивностью (пик3), или уровнем дохода(пик 4)? Как способность решать тесты в школе может быть связана с шансом сесть в тюрьму и вообще с любой интеллектуальной деятельностью, которая у этих людей получается лучше, хотя они всего лишь "поколение за поколением прогоняли через шкалы, где они тренировали способности решать различные тесты"?
Аноним 02/12/23 Суб 09:51:34 585562 397
>>585537
>черные доступа к образованию не имеют, но iq у них всё равно растёт, значит образование и iq не связаны, а значит растёт он у них по другим причинам
1. Программы благоустройства хоть и медленно, но работают, а значит, всё больше негров оканчивают школы и даже вузы.
2. IQ и образованность зависят ещё и от культуры. Даже имея доступ к бесплатной вышке, условный нигга (мага, вася) будет забивать хуй на саморазвитие, читать рэпчик и ходить на турнички — потому что пацанская жизнь для него имеет бóльшую ценность, чем лоховское ботанство.
Нужны глубокие переломы в менталитете и смена поколений, чтобы увидеть какой-то заметный прогресс.
мимо
Аноним 02/12/23 Суб 10:24:46 585564 398
>>585543
Нихуя маневры начались.

Ты мне ответь, почему IQ увеличился на 35 баллов, если он от генома зависит?

>>585544
Т.е. теперь уже у тебя не IQ, а какое-то надрачивание? Интересный манёвр.

Покажешь мне научные работы по этому надрачиванию? Его кто-то измерял? Что это, ехех, что, надрачивание-то, а?

>>585552
Найс порвак порвался. Понимаю, очень неприятно. Тебя годами тренировали что IQ это врождённое свойство барина, а тут нестыковочка, бух, пух, порвак порвался!

>>585561
Ты давай без манёвров, вопрос-то предельно простой.

Как так вышло что IQ выросло на 35 баллов за 100 лет? Гены сменились? Чо, как вообще в целом? Инополнетяне облучали и сменили геном? Леваки правым в шатны насрали? Кто говно подложил-то? Кто?
Аноним 02/12/23 Суб 11:55:02 585566 399
>>585564
> Как так вышло что IQ выросло на 35 баллов за 100 лет?
Культурное влияние.
Смотри, в ДНК заложен темперамент, который в равных культурных условиях даст разное IQ. IQ это логические закономерности и, соответственно, у логика он будет выше, при прочих равных условиях.
Но есть и культура, т.е. в каких условиях воспитывался индивид, если 100лет назад это было молится Богу и пасти коров, то сейчас, сначала через распространение техники, а потом и компьютеров, которые суть логические системы, человек воспитывается в логической среде, поэтому IQ на тестах и растёт, т.к. даже не предрасположенные природно к логике интуиты начинают эту логику понимать, а природные логики понимать еще глубже. Вот и все.
Аноним 02/12/23 Суб 12:30:11 585568 400
image.png 241Кб, 640x480
640x480
>>585566
> Культурное влияние.
Получается все те у кого низкое айсикью и нет физиологических отклонений - просто не подвержены культуре, на них это не повлияло. Загадка решена!

> IQ это логические закономерности
Не, логические закономерности не IQ, с чего ты это взял? Ты слово "логика" не встретишь рядом с "IQ" ни в одной научной работе.

> у логика он будет выше, при прочих равных условиях.
Неее, таких данных не существует. И что за логик ещё? Твой протык? Барин твой, ты так его уважительно называешь?

> человек воспитывается в логической среде, поэтому IQ на тестах и растёт,
Ну т.е. все у кого низкое айкью просто росли в нелогичной среде? Загадка решена!

> природные логики
Кто, блять? Что это за магические звери такие? Хоть кто-то их определил?

Тут это, понимаешь что произошло. Тут тест на IQ от "логической среды" зависит, откуда я тут знаю кто логик, а кто нет, я его определить не могу.
Аноним 02/12/23 Суб 12:39:58 585569 401
>>585568
>Получается все те у кого низкое айсикью и нет физиологических отклонений - просто не подвержены культуре, на них это не повлияло. Загадка решена!
Еблан, я же написал, что не одна культура, а и
>в ДНК заложен темперамент, который в равных культурных условиях даст разное IQ

>Не, логические закономерности не IQ, с чего ты это взял? Ты слово "логика" не встретишь рядом с "IQ" ни в одной научной работе.
Открой википедию, шизик.
>Каждый тест состоит из множества различных заданий нарастающей сложности. Среди них тестовые задания на логическое и пространственное мышление, а также задания других типов — в тесты обычно входят логические и арифметические задания, ориентировка в практических ситуациях — умение самостоятельно сопоставлять, обобщать известные факты (творческий подход, в том числе нестандартное мышление — допускается неоднозначный ответ, формулировка нескольких гипотез, разная аргументация), проверка оперативной памяти и т. п.
Аноним 02/12/23 Суб 14:17:48 585573 402
>>585569
> Еблан, я же написал, что не одна культура, а и
И как ты собрался отделять культуру от некультуры? Магическим компасом?

> в ДНК заложен темперамент, который в равных культурных условиях даст разное IQ
Шизофантазии которые ничем не подтверждены.

> Открой википедию, шизик.
Когда википедия стала научной статьёй? Или ты читать разучился, порвак?

Речь была про научную статью, ты осознаешь написанное мною или ты уже огородился в манямирок и не воспринимаешь реальность?

Притом на английской вики такого даже нет, только русская вики изобилует подобными шизофантазиями. Наверняка какой-то лахтёнок переписывал всю статью чтобы барину подлизнуть.
Аноним 02/12/23 Суб 16:52:46 585577 403
>>585535
Ты совсем дурачок? Айкю это комплексное явление, которое зависит от кучи факторов, в том числе и генетических.
Все просто, с дипломом проще устроится на работку. Что тут не понятного? Давно известно, что без бумажки ты хуй простой. Хотя исключений тоже хватает. Например Джобс даже шарагу не закончил. Да и по жизни был полным ублюдком. Что впрочем не помешало ему стать богатеем
>>585537
В школу то они ходят, да в шараги всякие
>>585566
Скорее уж наоборот. Это раньше люди решали логические задачи, как построить дом, выследить зверя, привести инструмент в рабочее состояние и т.д. А сейчас в школах 90% материала это абстракция и теория, которая имеет мало общего с практикой и прикладной логикой. В большинстве случаев долбоеб с красным дипломом, даже розетку дома починить не может.
Аноним 02/12/23 Суб 16:54:55 585578 404
>>585536
Соционика это лженаука, придуманная коммунистами.
Аноним 02/12/23 Суб 21:21:38 585585 405
>>585573
>Притом на английской вики такого даже нет, только русская вики изобилует подобными шизофантазиями. Наверняка какой-то лахтёнок переписывал всю статью чтобы барину подлизнуть.
Таблетки прими, шиз, везде тебе заговоры видятся - паранойя называется
Аноним 02/12/23 Суб 21:22:26 585586 406
image.png 623Кб, 1024x576
1024x576
>>584349
Птерозавры вообще охуевшими были. Они и бегать могли, и летать, и прыгать и всё это делали в стаях. Универсальные твари, прямо из какого-то фантастического романа. Им бы немного мозгов побольше, плодовитости и всё, пизда всему живому, приземляется такая хуйня и сразу же бежит на тебя отрывая нахуй голову.
Аноним 02/12/23 Суб 21:32:27 585589 407
>>585585
Какой это заговор? Ты что, не в курсе что википедию регулярно проправительственные организации атакуют и переписывают? Лол, да на эту тему даже статья из вики есть.

Это не какой-то заговор, это просто какая-то лахтохуйня по методичке шизу хуярит. Ну либо те кто начитался правых лахтосказок и спешат открыть миру правду через правки в вики.

Посмотри только, ты тоже нахавался этой хуйни и рассказываешь шизоидные сказки про IQ которые буквально звучат как бред. В ДНК темперамент, в ДНК культура, ДНК-надрачивание, просто какие-то бредовые фантазии. Кто только этим дерьмом срёт повсюду.
Аноним 02/12/23 Суб 21:43:41 585592 408
>>585586
Почему я не могу уместить в своей голове научность этой эволюционной хуйни? Что типа оно само вот так вот просто методом тыка, мутаций и отбора выстраивается, а потом ВНЕЗАПНО появляется возможность летать. Это же пиздец, когда организм должен выживать каждую секунду времени МАССОВО претерпевает такие изменение и переходы поведенчески и генетически. И отрастить крылья мало, нужны ещё перья, а ещё и лёгкие скелет, который может оказаться уязвимым для привычных нагрузок при охоте. Ну и просто крылья сами по себе не могут закрепиться никак на ровном месте, тем более без наличия все остальных особенностей для возможности полёта и активного их использования.
Аноним 02/12/23 Суб 21:50:23 585594 409
>>585589
> Ты что, не в курсе что википедию регулярно проправительственные организации атакуют и переписывают?
И чо, и чо? Физику уже переписали? Естествознание, научный метод, социогумманитарные науки и т.д.? Наверное это Дугинцы в темноту веков Википедию переписывают, чтоб на Руси все хороводы водили и радовались Богу солнца, чтоб извели морскую нечисть долларовую и единым порывом создали муравейник Хартланд счастливого общества людей без деняк. Что в этом плохого? Кроме газенвагенов и дахау для морской нечисти на переходной к счастью период. Цель оправдывает жертвы?
Аноним 02/12/23 Суб 21:59:27 585596 410
>>585594
>Цель оправдывает жертвы?
А что нет? На этом вся эволюция строится. Захотелось богу людей вместо динозавров - ебанул астероид. Захотелось новое человечество - утопил всех кроме горстки в ковчеге. Захотелось коммунизма миллионы расстрелянных и сгноенных в гулагах. Ядерная бомба туда же. В чём проблема?
Аноним 02/12/23 Суб 22:47:48 585600 411
>>585594
> И чо, и чо? Физику уже переписали? Естествознание, научный метод, социогумманитарные науки и т.д.?
Не, физику и естествознание переписывают капиталисты и их маркетологи. Несколько лет назад там ведро описывали как работающий двигатель, работающая гомеопатия, лекарства без доказанной эффективности лечат все болезни. Сейчас этой хуйни ещё больше стало. Соцгум почти весь в правильных интерпретациях, история тоже вся, включая исторические мифы поданные как правда или откровенная пропаганда.

> Что в этом плохого?
Что плохого в том что тебя кормят антинаучным бредом про IQ и летающие вёдра? Новые тёмные века довольно плохо, да. Поколение дегенератов которые не понимают что такое интеллект это очень плохо - дегенераты не способны ничего нормального создать.
Аноним 02/12/23 Суб 23:01:50 585602 412
>>585600
> Поколение дегенератов которые не понимают что такое интеллект это очень плохо - дегенераты не способны ничего нормального создать.
И что делать? Может шапочку из фольги одевать? Поможет?
Аноним 03/12/23 Вск 00:58:22 585609 413
>>585602
Создавать ресурсы на которых статьи проходят проверку научным советом двачеров, очевидно же. ща штаны подтяну и создам
Но ты, наверное, ничего не можешь, у тебя жопа горит.
Аноним 03/12/23 Вск 07:17:58 585610 414
>>585592
А вот атеизнутые материалисты уже более 1000 лет верят в самозарождение жизни. По их вере вселенная появилась случайно. Жизнь самозародилась случайно. Разум возник случайно. Охотник пошел в лес и случайно встретил добычу, а потом случайно попал из ружья
Аноним 03/12/23 Вск 09:29:45 585616 415
>>585592
Потому что это происходит не "внезапно"
Аноним 03/12/23 Вск 09:31:05 585617 416
>>585564
>Ты мне ответь, почему IQ увеличился на 35 баллов, если он от генома зависит?
Почему рост увеличился на 15 см, если он от генома зависит?
Аноним 03/12/23 Вск 10:55:17 585621 417
1699772411074277.jpg 2617Кб, 3664x2844
3664x2844
>>585592
>ВНЕЗАПНО появляется возможность летать
Вода и воздух одно и тоже, нет разницы между летать и плавать.
Аноним 03/12/23 Вск 11:16:02 585622 418
>>585617
Почему ты сравниваешь рост и интеллектуальную деятельность? По-твоему это корректное сравнение? Как это можно сравнить вообще? Или ты просто решил высрать аргумент из методички "как защитить айсикью барина для дебилов" не напрягаясь?
Аноним 03/12/23 Вск 11:51:29 585624 419
>>585622
Если твой аргумент работает только для интеллекта (но не для роста), то тебе нужны дополнительные пояснения, чтобы было понятно, почему для роста эта логика не работает
Аноним 03/12/23 Вск 12:09:23 585625 420
>>585624
Погоди, ты привёл аргумент про то что рост и айсикью это одно и то же, а я должен за тебя этот аргумент доказывать?

Нет, анон, это так не работает. Ты привёл аргумент - ты его и доказываешь. Не можешь доказать - значит твой аргумент инвалид умственного труда.
Аноним 03/12/23 Вск 12:49:45 585626 421
>>585625
И рост и ийсикью зависят одновременно от среды и от генов, собственно на этом всё.
Аноним 03/12/23 Вск 13:19:49 585629 422
>>585626
И как ты собрался влияние генов на айсикью измерить-то?

Среда может добавлять +/- 40 баллов айсикью, что ты измерять собрался? Входит ли значение айсикью в доверительный интервал? Ну входит, дальше что?
Аноним 03/12/23 Вск 16:31:28 585634 423
>>585629
Ты знаком с таким явлением как статистика? Ну там учёт среды и т.д.
Аноним 03/12/23 Вск 16:53:50 585635 424
>>585634
Ты вопроса не понял что ли? Как ты собрался корректировать на среду, или вычислять влияние генов, если +/- 40 баллов айкью подходят к границам доверительного интервала?

Не знаю даже, как упростить вопрос, чтобы понял даже верующий в айку.
Аноним 03/12/23 Вск 18:51:14 585638 425
>>585635
Мне надо тебе лекцию по математической статистике и теории вероятности сейчас бесплатно читать? У тебя математическое ожидание по выборке из нескольких тысяч человек тоже +- 40?
Аноним 03/12/23 Вск 22:33:28 585646 426
>>585638
Речь не про конкретный результат выборки, что ты несёшь-то, блядь? Какое матожидание? Матожидание применяются к одной выборке.

Речь про разные выборки. В разные временные промежутки (влияние среды, очевидно) выборки дают разные результаты. Айсикью сдвигается на две сигмы нахуй.

У тебя какой-то ментальный блок стоит на это или что?
Аноним 03/12/23 Вск 22:39:23 585647 427
>>585646
Т.е. найти людей из разной среды, и сравнить их средний IQ это слишком тяжело, да?
Аноним 03/12/23 Вск 23:37:04 585648 428
>>585647
Так тесты откалиброваны на другой выборке.
Аноним 04/12/23 Пнд 04:23:44 585653 429
>>584346 (OP)
Живу в МКД, слышно маргиналов соседей через вентиляцию. + ощущение что фонят стены. Решил рассверлить стену в комнате, и обнаружил, что в стенах присутствуют полости (либо каналы, не уверен точно, т.к. пока не сильно рассверлил стену), по ощущениям гладкие но с порами, примерно 2,5 см в ширину. Включил на кухне вытяжку - и из тех дырок которые я насверлил подул холодный ветерок.
Вопрос: каналы/полости 100% "запитаны" от вент каналов моего дома? Чем это обосновано, что это такое вообще в принципе?
И если нет, то как объяснить попадание воздуха из вент канала кухни в стену комнаты в которой так же есть вент канал
Аноним 04/12/23 Пнд 04:25:07 585654 430
>>585653
Алсо, поджигал спички и водил ими вокруг отверстий и засовывал прямо в них - в наиболее крупном спички затухали даже если их 2 и т.д., т.е. поток воздуха оттуда точно есть.
Аноним 04/12/23 Пнд 11:14:30 585661 431
>>585653
> Вопрос: каналы/полости 100% "запитаны" от вент каналов моего дома? Чем это обосновано, что это такое вообще в принципе?
Вытяжка тянет в вентиляцию, а из дырок через щели тянет с улицы, от соседей, от куда получится. Ты вытяжкой через комнату, через проделанные дырки, через полости откуда-то тянешь воздух.
Аноним 04/12/23 Пнд 14:10:58 585664 432
Аноним 04/12/23 Пнд 14:58:41 585666 433
IMG202312030144[...].jpg 1212Кб, 4080x3072
4080x3072
Не могу повторить действия в нужном порядке написанные на бумажке. Что это может быть?
Аноним 04/12/23 Пнд 15:00:24 585667 434
>>585664
Всякое бывает:
> Turritopsis dohrnii (лат.) — вид биологически бессмертных медуз, обитающих во всём мире, в водах умеренного и тропического климата.
Аноним 04/12/23 Пнд 21:32:03 585680 435
>>585621
С той лишь одной малюсенькой разницей, что при первой же ошибке в полёте летающее существо убивается ап землю, тогда как плавание можно оттачивать абсолютно безопасно во всех стилях.
Аноним 04/12/23 Пнд 21:39:27 585681 436
Аноним 04/12/23 Пнд 22:18:03 585685 437
>>585678
Промывка тупой школоты. Бабы запилили себе уютненький матриархат с феминизмом. Баба с ребёнком это вообще неприкасаемое существо, у которого прав больше чем у тебя. И живут они дольше тебя лет на 10. Стоимость писечки вообще улетела в небеса. Сейчас баба может лутать доллары даже не вставая с дивана, ибо куколды всегда найдется.
Аноним 04/12/23 Пнд 23:03:27 585688 438
>>585678
Корреляция не значит каузация.
Если X коррелирует с Y, и Y коррелирует с Z, то это не означает что X и Z будут с друг другом коррелировать.
Аноним 05/12/23 Втр 06:06:16 585699 439
>>585661
Потоки воздуха из дырки это всос вытяжки? Что-то бредом отдает. Даже при закрытой двери на кухню тянет ведь.
Аноним 05/12/23 Втр 07:18:54 585700 440
>>585689
Да. И более того. Выявление корреляций не достаточно, нужно еще смотреть на распределение величин и смотреть как они отклонятся от статистической модели. И даже тут есть ненулевая вероятность случайного совпадения.
Аноним 05/12/23 Втр 08:24:40 585701 441
>>585688
>Если X коррелирует с Y, и Y коррелирует с Z, то это не означает что X и Z будут с друг другом коррелировать.
Статистически то скорее всего будет, надо просто проверить как ковариация XZ связана с cov(XY) и сov(YZ) (мне лень проверять). Но вполне себе связи какой-то причинно-следственной между ними вообще не окажется, это ты прав.
Аноним 05/12/23 Втр 10:50:24 585703 442
>>584346 (OP)
>Однако, помните, что /sci/ - собирательный раздел для всех наук, но если вы глубоко заинтересованы обсуждением политики - вам в /po/, математики - /math/, философии - /ph/, психологии и психиатрии - /psy/, медицины - /me/, космоса и астрономии - /spc/.
Праина понимаю, что тут скрыт панч, что история не наука?
Аноним 05/12/23 Втр 11:31:06 585705 443
>>585678
Как уже сказано выше, с этим в матх. Тут такое местным не по зубам.
О корреляции можно думать, как о косинусе угла между векторами (центрированных) случайных величин, поскольку ковариацией можно ввести скалярное произведение на векторном пространстве. Простой геометрический рисунок показывает, что, зная углы между X и Z, и между Y и Z, угол между X и Y однозначно определить нельзя. Если с. в. не центрированы, то смысл не меняется (т. н. косинусовое подобие). Однако - точно так же, как с геометрическими векторами - можно дать оценку наименьшему/наибольшему возможному значению корреляции (расчёт использует т. н. частичную корреляцию). Но пользы от этого чаще всего мало, поскольку границы скорее всего будут очень широкие.

Это про твой теоретический вопрос. Что касается статистики, то такое последовательное "продвижение" корреляции может иметь смысл, потому что коэффициенты корреляции связаны с коэффициентами линейной регрессии (многоступенчатая оценка) и более сложных моделей вроде структурных. Но это имеет смысл только при разумных моделях - корреляции в общем случае не "транзитивны".

>>585688
Про причинность анон вообще ничего не говорил. Вот уж не понимаешь, о чём речь - сиди себе тихо.

>>585689
Зависит от конкретного применения.

>>585700
Корреляция - это понятие из теорвера, а не из матстата. "Ненулевая вероятность случайного совпадения" это какой-то нерелевантный бред. Корреляция это просто характеристика распределения. Здесь настолько же уместно говорить о "случайности" "совпадения", как и "случайности" "совпадения" углов между произвольными парами векторов.
Аноним 05/12/23 Втр 19:05:58 585714 444
Аноним 05/12/23 Втр 19:27:32 585717 445
>>585685
Им же хуже, тупеют, не развиваются. А могли бы уже в квантовой физике нобелевки брать, если бы письками не зарабатывали и проблемы не решали.
Аноним 05/12/23 Втр 21:37:35 585726 446
Аноним 05/12/23 Втр 22:46:18 585727 447
16950341722310.jpg 34Кб, 300x300
300x300
Аноним 06/12/23 Срд 15:51:05 585764 448
Какова вероятность того что у двух голубоглазых родителей родится ребенок с темными глазами?
Аноним 06/12/23 Срд 18:22:38 585768 449
>>585764
Цвет глаз определяется в первую очередь экспрессией генов и не привязаны к конкретным генам.
Вообще говоря, голубой цвет глаз это нарушение нормальной покраски. Гены в не той последовательности включаются. А почему это они делают может завит от множества факторов, в том числе не наследуемых.
Если брать самый распространённый вариант голубых глаз, то за них ответственны мутации в пяти разных генах, и достаточно выбить их комбинацию, а не иметь гомозиготные аллели.
В том случае вероятность заиметь карие глаза у голубоглазых родителей в районе 11% процентов.
Аноним 06/12/23 Срд 21:33:51 585780 450
>>585717
Бабы прекрасно себя чувствуют, в отличие от тебя, инцела) Ну можешь показать свою нобелевку, посмотрим к чему привело твое "саморазвитие"
Аноним 07/12/23 Чтв 04:08:30 585795 451
1. Бывает ли "быстрый" и "медленный" метаболизм?
2. Можно ли "подстегнуть метаболизм" хотя бы в теории, например чтобы это было таблеткой из хард-сайфай.
Гормональные болезни, генетически аномалии, старость - это всё исключаем.
Аноним 07/12/23 Чтв 04:33:20 585796 452
>>585785
Рекомендую поставить свечу в ближайшем Храме Господнем и приобрести Святой Образ в дом. Всех благ, молодой человек.
Аноним 07/12/23 Чтв 06:54:41 585798 453
>>585795
>1. Бывает ли "быстрый" и "медленный" метаболизм?
Бывает, но отклонение от нормы всё равно небольшое (скажем, 10%). Так что если ты колобок, то это не от медленного метаболизма, нужно просто жрать меньше. И наоборот.
Аноним 07/12/23 Чтв 07:26:40 585799 454
>>585798
Не 10% а в разы, в разы буквально больше жрут и вообще не толстеют.
Аноним 07/12/23 Чтв 09:18:17 585803 455
>>585786
>>585785
Сходи к врачу, пока что не так всё плохо, но выглядит так, что может развиться в шизу
Аноним 07/12/23 Чтв 09:19:56 585804 456
>>585798
Если людям гормоны колоть, даже стройных можно колобками легко сделать. Просто, если у тебя поломанный метаболизм, то жрать меньше просто тяжело, т.к. организм постоянно будет тебя ебать адовым чувством голода
Аноним 07/12/23 Чтв 16:07:16 585812 457
>>585678
>это нормально?
Ты можешь сколько угодно получать кореляций. Количество пиратов корелируют с потеплением, например, что дальше?
>т.е. приходят к тому, что мужчины умнее женщин,
Шизофрения. Тут никто не понимает что такое ум и интеллект, а ты говоришь про то что кто-то умнее или не умнее. Может и умнее. Но доказать ты это не сможешь.
Аноним 07/12/23 Чтв 16:23:03 585813 458
Чем лазер отличается от мазера?
Аноним 07/12/23 Чтв 16:29:21 585814 459
>>584346 (OP)
В чём суть претензий к Савельеву и оснований для его статуса фрика в сообществе очкариков? Кому не сложно, введите в курс дела, если вы в смежной с ним сфере трудитесь и разбираетесь в её аспектах.
Аноним 07/12/23 Чтв 18:05:42 585817 460
>>585813
Лазер светит в оптическом диапазоне (условно еще светом называют ИК и УФ, что близкие к видимому диапазону)
Мазер светить в микроволновом диапазоне, у него длина волны значительно больше чем и света.
Аноним 07/12/23 Чтв 18:45:40 585818 461
>>585817
хм, то есть мазерами можно давить спутники Машка?
Аноним 07/12/23 Чтв 19:35:02 585819 462
>>585795
>Можно ли "подстегнуть метаболизм" хотя бы в теории
Чем пиндосы и знаимались, подсаживая на таблетки для похудения из амфетамина
Аноним 07/12/23 Чтв 23:46:12 585823 463
>>585818
Можно, то на таких длин волн сильна дифракция.
Аноним 08/12/23 Птн 00:54:29 585824 464
>>585819
Подобные таблетки влияют на поведение (подавляют аппетит), а не на метаболизм.
>>585798
>Бывает, но отклонение от нормы всё равно небольшое (скажем, 10%).
Давай уточним: есть два здоровых человека одинакового веса, оба ведут себя идентично, находится в идентичной среде, питаются одинаково, допустим избыточно. Через год у одного +10 кг, а у другого +11 кг? Как это возможно? За счёт чего?
Аноним 08/12/23 Птн 02:37:20 585825 465
1699240720866998.png 1624Кб, 1280x1280
1280x1280
1699241753044416.png 1745Кб, 1280x1280
1280x1280
1699241873587895.png 1667Кб, 1280x1280
1280x1280
1699241575571176.png 1846Кб, 1280x1280
1280x1280
>>584346 (OP)
Как называются явления, сам концепт которых является проявлением сложности. И не имеет никакого смысла при декомпозиции, подобно стулу, сколоченному из грубых досок - ни в одной из досок по отдельности невозможно найти стул, более того невозможно понять когда они все вместе становятся стулом (стул, который нельзя собрать, стул, который нельзя определить, стул на котором невозможно сидеть и так далее). Не зацикливайтесь на стуле, то же самое можно сказать про пару мозг->сознание.
Аноним 08/12/23 Птн 03:10:15 585826 466
>>585825
>ни в одной из досок по отдельности невозможно найти стул
На доске тоже можно сидеть. Все свойства стула происходят из свойств досок. С сознанием это не работает.
Аноним 08/12/23 Птн 07:55:04 585828 467
>>585799
Нет, это миф. Лично знал двух таких человек - худющие, и действительно вроде как жрут кучу всего и что попало. А потом пересчитываешь всё на калории - и оказывается, что они в сутки в среднем хорошо если 1600 набирают, а иногда и на 1200 сидят.
Аноним 08/12/23 Птн 09:43:33 585829 468
>>585825
> Как называются явления, сам концепт которых является проявлением сложности. И не имеет никакого смысла при декомпозиции, подобно стулу, сколоченному из грубых досок - ни в одной из досок по отдельности невозможно найти стул,
Хуй знает что ты имеешь ввиду, может https://ru.wikipedia.org/wiki/Эмерджентность это?

>ни в одной из досок по отдельности невозможно найти стул
Зато можно найти скамейку и диван.
Аноним 08/12/23 Птн 18:41:50 585839 469
image.png 1591Кб, 1600x800
1600x800
>>585826
Почему у грузовых кораблей кбина сзади, а не спереди? Так же хуего видно куда плывешь.
Аноним 08/12/23 Птн 18:50:45 585840 470
>>585610
>Охотник пошел в лес и случайно встретил добычу
Нет, конечно. Не может охотник случайно встретить добычу - это слишком невероятно. Очевидно, что кто-то дергает охотника и добычу за ниточки.
Аноним 08/12/23 Птн 19:12:34 585841 471
>>585592
Мутации не могут быть случайны, но эволюции это не отменяет.
Аноним 09/12/23 Суб 10:03:53 585855 472
>>585814
Далёк от этой темы, но слышал что он просто токсичное хуйло
Аноним 09/12/23 Суб 22:52:50 585880 473
>>585840
> кто-то дергает охотника и добычу за ниточки.
Детерминизм большого взрыва
Аноним 10/12/23 Вск 00:21:09 585882 474
Все знают, что происходит когда тело движется со скоростью звука и выше, например сверхзвуковой истребитель, пилот не будет слышать никаких звуков от источников позади себя, но что будет, когда тело будет двигаться со скоростью света? Представьте что вы летите со скоростью света головой вперёд опустим моменты с тем, что вас растянет и разорвёт и т.п., правда ли что вы не будете видеть даже свои ноги? Что будет позади? Просто чернота, застывшая картинка?
Аноним 10/12/23 Вск 01:40:00 585884 475
погасит ли взрыв одной атомной бомбы энергию взрыва другой атомной бомбы, если их моща равна? какое будет давление на месте встречи?
мне для рассказа надо, я гуманитарий
Аноним 10/12/23 Вск 04:52:37 585885 476
1. Существует ли научное объяснение самоубийств у людей? Не общие слова и статистические корреляции, а именно общепринятая и работающая теория, описывающая механизмы явления.
2. Если нет, то почему психиатры, психологи, социологи и прочие шарлатны кукарекают, как будто что-то понимают?
Аноним 10/12/23 Вск 07:55:42 585888 477
>>585882
>Все знают, что происходит когда тело движется со скоростью звука
Ну ты вот не знаешь.

>сверхзвуковой истребитель, пилот не будет слышать никаких звуков от источников позади себя
Он все слышет, потому что звук передается внутри самолёта, а пилот относительно самолёта неподвижен. Ты что, на Конкордах не летал, портдж?
Аноним 10/12/23 Вск 11:47:22 585896 478
>>585885
1. Есть только разные гипотезы. Наиболее правдоподобное это слишком сильный перехват контроля полушарий над лимбической системой, отчего та начинает сопротивляться и уводит мозг в разнос на химической уровне. И рано или поздно полушарии и лимбическая система находят радикальный вариант разрешения конфликта.
2. Они как прошаренные нейросетки успешно угадывают и на этом строят свои теории. На самом деле это нормальная практика в научной среде.
Аноним 10/12/23 Вск 13:08:25 585899 479
>>585896
>1.
Ну фактически, это тоже просто общие слова. Типа как в физике 17 века уже знали про вакуум, но еще не существовало молекулярно-кинетической теории, поэтому вместо реальных причин для объяснения проведение жидкостей и газов ввели принцип, что "природа не терпит пустоты", который на самом деле по сути ничего не объясняет, а просто выражает наблюдаемые явления.
Или как в средневековой медицине, не имеющей представления о реальных причинах заболеваний, болезни объяснили какими-то циркуляциями желчи, крови и слизи.
Аноним 10/12/23 Вск 16:18:22 585903 480
allinone.jpg 201Кб, 1360x768
1360x768
Здравствуйте 🙂 посоветуйте пожалуйста качественную научно популярную книгу (но не без формул) по теории информации? Клод Шеноно и вот это вот всё. Чтобы был срединный путь между сжатием информации для не специалиста и подробностями для профессионалов.
Аноним 10/12/23 Вск 19:48:38 585905 481
37.png 615Кб, 2399x1262
2399x1262
Если представить огромные лезвия должны превышать длину рычага во много раз, картинка не в масштабе раскрытые ножницы в космосе, которые начать "закрывать" , то можно логически предположить, что скорость смыкания лезвий этих ножниц ближе к концу превысит скорость света.
а) как это будет выглядеть для стороннего наблюдателя?
б) если между двумя лезвиями поместить какой-нибудь объект, который будет выталкиваться лезвиями вперёд, сможет ли этот объект превысить скорость света?
в) почему это невозможно на практике?

Если не совсем понятно, то можно представить "ножницы", чьи лезвия будут проходить расстояние от Земли до Луны, что равняется примерно 1 световой секунде с копейками. На Земле мы смыкаем эти "ножницы" менее чем за секунду, что в итоге приведёт к тому, что на Луне заметят смыкание ножниц даже быстрее, чем пока об этом дойдёт сообщение с Земли, или чем это можно было бы разглядеть через телескоп.
Аноним 10/12/23 Вск 20:18:54 585906 482
>>585839
Ты был в океане? Там кругом вода. Просто сплошная вода. Там не надо никуда смотреть.
Аноним 10/12/23 Вск 20:26:06 585907 483
>>585905
Сто раз уже было.
Ножницы не будут мгновенно закрываться, они будут изгибаться дугой, и вот эта волна изгибания будет идти медленней, чем скорость света.
Аноним 11/12/23 Пнд 09:22:51 585913 484
>>585907
>Сто раз уже было.
Ну, еп...
>они будут изгибаться дугой
А почему? В дело вступят какие-то фундаментальные законы, не позволяющие разогнаться выше скорости света?
Аноним 11/12/23 Пнд 11:04:58 585914 485
>>585785
Не парься, этот кот - постфактум рационализация элементов сонного паралича, который бывает чуть ли ни у каждого первого человека. Бывает моменты когды ты вроде еще спишь, но чуть чуть проснулся, переходное состояние, но область мозга, которая отвечает за блокировку тела (чтобы во сне не сильно навредил себе) еще работает, а пошевелиться тело не может из за этого у тела возникает страх, который выдумывает у тебя кота, у кого то чертей сидящих на груди, тут фантазия обусловленна культурой. Но по факту это нормальная физиология, небольшой сбой который бывает почти у всех.

Я раньше думал я ебанулся из за похожих симптомов, уже думал о всякой религиознутой хуяте, а потом, как то на лециии описали физиологию сонного параличи и спросили поднять руку тех, у кого был он - почти вся аудитория подняла.
Аноним 11/12/23 Пнд 11:05:53 585915 486
>>585914
UPDATE: ничего не делал, само прошло лол
Аноним 11/12/23 Пнд 11:54:26 585917 487
Y-OLHSdzSc.jpg 170Кб, 1115x1576
1115x1576
HIanon. Нужна помощь в виде капли кристально чистой логики и структурированной информации для решения проблемы:

Где мне искать girlfrend IRL?

Не буду писать конкретную инфу о себе, мне не советы нужны, а интересует в принципе научный подход к решению этой проблемы.
Аноним 11/12/23 Пнд 16:01:31 585919 488
113.webp 140Кб, 957x1500
957x1500
114.webp 152Кб, 957x1500
957x1500
115.webp 130Кб, 957x1500
957x1500
116.webp 115Кб, 957x1500
957x1500
Аноним 11/12/23 Пнд 19:45:10 585922 489
Аноним 12/12/23 Втр 06:56:24 585933 490
>>585913
>А почему? В дело вступят какие-то фундаментальные законы, не позволяющие разогнаться выше скорости света?
Отнюдь. У тебя ножницы не цельные, а состоят из молекул. Если представить себе лезвие просто из цепочки молекул, то смыкание будет выглядеть так: сдвигается одна молекула, потом благодаря электромагнитному взаимодействию между молекулами сдвигается вторая, и т.д.
Электромагнитное взаимодействие распространяется со скоростью не выше c, поэтому и сдвиг ножниц будет передаваться не быстрее скорости света. А в реальности значительно медленнее.
Аноним 12/12/23 Втр 08:39:34 585934 491
16850087163990 [...].jpg 804Кб, 1440x1440
1440x1440
>>584346 (OP)
Может ли быть у планеты больше двух магнитных полюсов?
Аноним 12/12/23 Втр 09:38:06 585935 492
>>585922
>В клубасике.

Ты видишь размер моей головы на фотке? Какой клубасик? Клубасик из одного человека им.Гриши Перельмана?
Аноним 12/12/23 Втр 11:31:35 585936 493
>>585934
Да.
Пример Уран. у него почти квадрупольное магнитное поле.
Аноним 12/12/23 Втр 11:51:24 585937 494
>>585936
А шо если в его ядро шо-то въебётся, и квадруполь рассыпется в диполь, какие будут изменения по всей планете?
Аноним 12/12/23 Втр 12:39:06 585938 495
>>584346 (OP)
Чем отличается лоббизм от коррупции? Для самых маленьких, если можно.
Аноним 12/12/23 Втр 12:47:35 585939 496
>>585936
чем-то кол-во полюсов ограничено? обязательно должно быть четное количество
за тупые вопросы не бей. лучше обоссы - я гуманитарий
Аноним 12/12/23 Втр 13:56:52 585944 497
>>585938
Государство строиться из различных институтов.
Институты это набор устойчивых практик, которые не должны зависеть от людей, которые эти практики осуществляют. Коррупция это порча институтов или когда осуществление практики начинает зависеть от персоналий. Поскольку институты в государстве взаимосвязаны, то порча одного института автоматом портит другие.

Лоббизм это сам по себе институт, он позволяет некоторой группе продвигать свои интересы и консолидировать с другими в определенном поле, не шатая при этом другие институты.
Аноним 12/12/23 Втр 16:17:04 585946 498
>>585938
Тем, что лоббисты обязаны действовать открыто. В теории, действующего во вред избирателям политика снимут и не переизберут. Но на практике это, конечно, не работает.
Ну и работают лоббисты не только через взятки но и через пропаганду.
Аноним 12/12/23 Втр 20:00:45 585953 499
>>585935
Тогда на вписочке.
Аноним 12/12/23 Втр 22:50:00 585958 500
Почему
ЧЕЛОЧЕЧЕСТВО 20 ТЫСЯЧ ЛЕТ ОХОТИЛИСЬ
@
7 ТЫСЯЧ ЛЕТ В ЗЕМЛЕ КОПАЛОСЬ
@
ТОЛЬКО СЕЙЧАС НАЧАЛСЯ ГЕЙМПЛЕЙ А НЕ ЖИВОТНФЙ УРОВЕНЬ ЖИЗНИ
Аноним 12/12/23 Втр 23:46:23 585959 501
>>585958
В Северной Америке всё ещё охотятся, а в бСССР всё ещё копаются, если ты не в курсе.
Аноним 13/12/23 Срд 08:35:19 585969 502
как вообще началось развитие интеллекта у людей? типа может наши предки были слабыми да удалыми и решили пойти в командную работу и это дало толчок?
Аноним 13/12/23 Срд 09:19:43 585971 503
907b71267ba5c61[...].jpg 262Кб, 1080x810
1080x810
Аноним 13/12/23 Срд 09:44:14 585972 504
>>585971
Гриб сказал ему отправиться в путешествие где он и обрел способность адаптировался к любой среде, сверхгибкий разум.
Аноним 13/12/23 Срд 10:31:35 585973 505
>>585969
Жили были древесные обезьяны(Проконсулы), они как мартышки были ловкими и подвижными, что давало некоторый буст к общему интеллекту.
Однако в один момент леса начали редеть и превращаться в саванну. Бывшим древолазам пришлось менять рацион с плодов на мясную пищу, которую они крали у хищников. Переход на мясную пищу позволил им уменьшить челюстной аппарат и отодвинуть предел роста мозга.
Воровство еды у хищников довольно сложная задача, которая решалась коллективно. Каждому обезьяну требовалось прогнозировать поведение хищника и своих товарищей. Получался специфический групповой отбор, где выживали только самые сплоченные и большие группы.
Эпоха сменялась, саванны отступили и появились снова леса. Пришлось переходить на всеядный образ жизни, да еще обманывать новых хищников(леопардов). Тут опять усилился отбор на социализацию.
Аноним 13/12/23 Срд 10:41:51 585974 506
>>585969
Сначала нужны были условия для роста мозга и развития интеллекта - это прямохождение, что позволило расти голове без нагрузки на шею и освободило руки для орудийной деятельности (оперирование объектами необходимо для интеллекта). Дальше просто эволюционный отбор, в первую очередь внутривидовой - доминирование и выпиливание себе подобных, т.к. сверххищником человек стал очень быстро и давно. Но есть версия о скачкообразном росте количества нейронов из-за мутации гена и такого же отбора.
Если не рассматривать версию с мутацией гена, то какого-то начала или скачка нет. Почти вся история живого это эволюционное развитие нервной системы (=интеллекта).
Аноним 13/12/23 Срд 10:46:36 585975 507
4e49f369710981f[...].jpg 8402Кб, 6566x8215
6566x8215
>>585972
>Гриб сказал ему отправиться в путешествие

Грибы были в ахуе от того, где им приходится жить. Они посовещались (все 109 гендеров) и решили развить интеллект обезьян, чтобы создать цивилизацию, построить космические корабли и съебаться нахуй с этой конченной планеты.
Аноним 13/12/23 Срд 11:29:14 585976 508
>>585824
Метаболизм даже у одного человека может меняться ОЧЕНЬ сильно. Несколько лет ежедневных занятий спортом и уже совсем другой метаболизм.
Аноним 13/12/23 Срд 11:34:24 585977 509
>>585969
>как вообще началось развитие интеллекта у людей?
развитие интеллекта началось до людей
Аноним 13/12/23 Срд 11:52:05 585979 510
>>585976
>Метаболизм даже у одного человека может меняться ОЧЕНЬ сильно. Несколько лет ежедневных занятий спортом и уже совсем другой метаболизм.
Нет. Когда ты занимаешься спортом, то и ешь больше, потому что тратишь больше энергии. Метаболизм меняется незначительно.
Аноним 13/12/23 Срд 12:05:16 585980 511
>>585979
Сразу видно человека, который не занимается спортом.

Для остальных: гуглите про количество и КПД митохондрий и как эти показатели менятся в зависимости от типа тренировок. Например долгий бег с постоянной скоростью увеличивает количество митохондрий, а спринт увеличивает их эффективрность. Поэтому та же пробежка должна следовать правилу 80/20. Например 8 км бежишь в постоянном темпе, на увеличение количества митохондрий и 2 км с максимальной скоростью, на увеличение КПД митохондрий.

Например первое что выдал гугл:
https://openlongevity.org/mitochondria_medicine_2#:~:text=%D0%A4%D0%B8%D0%B7%D0%B8%D1%87%D0%B5%D1%81%D0%BA%D0%B0%D1%8F%20%D0%B0%D0%BA%D1%82%D0%B8%D0%B2%D0%BD%D0%BE%D1%81%D1%82%D1%8C%20%D0%B4%D0%BE%20%D1%81%D0%B8%D1%85%20%D0%BF%D0%BE%D1%80,%D0%BE%D0%B1%D1%8A%D0%B5%D0%BC%D0%BD%D1%83%D1%8E%20%D0%BF%D0%BB%D0%BE%D1%82%D0%BD%D0%BE%D1%81%D1%82%D1%8C%20%E2%80%94%20%D0%BD%D0%B0%2040%25.
Аноним 13/12/23 Срд 12:13:59 585981 512
45756785688.jpg 213Кб, 571x914
571x914
>>585958
Потому что ты долбоеб
Аноним 13/12/23 Срд 12:17:48 585982 513
423543535.png 69Кб, 713x409
713x409
>>585840
Очевидно что охотник применяет свои навыки для выслеживания добычи, а не прет в лес надеясь на удачу. Это только атеизнутые материалисты верят в удачу
Аноним 14/12/23 Чтв 01:35:15 585992 514
image.png 233Кб, 908x801
908x801
>>585919
так, блэт, требую пояснения
Аноним 14/12/23 Чтв 07:09:05 585998 515
Аноним 14/12/23 Чтв 07:38:52 585999 516
Аноним 14/12/23 Чтв 09:32:35 586000 517
chromemOmR6kt0zL.jpg 53Кб, 648x641
648x641
>>585999
Thanks!

Залип и влюбился со второй страницы:
<-
Аноним 14/12/23 Чтв 10:38:02 586001 518
На Земле могут закончиться металлы? Когда?
Аноним 14/12/23 Чтв 11:18:16 586002 519
chromeNWkFvz3wOf.jpg 426Кб, 2498x1353
2498x1353
>>586001
Кратко: да. Для разных металлов по разному.

Подробно: конечно металлы не покидают эту планету. Но это как с электрическим обогревателем - формально энергия электричества никуда не исчезла, она перешла в тепло, но по факту использать это тепло продуктивно проблематично.

С металлами, на примере меди: столетие назад разрабатывали месторожения около 15% меди в руде. Эти месторождения уже закончились, сегодня разрабатывают уже месторожения с парой процентво меди в руде, даже меньше. Но по мере уменьшения содержания полезного вещества в руде количество затрат и объемы руды, которую нужно переработать, растет почти по экспоненте. Да технологии совершенствуются, но тонна грунта, это всё еще тонна, и количество энергии на её перемещение не меняется. Поэтому на примере меди, довольно скоро (может быть меньше сотни лет) наступит момент, когда по мере введения в разработку всё более бедных месторожений, затраты (в том числе не прямые, типа экологии) превысят стоимость меди. Опять же, нельзя всё сводить только к деньгам.

В общем мысль понятно надеюсь.
Аноним 14/12/23 Чтв 11:51:10 586006 520
gynueb8nt4oy-39[...].png 789Кб, 1280x533
1280x533
>>586001
Важный момент, что даже бесконечный источник энергии не решит эту проблему, потому, что даже при бесконечной энергии будет накапливаться загрязнение и разрушение окружающей среды от производства и добычи, со временем превысив допустимый предел. И это относится не только к металлам.

Предел этот конечно разный для выживания миллиардов и миллионов людей. Первыми под нож пойдут достижения благополучной жизни - демократия и права человека. Но быдлу конечно дадут возможность восстановить справедливость, и почувствовать себя героем - вымещать бессильную злобу в игрушках виртуальной реальности, стреляя по elysium bugatti. LOL
¯\_(ツ)_/¯
Аноним 14/12/23 Чтв 12:48:08 586007 521
Аноны кто разбирается в юриспруденции объясните почему так

>УК РФ Статья 59. Смертная казнь

1. Смертная казнь как исключительная мера наказания может быть установлена только за особо тяжкие преступления, посягающие на жизнь.

2. Смертная казнь не назначается женщинам, а также лицам, совершившим преступления в возрасте до восемнадцати лет, и мужчинам, достигшим к моменту вынесения судом приговора шестидесятипятилетнего возраста.

>Статья 19

1. Все равны перед законом и судом.

2. Государство гарантирует равенство прав и свобод человека и гражданина независимо от пола, расы, национальности, языка, происхождения, имущественного и должностного положения, места жительства, отношения к религии, убеждений, принадлежности к общественным объединениям, а также других обстоятельств.
Аноним 14/12/23 Чтв 13:04:18 586008 522
>>586006
Имея
>бесконечный источник энергии
можно будет фильтровать что угодно на уровне атом вправо - атом влево.

Сейчас Машк понаделает ракет и можно будет металлы притягивать прямо из космоса. У каждого бомжа будет палладиевый унитаз.

Плюс вот-вот напридумывают всяких хитрых сплавов или еще каких материалов с самыми невероятными свойствами. Я уверен в светлом будущем.

Но быдло действительно лучше бы подсократить, ну до миллиарда хотя бы, чтобы дышалось полегче, толку от него все равно никакого.
Аноним 14/12/23 Чтв 13:08:27 586009 523
(JPEG Image, 18[...].jpg 6Кб, 187x270
187x270
>>586007
Все животные равны, но некоторые животные равнее других.

Это же писечки - ценный товар для кого надо, пынемать надо.
Аноним 14/12/23 Чтв 19:29:55 586017 524
Аноним 14/12/23 Чтв 20:30:14 586018 525
Почему время в любом девайсе, не подключенном к интернету и, соответственно, не получаещем апдейт с "правильным" временем всегда начинает спешить? Поискал в интернете но нашел только применимое к обычным кварцевым часам мням шмям на кварцы влияют внешние факторы. Какое время вообще настоящее? Или я просто шиз, если так то извиняюсь.
Аноним 15/12/23 Птн 09:22:27 586030 526
>>586018
> всегда начинает спешить?
Точные атомные часы штука громоздка и дорогая, все обычные часы идут с приличной, разной степени погрешностью. Если это не твоя шиза, про то что все часы спешат (я такой херней не занимался и первый раз об этом слышу), то можно только предположить, что учитывая априорную погрешность часов, их подкручивают на опережение что бы люди не опаздывали.

> Какое время вообще настоящее?
Как люди договорятся, то время и будет настоящим. Гугли: время по Гринвичу.
Аноним 15/12/23 Птн 12:07:48 586031 527
>>586017
>Сказки о кисейных берегах
>Рашка
Проиграно. Нет не дает. Но (в теории) из сверхпроводника можно сделать хранитель энергии.

>>586030
>Если это не твоя шиза, про то что все часы спешат
Нда... поколение пориджей в жизни не видевшие часов уже подросло и всюду лезет.
Аноним 15/12/23 Птн 13:21:18 586033 528
>>586031
> поколение пориджей в жизни не видевшие часов
Чел, я просто подвожу часы по необходимости, а отстали они или нет мне вообще не интересно, мне просто надо чтобы часы показывали правильное время.
Это у тебя шиза обострилась, во всякой ерунде связи искать, таблетки выпей
> Письмо на Балабановскую спичечную фабрику: «Я 11 лет считаю спички у вас в коробках - их то 59, то 60, а иногда и 58. Вы там ебанутые что ли все???»
Аноним 15/12/23 Птн 13:46:34 586035 529
>>586033
Поридж ты вообще с этой планеты? Кто то выпустил жпт? Охуительные истории часы он не глядя подводит, какой то бред про спички.
Аноним 15/12/23 Птн 14:02:08 586036 530
IMG202308260822[...].jpg 799Кб, 3264x2448
3264x2448
>>586033
этот прав

>>586035
>>586031
этот из:
> Письмо на Балабановскую спичечную фабрику: «Я 11 лет считаю спички у вас в коробках - их то 59, то 60, а иногда и 58. Вы там ебанутые что ли все???»
_
За полвека жизни, даже до появления телефонов, ни разу не обращал внимания спешат часы или отстают когда я их подвожу. Это же пиздец какой жалкой жизнью нужно жить, чтобы тратить время на такую хуету как в какую сторону часы сбиваются.
Аноним 15/12/23 Птн 14:14:08 586037 531
>>586036
Я думал речь идет про рассинхрон вообще. Снимаю тогда возражения.
Аноним 15/12/23 Птн 14:21:17 586038 532
>>586037
И кстати я тоже могу быть неправ, возможно нужно глубже копать. Что если все часы действительно сбиваются в одну сторону? инкто не обращал внимания, а ты обратил. Извини что обосрал. Что это? Тайный заговор мирового правительства мальчиков копателей из литиевых шахт в Африке или новый, неизвестный закон физики расширяющий общую теорию относительности?
Аноним 15/12/23 Птн 15:14:52 586039 533
>>586038
Я оп вопроса, имел в виду именно часы в девайсах - телефоны,планшеты. Например, у меня есть старый айпад, который я не подключал к интернету год и за это время он стал спешить ровно на 9 минут, при этом ни разу не выключался (стоит на зарядке как фоторамка). Механические часы понятно почему могут спешить или оставать, но почему компьютер отсчитывает неправильно, при условии что сам он работает без нареканий. Варианта два: или проблема в самой технике, или официальное время постоянно отводят немного назад, но второе это уже шиза, лол.
Аноним 15/12/23 Птн 15:54:23 586040 534
>>586031
>Нет не дает
Почему? Насколько я понимаю, чем больше электронов можно прогнать по проводу с учетом физ. свойств, тем мощнее будет электродвигатель. Или это не так?
>хранитель энергии
Какие там теоретические пределы емкости?
Аноним 15/12/23 Птн 16:43:00 586041 535
>>586040
У современных электродвигателей потери на механическое трение больше, чем омические потери в обмотках. Так же есть значительные потери при перемагничивание магнитопровода. Чтоб сделать йоба двигатели, нужно делать йоба подшипники и йоба железо.

Все упирается в размеры и материалов которые достаточно прочны. Чем больше контур тем лучше. Можно хоть вокруг планеты обмотать или на орбите развернуть и получить батарейку на десятые порядки джоулей.
Аноним 15/12/23 Птн 17:24:56 586043 536
>>586039
>официальное время постоянно отводят немного назад
Так и есть, чтобы никто не замечал неправильного движения солнца из-за того, что земля на самом деле не шар.
Аноним 15/12/23 Птн 18:06:42 586047 537
Почему никто ещё не додумался испытать на практике теоритическую модель автономного марсианского поселения, но не на Марсе, а в Антарктиде? Ведь Антарктида самое приближенное к условиям Марса место на Земле.
Аноним 15/12/23 Птн 18:28:19 586048 538
>>586047
Потому что Марс нинужная хуйня. Весь хайп был для того, чтобы ты на фильм сходил.
Аноним 15/12/23 Птн 19:04:55 586050 539
>>586048
А ведь анону который спросил - ему нужен Марс, так что получается что это ТЫ - никому не нужная хуйня.
Аноним 15/12/23 Птн 19:47:50 586052 540
>>586050
>шизоид спросил шизоидный вопрос
>ряяя вы фсе нинужные яскозал
понятно
Аноним 16/12/23 Суб 01:42:09 586056 541
>>586040
>Какие там теоретические пределы емкости?
На данный момент
https://en.wikipedia.org/wiki/Superconducting_magnetic_energy_storage
1–10 W·h/kg
Что не густо мягко говоря, на пару порядков меньше чем у аккумов, у которых в свою очередь на пару порядков меньше топлива.

Но в теории емкость ограничена критическим полем, которое ограничено.. неизвестно чем, потому что нет удовлетворительной теории высокотемпературных сверхпроводников. Так что i want to believe.
Аноним 16/12/23 Суб 14:10:48 586065 542
>>584346 (OP)
Кто-нибудь, скажите:
Современная физика позволяет предсказать свойства вещества по его химическому составу, такие как электропроводность или теплоемкость? Как это работает?
Аноним 16/12/23 Суб 22:17:00 586077 543
>>586065
Нет, не позволяет. не хватает вычислительной мощи. И этим не занимается физика.
Электропроводность по сути эмерджентное явление, и нет до сих пор внятной модели, только экспериментальные данные.
Аноним 16/12/23 Суб 22:44:59 586078 544
>>586077
>Электропроводность по сути эмерджентное явление
Что эмерджентного ты нашёл в движении электронов? Нет тут эмерджентности.
Аноним 16/12/23 Суб 23:01:36 586079 545
>>586078
Если брать металлы или полупроводники, то в них сами электроны никуда не двигаются и сычуют рядом с ядрами или в узлах решетки. Однако электроны могут участвовать в формирование обобщенной коллективной связи, которая размазана на весь кристалл. Вот у этой связи уже возникают эмерджентные свойства, которые не могут быть у электронов, состоящие их. Как пример перенос заряд через дырки или появление частиц, переносящие только спин.
Аноним 16/12/23 Суб 23:10:03 586080 546
Аноним 16/12/23 Суб 23:14:31 586081 547
>>586077
Чел, мне ноутбука хватит с мобильным чипом, чтобы посчитать теплоемкость какого-нибудь простого кристалла с +- 10% точностью, а если речь про газы, там вообще за 20 минут точность 0.1% можно добиться

Ну мы же не в 1995 году
Аноним 16/12/23 Суб 23:21:33 586082 548
>>586081
Давай считай параметры решетки железа исходя только из атомного веса и номера.
Аноним 16/12/23 Суб 23:26:49 586083 549
>>586082
Хватит только номера
Аноним 17/12/23 Вск 11:31:05 586089 550
>>586083
Ну как считается?
У железе минимум четыре кристаллическое решетки, а еще есть куча метастабильных.
Аноним 17/12/23 Вск 21:07:02 586098 551
>>586089
Лол, оно самую стабильную будет брать, чтобы найти всякие метастабильные или иные уже придется подрубать кластеры для расчетов, но опять же, банально взял 3,5,9 атомов, посмотрел какие там получаются расстояния и экстраполировал, получилось типа 2,85A

И это я брал самые простые и тупые методы, если захочется, можно заморочиться со спиновыми состояниями и взять методы посерьезнее, и там результат будет еще ближе к реальности

Ответ на вопрос:
>Современная физика позволяет предсказать свойства вещества по его химическому составу
Звучит как "зависит от сложности строения вещества, в простых случаях позволяет, в тяжелых нужно много вычислительной мощности, порой чересчур много"

А не как ты написал:
>Нет, не позволяет. не хватает вычислительной мощи.

Ну это же пиздеж, какую-нибудь NaCl и её теплоемкость считать можно в домашних условиях, c электропроводностью сложнее, но если структуру зон посчитать, то и электропроводность можно неплохо оценить. Если речь не про сложные системы со сверхпроводимостью или какие-нибудь суперзамудренные спиновые системы.
Аноним 17/12/23 Вск 23:19:59 586103 552
>>586079
> Если брать металлы или полупроводники, то в них сами электроны никуда не двигаются и сычуют рядом с ядрами или в узлах решетки.
А что делать со свободным электронами, они же электроны проводимости? Которые как раз и прыгают между атомами?
Аноним 18/12/23 Пнд 14:42:51 586110 553
>>584346 (OP)
Интересуют обзорные книги по природе и сути власти, подчинения, авторитетов, идолов, кумиров, рабства. В первую очередь антропология, социология, а потом уже всё остальное.
Делитесь тем, что считаете достойным внимания, что нельзя обойти стороной.
Аноним 18/12/23 Пнд 16:49:36 586111 554
>>586103
Электроны проводимости это квазичастицы, которые порождаются коллективной связью, и они не являются электронами сами по себе, тем более свободными.
Они не могут быть привязаны к конкретному атому или узлу решетки, они даже не "чувствуют" их, вместо этого они сычуют в разрешенной зоне, которую не могут покинуть и рассеваются на других квазичастицах, которые являются такими же порождением коллективной связи всех атомов в куске материала.
Аноним 20/12/23 Срд 05:45:12 586143 555
1. В СССР реально продавали в аптеках дерьмо летучих мышей (мумиё)? Это был дефицит?
2. Почему при первой же среднеазиатских "академиков" ввести это в официальную медицинскую практику им не постучали хуем по головам из Москвы? И АН и вся советская медицина это централизованная структура. В чём проблема? Брежнев любил обмазывался говном, и безопасности ради решили перейти с человеческого на летучемышиное? Как это объяснить?
Аноним 20/12/23 Срд 12:07:40 586146 556
Screenshot.png 660Кб, 1080x2400
1080x2400
>>586143
>В СССР реально продавали в аптеках дерьмо летучих мышей (мумиё)
Поридж, спок.
Аноним 20/12/23 Срд 20:45:33 586149 557
image.png 145Кб, 930x760
930x760
image.png 442Кб, 825x900
825x900
Аноним 20/12/23 Срд 20:51:16 586150 558
>>586149
Фонд, ассоциация, журнал … причём здесь наука? Знакомое слово в названии увидел?
Аноним 20/12/23 Срд 21:06:37 586151 559
>>586143
1. Мумиё (также известное как мумие) действительно продавалось в аптеках СССР. Это считалось естественным средством, имеющим различные преимущества для здоровья. Дефицита мумиё в аптеках не было.

2. Внедрение мумиё в официальную медицинскую практику в СССР, вероятно, было обусловлено несколькими факторами. Академики Центральной Азии, возможно, провели исследования и обнаружили потенциальные лечебные свойства мумиё. Кроме того, Академия наук и советская медицина не были полностью централизованными структурами, и могла существовать определенная степень автономии в принятии решений. Личные предпочтения политических лидеров, таких как Брежнев, обычно не являются единственной основой медицинской практики. Решение использовать мумиё, вероятно, было связано с научной оценкой и рассмотрением его потенциальных преимуществ.
Аноним 20/12/23 Срд 21:18:02 586152 560
>>586150
> Журнал аккредитован в Российском Индексе Научного Цитирования (РИНЦ).
В общем, в российской науке Рептилоиды теперь база.
Аноним 21/12/23 Чтв 06:09:02 586162 561
>>586149
А там написано как реаптилойдов отпиздить или это за гранью науки? Если так то я тогда тогда в религию.
Аноним 22/12/23 Птн 09:33:42 586181 562
Безымянный.jpg 76Кб, 564x564
564x564
Смотрел сейчас гуглкарты и не понял момент. Есть город Волгоград, есть волгоградский водоканал и его очистные сооружения.
Я так понимаю там очищают канализационную воду со всего Волгограда?
Но эти очистные сооружения находятся на острове в 2,5 км от Волгограда. Как туда доставляют канализационные стоки то? Их же не перевозят на танкерах, они что под Волгой на дне проложили гигантский говнопровод? И качают говно говняными насосами 24/7чтобы потом слить в ту же реку, но это же неэффективно как-то, нет?
Или водоканал там что-то другое очищает, типо питьевую воду для Волгограда?
Аноним 22/12/23 Птн 12:50:03 586184 563
>>586181
Не всю.
Вообще-то в каждом районе должна быть маленькая говночистка в виде нескольких больших подземных емкостей, туда сливают говно со всей округе, оно отстаивается и идет дальше по говно проводу к очистным сооружениям, там воду приводят в нормальный вид и отправляют в водохранилища, из которых уже забирают воду для водопровода.
Аноним 22/12/23 Птн 13:24:40 586185 564
>>586184
А спасибо, теперь понятно, там вода, просто дурно пахнущая. Я уж думал если прорвет на дне будет пиздец, всю Волгу засрут буквально.
Аноним 22/12/23 Птн 13:39:59 586187 565
>>586185
Очистительные сооружение это не глубокие пруды, главное тут площадь, а не объем.
Там нечему особо прорывается, чтоб хлынуло потоком. Хотя конечно есть вероятность протечки и заражения вод, но Волга и так не очень чистая, особой разницы не заметят.
Аноним 23/12/23 Суб 11:02:08 586200 566
homo-erectus-e1[...].png 495Кб, 873x720
873x720
c0139577-800px-[...].jpg 182Кб, 800x800
800x800
xk0rejzxth041.jpg 104Кб, 749x741
749x741
csmecht01cfdb72[...].jpg 94Кб, 900x900
900x900
>>584346 (OP)

У меня ПЛАТИНОВЫЙ вопрос - который наверное сюда.

А почему все Хомо вымерли? Логичная теория есть только про неандертальцев, которые фактически экстремофилы на пределе возможностей по жизни и рождению, чуток климат провернулся - чуток африканцы с копьеметалками пришли, и все.

Но почему работящий-прямоходящий и прочие вымерли? У них ареал остается. Их эволюционное преимущество над большими человекообразными обезьянами очевидно. Почему шимпанзе-орангутаны сегодня есть, а няши с пиков вымерли??? Как так?
Аноним 23/12/23 Суб 11:57:03 586203 567
>>586200
Потому что древние и современные хомо в одной экологической нише и, соответственно, вытесняли-выживали друг друга, а обезьяны остались на деревьях - в другой экологической нише. Шипманзе-орангутанги, кстати, тоже эволюционировали, и так же вытесняли своих предков. У нас с обезьянами один предок, от которого разошлись по экологическим нишам и стали разными, а сами предки, что людей, что обезьян - умерли.
Аноним 23/12/23 Суб 13:17:54 586205 568
>>586203

То есть на пустой Земле не было места для пары-тройки сверхприспосбленных видов?

Ну вот не верю, что прямоходящая обезьяна с зазубренной палкой и отщепом, которая может есть травку, фрукты, корешки, мясо, рыбу - не смогли выжить. Они уж точно лучше приспособлены чем обезьяны и хищники.
Аноним 23/12/23 Суб 15:14:15 586207 569
>>586205
> Ну вот не верю
Правильно, не верь, человека создал Бог, по образу и подобию своему.
Ебанько, зачем ты задаешь вопрос в науке, но не веришь научным объяснениям? Иди в раздел религии или эзотерики и там спрашивай, видимо тебе нужно магическое объяснение появления человека
Аноним 23/12/23 Суб 15:49:17 586208 570
>>586207

Всем известно, что рептилоида изнасиловал анунак = и так появились люди.

Я вполне научно задался вопросом:
- Есть вид, который всеяден, умеет в базовые орудия труда и стадность.
- Есть виды - которые не умеют всего этого, и тоже обитают в довольно узких нишах.
- Земля (африка) вполне достаточна для обитания ряда видов в одной нише, особенно столь продвинутых.

Условно говоря многие виды тех же хишников разделялись и эволюционировали а некоторые вообще крокодилы и им похуй на эволюцию параллельно и сейчас есть несколько видов в одних нишах.
Аноним 23/12/23 Суб 16:01:50 586209 571
>>586208
> - Земля (африка) вполне достаточна для обитания ряда видов в одной нише, особенно столь продвинутых.
В одной экологической нише не могут сосуществовать длительное время два одинаковых вида, более сильный вид вытеснит более слабого, слабый либо вымирает, либо уходит в другую экологическую нишу.
Аноним 24/12/23 Вск 11:40:18 586225 572
острие науки

Есть что-нибудь по самым передовым достижениям уровня фантастики? Типа: кристаллы времени, квантовые жидкости, конденсаты, компьютеры?
Аноним 24/12/23 Вск 23:48:38 586238 573
image.png 626Кб, 861x620
861x620
Почему бегемоты считаются парнокопытными если у них по 4 пальца на ноге?
Аноним 25/12/23 Пнд 21:11:28 586252 574
>>586225
Какие научные прорывы, с 1960-х ничего нового не изобретено.

Ученые-дармоеды в XXI веке не могут сделать ни телепорта, ни кошкодевочек. Только попиливают гранты и борются с конкурентами за свои теплые местечки ("Ученые против мифов", Комиссия по борьбе с лженаукой и прочая). Сейчас наука превратилась в средневековую церковь с догматами, инквизицией и лицемерными попами-педофилами.

Вот раньше, когда Королев в шарашке когда отлынивал, так получал пиздюлей от Лаврения Палыча, в итоге и запустил ракету в космос (что раньше считалось фантастикой, совершенно невозможным). Этих грантоедов бы в шарашки, у нас бы сейчас появился не только телепорт, но и звездолет до Альфы Центавра.
Аноним 26/12/23 Втр 20:20:27 586263 575
Почему поиск повторов в ДНК это важная задача?
Аноним 26/12/23 Втр 20:25:52 586264 576
>>586263
Вдруг боженька оставил комментарии в ДНК в стиле "КТО ПРОЧИТАЛ ТОТ ПИДАР"?
Аноним 26/12/23 Втр 21:28:27 586265 577
В фильме Оппенгеймер бытовала мысль, что во время испытания атомной бомбы может возникнуть цепная реакция и воспламенение атмосферы впоследствии, Энрико Ферми шутил якобы на эту тему. Как это вообще устроено и возможно ли? Или всё же режиссёр просто вешал лапшу для пущего эффекта и впечатления зрителя?
Аноним 27/12/23 Срд 02:09:52 586274 578
>>586263
>поиск повторов в ДНК
А если тебе текст дать на незнакомом языке без пробелов и расшифровывать заставить, ты это как делать будешь?
Аноним 27/12/23 Срд 07:03:56 586276 579
Почему ртуть запретили?
Аноним 27/12/23 Срд 10:36:06 586283 580
>>586276
Никто ничего не запрещал.
Нашли менее опасные для жизни аналоги.
Там где не нашли продолжают использовать.
Аноним 27/12/23 Срд 18:57:41 586293 581
>>586265
Собравшимся, впрочем, они говорили еще кое о чем, на что Теллер сам быстро указал. Ученые смотрели на каракули на доске, ошарашенные догадкой. При такой высокой температуре, более высокой, чем в центре Солнца, должны сливаться не только атомы водорода. При ней кулоновский барьер должны преодолевать не только атомы водорода в воде, но и атомы азота в воздухе. Она должна практически мгновенно зажечь водород в океанах и превратить в пламя воздух вокруг земного шара. Земля должна вспыхнуть меньше чем за секунду и навечно превратиться в безжизненную глыбу.

Никто из ученых, собравшихся в Беркли, не сомневался в теоретической осуществимости ядерного взрыва. Проблемы, которые, возможно, и не удастся преодолеть вовремя для практического использования во Второй мировой войне, были чисто техническими: например, никто не знал, можно ли удержать массу достаточно долго, чтобы цепная реакция привела к полномасштабному взрыву. Теперь же оказалось, что технические сложности создания бомбы – не главная проблема. Их преодоление могло обернуться не самой хорошей идеей.

Они начали проверять расчеты Теллера. Ошибку нашли довольно быстро. Он упустил критически важный фактор: передачу тепла атмосфере. Однако внесенные исправления не устранили возможности возникновения реакции, вызвавшей опасения.

Среди присутствовавших на презентации находился Ханс Бете, самый проницательный теоретик в группе, который впоследствии получил Нобелевскую премию за работу по термоядерным реакциям на Солнце. Опираясь на интуицию, он считал, что такой результат «невозможен».
https://military.wikireading.ru/hx6y14a27W
Аноним 27/12/23 Срд 20:17:34 586295 582
>>586274
Можно и без аналогий. То есть есть надежда, что отдельные последовательности отвечают за какие то конкретные свойства?
Аноним 27/12/23 Срд 21:40:37 586296 583
>>586265
Так или иначе, видимо, единственным итогом существоваших обсуждений является отчёт 1946 года «Ignition of the Atmosphere with Nuclear Bombs» за авторством Конопинского, Марвина и Теллера, в котором приводятся оценки с выводом о том, что как минимум тогдашних энергий для «поджига» атмосферы недостаточно, но что в районе температур от десятков то сотен мегаэлектронвольт (1011...1012 К) «зазор до поджига» становится небольшим и может быть перекрыт неточностями оценок.
https://certus.livejournal.com/54517.html
Аноним 29/12/23 Птн 01:34:02 586305 584
>>584346 (OP)
Почему существуют фиолетовые деревья с фотосинтезом? (Вот прямо в парке краснодара растут) Если такие есть, то зеленый цвет не преграда и можно запилить фотосинтез телесного цвета?
и ЭТО ПОДВОДИТ К МОЕМУ ВТОРОМУ ВОПРОСУ: Алла Пугачева и Мадонна питаются фотосинтезом?
Аноним 29/12/23 Птн 01:37:58 586306 585
Как поле Хиггса создаст новую вселенную? Протоны и нейтроны распадутся, кварки будут превращены в излучение чёрными дырами и разлетятся в пустоту.
Получается материи не останется? Так как "спонтанное нарушение электрослабого взаимодействия" создаст что-то из ничего?
Аноним 29/12/23 Птн 12:51:09 586309 586
>>586305
Фиолетовый или пурпурный цвет возникает из-за большего количества пигментов, которые перекрывает зеленый хлорофилл.
Эти пигменты производные карантинов и служат этакими аккумуляторами "электронов".
Обычно растения это делают, когда внутри клетках нужно пускать много энергию мимо основной системы фотосинтеза. А это уже делают от довольно большего количества причин.
Аноним 29/12/23 Птн 20:20:52 586310 587
16740661411371.png 1108Кб, 1280x720
1280x720
Поделитесь ссылкой на адекватный калькулятор для высчитывания возможных вариантов комбинаций из заданный чисел. В скотском интернетике для залётных пикабулей-маткапов из кали-юги по тарифу ростелекома насрато кликбейтом для их учащихся пиздюков-выблядков.

Я максимально не желаю думать самостоятельно: и мне поебать на раздел матана по комбинаторике, и то, насколько это просто (эта хуйня понадобилась мне лишь единожды — сейчас, почти в трицатник годиков, и не понадобится больше ни-ког-дааа). Мне лишь нужно узнать точное число вариантов написания чисел "0-9", если использовать "11" знаков.
Аноним 29/12/23 Птн 20:30:01 586311 588
Тут есть люди, которые интересуются изменением климата и читали статьи про "aerosol masking effect"? Очень хочется поговорить с интересующимся и разбирающимся человеком.
Аноним 29/12/23 Птн 20:30:22 586312 589
>>586310
Знаете... Похуй. Я просто сгенерю набор символов на компе и посчитаю варики в текстовом редакторе.
Аноним 29/12/23 Птн 20:33:18 586313 590
>>586310
Да ты охуел школьную программу не знать.
Хотя бы википедию открыл и узнал о перестановках, размещение и сочетание, чтоб узнать в каких случаях их использовать.
Аноним 29/12/23 Птн 21:18:52 586314 591
>>586312
Я охуел, потому что для генерации этого файла (с параметром разрешенных повторов чисел и их расположений как угодно) нужно ждать годы и иметь хард из 2050 года на охулиарды гигов. Без разрешения повторов — хуйня ,всего 100миллиардов
Аноним 30/12/23 Суб 03:48:41 586317 592
>>586313
Зачем ему знать все это? Для такой задачи достаточно немного подумать головой, тут не надо ничего учить.
Аноним 30/12/23 Суб 14:36:00 586323 593
Это правда, что обнаружили антигравитацию? На какой-то лекции про это сказали, но гугление ответов не дало.
Аноним 30/12/23 Суб 15:56:16 586325 594
>>586323
В период инфляции во вселенной инфлатонное поле действовало как антигравитация.
В первом приближении Темную Энергию можно считать антигравитацией.
Аноним 30/12/23 Суб 15:59:04 586326 595
>>586325
Я думал, что теория инфляции это довольно "зыбкая" теория.
> темную материю можно считать антигравитацией
Почему? Разве она наоборот не добавляет "массы" в пространство, чтобы формулы и наблюдения за гравитацией сходились?
Аноним 30/12/23 Суб 17:27:49 586327 596
>>586326
Теория инфляции стандартная гипотеза, лучше нее пока ничего толкового не придумали.

Так я про темную энергию говорил, а не материю.
Аноним 30/12/23 Суб 19:24:36 586328 597
>>586327
А, мозг подменил слова. Спасибо, значит это имел ввиду лектор

> Сущность тёмной энергии является предметом споров. Известно, что она очень равномерно распределена в пространстве[7], испытывает гравитационное отталкивание вместо гравитационного притяжения[7]
Аноним 31/12/23 Вск 21:23:16 586360 598
2059381132c9ec6[...].jpg 102Кб, 811x1023
811x1023
1. Как прикладные знания могут быть заложены в инстинктах? Например, бобры строят сложные плотины с учётом кучи факторов чисто инстинктивно, никто их этому не учит. Птицы аналогично строят гнёзда, термиты возводят свои термитники, муравьи занимаются сельским хозяйством, выращивая грибы и тлю и т.д.
2. Как далеко это может зайти? Возможно инстинктивное понимание инженерии, металлургии, химии? Типа орков, которые, будучи полными дегенератами, могут собирать космические корабли из говна и палок.
Аноним ## Mod ## 31/12/23 Вск 22:16:45 586361 599
Том.jpg 90Кб, 400x400
400x400
Друзья, вот и подходит к концу 2023 год. Совсем скоро мы преодолеем этот рубеж, и в очередной раз, надеясь на новогоднее чудо, перенесемся из прошлого в будущее. Наступили тяжелые времена: они, как никогда ранее, сплотили нас и научили многому. И я верю, что мы продолжим твердо и последовательно преодолевать трудности, возникающие на нашем пути. А собираться в уютный вечер за чашкой чая, чтобы подискутировать на разного рода интересующие нас темы здесь... Старый добрый /sci всегда открыт для каждого исследователя.

Вам же и вашим близким я желаю здоровья. Его никогда не бывает много и оно очень хрупко по отношению к другим имеющимся у нас ценностям. Берегите здоровье свое и своих близких: не огорчайте любимых вам людей и проводите с ними больше времени, уделяя заботу и внимание. Будьте сильны и идите с гордо поднятой головой вперед всем невзгодам и неудачам. Старайтесь терпеливо выдержать любое испытание, уготованное жизнью и преподнесенное вам. И помните о том, что жизнь прекрасна и заключается в мелочах. Живите каждым мигом и не думайте о плохом.

С праздником вас! Желаю множества научных открытий в наступающем Новом 2024 Году!

Модератор-кот-ученый
Аноним 01/01/24 Пнд 03:29:07 586363 600
>>586361
Спасибо, тебя тоже.

Ты гораздо лучше клоуна-модератора из /pr/, который включил автозамену самых употребительных терминов, и теперь там каждый второй пост невозможно прочитать.
Аноним 01/01/24 Пнд 05:01:43 586365 601
>>586361
Спасибо! С Новым Годом!
Аноним 01/01/24 Пнд 14:43:36 586369 602
Скажите, а извержение вулкана на Камчатке и землетрясение в Японии в один день как-то связано?
Аноним 01/01/24 Пнд 16:47:03 586371 603
Электрон квант какого поля?
Аноним 01/01/24 Пнд 16:54:53 586372 604
IMG202401010151[...].jpg 63Кб, 720x720
720x720
IMG202312291856[...].jpg 166Кб, 720x1280
720x1280
Аноним 01/01/24 Пнд 18:07:59 586374 605
IMG202401011800[...].jpg 128Кб, 854x1280
854x1280
>>586372
Бампает в тематике
Аноним 01/01/24 Пнд 18:11:33 586375 606
>>586369
Да. Там одна и та же плита подпирает и создаёт напряжения, при сбрасывание которых собственно возникают землетрясения.
Аноним 01/01/24 Пнд 18:14:34 586376 607
Аноним 01/01/24 Пнд 18:41:05 586378 608
IMG202312291854[...].jpg 80Кб, 640x480
640x480
>>586376
Отлично. Спасибо. Где почитать подробности кроме учебника?

Эта капча заставляет меня ощущать себя умным
Аноним 01/01/24 Пнд 21:24:25 586384 609
>>586378
Да нигде особо.
Можешь попытаться почитать лекции Дирака, они легче воспринимаются, но мало отличается от других учебников.
Аноним 02/01/24 Втр 09:19:31 586392 610
>>586360
Бампулька вопросику.
Аноним 02/01/24 Втр 11:35:32 586397 611
>>586360
Инстинкты (отдергивание руки когда прикоснулся к раскаленной поверхности) это всего лишь предобученные нейронные сети. Буквально спиной мозг, это предобученная нейронная сеть, которая умеет управлять кучей всего без участия сознания. Задняя часть мозга различает прямые линии даже у людей слепых от рождения (определяли по электронной активности).

Жизнь представлена на земле от муравьев до китов, от медуз до кротов, так что возможности для создания разной живой материи в том числе и инстинктов обширны.

Другое дело, что у предобученных нейронных сетей есть проблема того, что они обучены не на свежих данных, а на старых. Возможно очень старых. Домашние коты скребут паркет дома пытаясь "закопать" экскременты. При этом я даже готов поверить что сами коты понимают бессмысленность этого занятия, но оно им кажется правильным и естественным как для человека поднимать брови когда он удивлен и улыбаться когда услышал смешную шутку.

В общем, теоретически можно на инстинктах и ракеты строить, но практически это потребует создание второго мозга в теле, который будет заниматься постройкой ракеты пока твоё сознание удивлённо будет смотреть на схемы и оборудование (или не удивлено, ведь как может быть иначе для такого существа, оно просто будет знать, что вот эта схема хорошая, а вот эта плохая, это же очевидно как для нас 2х2=4). Также это потребует как-то обновлять эти предобученные нейронки (вернее ДНК), чтобы строить не одно и тоже, а что-то новое.
Аноним 02/01/24 Втр 12:45:35 586401 612
1280px-ESTCube-1.jpg 155Кб, 1280x960
1280x960
Вопрос по космическим двигателям без рабочих тел.
Есть солнечный парус, использующий давление света.
Есть электрический парус, использующий давление протонов.

Почему нет активного электромагнитного паруса? Грубо говоря, сверхпроводниковой АФАР, нацеленной лучами на космические источники электромагнитного излучения, притягиваясь к одним и отталкиваясь от других силой Ампера?
Аноним 02/01/24 Втр 12:45:46 586402 613
>>586360
Если я правильно помню, инстинктивно бобрам только хочется копать при звуках воды. Строить плотины они учатся в детстве.
Птицы тоже многому учатся у родителей и других другие птиц, у них даже есть подобие разных культур.
Если сделать человека которому очень нравятся сложные устройства, то он с высокой вероятностью станет хорошим инженером.
Это как, когда поток частиц в мозгу напоминает нужное явление, мозг жмет на кнопку удовольствия. И человеку хочется работать с этим явлением. Или наоборот.
Аноним 02/01/24 Втр 12:48:23 586403 614
>>586401
>космические источники электромагнитного излучения
Это излучение же фотоны переносят.
Аноним 02/01/24 Втр 12:49:24 586404 615
>>586403
Ну квазары, например, вполне радиоволны генерируют
Аноним 02/01/24 Втр 13:14:27 586405 616
>>586360
> 1. Как прикладные знания могут быть заложены в инстинктах? Например, бобры строят сложные плотины с учётом кучи факторов чисто инстинктивно, никто их этому не учит.
В инстинктах заложены поведенческие, мотивационные паттерны. Это нельзя называть знанием (совокупность сведений, информации). У бобра нет чертежа в голове. Бобер действует как бы интуитивно, решая возникающие проблемы по месту, а удачные решения закрепляются условным рефлексом. Плюс наверняка они зеркалят взрослых и чему-то научаются у них. Т.е. научение в некоторой степени у них имеется: отзеркаливание/повторение и условные рефлексы.

> 2. Как далеко это может зайти? Возможно инстинктивное понимание инженерии, металлургии, химии?
У них нет понимания, знания в человеческом смысле. Развитие идёт методом рандома и отбора поведенческих паттернов. На отборе инстинктов и рефлексов не дойти до уровня человеческого понимания, это качественно другой уровень.

> Типа орков, которые, будучи полными дегенератами, могут собирать космические корабли из говна и палок.
Нет. Можно только гипотетически представить, что их кто-то научит, произведёт искусственный отбор на нужные поведенческие паттерны и т.п. В общем, утрируя, человек и есть такой орк, собирающий ракету слесарь просто делает несложную операцию, а знание как летают ракеты ему и не нужно.
Аноним 02/01/24 Втр 13:40:25 586406 617
>>584346 (OP)
Интересно, возможен ли гипотетически и практически запуск цепной реакции синтеза без помощи ядерного взрыва? Чисто термоядерная бомба, без ступени деления.
Аноним 02/01/24 Втр 13:49:23 586407 618
>>586406
В экспериментальных энергетических установках лазером зажигают. Но для взрыва фокусирующие оболочки (или как там их называют) все равно понадобятся.
Аноним 02/01/24 Втр 14:04:55 586408 619
>>586407
Мне кажется лазер для бомбы это тупиковый путь. Во всех этих экспериментальных установках выход энергии меньше чем затрачено, цепная реакция гаснет, а сами установки имеют нереальные размеры.

Проблема в огромной плотности мощности, требуемой для удержания нагретой плазмы в компактной кучке, пока идет синтез. Интересно нельзя ли тут как-то использовать кинетическую энергию, в ней ведь можно запасти потенциально неограниченное количество энергии, можно даже не за один раз. Что-то типа боеголовки космического базирования на высокоэллиптической орбите, или вообще на ретроградной околосолнечной. Потом в процессе входа в атмосферу на нескольких десятках километров в секунду кинетика конвертируется в тепло, которое обжимает капсулу с горючим.
Аноним 02/01/24 Втр 16:51:17 586412 620
>>586384
А чому в википедии не пишут про это?
Аноним 02/01/24 Втр 17:24:38 586413 621
Объясните на пигмейском, из чего состоит материя?
Вот есть атом. Атом — это такая хуёвина с ядром и летающими вокруг него на световой скорости электронами. В ядре протоны и нейтроны. Что дальше? Почему там уже начинаются какие-то математические абстракции и гипотетические квазичастицы? Типа оно настолько мелкое, что уже ни увидеть, ни описать нельзя — только теории строить? Или как?
Аноним 02/01/24 Втр 17:28:06 586414 622
Аноним 02/01/24 Втр 18:59:15 586417 623
>>586405
>их кто-то научит
А кто научил бобров плотины строить? Могли бы грызть себе кору и в ус не дуть. Прочие грызуны так и делают — и ничего, процветают. Птицы аналогично. Зачем ткачу целая хижина на дереве, если голубю пары веток хватает?
Я к тому, что такие паттерны поведения, видимо, возникают рандомно — а дав какое-то преимущество, закрепляются и совершенствуются. Пять миллионов лет назад бобр строил хатку в случайной куче веток, а ещё через пять миллионов лет начнёт таскать в свою плотину камни для укрепления фундамента. Через десять научится углублять дно водоёма и изменять рельеф берега, а через двадцать освоит кирпичи из сырой глины. А там уже до освоения огня и начала медного века всего ничего — пара сотен миллионов — остаётся.
Грубо говоря, конечно.
Аноним 02/01/24 Втр 19:24:22 586418 624
>>586413
> Типа оно настолько мелкое, что уже ни увидеть, ни описать нельзя — только теории строить?
Типа да. Настолько мелкое, что у нас нет инструмента для наблюдения, не оказывая влияние на него. Любое наблюдение это взаимодействие. Чтобы узнать положение или скорость фотона надо в этот фотон пульнуть другим фотоном, в результате мы видим результат взаимодействия измеряемого фотона (за которым пытаемся наблюдать) с фотоном от измерителя, а не измеряемого фотона самого по себе. Для измерения без влияния нужен измеритель, который воздействовал бы на измеряемое значительно более мелкой частицей, а такого пока нет и не предвидится.
Аноним 02/01/24 Втр 19:27:15 586419 625
>>586413
> увидеть
Как увидеть то, что не отражает свет?
> описать
Описывают на основе наблюдений. Наблюдать за частицами можно посредством их взаимодействия с другими частицами.
> только теории строить
Если теория успешно предсказывает, то в чем проблема?
> из чего состоит материя
Правильный вопрос: чем является материя. Материя является формой энергии. Электрон как бы тоже не "шарик", а именно энергия в какой-то точке пространства.
Аноним 02/01/24 Втр 19:34:51 586420 626
>>586417
> Через десять научится углублять дно водоёма и изменять рельеф берега, а через двадцать освоит кирпичи из сырой глины.
Такое можно гипотетически представить. Но чтоб бобры начали понимать металлургию, химию и построили ракету, такое методом перебора инстинктивных действий не получить, тут нужно абстрактное мышление, накопление информации (язык, письменность), специализация труда и прочее.
Аноним 02/01/24 Втр 21:51:54 586424 627
>>586412
> А чому в википедии не пишут про это?
Напиши сам.
(кто? Я?)
Аноним 03/01/24 Срд 00:10:10 586426 628
>>586412
Вообще-то пишут, например в статье про уравнение Дирака например.
Аноним 03/01/24 Срд 02:01:13 586427 629
>>586414
> Из энергии.
* E0=m•c2 недвусмысленно об этом говорит, если ты ещё не понял. Именно в этом её "фишка".
Аноним 03/01/24 Срд 02:48:01 586429 630
>>584346 (OP)
Анонэс! Доброй ночи. А конкретно- анаоэс-физики. Обращаюсь к вашим светлым( и темным) мозгам, потому что мои мозги уже потрескивают и поплыли. Сказу прямо и честно-я тупо гуманитарий. Закончил в свое время истфак(правда историком нихуя никогда нигде не работал). Короче я к тому, что я нихуя не физик, а скорее все же лирик. И вот с годами я начал в ютубчике посматривать и в интернетиках почитывать всякую интересную инфу про квантовые флуктуации, природу материи, сингулярность и всякие там бозоны Хиггса( благо этой хуйни там счас навалом). Ясен хуй, теоретической базы у меня никакой, из физики я помню ток mc квадрат, поэтому до конча понять этих прекрасных людей со светлыми очами мне трудновато. И все же я сам для себя начал делать кой-какие выводы. Вот хочу у вас уточнить кой-чаво.
Вот допустим для меня открытием стало что мы нихуя не тратим никакие энергии, которые используем. Они просто переходят из одного состояния в другие( кинетические в статические, энергия в массу), но в принципе все энергии сохраняются в своем первоначальном виде. Так это что получается, аноны? Вся жизнь на планете земля существует за счет перехода эпергий и масс из одного состояния в другое? насколько я понял вумных дядек, вся энергия во вселенной пытается перейти из концентрированного состояния в рассеянное( простите, если некорректно термины употребляю) и в принципе все это называется энтропия. То есть по сути мы существуем за счет энтропии получается?
Аноним 03/01/24 Срд 06:40:03 586433 631
>>586429
Да. То, что обыватели называют использованием энергии, является получение нужного результата за счёт увеличения энтропии.
Но вообще закон сохранения энергии это середина школы. Как ты это пропустил?
Аноним 03/01/24 Срд 06:47:35 586434 632
>>586429
Это с точки зрения вселенной. Земля не является замкнутый системой, можешь расслабиться.
Аноним 03/01/24 Срд 11:22:05 586436 633
>>586434
> Это с точки зрения вселенной.
Вселенная замкнутая система?
Аноним 03/01/24 Срд 12:01:26 586439 634
>>586436
Предположительно, да.
Аноним 03/01/24 Срд 12:23:15 586443 635
>>586433
А как энтропия связана с течением времени? Я так понял, что сам факт течения времени- это следствие повышения уровня энтропии. Или, наоборот, повышение энтропии происходит за счет течения времени, Или мухи отдельно, котлеты отдельно?
Аноним 03/01/24 Срд 12:55:42 586444 636
>>586418
>Для измерения без влияния нужен измеритель, который воздействовал бы на измеряемое значительно более мелкой частицей, а такого пока нет и не предвидится.

Проблема не в этом, если я тебе в классической физике ограничу способы измерения параметров какой-нибудь системы только теми, которые эту систему хуярят сильно, то ты всё-равно сможешь косвенно и с помощью математики извлечь в конце концов все нужные физические наблюдаемые с точностью, которую тебе приборы позволят. Потому что сами параметры и физические величины имеют свои значения, независимо от того измерял ты их или нет

В квантовой же механике уже из самого мат.аппарата вытекает, что физические наблюдаемые не всегда имеют значение, а измерение (сколько бы слабым ты его не делал, хоть косвенным вообще) и сколько статистики одинаковых систем не собирал, не дает тебе определить однозначную физическую наблюдаемую. Это в своё время вызвало споры о том, что квантовая механика лютый пиздеж и её надо напильником дорабатывать, но потом поставили эксперименты, которые четко указали, что дело не в измерении.

"Измерение которое влияет на систему, поэтому измерить точно нельзя" это научпоп-хуита для обывал, которым кто-то это скормил. Не надо так думать.
Аноним 03/01/24 Срд 12:59:30 586445 637
>>586443
Вещи плохо связанные. Какой-нибудь маятник с колебаниями свою энтропию не меняет, но ведь время всё-равно течет. Энтропия это про другое, это про потерю информации о системе, т.е. чисто фишка, которая возникает из мысли "ну мы неточно знаем начальные условия, а еще окружающий мир хоть немного но влияет на систему всегда, поэтому информация утечет".
>>586436
Предположительно да, но не сохраняет энергию, так что не парься
Аноним 03/01/24 Срд 13:54:59 586446 638
>>586436
Неизвестно. Когда мы говорим вселенная чаще всего имеется наблюдаемая вселенная. Что там дальше за наблюдаемой вселенной непонятно.
>>586444
То есть у частицы нет местоположения или все же местоположение неоднозначно в каком-то ограниченном пространстве? Вроде же второе, то что да мы не знаем и не можем знать где находиться частица, но с вероятностью такой-то она находиться вот здесь, а с другой вероятностью вот здесь.
Аноним 03/01/24 Срд 14:47:05 586447 639
>>586439
> Предположительно, да.
>>586445
> Предположительно да
>>586446
> Неизвестно.

Поэтому меня всегда бомбит, когда понятие «энтропия» применяют ко вселенной и начинают из этого делать какие-то выводы и предсказания. Скажите тогда что вселенная это организм и он умрет или поделится на несколько вселенных, как клетка, это точно также «предположительно» как и энтропия.
Аноним 03/01/24 Срд 15:10:20 586448 640
>>586447
Точные ответы только в библии. В физике всё что не проверили сотню раз в контролируемых условиях "предположительно".
Аноним 03/01/24 Срд 15:22:18 586450 641
>>586448
> В физике всё что не проверили сотню раз в контролируемых условиях "предположительно".
«Предположительно» в науке это гипотеза, а чтобы считаться научной гипотеза должна быть фальсифицируемой. Закрытость вселенной (и соответственно энтропия) не может считаться научной гипотезой, она не фальсифицируема, ты не можешь вылететь за пределы вселенной и проверить закрытость вселенной. Так что энтропия вселенной это не наука, а диванные домыслы, метафизика если философски.
Аноним 03/01/24 Срд 15:32:31 586451 642
>>586447
Ты в ближайшие сто лет умрёшь с гарантией. Очень вероятно, человечество не осилит даже следующие 20 тысяч и вымрет. Конечно, если вдруг научимся терроформациям и межзвездным перелётам, возможно, пару миллиардов протянем (не совсем уже мы), но это крайне оптимистичные предположения. Из разряда чудес.
Аноним 03/01/24 Срд 16:02:55 586452 643
>>586446
>у частицы нет местоположения или все же местоположение неоднозначно в каком-то ограниченном пространстве?
Само понятие местоположения мало смысла имеет просто. Это что-то из нашего мира, что-то статистическое.
Аноним 03/01/24 Срд 16:09:33 586453 644
>>586452
Не могу понять. Есть камень, он состоит из атомов, они из частиц. Где частицы? Там где камень. Вроде всё логично.
Аноним 03/01/24 Срд 16:13:19 586454 645
>>586446
>То есть у частицы нет местоположения или все же местоположение неоднозначно в каком-то ограниченном пространстве?
Третье. Тело частицы бесконечно расширяется. Пока она не решит провзаимодействовать с другой пересекшейся с ней частицей. После чего исчезает. И новые частицы начинают расширяться уже из точки взаимодействия.
Аноним 03/01/24 Срд 16:55:33 586455 646
Аноним 03/01/24 Срд 17:09:00 586456 647
442A7F15-D29C-4[...].jpeg 174Кб, 1530x675
1530x675
>>585484
>Почему в русскоязычной среде принято отрицать/приуменьшать значение IQ?
Потому что пикрил. Корея — хз, но Китай, где обезьяны срут на улицах и живьём жарят собак на говняном масле? Иран, который буквально воплощение всех стереотипов о муслимских парашах пятого мира? Для полноты картины там только какого-нибудь Чада или Конго в пятёрке лидеров не хватает.
Аноним 03/01/24 Срд 17:27:42 586457 648
>>586456
>>585484
Потому что IQ изначально придуман как тест показывающий недостаточный интеллект. Он не говорит, что один человек умнее другого, он говорит, что один человек имеет проблемы с интеллектом (IQ < 85), а другой нет (IQ > 85).
Аноним 03/01/24 Срд 23:02:27 586459 649
>>586453
Ну ты когда говоришь "камень" ты довольно большой объем описываешь, при этом частиц там тоже много, поэтому среднее значение нахождения этих частиц отлично совпадает с твоими наблюдениями.

А если возьмешь какой-нибудь атом водорода, то электрон этот ебучий не будет уже находиться где-то конкретно, у него будет своё состояние квантовое, уже из которого ты можешь найти распределение его нахождения в этом атоме.

При этом это не классическая вероятность, где мы просто не знаем, там в принципе природа как-бы не в курсе где он.
Аноним 03/01/24 Срд 23:12:07 586460 650
>>586459
То есть при большом скоплении частиц они становятся устойчивыми? Хотя нет, не так. Большое (интересно сколько) скопление частиц обладает свойством устойчивости в пространстве. Малое скопление частиц (до одной) не имеют свойства вообще находиться в конкретном месте.

Получается "местоположение" есть результат взаимодействия многих частиц друг с другом. То есть сосредоточенная энергия (а что есть частица как не энергия) превращается в массу, материальный объект, в то время как распределённая энергия в пространстве представляет собой некое энергетическое поле теряющие свойства привычной материи.
Аноним 03/01/24 Срд 23:13:48 586461 651
>>586459
А правда, после какого момента концентрации частиц у них как у скопления появляются привычные нам координаты в пространстве?
Аноним 04/01/24 Чтв 00:30:51 586462 652
>Большое (интересно сколько) скопление частиц обладает свойством устойчивости в пространстве.
Там всё сложнее, смотря как ты определяешь устойчивость. Просто когда ты берешь камень, тебе немного похуй на погрешности в 10^-10 м, т.к. это 10^-10 %. А если у тебя объект размером с 10^-9, то на подобные штуки глаза уже не закрыть.

Плюс еще вопрос в том, как частицы взаимодействуют, свободная частица имеет тенденцию "расплываться", а если связанная, то распределение вероятностей не меняется либо циклично меняется.

>То есть сосредоточенная энергия (а что есть частица как не энергия) превращается в массу, материальный объект, в то время как распределённая энергия в пространстве представляет собой некое энергетическое поле теряющие свойства привычной материи.

Массу и энергию не разделяют особо на разные категории, но туда лучше не лезть, это уже философия.
Аноним 04/01/24 Чтв 02:02:56 586464 653
>>586459
Электрон это понятно, а кварки что?
Аноним 04/01/24 Чтв 11:35:50 586476 654
>>586462
То есть весь мир по сути нестабилен с точки зрения местоположения? Мы просто слишком большие, чтобы заметить это "дрожание" вселенной. Воодушевляет на самом деле. Столько ещё можно осознать.
Аноним 04/01/24 Чтв 11:40:38 586477 655
fb.png 1085Кб, 1200x630
1200x630
>>586476
Ещё осознай, что всё есть пустота с некими всплесками энергии
Аноним 04/01/24 Чтв 13:07:14 586478 656
>>586476
> То есть весь мир по сути нестабилен с точки зрения местоположения?
Не только положения, но и по куче других параметров. На микроуровне все "плавает", вернее колеблется.
Аноним 04/01/24 Чтв 15:42:12 586484 657
>>586476
Если начать немного копаться, внезапно появляется явление энтропии из этого.
Аноним 05/01/24 Птн 07:00:18 586495 658
Не лучше ли говорить, что пространство-время не 4мерно, а 3.5мерно, потому что на данный момент ни наука, ни даже философия ничего не могут сказать о путешествиях в прошлое, т.е. пока что можем говорить только о движении вперёд. Причём путешествие в прошлое чаще всего представляют как скачок, телепорт,- а с плавным движением назад всё ещё запущенней. Что будет видеть путешественник и как будут видеть его? К чему заморачиваться - ну а типа вдруг для описания движения назад во времени неизбежно придётся вводить новые координаты, потому что стрела времени может быть прямой только в будущее, а в прошлое неизбежно пойдёт загогулинами дробной размерности, так что лучше сразу подчеркнуть, что врпмя может быть не 1-,2- и тд целомерными, а именно нецелой размерности. ладно, вы думойте, а мне спать пора
Аноним 05/01/24 Птн 10:28:03 586499 659
>>586495
> наука ничего не может сказать про перемещение во времени в прошлое
Альберт Эйнштейн всё сказал уже про это. Ищи червоточину, создавай отрицательную гравитацию, чтобы компенсировать своё влияние гравитации и попадешь в прошлое. Нашли правда пока что ноль этих червоточин, но рано или поздно найдут или сами сделают.
Аноним 05/01/24 Птн 13:45:09 586501 660
>>586495
Если ты начнешь время назад отматывать, не заметишь разницы, разбитая ваза не соберется, ты не в своё прошлое попадать начнешь, а в самое вероятное
Аноним 05/01/24 Птн 13:54:17 586502 661
Аноним 05/01/24 Птн 18:39:54 586509 662
>>586499
>создавай отрицательную гравитацию
Это не поможет. Вселенная расширяется на из-за положительного знака гравитации.
Аноним 05/01/24 Птн 18:42:25 586510 663
>>586499
> Ищи червоточину, создавай отрицательную гравитацию, чтобы компенсировать своё влияние гравитации и попадешь в прошлое.
только это уже не физика, а математика, не затрагивающая проблему причинности типа парадокса дедушек или как его. А физика понятия не имеет, как поставить соответствующий эксперимент для разрешения парадокса, ни про существование отрицательной/мнимой энергии для этого и как добыть её. Т.е. как я представляю хорошее продвижение в этом направлении: пишут статью - ставят соотв эксперимент, закрепились, повторили и тд маленькими шажками. А по факту тут с экспериментами туговато, городят пирамиду спекуляций одна на другой. А матаном можно отрицательную, мнимую, кватернионную и какую хочешь гравитацию ввести и телепортироваться по вселенной как хочеш
Аноним 05/01/24 Птн 19:20:43 586511 664
>>586510
> парадокс дедушки
Что-то физики спокойно живут с парадоксом телепортации частиц в пространстве в места где их не могло быть по всем законам логики.
> с экспериментами туговато
> спекуляции
Всё так, но какой вопрос такой ответ. Лучше всё равно ничего нет кроме очевидного: пока никто не путешествовал, а значит мы не знаем возможно это или нет.
Аноним 05/01/24 Птн 20:05:27 586512 665
>>586511
>> парадокс дедушки
>Что-то физики спокойно живут с парадоксом телепортации частиц в пространстве в места где их не могло быть по всем законам логики.
Неправда, никаких наблюдаемых парадоксов не существует.
Аноним 05/01/24 Птн 20:06:55 586513 666
>>586512
Объяснишь как материальный объект телепортируется?
Аноним 05/01/24 Птн 20:26:05 586514 667
>>586513
Я не физик, я математик.
Я имею в виду, что ты неправильно понимаешь, что такое парадокс. Парадокс, это когда из А можно вывести Б и не-Б одновременно. Это говорит о том, что А ложно, и на этом строится доказательство от противного.
В реальном мире нет и быть не может фактов, приводящих к парадоксам, потому что законы логики выполняются неукоснительно.
Если тебе кажется, что телепортирующиеся частицы нарушают какие-то законы логики, то нет, это не так. Просто твои представления о том, как эти частицы должны себя вести неверны.
Аноним 05/01/24 Птн 20:35:44 586515 668
>>586514
Понял, но как это объясняет невозможность ситуации с дедушкой? Убил дедушку, как я родился в этом случае? А какая разница, родился и родился. Телепортировалась сквозь непроницаемую стенку? А как это возможно? А какая разница, возможно и возможно.

Первое это парадокс, второе это квантовая физика. Я понимаю, что второе квантовая физика это потому что мы наблюдали телепортацию. А первое парадокс потому что гипотетическая ситуация.

Ладно, в принципе я сам ответил на свой вопрос. Спасибо за внимание.
Аноним 05/01/24 Птн 20:49:32 586518 669
Силаев П. К. -[...].webm 20325Кб, 1280x720, 00:00:50
1280x720
>>586513
Ты сам то знаешь, что называют квантовой телепортацией?
Аноним 05/01/24 Птн 20:53:48 586519 670
Аноним 06/01/24 Суб 00:29:13 586522 671
>>584346 (OP)
Когда железо или сталь калят, что за чешуя сыпется с них?
Аноним 06/01/24 Суб 08:48:16 586524 672
>>586522
Окалина или оксид железа.
Аноним 06/01/24 Суб 09:39:19 586528 673
>>586515
Ничего ты не понял.
Существование частицы никак не противоречит её телепортации за стенку, потому что частица не шарик, а поле, которое стенка не экранирует.

А вот мёртвый дедушка твоему существованию противоречит. Разве что ты "тунеллировал" вместо нормального рождения. Но это как раз одно из решений этого парадокса через многомировую интерпретацию: ты попадаешь в не свой мир.
Аноним 06/01/24 Суб 11:53:03 586532 674
Если вселенная бесконечна, значит после моей смерти рано или поздно все молекулы разложившегося меня вновь соберутся обратно и я снова стану живым. В чём я не прав?
Аноним 06/01/24 Суб 13:21:58 586535 675
Аноним 06/01/24 Суб 13:44:13 586536 676
>>586535
Ну моё тело и моё сознание в любой момент моей текущей жизни епта
Аноним 06/01/24 Суб 14:38:49 586538 677
>>586536
Ладно, что такое сознание?
Аноним 06/01/24 Суб 15:41:34 586541 678
>>586538
Ты мне зубы то не заговаривай. По факту мне напиши в чём я не прав или извинись в случае моей правоты.
Аноним 06/01/24 Суб 17:48:50 586551 679
>>586541
Вселенная конечна и имеет начало. Земля конечна и имеет конкретные границы. Ты сдохнешь навсегда.
Аноним 06/01/24 Суб 19:32:57 586557 680
>>586541
Ты не определил понятия, а значит непонятно о чем говорить. Да и вселенная не бесконечна, это в точь так же как считать комнату бесконечной после того как сделал один шаг в темноте.
Аноним 06/01/24 Суб 19:34:48 586558 681
>>586532
Школьник прочитал Ницше и прибежал искать научное подтверждение вечному возвращению? Не будет его. Начни с того, что ты секунду назад был другой, а организм полностью обновляется за 5-7 лет, в нем не остаётся ни одного атома прежнего тебя. Кто должен восстановится в будущем, если этого стабильного «кто» в принципе не существует?
Аноним 06/01/24 Суб 19:37:25 586560 682
>>586557
>Да и вселенная не бесконечна
Школьник опечатался, для вечного возвращения вселенная должна быть конечной, если вселенная бесконечная, то ни каких повторений форм не будет.
Аноним 06/01/24 Суб 19:42:57 586562 683
>>586560
Плевал я на бредни Ницше. Пока нет доказательств бесконечности вселенной. Есть лишь понимание, что если мы не изобретаем устройство способное на телепортацию, то никогда (предположительно) не поспеем до её края из-за скорости её расширения.
Аноним 06/01/24 Суб 19:44:19 586563 684
Аноним 06/01/24 Суб 21:08:57 586566 685
>>586560
Это не важно.
Если вселенная возникла, то ее возникновение возможно. И она возникнет вновь.
Через достаточное количество повторений, ребенок с памятью Анона может сформироваться случайно. Даже если это просто клон.
Аноним 06/01/24 Суб 22:00:22 586568 686
>>586566
>ребенок с памятью Анона может сформироваться случайно
Дальше что? Анон воскреснет из мертвых? Душа Анона спустится с небес и начнёт опять жить земную жизнь в теле-копии?
Аноним 06/01/24 Суб 22:23:56 586569 687
>>586568
>Душа Анона спустится с небес и начнёт опять жить земную жизнь в теле-копии?
Это не важно. Душа в теле-копии будет искренне верить что она и есть душа Анона.
Анон же, это не тело с его молекулами. И не сознание с его квалиа. Анон это личность и память.
Аноним 06/01/24 Суб 22:25:13 586570 688
Душа и тело Анона - всего лишь среда в которой Анон существует.
Аноним 06/01/24 Суб 22:43:10 586571 689
>>586569
> Анон это личность и память.
Для наблюдателя со стороны, с перспективы третьего лица. А от первого лица ты умрешь и от того, что когда-то появится твоя копия, ни чего для тебя не изменится, ты не начнёшь жить заново, не воскреснешь или что ты сам себе выдумываешь.
Аноним 06/01/24 Суб 22:45:08 586572 690
Да нет никакого анона. Это всё самообман. Буквально. Самообман. То чего нет, но во что выгодно верить. Я не более чем смешное поле частиц и моя "смерть" значит июне больше чем пустяк.
Аноним 06/01/24 Суб 23:16:21 586573 691
>>586572
> но во что выгодно верить.
Старикам выгодно, чтобы смерти не боятся, как самоуспокоение. И выгодно пролам насаждать такое мышление, чтоб работали и не роптали и в окопы смело шли умирать. А в молодом и зрелом возрасте это деструктивно, лучше жить с пониманием что жизнь у тебя одна и относится к ней с соответствующей ответственностью.
Аноним 07/01/24 Вск 02:02:42 586578 692
>>586576
Не будет, скорость света всегда скорость для наблюдателя. Как угодно изворачивайся, а выше этого скорость невозможна. Не баг, а фича. Константа в программном коде.
Аноним 07/01/24 Вск 06:03:05 586584 693
>>586566
Нет. Это неопределенность бесконечность к бесконечности
Аноним 07/01/24 Вск 10:19:47 586586 694
>>586574
Смысл в проживании здесь и сейчас - Дазайн.
Аноним 07/01/24 Вск 14:02:55 586591 695
>>586584
Без принятия на веру таких недоказуемых вещей как реальность прошлого, существование других людей и наличие причин у событий мышление становится бессмысленным и обсуждать просто нечего.
Аноним 07/01/24 Вск 22:02:07 586603 696
3270330original[...].jpg 18Кб, 520x245
520x245
11111.JPG 19Кб, 897x138
897x138
>>584346 (OP)
Анчоус, что за еврейские фокусы? Это на каждом сухом мороженном, плюс-минус. На НГ изьебнулся, но получилось только сто грамм добавить, а тут обещают из ста смеси и трехсот миллилитров шестьсот продукта. Откуда двести то беретcя? Родители пиздели и Дед Мороз существует? Может Шваб в пизду со своими жуками идет, мы тут на самогенеряшимся мороженом протянем?
Аноним 08/01/24 Пнд 01:45:34 586606 697
Прошу пояснить причину тряски касательно проблемы измерения в КМ, и особенно разгона про СОЗНАНИЕ.

Если измерение это тупо встреча фотона инструмента с испытуемым электроном, то чем это отличается от встречи электрона с левым фотоном, который просто мимо летел? В обоих случаях схлопнется волновая функция. И причём тут тогда наблюдатель с СОЗАНИНИЕМ? У хомо сапиенс монополия на испускание фотонов? Смешно
Аноним 08/01/24 Пнд 03:12:48 586608 698
>>586606
Тут проблема в том, в КМ постулируются два типа систем. Классические - у них строго детерминированы динамические переменные. Они образуют прибор/наблюдатель/мамашу-шлюху/наблюдаемую вселенную.
И квантовые - они живут в своем манямирке, который не имеет физического смысла, пока не испытает возмущение со стороны классической системы.
Однако по идеи классические системы являются продолжением квантовых систем, но у них не из чему взяться "классичности" кроме как другой классической системы. А конечном итоге все сводиться к тому, что изначальной классичностью обладает некая магическая хуита, которая интерпретирует измерения. От сюда и лезет шиза с сознанием.
Аноним 08/01/24 Пнд 04:39:05 586609 699
>>586603
Ты путаешь массу и объём.
Аноним 08/01/24 Пнд 04:48:33 586610 700
>>586606
>В обоих случаях схлопнется волновая функция.
Там кек в том, что схлопывания в уравнении нет. Т.е. если у тебя электрон ебучий в суперпозиции находится, и на него фотон налетает, то фотон тоже становится в суперпозиции, и если ортодоксально подходить к квантовой мехнике, то когда потом этот фотон в детектор залетает, сам детектор тоже в суперпозиции, и ты потом смотришь в него и тоже в суперпозиции. И если подходить так, только в этот момент ты заметишь какую-то хуйню в результатах.

Причем если я не ошибаюсь, там объекты довольно классического размера точно в квантовое состояния загоняли уже в экспериментах.
Аноним 08/01/24 Пнд 08:48:21 586612 701
>>586609
Когда инструкцию читал, тоже об этом думал, хитрожопый маркетолог аккуратно жонглирует. Но я же сделал его, взвесив все этапы на весах. В моем понимаем масса в простом процессе не может появится, мы не меняем кардинально вещество, мы буквально делаем смешение, однородность и меняем агрегатное. Добавилось сто грам - было 397 грамм из смеси и сливок, а стало 501 грамм. Возможно я рукожоп или маркетолог смело припизднул еще сто грамм, не принципиально, но сотка то добавилась. Откуда?
Аноним 08/01/24 Пнд 09:26:19 586613 702
>>586612
У тебя в инструкции написано 600 мл, а не г.
А сотка могла легко взяться из воздуха.
Аноним 08/01/24 Пнд 10:34:00 586614 703
>>586495
> Не лучше ли говорить, что пространство-время не 4мерно, а 3.5мерно, потому что на данный момент ни наука, ни даже философия ничего не могут сказать о путешествиях в прошлое, т.е. пока что можем говорить только о движении вперёд.
Никакой связи мерности и путешествий в прошлое нет. Мерность возникает совсем из других соображений.
Аноним 08/01/24 Пнд 10:39:56 586615 704
>>586576
> Если из другой галактики создать световой пучок маятник который будет шатать со световой скоростью то световое пятно на поверхности другой галактики будет двигаться выше скорости света! Думойти!
Это известный факт. И он никак не противоречит физике, тк никакая материя или энергия тут не движется быстрее скорости света.
Аналог этой ситуации - сверхбольшие ножницы, у которых точка соприкосновения лезвий может двигаться со сверх световой скоростью. Или можно просто выставить ряд лампочек и зажечь их все почти одновременно так, что "точка зажигания лампочек" движется со сколь угодно большой скоростью.
Все эти примеры не позволяют передать энергию или информацию со скоростью большей скорости света.
Аноним 08/01/24 Пнд 10:42:18 586616 705
>>586607
> инфополе вселенной
Что это такое?
Аноним 08/01/24 Пнд 10:43:07 586617 706
>>586607
> инфополе вселенной
Что это?
Аноним 08/01/24 Пнд 10:54:18 586619 707
>>586613
>из воздуха.
Наверное да, в этом ответ. Куб воздуха при нулевой 1,29 кг весит. Мороженое мы взбиваем, образую множество камер-пузырьков. Фокусы не перестали быть еврейскими, поскольку вес готового продукта не меняется, а просто добавляется масса воздуха, но откуда добавилась масса мы решили. Спасибо.
Аноним 08/01/24 Пнд 11:37:58 586620 708
>>586610
>сам детектор тоже в суперпозиции
это ж недоказано
Аноним 08/01/24 Пнд 12:27:00 586621 709
>>586608
>>586610

Хотите сказать, что если на испытуемый электрон налетит случайный фотон (не от наблюдателя), то коллапса не будет? Прикол. Я думал что разрушение квантовых систем левыми частицами случайным образом — и есть деконеренция
Аноним 08/01/24 Пнд 12:44:15 586622 710
Интеллект действительно начинает снижаться с 30 лет? Насколько сильно? Можно ли отсрочить деградацию?
Аноним 08/01/24 Пнд 12:51:12 586623 711
>>586621
Суперпозицию разрушает именно прибор - классическая система.
Если на твой электрон нелетит левый фотон, то суперпозиция незначительно измениться.
Однако если ты будешь попытать одновременно измерить это левый фотон и электрон. Этот налет уже уничтожить суперпозицию.
Декогеренция это про то, что прибор измеряет не те частицы, которые мы приготовили.
Аноним 08/01/24 Пнд 12:57:16 586624 712
>>586622
Смотря, что считать интеллектом.
Если считать способности к быстрому обучению чему-то новому, то да. Еще страдает скорость осмысленных реакции и принятия решений. В какой-нибудь спорте будешь всасывать у молодых.
Насколько сильно это проявляется тут сильно индивидуально.
Да. заниматься максимально разнообразной деятельностью, общаться с молодыми.
Аноним 08/01/24 Пнд 13:05:57 586625 713
>>586623
а что такое прибор? можно считать прибором экран за двумя щелями, где подсвечиваются попадания электрона? и опять же причем тут сознающий человек, на экране и без него все подсветится
Аноним 08/01/24 Пнд 13:22:21 586628 714
>>586625
Выше я писал, что прибор это любая система, наделенная классическими свойствами. А то как это наделяется это уже философский вопрос.
На практике же прибор это все что "классически" действует на квантовую систему. Это одновременно и две щели, экран и пустота между ними.
Если уж воспринимать эту шизу с сознанием, то оно делегирует свою классичность прибору только одним знанием о нем.
Аноним 08/01/24 Пнд 16:51:58 586629 715
>>586628
наблюдатель увидит либо стрелку вверх либо вниз, хотя в манямире они в суперпозиции — это я понял

обезьяна тоже увидит только один исход, полагаю

а собака? а рыба?

а может мера создание в существе есть способность схлопывать волновую функцию, ну или выбирать какой-то мир в альтверсе, если ММИ права
Аноним 08/01/24 Пнд 17:21:01 586630 716
>>586629
> хотя в манямире они в суперпозиции
Суперпозиция это состояние до измерения (воздействия), о котором мы, соответственно, ни чего не знаем, оно нам недоступно. Измерив (оказав воздействие) мы наблюдаем иное состояние. Просто поменяй слово «суперпозиция» на «не известное(не наблюдаемое), до измерения (до воздействия)» и вся шарлатанская магия НАБЛЮДАТЕЛЯ пропадёт.
Аноним 08/01/24 Пнд 18:26:13 586631 717
>>586629
Даже бездушная машина или компьютер, если он классический, увидит только один исход.
Коллапс волновой функции чисто магический, тут много чего можно напридумать, ибо его механизм не доступен нам через эксперимент.
Аноним 08/01/24 Пнд 18:52:34 586632 718
>>586620
Да, недоказано, это просто как следствие КМ идет

Но на детекторе LIGO они линзу 40 кг засунули в квантовое состояние, а еще какие-то чуваки, маленькие барабаны алюминивые переводили в запутанное состояние.
Аноним 08/01/24 Пнд 20:13:18 586637 719
>>586631
Да блин, что делает объект классическим, может кто-то объяснить твёрдо и четко? Только не говорите что его способность схлопывать волновуху, а то уже рекурсия какая-то
Аноним 08/01/24 Пнд 21:14:12 586639 720
>>586637
В каждой интерпретации свое объяснение. Разве кроме многомировой, где коллапса нет как таково.
Аноним 08/01/24 Пнд 22:10:37 586640 721
>>586632
>они линзу 40 кг засунули в квантовое состояние
Они ее просто охладили.
>переводили в запутанное состояние
...
Аноним 08/01/24 Пнд 22:15:51 586641 722
>>586606
>это тупо встреча фотона инструмента с испытуемым электроном
На каком расстоянии происходит эта "встреча"? 1 метр? 1 нанометр?
Напомню на всякий случай - размеры точечных частиц равны нулю. С другой стороны, радиус действия электромагнетизма - бесконечность.
Аноним 08/01/24 Пнд 22:36:12 586644 723
>>586640
>Они ее просто охладили.
И что это меняет?
>>586641
Тебе придется дать определение "расстояния" для квантовой электродинамики, что потребует от тебя пары лекций и довольно смешного ответа, который только больше тебя запутает
Аноним 09/01/24 Втр 01:58:45 586647 724
>>586557
Если вселенная достаточно долго будет существовать, то в ней возможно что угодно - появление больцмановского мозга из флуктуаций или анона сдохшего триллионы лет назад.
Аноним 09/01/24 Втр 02:25:25 586648 725
>>586644
А расстояние от земли до луны например это тоже "смешная" концепция или она каким то невероятным образом вдруг проясняется?
Аноним 09/01/24 Втр 03:04:56 586649 726
>>586648
Оно проясняется очевидным образом
Аноним 09/01/24 Втр 10:08:25 586655 727
>>586647
Ключевое слово "если". Энтропия пока что выглядит как причина по которой не всё что угодно может произойти в будущем.
Аноним 09/01/24 Втр 11:44:32 586657 728
>>586649
Нет, не проясняется.
Аноним 09/01/24 Втр 16:39:57 586662 729
>>586619
Пузырьки не влияют на массу, но газы из воздуха могут вступать в реакцию, образуя вещества, которые уже будут влиять.
Аноним 10/01/24 Срд 03:13:08 586667 730
Что можно делать с тяжелой водой? Кроме как налезать на бутылку
владимир 10/01/24 Срд 09:33:44 586670 731
1000015266.jpg 396Кб, 1080x2400
1080x2400
Какой радиус Шварценеггера для планеты Земля?

Каковы доказательства, что частицы попав в чёрную дыру, стремятся к центру, а не вечно движутся по окружности радиуса, в который попали?
Аноним 10/01/24 Срд 09:55:43 586671 732
>>586622
> Интеллект действительно начинает снижаться с 30 лет? Насколько сильно?
Вот этот >>586624 анон прав, вопрос в том, что считать интеллектом. Быстрое обучение - да, будет деградировать с возрастом. Но если считать интеллект как приспособляемость к изменениям, то тут уже не так однозначно. У взрослого есть опыт и знания, которые будут компенсировать способность к обучению, но это будет сильно зависеть от опыта и знаний конкретного индивида, от ширины когнитивной карты (картины мира) и гибкости мышления (способности к восприятию нового). И в таком понимании интеллекта (приспособление) он растёт (не у всех) до старческих естественных дегенеративных изменений в мозге, которые не всегда приходят раньше смерти.
> Можно ли отсрочить деградацию?
Занимайся умственным трудом, познавай, меняй деятельность, развивай гибкость ума и т.п. Всякие альцеймеры ебут работяг, всякие кризисы выпиливают их же.
Аноним 10/01/24 Срд 10:01:10 586673 733
Аноним 10/01/24 Срд 12:39:37 586674 734
Аноним 10/01/24 Срд 12:43:51 586675 735
>>586674
Ну разок накатил, за науку а потом что? Типа коллекционировать для внуков, профитно получится?
владимир 10/01/24 Срд 13:54:34 586677 736
>>586675
Можешь попробовать наладить плазменный реактор.
Аноним 10/01/24 Срд 14:01:53 586679 737
Аноним 11/01/24 Чтв 07:04:04 586696 738
В связи с последними событиями вопрос: а нахуя нужны общие городские котельные? Там же между котельной и потребителями в трубах наверняка огромные потери энергии. Разве не энергоэффективней каждому дому иметь свою мини-котельную? Или необходимость подвоза топлива к каждому хрущу перекроет любой выигрышь в энергии?
Аноним 11/01/24 Чтв 07:17:47 586697 739
>>586696
Люди долбоебы, пересрутся на вопросе как и кому сколько топить, куда дым несет и тд. Есть невидимое нечто дающее тепло и греющее пустоту (похуй) - все привыкли. Конечно топить индивидуально разумнее, но вот отказать от "услуги" тот еще квест.
Аноним 11/01/24 Чтв 07:49:14 586699 740
Привет, двач.

С чего начать изучение кибернетики? Может есть какие-то гайды? Типо "вот сначала эта книга, потом эта книга, потом вот это видео" и тд?

Немного посмотрел зарубежный ютуб на тему кибернетики, попадаются только рекламы института кибернетики в Австралии, а учебную программу не показывают.
Аноним 11/01/24 Чтв 07:57:30 586700 741
Вчера наткнулся на такой термин как CSE. Сначала подумал, что это Computer science engineering. Оказалось, что это computational science and engineering
https://en.wikipedia.org/wiki/Computational_engineering

Есть что-то годного на русском языке по теме? А на английском есть годнота для начинающих?
Как понимаю, то нужно неплохо знать матан на уровне бакалавра физмата, знать физику, программирование, CS (computer science) и несколько компьютерных программ?
Ничего не упустил?

С меня луч добра!
Аноним 11/01/24 Чтв 11:19:43 586704 742
>>586696
Не знаю, но нужны во всех странах Европы где зимой холодно тоже центральное отопление (за исключением Польши, которая пошла по второму пути и теперь там нечем дышать от гари в некоторых городах)
Аноним 11/01/24 Чтв 12:25:42 586706 743
>>584346 (OP)
Фоновые нейтрино влияют ли на бета-распад ядер?
Аноним 11/01/24 Чтв 12:44:51 586707 744
>>586706
Если не фоновые влияют, то и фоновые влияют
Аноним 11/01/24 Чтв 13:10:50 586708 745
>>586707
А не фоновые таки влияют?
Вроде нейтрино сильно влияют на синтез сверхтяжелых ядер при взрывах сверхновых или ядерными реакциями в белых карликах.
В любом случае фоновых нейтрино на десятки порядков больше, чем от всяких ядерных реакторов и прочего рукотворного говна, при этом мы не можем экранироваться от нейтрино как от фотонов, чтобы с чем сравнивать бета распады.
Аноним 11/01/24 Чтв 13:32:23 586709 746
>>586708
>Вроде нейтрино сильно влияют на синтез сверхтяжелых ядер при взрывах сверхновых
Смотря как считать
Аноним 11/01/24 Чтв 14:36:12 586711 747
>>586709
Давно все посчитали.
Ядра умирающих звезд непрозрачны для нейтрино, отчего в нем идут специфические реакции с рождением нейтронов(нейтрино встречается с кварков, меняет его аромат, а сам превращается в позитрон), которые захватывают ядрами на периферии, отчего они могут мутировать аж до урана. Если недра звезды достаточно плотные, начинается лавинный процесс нейтронизации вещества, где нейтрино выступают катализаторами.
Аноним 11/01/24 Чтв 14:39:28 586712 748
>>586711
Интересно, может ли быть такой поток нейтрино, который бы смог убить человека?
Аноним 11/01/24 Чтв 14:51:08 586713 749
>>586712
Вполне может, но ты быстрее помрешь от того, что его создает.
Аноним 11/01/24 Чтв 17:32:54 586717 750
>>586711
>Давно все посчитали.
Я другое имел ввиду и при другом процессе.
Если про ядро речь, и непрозрачно, то там похоже действительно где-то 1к1 должно получаться.
Вообще я не уверен что тот другой процесс вообще существует, а не просто популизаторы так криво и уёбищно про него рассказывают, что полностью теряется смысл в тексте о том что они вообще не о таком процессе говорят а о совсем другом.
Аноним 11/01/24 Чтв 21:39:15 586722 751
>>586712
Да, даже кто-то считал, нужно,чуть ли не впритык к сверхновой подойти в момент разгара пиздеца
Аноним 11/01/24 Чтв 23:26:57 586723 752
>>586711
Нейтрино без заряда попадает в протон и превращает его в нейтрон?

Ну и с чего это катализатор, если это тратящийся компонент реакции.
>начинается лавинный процесс нейтронизации вещества, где нейтрино выступают катализаторами.
Аноним 11/01/24 Чтв 23:54:13 586726 753
>>586722
Ну вот тот второй процесс о котором я говорил, типа популизаторы про него рассказывают
Типа поток нейтрино именно, а не нейтроны, создают во внешних слоях при взрывах звёзд расходящуюся ударную волну, и синтез идёт именно из-за давления, а не от потока нейтронов. При этом от этого якобы такое давление что выше чем в ядрах самых больших звёзд, и выше чем при любых взрывах, и это не в центре а во вшнешних слоях так.

Но чёт я так прикинул, сомнительно это всё, возможно популяризаторы просто дегенераты и не понимают о чём говорят.
Аноним 12/01/24 Птн 16:48:39 586750 754
>>586723
Нейтрино попадает в протон, тот превращается в нейтрон плюс позитрон.
Аноним 13/01/24 Суб 00:57:49 586758 755
Посоветуйте книгу по истории философии, плиз. Что-то годное, только не Бертрана Рассела.
Аноним 13/01/24 Суб 01:39:25 586761 756
>>586758
Критерии годности какие?
Аноним 13/01/24 Суб 14:04:24 586765 757
>>586761
Чтоб поменьше говниной воняло естественно
Аноним 13/01/24 Суб 22:05:02 586770 758
>>586765
Тогда тебе придется помыться перед чтением
Аноним 14/01/24 Вск 10:49:23 586787 759
Почему биоооужие невозможно?
Все сука стремится к равновесию дпже шмели в лабораториях
Аноним 14/01/24 Вск 12:39:39 586796 760
>>586787
Почему невозможно? Чем тебе вирусы не биооружие?
Аноним 14/01/24 Вск 13:08:33 586799 761
>>586796
Он имел ввиду биоружие из рассказах кремлевских старцев.
Типо берем зверушку, заклинаем ее бациллами, выпускаем к врагам. Зверушка контачит врагов, бациллы перекидываются на них и они от этого дохнут от болячек. Но раз бациллы заговоренные, то они не берут четких пацанов.

Из вирусов плохое оружие, ибо они эволюционно самообезвреживаются, приходя к устойчивому равновесию с популяцией.
Вирусы не избирательны и не дают надежных результат.
Очень высока вероятность, что применяющий первый пострадает от бяки, а цель вообще никак не пострадает.
Аноним 14/01/24 Вск 18:03:13 586806 762
Аноним 14/01/24 Вск 18:08:17 586807 763
>>586806
>фундаментальная наука
Бесплатно дала бомжу?
Аноним 14/01/24 Вск 18:53:30 586810 764
>>586806
Красиво стелет про склонность к онкологии определенных этнических групп в зависимости от мутации генома.
Аноним 14/01/24 Вск 19:19:51 586811 765
>>586806
> дочь Путина
У меня у одного отторжение слушать её хотя по сути к действиям отца она не имеет отношения?
Аноним 14/01/24 Вск 19:32:13 586812 766
>>586811
>отторжение
Отторжение, не отторжение, а работать будешь рабочая шлюха☝️
☝️☝️☝️
Аноним 14/01/24 Вск 21:30:11 586813 767
>>586811
Хз.
На фоне сына байдена там хотя бы есть что слушать.
Аноним 14/01/24 Вск 21:37:06 586815 768
>>586806
Странное у неё видение по поводу удорожания технологий.
Взять тот же ферментативный катализ, например.
До него получение некоторых веществ было не просто дорого а в промышленном объеме невозможно.
Да он стоит на каком-то этапе дороже, чем в средних веках заячьим пометом обмазаться, но он открывает перспективы, которые заячьему помету в принципе не доступны.
Плавает, короче, студентка.
Аноним 14/01/24 Вск 22:14:52 586818 769
Аноним 15/01/24 Пнд 14:37:01 586839 770
Почему живые существа имеют уникальный ДНК?
Если ДНК - это молекула, то они ведь должны быть идентичны друг другу, скажем как молекула воды или углекислого газа, разве нет?
Как тогда работает тест?
Аноним 15/01/24 Пнд 15:11:44 586840 771
>>586839
Мутация и эволюция же. Это классика! Это знать надо!
Аноним 15/01/24 Пнд 15:17:41 586841 772
>>586840
>Мутация
И всё равно не понятно. Если структуру ДНК изменить, то это уже будет не днк, правда ведь?
Или энта молекула может обрастать некоторыми уникальными свойствами? В чём они заключаются?
Аноним 15/01/24 Пнд 15:51:39 586845 773
>>586841
ДНК это про состав, а состав у них у всех примерно одинаков. Если структуру воды или углерода изменить, они перестанут быть?
Можешь считать ДНК видом молекул если тебе так проще.
Аноним 15/01/24 Пнд 15:56:07 586846 774
>>586841
Структура такова, что позволяет один из ее частей(азотистые основания) имеет произвольные дискретные вариации. Их можно в любом порядке располагать, это не изменить структуру ДНК.
Мутации это изменение закодированной последовательностей, на структуру это не влияет. Хотя ДНК имеет комплементарную пару и она чувствительна к последовательности, но биомашины автоматом фиксят это.
Аноним 15/01/24 Пнд 16:04:34 586847 775
>>586839
Если есть дерево, то любое дерево должно быть идентичны друг другу, разве нет? Иначе не дерево, правда ведь?
Аноним 15/01/24 Пнд 17:43:00 586850 776
>>586847
дерево - не молекула
Аноним 15/01/24 Пнд 22:49:04 586867 777
>>586839
Газ это ведь молекулы, хули тогда говорят что есть штук 50 разных газов в таблице Менделеева, они что, ебанутые? Раз газ это молекулы, то должен быть всего один газ!
Аноним 16/01/24 Втр 11:00:27 586886 778
>>586867
Газ - это состояние вещества.
Дезоксирибонуклеиновая кислота имеет молекулярную структуру, а это значит что взяв один образец у разных живых организмов, они должны 100% совпадать.
Мой глупый вопрос к тому, почему ДНК-тест называют ДНК тестом, если это по сути сравнение молекул воды или метана?
Что на самом деле тестируется, и на каком уровне? Ген, теломера, хромосома?
Аноним 16/01/24 Втр 12:11:04 586887 779
>>586886
Разрешите доебаться.
ДНКой чисто из определению называется любая молекула состоящая из сахара дезоксирибозы, кислотной функциональной группы(фосфорная) и нуклеотид - азотистые основания.
Так ДНК можно назвать целый класс молекул, который не встречаются в естественном виде, ибо кислотный остаток и азотистое основания можно заменить на другие, вариаций куча.
Аноним 16/01/24 Втр 13:35:02 586892 780
>>586886
> они должны 100% совпадать
ДНК это полимер, она собирается из нескольких мономеров (нуклеотидов), то в какой именно последовательности эти мономеры соединяются в полимер и кодирует генетическую информацию. У разных молекул днк на одном и том же участке могут быть разные последовательности нуклеотидов чем они и отличаются. Процес определения этой последовательности называют секвенированием.
Аноним 16/01/24 Втр 22:16:43 586897 781
>>586886
>Дезоксирибонуклеиновая кислота имеет молекулярную структуру
Чем это отличается от слова "газ"?
>они должны 100% совпадать.
То что какой-то шизик-долбаёб у себя в голове нафантазировал что что-то ему "должно", ну хули, наверное так оно и есть)) но чёт не совпадает.
Аноним 16/01/24 Втр 23:00:54 586902 782
>>586886
>почему ДНК-тест называют ДНК тестом
Потому что это днк-тест
Во-первых, тест газов вполне логично называть тестом газов. Тестируют ведь газ.
Во-вторых, днк разные, у тебя системная ошибка, ты не просто нафантазировал себе другое опредление у слова сам, вместо того чтобы разобраться что оно означает, а ты в принципе не понимаешь что бывают общие очень общие определения, которые большую разновидность всего разного собою описывают. А бывают специальные определения, которые только одну какую-то конкретную вещь означают.
Ты думаешь что все определения специальные, и описывают только одну какую-то конкретную вещь.
Аноним 17/01/24 Срд 03:03:57 586908 783
>>586655
А чё бы нет? По меркам вселенной она существует ничтожную долю времени от всего возможного.
Аноним 17/01/24 Срд 05:30:48 586911 784
>>586670
У тебя ж там результат на картинке.
Доказательств никаких нет, а движение к центру вообще как бред звучит.
Так-то в чёрной дыре растяжение пространства, а не просто движение. Плюс скорее всего это вообще не имеет смысла, такая концепция, т.к. время бесконечно замедленно, т.е. внутренностей у чёрной дыры нет и никакого движения к центру нет, потому что чёрные дыры одновременно и не существует и существуют при этом.
Аноним 17/01/24 Срд 14:22:49 586920 785
Посоветуйте, статьи/видосы/подкасты о проблеме демаркации, научности и истинности. Пока отобрал только сочинения отдельных авторов типа Поппера и Фейерабенда, но хотелось бы на общий анализ посмотреть и дополнительно узнать о истории смены этих критериев в научной среде.
Аноним 17/01/24 Срд 19:47:21 586930 786
>>586920
Курс философии науки на степике или типа того
Аноним 18/01/24 Чтв 22:01:41 586953 787
возможно ли предсказать через сколько десятилетий CRISPR-Cas9 будет способна лечить уже психические болезни?
Аноним 19/01/24 Птн 00:18:12 586954 788
>>586953
Ты трясущийся тупой тревожник? Или ещё каких-то "болезней" себе нафантазировал?
Аноним 19/01/24 Птн 09:10:58 586962 789
>>586954
Он указал криспс так что или ОКР или шизофрения. Это самые популярные психическое заболевания, которые зависят от генетики.
Аноним 19/01/24 Птн 13:54:31 586976 790
>>586962
Самые популярные выдуманные заболевания зависящие от генетики, лол.
Аноним 19/01/24 Птн 13:55:48 586977 791
>>586976
А ты никогда от ОКР не страдал по всей видимости. Ну, ничего, родился у тебя ребенок с шизофренией по другому заговоришь.
Аноним 19/01/24 Птн 14:02:56 586981 792
Аноним 19/01/24 Птн 15:33:41 586990 793
Да когда уже сингулярность блеать?!
Аноним 19/01/24 Птн 15:59:59 586994 794
>>586990
Это типа когда всё суперэффнктивно будут делать роботы и ии в неограниченных(условно) количествах, а таких как ты рабочих закинут в биореактор как бесполезный биомусор?
Аноним 19/01/24 Птн 16:13:14 586996 795
>>586994
>роботы и ии в неограниченных(условно) количествах
>а таких как ты рабочих закинут в биореактор как бесполезный биомусор
А зачем роботам делать что-то в неограниченных количествах, если это некому будет потреблять?
Аноним 19/01/24 Птн 16:50:55 587000 796
>>586996
>если это некому будет потреблять?
Ну смотри, сейчас ты тупорылый опущенный хуесос работаешь за околобесплатно, а потребители кроме тебя несмотря на это находятся.
Аноним 19/01/24 Птн 16:57:54 587003 797
>>587000
А кто сказал, что золотой миллиард тоже не покромсают?
Аноним 19/01/24 Птн 19:53:06 587028 798
>>586996
Роботы и станут новыми потребителями, а людишки станут нинужны.
Аноним 19/01/24 Птн 20:54:26 587029 799
>>587028
Кто роботам цены на электричество в розетке будет устанавливать? Нет, если в них проснётся квалиа, то они это поймут и без людей подключаться к солнцу, но пока ключи от Чернобыля у людей, роботам не о чем переживать
Аноним 20/01/24 Суб 00:26:50 587031 800
>>587029
Приказываешь своим роботам построить электростанцию. Профит.
Аноним 20/01/24 Суб 01:02:29 587036 801
image.png 1144Кб, 4963x3508
4963x3508
>>584346 (OP)
Как распределяется энергия в электросети?
Допустим есть разные потребители с разной мощностью на разном расстоянии от источника, подключенные ЛЭП с разным сопротивлением.
При недостатке энергии все будут получать одинаковое количество или одинаковый процент от мощности или на это как-то еще влияют параметры ЛЭП, подстанций, РУ?
При избытке энергии и наличии нескольких источников, все источники будут загружены одинаково или на одинаковый процент от мощности или на это как-то еще влияют параметры ЛЭП, подстанций, РУ?
Интересуюсь с целью симуляции энергосистемы в игре.
Аноним 20/01/24 Суб 08:58:55 587046 802
>>587036
На ютубе ищи "Электроликбез".
Аноним 20/01/24 Суб 15:55:54 587059 803
>>587036
Чтобы правильно рассчитывать сети тебе надо отдельную специальность закончить в институте.
Сам расчет относительно простой, но там есть нюансы с ситуациями, когда что-то выходит из строя.
Наверняка есть мод для факторио.
Аноним 21/01/24 Вск 12:00:12 587072 804
>>587046
Я бы предпочел информацию в текстовой форме
Аноним 22/01/24 Пнд 03:40:36 587096 805
У графита высокая проводимость. Много свободных электронов?
Тогда почему он не считается металлом? И проводимость есть, и свободных электронов много, блестит как металл
Похоже по всем параметрам на хрупкий металл без свойства "ковкость", ну хули, многие металлы не ковкие.

Выделяют ещё вроде какую-то особую металлическую связь, а у графита выделяют связь углерод-углерод, неужели это настолько разные вещи?
Аноним 22/01/24 Пнд 10:26:59 587099 806
>>587096
"Металл" это оценочная характеристика химического элемента. Графит не химический элемент, это модификация углерода.
Аноним 22/01/24 Пнд 14:07:59 587112 807
Почему нельзя сделать чисто оптические без электрики чернобелые очки?
Аноним 22/01/24 Пнд 15:04:35 587113 808
>>587096
Про металл уже сказали.
Ток могут проводит так же полупроводники и ионные жидкости.
Проводимость графита не описывается в рамках модели зонной проводимости, там уже нужные другие модели.
Металлическая связь это условность химиков. Как вообще любые химические связи, ибо они не хотят по честному считать волновую функцию всех электронов.

Если уж хочешь простое объяснение проводимости графита, то у него электроны образовывают ароматическую связь, которая при больших масштабах представляется как шатающихся заряд вдоль слоя.
Аноним 22/01/24 Пнд 15:37:11 587114 809
>>587112
> чернобелые очки
Если я правильно понял что ты подразумеваешь под "чернобелыми очками" то нельзя потому что не нашли какого-то физического процеса который напрямую позволяет уровнять мощность излучения по всем длинам волн. Тебе нужна какая-то хуйня которая под каждую длину волны будет "смотреть" на ее мощность и в зависимости от этого либо усилять либо уменьшать чтобы все усреднить. Это очень спецефическое воздействие.
Аноним 22/01/24 Пнд 16:43:37 587116 810
Детский вопрос к биологам. Наверное платина. Раки и крабы чувствуют боль когда их варят в кипятке ? А когда крабам отрывают конечности ? Забавно что в моём детстве всегда про раков спрашивали, а сейчас чаще про крабов.
Аноним 22/01/24 Пнд 17:17:40 587120 811
>>587113
>
>Если уж хочешь простое объяснение проводимости графита, то у него электроны образовывают ароматическую связь, которая при больших масштабах представляется как шатающихся заряд вдоль слоя.
Т.е. свободных электронов как в металлах нет? Класс, спс.
Аноним 22/01/24 Пнд 18:20:13 587121 812
>>587116
Не так давно признали у членистоногих сознание, т.е. наличие чувств. Теперь их положено убивать, прежде чем кидать в кипяток.
Существует ли время Аноним 23/01/24 Втр 00:12:24 587135 813
Суп срач. Неофит итт. Насмотрелся научпоп видосов, и нейроны в мозгу зашевелились. Вот стало интересно. Постараюсь изложить тезисно. Если в тезисах заблуждаюсь - поправь.

Время - это энтропия. А точнее, это однонаправленное движение энергии в пространстве.
На время влияет скорость и Гравитация. Если тело ускоряется, то время для него замедляется. Также, чем больше массы у тела, тем медленнее для него течет время.

Тело обладающее массой не может достичь скорости света, т.к. для этого нужно бесконечное количество энергии.

Учитывая все вышесказанное, можно ли заключить, что время "замедляется" потому, что, например, электронами на орбитах атомов становится "тяжелее" вращаться вокруг орбит. Поэтому замедляются все процессы в микромире, а в следствии этого и в макромире?

Получается ли из этого, что время не вещь в себе, а лишь следствие "перемещения" энергии в пространстве?
Аноним 23/01/24 Втр 00:43:35 587136 814
>>587135
>Время - это энтропия.
Время это время, энтропия это энтропия. Разные категории, связать у тебя их не выйдет.
>Учитывая все вышесказанное, можно ли заключить, что время "замедляется" потому, что, например, электронами на орбитах атомов становится "тяжелее" вращаться вокруг орбит. Поэтому замедляются все процессы в микромире, а в следствии этого и в макромире?
Дефайн "тяжелее"?
>Получается ли из этого, что время не вещь в себе, а лишь следствие "перемещения" энергии в пространстве?
А пространство это вещь в себе? А энергия это вещь в себе?

Прекращай смотреть научпоп. Лучше потрать лет 5 на обучение, тогда эти вопросы сможешь уже рассмотреть получше. Научпоп делает тебя глупее
Аноним 23/01/24 Втр 00:47:13 587137 815
>>587135
Есть много эффектов, и слишком уж одинаково всё замедляется, такое ощущение что.. ну это довольно существенный намёк что это всё-таки какая-то вещь в себе.
Ну понял да, если бы был только один какой-то эффект и механизм "замедления", то логично что всегда одинаково всё замедляется. Но там вариаций много, и вроде как всё абсолютно одинаково замедляется, а когда много разных и всё одинаково, это уже странно.
Аноним 23/01/24 Втр 02:34:39 587143 816
Сап двач. Насмотрелся видосов по микробиологии, возник вопрос. В каждой клетке человека в ядре содержится весь геном. В процессе митоза дублируется весь геном. А как тогда клетка узнает, что ей необходимо разделиться на идентичную клетку, а не на любую другую рандомную клетку? Или процесс деления каждой разновидности клеток создан так, что ни на что другое она поделиться не способна?
Аноним 23/01/24 Втр 04:30:40 587145 817
>>587143
Клетка собирает сигналы из окружающей среды. Химические, электрические. На их основе решает куда двигаться и когда в кого делиться.
Некоторые колонии бактерий тоже обмениваются сигналами, они как разреженное многоклеточное.
Аноним 23/01/24 Втр 10:35:01 587150 818
>>587135
>Получается ли из этого, что время не вещь в себе, а лишь следствие "перемещения" энергии в пространстве?
Время это отражение течения процессов (изменений) окружающего мира в нервной системе организма. Наблюдаемые цикличные процессы (оборот земли, оборот вокруг солнца) человек абстрагирует до понятия время (день, год).
Да, время это не вещь_в_себе, время это следствие фиксации (наблюдения) процесса (изменений, событий) другим процессом.

>Учитывая все вышесказанное, можно ли заключить, что время "замедляется" потому, что, например, электронами на орбитах атомов становится "тяжелее" вращаться вокруг орбит. Поэтому замедляются все процессы в микромире, а в следствии этого и в макромире?
Не готов утверждать, что именно и на каком уровне замедляется. Но в общем примерно так и есть. Скорость и гравитация замедляют процессы, поэтому в «парадоксе близнецов» путешественник по возвращении попадает «типа» в будущее, а часы на спутниках приходится корректировать.
Аноним 23/01/24 Втр 10:52:54 587151 819
Снимок.PNG 28Кб, 1119x408
1119x408
А существует ли вообще пример такой бесконечной последовательности целых чисел f(n) для которой доказана конечность количества простых чисел в ней?
Аноним 23/01/24 Втр 11:49:04 587152 820
>>587151
f(n) = 2 * n (n - натуральное число) например
Аноним 23/01/24 Втр 12:20:49 587153 821
1706001636268.jpg 69Кб, 600x600
600x600
Аноним 23/01/24 Втр 13:57:23 587154 822
>>587152
Ну ладно. формально есть один пример. Просто интересует что-то не такое тривиальное - дохулион простых чисел в некой последовательности, а потом раз и после N-го номера ни одного больше не попадётся и это доказано.
Аноним 23/01/24 Втр 14:14:55 587156 823
>>587154
> Ну ладно. формально есть один пример.
Какой пример? Он привёл последовательность чётных чисел. В ней вообще нет ни одного простого числа.
Аноним 23/01/24 Втр 14:25:27 587157 824
Аноним 23/01/24 Втр 14:27:24 587158 825
>>587157
Ах ты бля. Ok. В натуре.
Аноним 23/01/24 Втр 16:18:15 587160 826
>>587158
Я тут мимо проходил и хотел просто спросить у тебя один вопрос -
Ты такое тупорыло чмо, почему ты вообще считаешь что имеешь право разевать свою тупорылую ротешу и чего то там из нее кукарекать на людей заведомо умнее тебя?
Аноним 23/01/24 Втр 16:19:28 587161 827
Аноним 23/01/24 Втр 21:21:14 587174 828
Реально провести "генный реверс-инженеринг" птиц и вырастить жизнеспособного цыплёнка курицы, например, без перьев, но в чешуе? С зубастой пастью вместо клюва? С длинным хвостом?
Почему так не делают? Уверен, домашние динозавры будут пользоваться огромным спросом независимо от цены. Пусть будут стерильными, если зоошизики боятся, что они сбегут и расплодятся.
Аноним 24/01/24 Срд 00:02:33 587180 829
>>587136
"Тяжелее", значит, что скорость света константное, и чем ближе к ней приближается тело, тем больше энергии требуется для любого движения.
>>587150
Просто, если это так, то почему об этом научпоперы не говорят? Я такие тезисы никогда не слышал. Получается и путешествия во времени тогда невозможны, как невозможно провернуть фарш назад.
Помощь отечественной науке Аноним 24/01/24 Срд 00:13:09 587181 830
image.png 123Кб, 274x184
274x184
Такое дело.

Есть желание оказать посильную помощь единственной достойной сфере человеческой жизнедеятельности - науке, желательно отечественной.

Сам умом я, как вы понимаете, не вышел, так что непосредственное участие исключается.

Но могу, например, донатить какую-то не слишком крупную сумму денег, или отдать пека под вычисления.

Я погуглил на эту тему, и чёт ничё вменяемого не нашёл.
Из тех проектов, где нужна помощь волонтёров, гуглятся только всякие натуралистические проекты по каталогизации живой природы, а беготня с сачком за бабочками и сбор гербария мне совсем не привлекают.

У меня больше стоит на разного рода технику, физику, космос, вот это всё.

По донатам тоже ничё не понятно.

Может подскажете приличные варианты?
Либо донаты (но только так, чтобы они не осели в карманах мойшенников), либо какая-то условно-пассивная натуральная помощь вроде выделения машинного времени.
Аноним 24/01/24 Срд 00:19:35 587182 831
>>587174
Нет, мы ещё очень далеко от этого.
Аноним 24/01/24 Срд 10:48:01 587187 832
Я уже задавал здесь этот вопрос, никто не смог ответить. Задам ещё раз.
Когда проводят соцопросы, то в норме до 80-90% респондентов либо отказываются отвечать на вопросы, либо отвечают не на все (например не о себе), и их ответы не засчитываются. Нвероятно, но людям нахуй это не надо.
Вопрос: каким, блядь, образом, социологи претендуют на отражение "общественного мнения", если с ними никто, сука, не разговаривает?
Ну нет у них репрезентативной выборки, у них выборка на 100% состоит их странных людей с отклоняющимся поведением.
Аноним 24/01/24 Срд 10:54:18 587188 833
>>587187
Я более того скажу, большинство соцопросов до сих пор это телефонные обзвоны. Ну, ты знаешь, когда тебе звонят незнакомые люди и такие, это соцопрос.

И говорит какая-нибудь старушка, либо женщина с не очень приятным голосом. Можете ли вы уделить 15 минут своего времени, чтобы ответить на вопросы, про то, какие последние телеканалы вы смотрели. И если твой ответ, их не удовлетворяет, то есть ты вместо того, чтобы говорить конкретно заданные фразы. Говоришь, я не помню, тебя перебивают и говорят, отвечайте строго по тому сценарию, который мы вам проговорили.
Аноним 24/01/24 Срд 12:43:32 587190 834
>>587187
> каким, блядь, образом, социологи претендуют на отражение "общественного мнения"
Какое есть, другого не подвезли. Ты как ребёнок, который узнал, что мистер пропер не прилетает и не делает уборку, да и само СУПЕРсредство ни чем не отличается от дешевого, только в красивой таре и с вкусной отдушкой.
А вообще, понятие «общественное мнение» и не подразумевает 100% опроса, это мнение ОБЩНОСТИ, т.е. достаточно опросить часть, чтобы иметь представление об ОБЩЕМ состоянии дел.
Аноним 24/01/24 Срд 13:28:12 587191 835
>>587187
Так ведь соцопросы по сути опрос на лояльность ангажированной группы, а не что-то уровня маркетинговых исследований. Ценность не в их результатах, а в том факте что они проводятся и для кого. Этакий сорт оф ритуал.
Аноним 24/01/24 Срд 14:31:55 587193 836
>>587190
Ты не прав. Чтобы иметь представление об общем состоянии дел нужно спросить часть общества это верно, но окрашиваемая часть общества должна состоять из всех групп населения, а не исключительно шизы готовые говорить про войну по телефону с незнакомым человеком.
>>587191
А как же "независимые" соц. опросы?
Аноним 24/01/24 Срд 15:54:45 587194 837
>>587193
>А как же "независимые" соц. опросы?
Тоже самое. Их "независимость" только в другой целевой группе.
Аноним 24/01/24 Срд 17:32:19 587196 838
>>587193
> а не исключительно шизы готовые говорить про войну по телефону с незнакомым человеком.
Я же тебе написал: что имеем, то имеем.
Что ты предлагаешь? Запретить опросы. Или. Законодательно принудить граждан обязательно проходить опросы под детектор лжи (а то шутники будут искажать данные и тебе опять не понравится), с уголовной ответственностью за отказ и за ложь.
К чему твой срыв покровов?
Аноним 25/01/24 Чтв 15:37:05 587219 839
🌑 Невероятный ф[...].mp4 4147Кб, 720x1280, 00:00:38
720x1280
У кого спираль есть? Сделайте так же.
Аноним 25/01/24 Чтв 19:49:00 587232 840
>>587219
Как мне казалось.
В принципе это всё, что вам необходимо знать о современном образовании инженеров.
Аноним 25/01/24 Чтв 19:49:59 587233 841
>>587219
Я так тоже показал однажды один эффект одному доктору технических наук.
Дословно он сказал - этого не может быть, потому что Я не понимаю как это работает.
Аноним 25/01/24 Чтв 19:52:25 587234 842
Аноним 25/01/24 Чтв 20:52:02 587242 843
>>587232
>>587233
Тупые дауны/шизы/фантазёры
Он потом сам про леску говорит
Ахаха, как же много этот чувак долба6ов словил этим приёмом.
Аноним 25/01/24 Чтв 22:22:44 587245 844
>>587242
На 11 секунде леску видно
Аноним 26/01/24 Птн 01:46:25 587250 845
Почему индусня постоянное кей-кает? Они сами не понимают что это пиздец как нелепо и раздражает?
Аноним 26/01/24 Птн 09:05:51 587255 846
>>587242
Речь вообще не об опыте шла, а о его пиздобольстве.
Аноним 27/01/24 Суб 05:06:08 587276 847
Поясните за автопилот, особенно фирму Ольги Усаковой - Cognitive technology.

Неужели настолько крутая, что аж сам Илон Маск с его автопилотом курит нервно в сторонке?
Аноним 27/01/24 Суб 13:03:56 587286 848
Аноним 27/01/24 Суб 13:05:20 587288 849
>>587276
Звучит как хуйня какаето,и автопилот чего¿
Транспорта?
Аноним 28/01/24 Вск 17:23:54 587317 850
JstgP5U6kKg.jpg 135Кб, 720x720
720x720
1. Дети проводят за видеоиграми, изображающими убийства, сотни часов в год. Годами сидят в виртуальных коробках Скиннера и получают поощрения за имитацию убийств, получают удовольствие от изображений агонии, крови, оторванных конечностей. Года за годом, с раннего детства.
2. Никакого влияния такого времяпрепровождения на их психику наука не фиксирует.
Внимание вопрос: разве отсутствие данных само по себе не говорит нам о чём-то важном? Ну например либо социальная "наука" - бесполезное говно, либо тысячи часов научения и цмилилоны циклов подкрепления ни на что влияют.
Аноним 28/01/24 Вск 17:40:39 587318 851
>>587317
1. Волки всю жизнь убивают живых существ
2. Волки всю жизнь играют друг с другом имитируя укусы

Даже волки способны отличать игру от жизни, с чего люди бы не могли?
Аноним 28/01/24 Вск 17:47:49 587319 852
>>587318
Видеоигры это не игровое поведение животных и людей, вообще ни разу, держу в курсе.
Аноним 28/01/24 Вск 19:49:40 587321 853
>>587319
Уже давно всё исследовано, и видеогры не повышают насилие, а в некоторых случаях даже понижают, потому что позволяют выплеснуть агрессию в игре вместо реального мира.
Аноним 28/01/24 Вск 20:04:58 587323 854
>>587321
>видеогры не повышают насилие
1. Вопрос был не про "повышение насилия", а про их воздействие на психику детей.
2. Доказательств, что видеоигры "не повышают насилие" - нет. Удивительно, понимаю.
Аноним 28/01/24 Вск 21:33:03 587324 855
>>587317
>>587318
Многомерные внематериальные сущности гаввах качают и им не важно игра или не игра. Важно состояние ума и так сказать у многих геймеров бывает сильно бомбит не хуже "реального" убийства.
Аноним 28/01/24 Вск 21:54:01 587326 856
>>587317
>Никакого влияния такого времяпрепровождения на их психику наука не фиксирует.
Конечно не фиксирует, так как получается нормальная лысая агрессивная обезьяна. В своём становлении молодая особь проходит необходимый этап древней биопрошивки подавлять другие виды и доминировать в стае себеподобных, оттуда идет детская и подростковая жестокость, довидеоигровую эпоху детишки и подростки ходили на стрелки дворами, пиздились в шкалке, вешали в подвалах котят, резали и надували лягушек на болотах, отрывали тараканам ноги и так далее, не повсеместно, не все, но это всегда было свойственно части детенышей человека разумного, всё это видимо нужно для деконструкции угроз животного окружающего созревающей психикой и нечто вроде такого, хз, я не психолог и вообще не гуманитарий, но давно задумываля об этом. А ты задумайся вместе с психологами и социологами, что если человека с детства изолировать не только от насилия, но и от любой информации о насилии в отношении своего и других видов, включая всякие литературные произведения, какой получится человек?
Аноним 28/01/24 Вск 22:20:45 587327 857
>>587326
Дети не надувают лягушек по два часа в день каждый день на протяжении 10 лет подряд.
И видеоигры это не агрессивное поведение. Это обучение. Картинка - действие - поощрение.
Мне предлагается поверить, что ребёнок тратит тысячи часов на обучение в коробке Скиннера и это никак на него не влияет.
Аноним 28/01/24 Вск 22:59:17 587329 858
>>587327
Почему не влияет? Влияет, просто не так, как ты думаешь. Ребенок узнает, что на WASD обычно передвижение в играх. Узнает, что на букву R – перезарядка оружия, а на букву G – бросок гранаты. И тому подобные вещи. Он учится объединять движения левой и правой руки для достижения нацеливания на противника. В общем, он лучше начинает играть в игры. Из-за того, что он играет в игры. Ребенок знает чем НПЦ отличается от человека и почему убивать НПЦ можно, а людей нет. Потому что он понимает различия между реальной жизнью и игрой. Если же не различает, то это болезнь, но с таким же успехом он может не отличать книжный мир от реального и верить, что если спрыгнуть с крыши на метле, то он полетит.
Аноним 28/01/24 Вск 23:27:23 587331 859
>>587329
Ребенок учится играть в шутеры за 10 часов.
Остальные 8000 часов никак на него не влияют? Миллионы повторений цикла стимул - действие - поощрение - и ничего.
Это пиздец какое мощное допущение, практически фантастика.
Аноним 28/01/24 Вск 23:33:31 587332 860
>>587331
Вызывает зависимость, да. Сильную ли? Нет, не сильную. Хотя некоторые люди более склонны именно к игровой зависимости, но из моего обывательского опыта таких меньшинство.
Аноним 29/01/24 Пнд 00:36:58 587334 861
>>587332
Очередной гениальный тейк. Дети тратят тысячи часов чтобы сидеть на месте, пыриться в экран и постоянно повторять однотипные действия. Это зависимость! Невероятно.
Аноним 29/01/24 Пнд 00:41:48 587335 862
>>587334
Ты отрицаешь существование игровой зависимости?
Аноним 29/01/24 Пнд 06:10:07 587340 863
>>587331
>Остальные 8000 часов никак на него не влияют?
Первые 4000 приобретаются полезные для игры и для жизни навыки. Последние 4000 теряются полезные для жизни но не для игры навыки.
Аноним 29/01/24 Пнд 14:09:16 587344 864
Почему птиц на проводах не убивает ток??? я в курсе, что он убьет если наземное животное притронется и создаст разнсоть потенциалов, которая будет идти черещ него. Но ведь когда птица сидит на проводе, ток все равно ВХОДИТ в нее через лапки, просто никуда не идет дальше для прожарки. Так почему не убивает
Аноним 29/01/24 Пнд 14:41:38 587345 865
>>587344
Он не входит. У птицы нет потенциала, чтобы в неё шел ток, так что он спокойно себе движется в направлении потенциала на другом конце провода.
Аноним 29/01/24 Пнд 14:49:47 587346 866
>>587345
А если человек повиснет на проводе ЛЭП, как на турнике – его убьёт?
Аноним 29/01/24 Пнд 14:56:43 587347 867
>>587331
Чел, твои вопросы ни о чем. У тебя вся модель неправильная - с какими-то адовыми детьми-психопатами, которые бесконечно гоняют в человеконенавистнические стрелялки.

Во-первых, процент людей, предпочитающих исключительно один жанр компьютерных игр, невероятно мал. Это характерно только для нишевых вещей типа симуляторов поездов и проч., которыми увлекаются лютые аутисты и машинисты на пенсии. Как правило, люди играют в игры нескольких различных жанров, среди которых шутеры занимают очень скромное место.

Во-вторых, 70% игроков вообще никогда не играют в шутеры. Из тех 30%, которые в них регулярно играют, только несколько процентов особо упорных адреналиновых маньяков набрали к совершеннолетию целых 8000 часов игры. Остальные чуть-чуть побегали по локациям в период с 12 до 17 лет - и, немного повзрослев, завязали с этим низкопробным собачьим кайфом.

В-третьих, шутеры-симуляторы - то есть ебучие имитаторы жизни солдата-спецагента-маньяка-насильника, приближенные к реальности с ее солдафонством, гноем и окопным говном - вообще никогда не взлетают. Потому что на самом деле людям очень неприятен подлинный реализм. Популярные шутеры - это тщательно спроектированные и подсвеченные в нужных местах ландшафты и коридорчики с визуально-акустической эпилептической гиперстимуляцией и псевдофизикой, в которых все красочно взрывается, пердит, булькает и разбрызгивает по сторонам невозможные ведра кровищи. То есть популярные шутеры - это МУЛЬТИКИ, они намеренно абстрагированы до уровня схем, в которых одни моменты реальности подчеркнуты, а другие нарочно скрыты.

В-четвертых, целью современных игр является не дрессировка с вознаграждением, а вовлечение игрока в состояние потока, которое отличается от мучений голубя в ящике Скиннера примерно так же, как палец отличается от жопы. В учебных курсах типа "Game development essentials" все это подробно объясняется в главах про игровую механику. В состоянии потока человек не воспринимает реальности, он в полубессознательном состоянии движется по градиенту возрастания комплексности игровых паттернов, подбирая ключики к постоянно меняющимся и усложняющимся замочкам, постоянно приобретая все новые и новые способности и альтернативные возможности для осуществления свободного выбора. Как шахматист видит не конкретные фигуры на доске, а динамические паттерны угроз-возможностей и потенциальные ветви дерева игры - так и рассекающий на части и нагибающий врагов игрок видит только абстрактные динамические игровые паттерны, а не "существ", которых нужно "убить".

Короче, для начала подучи матчасть и только потом задавай вопросы.
Аноним 29/01/24 Пнд 15:02:20 587348 868
>>587346
С ним ничего не будет пока он не коснется земли. У земли больше потенциал чем у чего бы ни было ещё, так что ток пойдет через человека к большему потенциалу (к земле) и человек сгорит.

Думаешь почему требование при работе с электротехникой стоять на резиновом коврике? Чтобы ты для тока был неинтересен, так как стоишь на материале, который изолирует тебя от земли, так как не имеет токопроводности.
Аноним 29/01/24 Пнд 15:28:28 587349 869
moke-working.gif 1922Кб, 250x172
250x172
Привет анон! Как называется использование эволюционных рефлексов/механизмов организма себе на пользу?

По типу умывания холодной водой нырятельный рефлекс млекопитающего, смотреть на солнце 1-2 минуты утром как проснешься, питаться раз в сутки для мобилизации сил организма, не светить себе синим светом в ебало перед сном, и тд.

Слышал у Хьюбермана на подскате, что то типа "Evolutionary neuromethods" или что то такое.
Аноним 29/01/24 Пнд 16:18:46 587351 870
>>587349
Термин, который вы описываете и который упоминался Хьюберманом, скорее всего относится к "эволюционным методам нейрохакинга" или "эволюционным нейрометодам". Этот подход включает использование понимания эволюционно сложившихся рефлексов и механизмов нашего тела для улучшения здоровья, благополучия или повышения эффективности. Примеры, которые вы привели, как холодные ванны для стимуляции нырятельного рефлекса или контроль светового режима для регулирования циркадных ритмов, являются типичными примерами этого подхода.

Эти методы часто основываются на идее, что, воспроизводя условия, в которых эволюционировал человек, мы можем стимулировать организм функционировать более эффективно и здорово. Это включает в себя различные аспекты: от питания и физической активности до сна и светового воздействия.
Аноним 29/01/24 Пнд 16:40:39 587355 871
>>587345
🤯
а полчему не входит? у живых существ же есть электричество внутри
Аноним 29/01/24 Пнд 16:50:31 587356 872
>>587355
Представь себе, что ты электрический ток. У тебя есть выбор пойти по поводу у которого сопротивление 0,01 Ом или по человеческому телу, у которого сопротивление 500 Ом. Куда как ты думаешь пойдет 99,9 % тока? Правильно по поводу. У резинового коврика сопротивление вообще 9999999999 Ом, в его сторону пойдет ещё меньше тока.
Аноним 29/01/24 Пнд 16:57:08 587357 873
>>587348
Кстати, в гуро-тредах бывали видосы, как зацеперов хуячило током без контакта с проводом.Типа электрический пробой.
Аноним 29/01/24 Пнд 16:59:09 587358 874
>>587351

Не оно но близко. Спасибо гопота.
Аноним 29/01/24 Пнд 18:20:25 587362 875
>>587347
> Популярные шутеры - это тщательно спроектированные и подсвеченные в нужных местах ландшафты и коридорчики с визуально-акустической эпилептической гиперстимуляцией и псевдофизикой, в которых все красочно взрывается, пердит, булькает и разбрызгивает по сторонам невозможные ведра кровищи. То есть популярные шутеры - это МУЛЬТИКИ, они намеренно абстрагированы до уровня схем, в которых одни моменты реальности подчеркнуты, а другие нарочно скрыты.
В такие шутеры действительно много не наиграешь. Залипают в сетевые пабги, контрстрайки, гта и т.п. вот там реально сутками сидеть.
Аноним 29/01/24 Пнд 18:23:14 587363 876
>>587362
> В такие шутеры действительно много не наиграешь.
В юности зависал в Quake и Unreal. И не один я. Так что не гони
Аноним 29/01/24 Пнд 18:32:35 587365 877
>>587363
Тоже залипал, не спорю. Но тогда не было альтернативы. Сейчас в офлайн игры залипать ну такое себе, онлайн на порядок интересней.
Аноним 29/01/24 Пнд 20:40:01 587369 878
>>587347
>целью современных игр является не дрессировка с вознаграждением
>для начала подучи матчасть
Хуйчасть, придурок. Откуда вы лезете, блядь.
Аноним 29/01/24 Пнд 21:21:03 587372 879
>>587365
Смотря для кого. Я пробовал онлайн, и максимум играю в них как в оффлайн. Взять диабло, он для меня от сингла отличался только возможностью разменов шмота. А так ну играешь и играешь, гавкаешь в чат сокращения на игровом сленге как дрессированный обезьян и всё. Это нахуй ничего не значит. Что-то "околоспортивное" кс там или старкрафт, на это нужен талант и постоянный наигрышь часов, а в остатке только нервы и мозгоебизм. Мне сюжетки, песочницы, и метроидвании в разы больше нравятся любого сетевого дрочева.
Аноним 30/01/24 Втр 05:04:53 587384 880
Белый самец 100iq спаривается с белой самкой 100iq, у них рождается ребёнок 100iq, верно? Теперь белый самец 100iq спаривается с чёрной самкой 100iq - будет ли у ребёнка iq меньше?
Аноним 30/01/24 Втр 05:34:54 587385 881
>>587384
Ты не учёл генетический параметр "вероятность появления у особи потомства с iq размером Х"

Узнать его можно например по предкам.
Если в роду все были дауны 70-90 iq, например 10 поколений, и у них вдруг родился ребёнок 120iq, то у этого ребёнка конечно будет повыше шанс появление ребёнка с iq выше чем 70-90, чем если бы был род 10 поколений с iq70-90 и его 11-й представитель завёл бы ребёнка.
Но например скорее всего эта вероятность намного меньше чем если был род 10 поколений 120iq, у них родился вдруг 80iq дебил и чтобы у этого 80iq дебила родился ребёнок 110-120iq. Это более вероятно будет намного мне кажется.

Так что важен РОД.

А конкретно по поводу твоего вопроса-я ни разу не встречал информации, которая бы указывала что метисы получаются какими-то особо тупыми по сравнению с родителями, если конечно учесть род, ну или просто большую выборку усрелняющую.
Так что да, В СРЕДНЕМ, 100+100 будет давать 100, и не важно метис или белый+белый или чёрный+чёрный.
Аноним 30/01/24 Втр 08:45:29 587386 882
>>587384
Из-за концентрации содержания мелатонина в коже IQ будет ниже? Чего блять?
Аноним 30/01/24 Втр 09:33:25 587391 883
>>587386
В целом такова статистика.
>в целом
Если не смотреть про что конкретно он спрашивал.
Аноним 30/01/24 Втр 09:34:27 587392 884
>>587384
>https://en.wikipedia.org/wiki/Regression_toward_the_mean
>The concept of regression comes from genetics and was popularized by Sir Francis Galton during the late 19th century with the publication of Regression towards mediocrity in hereditary stature.[8] Galton observed that extreme characteristics (e.g., height) in parents are not passed on completely to their offspring. Rather, the characteristics in the offspring regress toward a mediocre point (a point which has since been identified as the mean). By measuring the heights of hundreds of people, he was able to quantify regression to the mean, and estimate the size of the effect. Galton wrote that, "the average regression of the offspring is a constant fraction of their respective mid-parental deviations"
Как мы помним, Гальтон - нацист, так что IQ будет таким же. И вообще IQ не работает. Генетика - лженаука. В современном мире главное - эмоциональный интеллект! Нацизм - не пройдёт. Я люблю негров!
Аноним 30/01/24 Втр 11:31:29 587396 885
>>587384
>Теперь белый самец 100iq спаривается с чёрной самкой 100iq - будет ли у ребёнка iq меньше?
Неизвестно, их iq может быть закодировано разными генами. Ребенок может родиться с как меньшим, так и с большим iq.
Аноним 30/01/24 Втр 11:47:31 587397 886
>>587396
>их iq может быть закодировано разными генами
Смешивалка же смешивает рандомно, а интеллект наверняка кодирует очень большое количество генов.
Т.е. с высокой вероятностью мозг будет очень сильно перемешен до равномерности.

Хотя вот конечно бывают что дети прям на 100% вылетый один из родителей, бывает почти на 100% другой, ну и соответственно 50 на 50 и другие соотношения.
Хрен короче знает как эта смашивалка работает.
Аноним 30/01/24 Втр 12:14:23 587401 887
>>587397
> Хрен короче знает как эта смашивалка работает.
Ни как не работает. Генетическая смешивалка работает только для темперамента, который лишь косвенно и незначительно влияет на интеллект, а интеллект определяет социокультура. Для нормы, без учета болезней типа аутизма и т.п.
Аноним 30/01/24 Втр 12:47:13 587404 888
>>587401
Я думаю они не просто так говорят конкретно про IQ в сообщениях выше. Интеллект что такое до сих пор срачи ходят. А то что уровень IQ коррелирован по наследству с уровнем 0,8 это проверенный факт.

Другое дело, что IQ вообще не имеет отношения к оценке уровня интеллекта, так как спроектирован давать ответ на вопрос есть ли проблемы с интеллектом, а не для того, чтобы сравнивать интеллект двух индивидов.
Аноним 30/01/24 Втр 13:09:32 587406 889
>>587404
> А то что уровень IQ коррелирован по наследству с уровнем 0,8 это проверенный факт.
Удивительно, не может такого быть. Родители передают не только гены, но и воспитанием передают социокультуру. Вот почему-то детям маугли IQ генами ни как не передаются. Магическое исключение.
Аноним 30/01/24 Втр 13:44:30 587407 890
>>587406
Это учитывают исследования. Коэффициент корреляции до 0,8 между близнецами выращенными в разных условиях. Хотя, я повторюсь ещё раз, IQ не оценивает интеллект.

> Да, этот вывод о влиянии генетики на интеллект основан на исследованиях близнецов. В этих исследованиях анализируется различие в интеллектуальных способностях между однояйцевыми и разнояйцевыми близнецами. Однояйцевые близнецы генетически идентичны, в то время как разнояйцевые близнецы, как и обычные братья и сестры, имеют примерно 50% общих генов.

> Исследования показывают, что когда интеллектуальные различия между однояйцевыми близнецами, выращенными в разных условиях, меньше, чем между разнояйцевыми, это указывает на значительное генетическое влияние на интеллект. Оценки генетического влияния (известные как «наследуемость») на интеллект варьируются от 50% до 80%, что означает, что эта доля вариабельности интеллекта в популяции может быть объяснена генетическими различиями.

> Эти исследования являются частью области бихевиористской генетики и помогают понять, как взаимодействуют генетические и окружающие факторы в развитии когнитивных способностей. Однако важно отметить, что наследуемость относится к вариации в группе, а не к индивидуальным способностям, и что окружающая среда также играет значительную роль в развитии интеллекта.
Аноним 30/01/24 Втр 13:53:53 587408 891
>>587407
Дай ссылку на исследование, я вот вижу обратные результаты

> Тесты показали, что IQ приемной близняшки на 16 пунктов ниже, чем у сестры, хотя обычно однояйцевые близнецы показывают разницу в IQ не более семи баллов. Хотя ученые до конца не уверены, что вызвало такую резкую разницу в IQ, в исследовании отмечается, что у приемной девочки в детстве наблюдалось больше проблем в семье.
Аноним 30/01/24 Втр 14:24:25 587409 892
>>587408
Вот несколько исследований, которые изучают влияние генетики на интеллект на основе исследований близнецов:

1. Исследование [Hiroto Takahashi et al.](https://dx.doi.org/10.1017/thg.2022.10) показало, что однояйцевые близнецы более схожи в плане поведенческих черт, включая интеллект, по сравнению с двуяйцевыми близнецами, что указывает на влияние генетики.

2. В мета-анализе [S. Sniekers et al.](https://dx.doi.org/10.1038/ng.3869), включающем 78,308 человек, было идентифицировано 336 SNP и 22 гена, влияющих на интеллект, 11 из которых были новыми находками.

3. Исследование [S. Franić et al.](https://dx.doi.org/10.1017/thg.2014.26) показало, что фенотипическая стабильность вербального, невербального и общего интеллекта в детстве и подростковом возрасте в основном обусловлена высокой стабильностью аддитивных генетических факторов.

4. [A. Vinkhuyzen et al.](https://dx.doi.org/10.1007/s10519-011-9507-9) обнаружили, что влияние генетики на интеллект в зрелом возрасте оценивается в 44% из-за аддитивных генетических факторов, 27% из-за неаддитивных генетических факторов и 11% из-за ассортативного скрещивания.

Эти исследования подчеркивают важность генетических факторов в развитии интеллекта, но также подчеркивают роль окружающей среды и взаимодействия генов и окружающей среды в этом процессе.
Аноним 30/01/24 Втр 14:37:35 587410 893
IMG202312220833[...].jpg 136Кб, 979x1080
979x1080
>>584346 (OP)
Всём привет.
Есть закон диалектики в котором количественные изменения приводят к качественным.
Например - лед нагреваясь превращается в воду, вода в пар, а пар в плазму. Что будет если дальше нагревать плазму? Следующее агрегатное состояние вещества? Или дальше просто плазма плазмой остаётся?
Аноним 30/01/24 Втр 14:54:07 587411 894
>>587410
нагревание суть ускорение
Аноним 30/01/24 Втр 15:10:43 587412 895
>>587409
> изучают влияние генетики
Вот именно - ВЛИЯЕТ. О чем я не спорил. А не определяет, как социокультура.
Аноним 30/01/24 Втр 15:12:20 587413 896
>>587410
Дрожание атомов дойдет до скорости света, если нагреть ещё больше, то начнут появляться новые частицы. В БАКе экспериментально это подтвердили лет 10 назад.
Аноним 30/01/24 Втр 15:57:39 587420 897
>>587412
> Что влияет на вкус каши? Соль.
> Что определяет вкус каши? Соль.
Ты понимаешь, что уже начал натягивать сову на глобус?
Аноним 30/01/24 Втр 16:47:40 587424 898
>>587411
Ну наверно.
>>587413
Как массовые частицы дойдут до скорости света? Да и в коллайдере не появляются новые частицы, просто их сталкивают на большой скорости и мы наблюдаем запчасти из которых эти атомы состоят.
Аноним 30/01/24 Втр 17:03:46 587427 899
>>587424
> как массовые частицы дойдут до скорости света?
Их разгоняют с помощью электромагнитного поля до скорости близкой к скорости света. До самой скорости света разогнаться невозможно.
> мы наблюдаем запчасти из которых частицы состоят
Бозон Хиггса это запчасть частицы? Какой если не секрет?
Аноним 30/01/24 Втр 17:53:19 587431 900
>>587427
Я не собираюсь спорить на эту тему. Мне интереснее было о следующем агрегатном состоянии вещества после плазмы если законы диалектики верны.
Аноним 31/01/24 Срд 04:10:26 587439 901
>>587431
>Я не собираюсь спорить на эту тему. Мне интереснее было о следующем агрегатном состоянии вещества после плазмы
Нет никакого другого агрегатного состояния. Агрегатные состояния образуются за счёт электромагнитных сил. В плазме у газа просто отхерачились (и улетели нахуй) электроны. Больше ты ничего тут электромагнитно не поменяешь.

> если законы диалектики верны
Лол нет.
Аноним 31/01/24 Срд 13:24:04 587448 902
>>587439
>Агрегатные состояния образуются за счёт электромагнитных сил
В общем случае нет. На квантовом уровне работает принцип Паули, что ведет в появлению новых фаз, фактически других агрегатных состояний.
К примеру любой кусок металла, это кристаллическая решетка обычного твердого тела, внутри которой "разлита" ферми-жидкость из электронов.
Аноним 31/01/24 Срд 21:32:58 587458 903
Сап, вот есть атмосферная коррозия металлов, из-за присутствия влаги в воздухе, выпадения росы и тд даже без дождя металлические конструкции будут медленно ржаветь, вопрос в следующем, если скажем металл попал под дождь несколько раз и на нем быстро появился налет ржавчины, а потом мы исключим прямой доступ осадков на поверхность металла, то коррозия вернется к своему обычному темпу, либо налет каким-то образом ускорит процесс? Просто есть велик из хромо-молибденовой стали - пару-тройку раз прокатился в дождь, внутри на трубах появился рыжий налет, больше я кататься в дождь не буду, мне нужно убирать ржавчину во избежание ускоренного гниения рамы или без доступа воды он еще внукам достанется покататься? Спасибо за ответы.
Аноним 01/02/24 Чтв 12:13:30 587477 904
>>587458
Для обычный сталей ржавчина это гидроксид железа. Из-за особенностей электрохимии ржавчина способна автокатализировать коррозию атмосферным кислородом и влагой.
С случае легированных(хромом или молибденом) сталей все сложнее, ибо там действует другие процессы. Коррозия идет через перекристаллизацию и уже реакция не автокаталитическая. Впрочем лучше все таки зачищать поверхность.
Аноним 01/02/24 Чтв 21:46:14 587510 905
SEM рачок-водор[...].gif 2148Кб, 400x300
400x300
>>587477
Спасибо, добра, лучше ответа и представить сложно, мой любимый сайентач не подвел! Пикрандом.
Аноним 01/02/24 Чтв 21:50:17 587511 906
>>587510
Как получают такие изображения? Это же просто шок какое пррблржение
Аноним 01/02/24 Чтв 21:59:16 587512 907
>>587511
Сканирующая электронная микроскопия.
Аноним 01/02/24 Чтв 21:59:47 587513 908
Аноним 01/02/24 Чтв 22:11:00 587514 909
>>587513
Я знаю не больше гугла. Исследуемый объект специально готовят осаждая на нем тонкую пленку металла (вроде золото обычно), поэтому живые объекты не поизучать, потом помещают в вакуумную камеру, и обстреливают пучком сфокусированных электронов, которые отражаются от металлической пленки и попадают в детекторы, которые формируют изображение, длина волны электронов очень маленькая - соответсвует атомным размерам поэтому можно получить гигантские увиличения в миллионы раз, в отличие от оптической микроскопии.
Аноним 01/02/24 Чтв 22:17:20 587515 910
>>587514
Охренеть. Круто, спасибо
Аноним 02/02/24 Птн 12:06:33 587543 911
>>587514
А я думал там тупо иголкой водят и напряжение на ней записывают. Я в чем то не прав в своем упрощении?
Аноним 02/02/24 Птн 12:46:53 587544 912
>>587543
С иголкой это атомно-силовая микроскопия.
Напряжения не всегда записывают, чаще всего снимают лазером колебательные моды с кристалла иглы.
Аноним 03/02/24 Суб 10:19:42 587590 913
>>587544
Немножко поправлю. С иголкой это сканирующая туннельная, ей можно хоть атомы отрисовать на поверхности, лишь бы проводила. В атомно-силовой все ж кантилевер, с которого отражение лазера фотодетектор снимает.
Аноним 03/02/24 Суб 19:57:41 587602 914
17069792219990.mp4 35Кб, 148x192, 00:00:06
148x192
Зачем нужна математика?
Это фентези, если математика работала в жизни шоколад был бы бесконечным.
Аноним 03/02/24 Суб 20:01:20 587603 915
>>587602
Части шоколадки отламываются в виде крошек и остаются на ноже, которым им разрезают. Хоть на атом, но будет уменьшаться. ИМХО, математика нужна как способ счисления, а не как что-то самостоятельное, НО я слышал и обратное.
Аноним 03/02/24 Суб 21:21:15 587606 916
podxg4vuco0-768[...].jpg 27Кб, 768x326
768x326
Пытаюсь понять, что из себя представляет наш мир. Читаю в основном википедию и хабр. Самая непонятная хуйня, которая периодически попадается, что частицы это не частицы и не поле/поля. Где-то мне попалось утверждение, что это вообще взаимодействия. Т.е. не взаимодействия чего-то, а просто взаимодействия в чистом виде, но для удобства это иногда представляют в виде частиц, а в других случаях в виде поля.
Еще часто попадается мысль, что пространства тоже нет. Как вариант пространства-времени. Ну что пространство это тоже удобная модель для физиков и математиков, а так его нихуя нет. А у нас возникает иллюзия пространства из-за особенностей восприятия.

Где-то можно об этом по-подробнее почитать? Если оба утверждения не бред, конечно.

Как может быть, что для фотона все время существования вселенной один миг, если его скорость конечна? Это как-то связывают с тем, что у фотона нет массы и поэтому у него такое "восприятие" пространства-времени.

Алсо, посоветуйте каких-нибудь научпоп книг, чтобы наконец понять, что из себя представляют, хотя бы примерно прочувствовать частицы, суб частицы, пространство это ебаное блядь. Только я гуманитарий, все мои "познания" из википедии.

Собираюсь прочитать "Вселенная. Происхождение жизни, смысл нашего существования и огромный космос" и еще
"Квантовые миры" Шона Кэрролла. Норма вариант?
Аноним 03/02/24 Суб 22:00:06 587607 917
>>587606
Если реально гуманитарий, читай какого нибудь Фаербенда или кто там любил капченых хуесосить. Поймешь (и другим будешь разъяснять) что ученые тоже люди и нихуя толком не понимают, сочиняют всякую хуиту и любят переобуваться в прыжке.
Аноним 03/02/24 Суб 22:37:58 587609 918
Как научно объясняется появление жизни? По какому принципу и зачем материя управляемо начинает накапливать энергию своими силами и распоряжаться ресурсами? Есть хотя бы догадки? Ни одна из религиозных теорий о боге и близко не подходит. То есть наши религиозные представления с театральной подачей реальности это какая-то яма вроде слепого пятна на сетчаке, куда мы заруливаем сознанием исходя из эмоционального реагирования на мир и интерпретаций на основе такого опыта, и в следствие этого у нас формируется ложная картина реальности, из которой нам не доступно видеть объективное. Если полагаться на эмоциональный опыт нельзя, поскольку он формировался только для субъективного и коллективного выживания в определённых условиях, как тогда объясняются законы эволюции вне естественной среды? Почему мы вынуждены идти контринтуитивным путём через противоречия, проходя путь обратным образом от выживания к знаниям и наоборот, реформируя мотивы и меняя цели? Это же шизофрения, так как мозг не способен работать в условиях постоянной неоднозначности и отсутствия привязки к определённым значениям, а сложившиеся стереотипы становятся абсурдным мусором из религий, стереотипов, суеверий. и так по кругу. Как тогда составить адекватную модель восприятия жизни и развития в нём?
Аноним 04/02/24 Вск 03:58:29 587612 919
>>587606
>Ну что пространство это тоже удобная модель для физиков и математиков
Так оно так и есть же
Аноним 04/02/24 Вск 04:04:54 587613 920
Физикам вопрос. Объясните, что происходит с электронами и орбиталями в нейтральном атоме при реакции его деления или спонтанном распаде? По сути я предполагаю, что электроны могут перестраиваться на орбитали новых атомов, но весь процесс технически представить сложно.
04/02/24 Вск 09:25:36 587616 921
>>587613
>орбитали
>>587613
Отвечу как инженер микроэлектронщик который изучал курс атомной и ядерной физики один семестр.
При образовании нового элемента, или при его распаде получаются новые вещества. Например калий 19/38 при бета распаде превратится в инертный газ аргон 18/38. Как видно число нуклонов в ядре осталось таким же, поменялся лишь атомный номер, в результате этого ядро стало слабей притягивать электроны в атоме, отсюда происходит и смещение электронных энергетических уровней. 4s электрон на внешней оболочке в результате такого смешения оказывается свободным и покидает атом, оставляя Аргону законченную электронную конфигурацию благородного газа.
04/02/24 Вск 10:25:39 587619 922
>>587096
Металлы по природе эластичны благодаря наличию электронного газа (электроны обобществлены, т.е валентный электрон одного атома металла одновременно принадлежит всем атомам кристалла) , который в свою очередь образуется благодаря металлической связи между атомами вещества. В углероде атомы соединены ковалентными связями, например одна из аллотропных модификаций углерода - алмаз является диэлектриком и не проводит электрический ток, так как нет свободных электронов проводимости как в металле. ( с допущениями алмаз можно рассматривать как полупроводник с большой шириной запрещенной зоны). Проводимость графита обусловлена его слоистой структурой. Если в алмазе атомы углерода соединены друг с другом сильной ковалентной сигма связью, то в случае с графитом три атома углерода соединены сигма связью а четвертый пи связью, электрон в этой связи более подвижен. Благодаря этому графит проводитэлектрический ток. P.s химики простите если где то накосячил в универе с натяжкой получил четверку.
Аноним 04/02/24 Вск 14:30:37 587622 923
>>587619
> Металлы по природе эластичны благодаря наличию электронного газа
Ну это не совсем так. Пластичность и тип связи вещи не всегда совместимые. К примеру хром, рутений и т.п. скорее в порошок уйдет при деформировании, ну а органика к примеру ковалентная будет вполне себе деформироваться.
> Если в алмазе атомы углерода соединены друг с другом сильной ковалентной сигма связью, то в случае с графитом три атома углерода соединены сигма связью а четвертый пи связью, электрон в этой связи более подвижен.
Тут прям так связи нельзя упрощать. Имеет место sp3 и sp2 гибридизация для алмаза и графита. Между слоями графита ван-дер-ваальсовы связи, в листе графита все электроны находятся в разрешенной зоне, что подтверждается отсутствием прозрачности (аля как металл), даже в графене есть дираковские точки на границе зоны Бриллюэна. Проводимость графита не оч хорошая, тк в отличие от металла проводимость только в слоях и рассеяние на границах чешуек достаточно большое, что тянет повышение удельного сопротивления.
Аноним 04/02/24 Вск 16:01:25 587627 924
>>587622
Не уверен, что хром и рутений это идеальный пример чисто металлической связи. Там d-d взаимодействия неплохо так дают прикурить. А чисто металлическая параша это какой-нибудь кусок калия.

Мне кажется тут все упирается в семантику, что имеется ввиду под "металлами". В целом металлы, или наши выдуманные модельные металлы.
Аноним 04/02/24 Вск 16:12:05 587628 925
>>587627
Да, это я все согласен, если по-хорошему, то пластичнось это величина барьеров Пайерлса-Набарро, то бишь "легкость" скольжения дислокаций по плоскостям скольжения (спайности) с наибольшей ретикулярной плотностью, что для ковалентных и ионных кристалло автоматически может давать разрушение если спайность совершенная или весьма совершенная.

> что имеется ввиду под "металлами"
Ну, если мы уйдем от электропроводности, где металлы это как правило "проводники", а перейдем к зонной структуре, то "металлы" это вещества, где уровень ферми лежит в первую очередь в разрешенной зоне, а во-вторых эта зона должна быть заполнена частично. По идее так.
Аноним 05/02/24 Пнд 12:30:18 587645 926
809749d31a24014[...].jpg 12Кб, 512x512
512x512
>>584346 (OP)
Анон, думаю второй день над антропологической природой звука. Проблема даже в формулировке поиска, чтобы найти материалы, поэтому задам тебе вопрос, возможно совместный ход размышлении даст ответ. И подозрение что он вообще на поверхности.

Человечество использовало и развило звуковую систему до определенных высот. От социальных (речь, музыка, сигнально-маркерно-информационные звуки, интонационный окрас) до технических (гидролокация, ультразвук в науке-производстве-медицине, военные и психологические аспекты применения и т.д.) Но всё это следствие формирования и развития артикуляционного аппарата у нашего вида, понимание и изучение природы звука. У животного мира такой же механизм - акустическая коммуникация, маркеры угрозы, защиты и привлекательности. Но опять же это следствие и развитие аппарата, не изначальная данность.

Зачем природа создала речевой аппарат и звуковые коммуникации? Приятная побочка пищеварительно-дыхательной системы в дальнейшем развитый когнитивными аспекатми развития мыслительной системы?

Пример: характеристики экстермофилов обоснованны средой обитания, оволосение терморегуляция, уплотнение костного-кожного аппарата и хитинирование защита и т.д. Но пиздеть зачем природа нам разрешила? Зачем сделала, что все пиздят? Пиздят, пиздят, пиздят. Рыбы же не пиздят, максимум звуки жизнедеятельности. Получается просто побочка, выросшая в одну из фундаментальных систем жизнедеятельности?
Аноним 05/02/24 Пнд 12:39:09 587646 927
>>587645
У тебя есть система клеток. Когда эту систему тревожит, определенным образом, определенная разновидность помех. За этим следует вредное или полезное явление.
А когда все уже умеют слышать, умение пиздеть становится крайне выгодно.
Аноним 05/02/24 Пнд 13:01:50 587647 928
>>587646
>все уже умеют слышать
Да, система звука интуитивно организовалась из-за координационной системы слуха. У нас есть один из механизмов ориентации, координаты в виде высоты-силы-спектра и есть протоаппарат взаимодействия, и мы естественно его разовьем. Отлично, есть ответ. Теперь буду размышлять нахуя нам слух дала природа? Понятно колебания и вибрации, но все равно это интерпритации звукового аппарата мозгом, без него это просто раздражитель определённых участков имеющий градиент силы колебания. А тут система посложнее.
Аноним 05/02/24 Пнд 14:17:17 587650 929
IMG7022.jpeg 392Кб, 1920x1880
1920x1880
А прикиньте как жилось людям которые так или иначе относились к программе Аполлон?
Тогда вообще нереально всех заливали деньгами, в пересчете на сегодняшний день это как почти 300 миллиардов долларов.
Почти все кто занимался этим мгновенно стали мажороми с большим домом, семьей, собакой и спортивной машиной.
Аноним 05/02/24 Пнд 14:49:04 587653 930
>>587616
Чего? Бета распад, ядро стало более заряженным на +1, а ты говоришь что из оболочки один электрон при этом улетел?
Аноним 05/02/24 Пнд 14:53:25 587654 931
>>587653
А понял, ты просто не указал что это бета+ распад
По дефолту ж считают бета распал бета- распадом.
Аноним 05/02/24 Пнд 16:55:14 587662 932
>>587612
Поясни свою мысль, будь добр. И где об этом можно почитать?
Аноним 05/02/24 Пнд 17:11:36 587668 933
>>587645
Все фичи всех ДНК организмов есть результат случайных мутаций. Если мутация давала какие-то преимущества в размножении особи, то мутация закреплялась в виде. Если не давала, то мутация исчезала из вида.

> зачем природа разрешила
Природа не является субъектом, чтобы иметь намерения.
Аноним 05/02/24 Пнд 17:50:42 587675 934
>>587662
Я не думаю, что это научный вопрос - и я не ученый если что - просто наука же этим и занимается вроде - приближенным описанием мира. Из наблюдений и тд конечно, но типа карта же это не мир вокруг тебя, а описание его
Аноним 05/02/24 Пнд 19:07:46 587686 935
>>587668
>Природа не является субъектом, чтобы иметь намерения.
Природа - это не абстракция. Твои мама и папа это Природа относительно тебя.
Аноним 06/02/24 Втр 01:13:12 587717 936
>>585592
Потому что это псевдонаучная хуита, целью которой является поддержка капиталов калпидорглистов, а именно - показать конкуренцию, чтобы за всякую хуйню люди грызлись между собой, а калпидорглисты с них ржали, и кормили их этой макулатурой жидомасонской, что лезет сотнями миллиардов ежегодно - с печатных станков.
Аноним 06/02/24 Втр 02:14:13 587718 937
>>587717
Ты веришь в капиталистов? Лох. Это рептилоиды.
Аноним 06/02/24 Втр 15:24:48 587765 938
>>587717
какова этмилогия слова механика в квантовой механике?
механика - как взаимодействие в смысле "взаимодействие заявителя и нотариусе" ?
Аноним 06/02/24 Втр 15:58:35 587771 939
mus2.png 125Кб, 233x257
233x257
>>587765
можно ли снять с себя статическое напряжение, если помыть руки под крано?
Аноним 06/02/24 Втр 16:37:04 587775 940
>>587765
Слово «квант» происходит от латинского quantum («сколько, как много») и английского quantum («количество, порция, квант»). «Механикой» издавна принято называть науку о движении материи. Соответственно, термин «квантовая механика» означает науку о движении материи порциями (или, выражаясь современным научным языком науку о движении квантующейся материи). Термин «квант» ввел в обиход немецкий физик Макс Планк (см. Постоянная Планка) для описания взаимодействия света с атомами.

https://elementy.ru/trefil/20/Kvantovaya_mekhanika
Аноним 06/02/24 Втр 18:51:38 587782 941
Кто вкурсе размерностей и способностей измерительной физической техники
Насколько явно замеряема или незамеряема разница между двумя длинами волн излучения, которые различаются меньше чем на постоянную планка?
Аноним 06/02/24 Втр 21:51:19 587790 942
>>587606
Брайн Грин "Ткань космоса".
Дэвид Дойч "Структура реальности".

Обе книжки написаны для гуманитариев без всяких формул.
Аноним 06/02/24 Втр 22:33:37 587791 943
>>584795
>Время работает точно так же как и еще одна добавленная к пространству ось.
Не работает, т.к. не связано с пространством.
Иначе релятивисткое замедление времени пойдёт по пизде.
Аноним 06/02/24 Втр 22:37:40 587792 944
>>585178
>чё всё? лучшие умы доски ничего не смогли ответить внятного?
Отвечать не на что
>>584809
Сырой не означает невозможность воспламенения.
Аноним 06/02/24 Втр 22:43:41 587793 945
>>584818
>Почему человек (да и большинство животных в принципе) не может есть (полноценно усваивать) траву, листья, кору, корешки, древесину?
Есть может.
Люди даже глину едят в африке.

Полноценно усваивать человек вообще мало что может в сыром виде. Фрукты какие-нить, типа бананов, разве что. С обработкой ты можешь хоть кору жрать, гугли берёзовую муку, например и берёзовый хлеб.
Аноним 06/02/24 Втр 23:19:28 587796 946
>>585825
Нормально ты так дунул, братиш.
Стул - это объект, обладающий свойствами, в основном это:
-можно сидеть
-можно передвинуть
-можно опереться на спинку.

Ты не можешь разобрав (декомпозировав) стул на доски, найти свойства стула, потому что это уже не стул.

>более того невозможно понять когда они все вместе становятся стулом
Когда что-то собирается в один объект, обладающий свойствами выше.

Сознание и мозг это не составные части одного и того же, это как лампочка, которую включили и она излучает свет.
Аноним 06/02/24 Втр 23:29:30 587797 947
>>586018
>Почему время в любом девайсе, не подключенном к интернету и, соответственно, не получаещем апдейт с "правильным" временем всегда начинает спешить?
Потому что китайский ширпотреб.
У тебя кварцевый генератор копейки стоит, хуле ты хотел?
Хочешь точность - покупай хронометр.
Аноним 06/02/24 Втр 23:32:00 587798 948
>>586047
>Почему никто ещё не додумался испытать на практике теоритическую модель автономного марсианского поселения
По новостям уже которую партию людишек на 2-3 года в герметичных капсулах запирают.
В них челы дрочат рекуперацию, и выходят погулять наружу только в скафандрах.

Хуй знает, как ты всё это пропустил.
Аноним 06/02/24 Втр 23:41:04 587800 949
>>586310
>Мне лишь нужно узнать точное число вариантов написания чисел "0-9", если использовать "11" знаков.
Числа с 0 по 9 имеют единственный вариант написания в системе из 11 цифр, если счисление по арабской системе
Аноним 06/02/24 Втр 23:44:55 587801 950
>>586401
>Почему нет активного электромагнитного паруса?
Гугли микроволновый двигатель
Аноним 06/02/24 Втр 23:50:59 587802 951
>>586532
>значит после моей смерти рано или поздно все молекулы разложившегося меня вновь соберутся обратно и я снова стану живым. В чём я не прав?
В том, что это максимально бесполезная информация, поскольку свой жизненный опыт, который определяет тебя, ты не перенесёшь.
Аноним 06/02/24 Втр 23:55:36 587803 952
>>587344
>Почему птиц на проводах не убивает ток???
Разница сопротивлений.
Лапы птиц являются параллельной цепью, а при параллельном подключении количество тока обратно пропорциональна сопротивлению участка цепи.

Короче, ток через птицу настолько мал, что его можно игнорировать.
Аноним 06/02/24 Втр 23:59:06 587804 953
>>587384
>Белый самец 100iq спаривается с белой самкой 100iq, у них рождается ребёнок 100iq, верно?
Не верно, рисунок коры головного мозга формируется под чисто механическими причинами.
Иными словами, даже клонируя 100iq человека мы можем получить как 120iq, так и 80iq, тупа изза особенностей внутриутробного развития.

Отпечатки пальцев будут одинаковыми, но не рисунок коры головного мозга.
Аноним 07/02/24 Срд 00:05:43 587805 954
>>587602
Смотри на плитку из четырёх кусочков. Микрошип на месте отреза магически удобно удлинняется при стыковке на новом месте.

Красивый видеомонтаж, короче.
Аноним 07/02/24 Срд 00:08:56 587806 955
>>587609
>Как научно объясняется появление жизни?
Тебе теория эволюции уже всё расписала.
Учёные мужи уже и первичный бульон варили, и протобактерию в первичном бульоне при подаче электроразрядов получали.
Все исследования можно нагуглить
Аноним 07/02/24 Срд 03:44:40 587809 956
>>587804
Ты сейчас опровергнуть факт генетической наследственности интеллекта опровергнуть пытаешься или просто к условностям в цифрах приебался?
Аноним 07/02/24 Срд 06:37:03 587810 957
>>587782
Постоянная планка и длина волны разные размерности, что ты имеешь ввиду, если разница между ними не может быть равна постоянной планка?
Аноним 07/02/24 Срд 07:31:25 587811 958
>>587810
Ну орбиту электрона же квантуют постоянной планка.
У фотонов разной длиной волны разная рюэнергия, значит тоже можно как-то постоянной планка проквантовать.
Аноним 07/02/24 Срд 07:48:19 587813 959
>>587811
>Ну орбиту электрона же квантуют постоянной планка.
И кстати вот кажется, типа планковские величины это такие маленькие штуки(в основном) и постоянная планка это про самый-самый микромир.
Но вот высоты орбиталей электронов квантуют постоянной планка, и переходы электронов между ними сопровождаются излучением вполне себе мощных фотонов.
По-идее из этого жёстко следует что постоянная планка нифига не маленькая.
Или я что-то не так понял?
Аноним 07/02/24 Срд 08:23:23 587820 960
>>587809
>Ты сейчас опровергнуть факт генетической наследственности интеллекта опровергнуть пытаешься или просто к условностям в цифрах приебался?
Второе.
Генетическая наследственность интеллекта показывает результат только в статистике, т.е. когда ты будешь анализировать тысячи особей. Иными словами в твоём случае дети белых В ЦЕЛОМ будут умнее.
При разборе единичного случая факторы развития оказывают такое же, если не большее воздействие на итоговый интеллект, поэтому хуй ты чё спрогнозируешь.
Аноним 07/02/24 Срд 10:36:53 587827 961
smfull.jpg 200Кб, 1200x800
1200x800
>>587820
>Генетическая наследственность интеллекта показывает результат только в статистике

>При разборе единичного случая
Аноним 07/02/24 Срд 10:51:59 587828 962
>>587827
Ты путаешь генетическую болезнь и общий случай.
Генетическая болезнь может характеризоваться особенным, лол, коэффициентом вероятности повторения в следующем поколении.
Который будет сильно отличен от веротяности повторения вариаций развития нормы.
У 10 поколений iq 80 обоих родителей, если появится ребёнок iq100-очень маленький шанс что у его детей будет тоже iq близкий к 100. Но конечно выше чем был шанс появиться у этого iq 100 с родителями iq 80.
Аноним 07/02/24 Срд 10:56:28 587829 963
>>587828
>ты путаешь генетическое влияние на интеллект и генетическое влияние на интеллект

Есть за мной такой грешок.
Аноним 07/02/24 Срд 11:16:44 587831 964
>>587828
Крч, нужно искать высокую, большегрудую, большежопую, худую, с правильными чертами лица, голубоглазую блондинку, у которой мама и бабушка такие же. А и ещё здоровую и не стареющую.

В целом, всё осталось таким же как и до научных исследований в этом вопросе.
Аноним 07/02/24 Срд 11:28:19 587832 965
>>587829
Ты тупой, генетическая болезнь это набор свойств
В том числе и такого свойства как коэффициента влияния на следующие поколение.
Поэтому твоя картирка та к тебе подходит только разве что, а не как оспаривающий пример.
Аноним 07/02/24 Срд 11:29:26 587833 966
>>587831
Естественно, вот только зачем тот даун спорил с очевидной вещью что интеллект это про среднее, а не про жёсткую передачу от родителей к детям, вопрос.
Аноним 07/02/24 Срд 11:37:09 587834 967
Кстати если рассматривать тупизну, как попроченность структур и строяния мозга, в том числе и размера
В принципе это то же самое что и у даунов.
И вот у даунов высокая крайне вероятность появления детей даунов.
Так что сложение 80+100=90 так скорее всего не работает.
Порченная в большом количестве мест структура намного больше имеет вероятность повториться порченной, потому что порченность это беспорядок
А вот правильно функционирующая структура это порядок.

Так что усли усреднять, то допустим было 10 поколений 100, потом родился 80, то и дальше с высокой вероятностью будут появляться около 80-ти.
Или например 10 поколений 100, из потомок тоже 100 смешался с потомком 10 поколений у которых было 80
То полчится тоже около 80-ти, а не 90.
Аноним 07/02/24 Срд 11:55:09 587836 968
>>587832
>интеллект не зависит от генов
>дауны такие потому что их такими воспитали
ясно
Аноним 07/02/24 Срд 12:00:56 587837 969
>>587836
Ох, тупой даун
Интеллект жёстко определяется генами.
Передача генов от родителей к детям не жёсткая, жёсткая передача от совокупности предков детям с некоторой вероятностью колебаний в + или -
Ты настолько тупой что не отлечаешь жёсткое определение и жёсткую передачу.
Аноним 07/02/24 Срд 12:27:02 587839 970
>>587820
>При разборе единичного случая факторы развития оказывают такое же, если не большее воздействие на итоговый интеллект
>Передача генов от родителей к детям не жёсткая, жёсткая передача от совокупности предков
Очевидно, что у умной тян скорее всего и умные предки, так что и колебания там особо никакого не будет, учитывая что интеллект передаётся как раз по материнской линии, по крайней мере в случае с рождением сына.
Аноним 07/02/24 Срд 13:33:07 587846 971
>>587839
>интеллект передаётся как раз по материнской линии, по крайней мере в случае с рождением сына.
Охуенные истории.
Нормальные исследования об этом есть, а не журналы для тупых шалав?

Почти всю историю человечества интеллектуальным качествам женщин вообще мало внимания уделяли, используя просто как инкубатор.
И что-повально дауны-мужчины не рождались.
Если всё не с точностью до наоборот, то как минимум такое же влияние, 50/50.
Аноним 07/02/24 Срд 13:37:51 587847 972
Аноним 07/02/24 Срд 14:58:40 587857 973
714967.gif 4572Кб, 396x299
396x299
Аноним 07/02/24 Срд 15:13:27 587859 974
>>587645
Это была побочка. У тебя есть горло и есть воздух, который по нему гоняют легкие. Получается незадокументированная фича "дико орать". Самки текут от альфачей, а альфачи быкуют и орут. Омеганы сидят тихо и не отсвечивают. В итоге самки дают альфачам, которые орируют погромче, а те что молчат - не дают потомство.


>Рыбы же не пиздят
рыбам нечем - у них нет легких
Аноним 07/02/24 Срд 15:15:15 587860 975
>>587839
>интеллект передаётся
Нет.
Никакого интеллекта никому не передается.
Генетически возможно запрограммирована биохимия мозга.
Но даже имея идеальную биохимию, смотря опять же что иметь в виду, если ты родишься в семье которая продаст тебя в 5 лет в публичный дом бангладеша, никакого интеллекта в тебе не образуется.
Аноним 07/02/24 Срд 15:47:03 587866 976
>>587827
>>587839
>Очевидно, что у умной тян скорее всего и умные предки
Вообще не обязательно.

Почему я вот тут >>587820 сказал, что это всё роляет лишь на статистике?

Потому что внешние факторы влияют на интеллект куда больше, чем генетическая предрасположенность к интеллекту (если это не заболевание).

Широкий таз или узкий влияет на травмы черепа при родах. Рандомность образования извилин зависит тупа от того, как клетки скрючились механически. Единственный реально коррелирующий параметр на данный момент с айкью - объем черепной коробки, но опять же, он коррелирует только на больших числах. Т.е. В СРЕДНЕМ люди с меньшим мозгом тупее по статистике, но на практике роляет и его структура и умение им пользоваться. У Тургенева 2кг мозга, у Анатоля Франса 1кг - оба писатели.

А ещё этап развития мозга не завершается после рождения, он идёт ещё лет 5 активно потом. Есть задокументированная особенность человека, что если он не научился говорить в детстве, то больше он не научится никогда - есть примеры "диких" людей, которых уже во взрослом возрасте пытались вернуть в цивилизацию. Они так и не научились говорить и понимать речь, максимум - разучивали отдельные слова как собака команды.

На данный момент не обнаружено гена "умности", с помощью которого можно было бы селекционировать людей как мопсов и выращивать сверхразум. Потому что сам мозг оказывается формируется очень странным образом, не соотносящимся с каким-то геном.
Аноним 07/02/24 Срд 15:47:41 587867 977
>>587860
Тебя продали и ты так себя теперь успокаиваешь?
Аноним 07/02/24 Срд 15:51:33 587868 978
>>587866
>А ещё этап развития мозга не завершается после рождения, он идёт ещё лет 5 активно потом.
Это я к чему - можно в этот промежуток времени заниматься со своими детьми, что сделает их более эрудированными на всю оставшуюся жизнь, а можно проебать момент и будут не такими умными с меньшим айкью
Аноним 07/02/24 Срд 16:00:05 587869 979
>>587645
Тут скорее эволюционная выгода от превращения организма в передатчик сигналов плюс хорошее дальнодействие, подходящая ширина канала и повсеместное наличие медиатора (воздуха, воды).

Как приемник сигналов в зависимости от дистанции хорошо работает гаптика, вкус, запах, звук и свет. Как передатчик - нихуя. Касаться, лизать и нюхать можно только рядом расположенные предметы. Метить территорию можно только на длину струи. Пердеж и феромоны улетают по ветру в непредсказуемом направлении. Для того, чтобы вибрировать или топать ножкой, нужно большая масса тела или наличие твердого грунта. Мигать светом как светлячок можно только ночью, под землей или под водой, потому что супротив солнечного засвета не хватит мощности. А вот орать можно круглосуточно, в широкой полосе частот и на многокилометровую дистанцию. Если бы на небе вместо солнца и луны висело гигантское ОРАЛО, которое круглосуточно пиздело бы как репродуктор на всех частотах - тогда орать и правда было бы бессмысленно. А если бы с неба постоянно ссало разнообразной органикой космическое ХУЙЛО, то тогда бы собакенам не было смысла нюхать чужие газы и метить территорию. Ну ты понел логеку.
Аноним 07/02/24 Срд 17:08:54 587875 980
>>587868
Как ты объяснишь умных детей алкашей, наркотов и прочих бомжей? Которыми естественно вообще никак не занимались.
Кроме объяснения того что у тебя низкий интеллект, и ты пишучи такое здесь, самому себе фактически рассказываешь эти сказки пытаешься себя успокоить, но и нам приходится их здесь видеть?
Аноним 07/02/24 Срд 17:14:22 587877 981
>>587875
>Которыми естественно вообще никак не занимались.
Улица воспитала, ауф
Аноним 07/02/24 Срд 17:25:39 587879 982
>>587866
>Потому что внешние факторы влияют на интеллект куда больше, чем генетическая предрасположенность к интеллекту
Ты же понимаешь что ты просто комнатный пиздабол, спорящий сейчас с мочёными https://www.mensa.org/iq/genes-and-environment ?

>в этот промежуток времени заниматься со своими детьми, что сделает их более эрудированными на всю оставшуюся жизнь
Эрудированность не равна интеллекту. Конкретно интеллект ты дальше генетически заложенного потенциала у ребёнка развить никак не сможешь, так же как и не сможешь замедлить деградацию мозга с возрастом засчёт интеллектуальной нагрузки.

>На данный момент не обнаружено гена "умности"
Коупмонстрище... В говне мочёные уже более 1к генов нашли, ответственных в той или иной степени за когнитивные способности
Аноним 07/02/24 Срд 17:38:06 587881 983
>>587866
>На данный момент не обнаружено гена "умности", с помощью которого можно было бы селекционировать людей как мопсов и выращивать сверхразум
Тупая одебилевшая трансошалава, селекция точно так же и замечательно работает без всяких обнаружений какого-то гена, выблядок ты тупых уёбищ без хуя.
Аноним 07/02/24 Срд 17:42:38 587883 984
Лол, походу эта шалава, не скажу думает, считает, что умных людей невозможно отличить от тупых, кроме как если не покапаться в их гене, и не найти есть ли у конкретного человека ген умности, или нет
Ведь все равны
Ахахахаэаэ
Аноним 07/02/24 Срд 21:04:27 587912 985
>>587883
Вот, сразу видно, твои родители умные люди, такого умного человека вырастили, ну прям совсем не долбоеб-дебил
Аноним 07/02/24 Срд 22:26:35 587916 986
>>587875
>Как ты объяснишь умных детей алкашей, наркотов и прочих бомжей?
Так я уже обьяснил, что интеллект с наследственностью слабо коррелирует.
Очевидно, что бомжи-алкаши-наркоши есть биомусорный скот, который по определению тупой. Если бы была жёсткая наследственность интеллекта, то и их дети были бы тупым алкоскотом.

>>587879
>Professor Sir Cyril Burt, former Mensa World President and one of the world's leading psychologists at the time, is famous for his twin studies, in which he discovered that the correlation between the IQs of identical twins reared apart was an enormous 0.77, which almost irrefutably supports a large genetic influence on IQ.
>0.77
>Большая корреляция
Про что я и говорю. Будь зависимость интеллекта от генов жесткой, этот параметр был бы равен единице.
Камон, у однояйцевых близнецов, считай, законных клонов человека, разный айкью. Какого, спрашивается, хуя.

>A child may be born with genes for high intelligence, but if that child grows up in a deprived environment where he/she is malnourished orlacks access to mental stimulation, the child may not score well on measures of IQ.
Про что я и говорю, буквально.
>Конкретно интеллект ты дальше генетически заложенного потенциала у ребёнка развить никак не сможешь
Зато можешь его проебать, чекай пример с языком.
Важно то, что ты можешь тренировать само мышление, соображалку.
>не сможешь замедлить деградацию мозга с возрастом засчёт интеллектуальной нагрузки.
Нихуя себе новости. Первый раз о таком слышу, пруфы будут?

>В говне мочёные уже более 1к генов нашли, ответственных в той или иной степени за когнитивные способности
Ты ничем не опроверг моё заявление, а лишь подтвердил.
Это всё гены уровня "размер черепа", "форма черепа", особенности кровеносной системы и т.д., каждый из которых влияет косвенно, а не прямо.

>>587881
>Тупая одебилевшая трансошалава, селекция точно так же и замечательно работает без всяких обнаружений какого-то гена
Чел, это подтверждает теория Дарвина и происхождение человека

>>587883
>Лол, походу эта шалава, не скажу думает, считает, что умных людей невозможно отличить от тупых, кроме как если не покапаться в их гене, и не найти есть ли у конкретного человека ген умности, или нет
>Ведь все равны
Кажется, ты абсолютно не понял тему моего поста. Переформулирую:

В связи с отсутствием генов, непосредственно влияющих на интеллект, невозможно однозначно судить об уровне будущего интеллекта у ребёнка в единичном конкретном примере ввиду сильного воздействия на интеллект внешних факторов среды, в которой появляется ребёнок.

Не, ну если тебя устраивают предикты в виде "ну, у белого самца с белой самкой 80% шанс получить ребёнка с iq в промежутке 80-120, а у белого самца с чёрной самкой 80% шанс получить ребёнка с iq в промежутке 75-115" - то пожалуйста, кушайте не обляпайтесь.
Аноним 07/02/24 Срд 22:35:17 587917 987
>>587883
Про равность я кстати нихуя не понял, откуда ты это высрал, мы тут буквально про разные значения айкью говорим.
Аноним 07/02/24 Срд 22:40:22 587918 988
>>587916
>Какого, спрашивается, хуя.
Эффект сперматозоида.
Аноним 07/02/24 Срд 22:42:47 587919 989
>>587806
То есть нет ничего сверх? Всё можем делать и мы сами своими руками?
Аноним 07/02/24 Срд 22:51:50 587920 990
>>587919
>То есть нет ничего сверх?
А чего тебе надо сверх?
>Всё можем делать и мы сами своими руками?
Тут как с телефоном в твоей руке - ты можешь примерно представлять, как он работает, но хуй ты его сделаешь своими руками.

На данный момент мы не можем проектировать организмы с нуля, но можем модифицировать существующие организмы.

Если для производства телефонов у нас есть куча высокотехнологичного оборудования, то для создания организмов с нуля пока такого оборудования нет.
Аноним 08/02/24 Чтв 04:17:08 587923 991
>>587916
>Очевидно, что бомжи-алкаши-наркоши есть биомусорный скот, который по определению тупой.
Интеллект определяет успех только на 20%. Остальное это личностные качества и среда.

>В связи с отсутствием генов, непосредственно влияющих на интеллект
Соре, но гены на интеллект пожалуй отсутствуют конкретно у тебя, учитывая что тебе в ебальник буквально кинули инфу о 1к генов ответственных за интеллект и исследования корреляции генетики с интеллектом аж в 77%.

>каждый из которых влияет косвенно, а не прямо.
Шиз, нет никакого гена ума, есть совокупность генов, которая и формирует определённый уровень интеллекта.

>Первый раз о таком слышу, пруфы будут?
Смысл в том что что активные нейроны чаще подвергаются мутациям, а при интеллектуальных нагрузках большее кол-во нейронов активно. Исследования сам ищи
Аноним 08/02/24 Чтв 05:24:17 587924 992
Аноны. Заканчиваю заочно специалитет геология нефти и газа. Имею опыт работы по специальности 5+ лет. Задумываюсь о поступлении в аспирантуру, но чё то сомневаюсь в своих силах. Я даже не представляю, что там делают, там же на лекции особо не нужно ходить пердеть, так ведь? Какую-то исследовательскую работу проводить надо, хуе мое...Аноны аспиранты, вот чем вы занимаетесь? Спрашиваю всех, но если есть геологи, то ещё лучше. Ведь рано или поздно я все равно поступлю в аспирантуру, мне только нужна более лучшая теоретическая подготовка.
Аноним 08/02/24 Чтв 08:03:46 587926 993
>>587923
>Интеллект определяет успех только на 20%. Остальное это личностные качества и среда
Люди, добровольно принимающие алко и наркоту автоматически выписываются из категории умных

>тебе в ебальник буквально кинули инфу
Я тебе цитаты из твоей же статьи кидал

>есть совокупность генов
Правда чтоли?))) Как это противоречит моим словам?

>Смысл в том что что активные нейроны чаще подвергаются мутациям, а при интеллектуальных нагрузках большее кол-во нейронов активно. Исследования сам ищи
Мутировал с этой информации
Аноним 08/02/24 Чтв 08:32:14 587928 994
>>587926
>Люди, добровольно принимающие алко и наркоту автоматически выписываются из категории умных
Как бы так сказать... Ты несколько не в том положении чтобы определять кого-то в умных или тупых на основе косвенных признаков. Типа это как если бы бомж с помойки подходил бы к людям и рассказывал бы им как стать успешным.

>Я тебе цитаты из твоей же статьи кидал
Так ты бы их прочитал для начала, прежде чем в штаны срать. Там прямым текстом пишется что генетическая наследственность интеллекта огромная
>irrefutably supports a large genetic influence on IQ

>Как это противоречит моим словам?
В твоём копиумном манямире пожалуй никак. В реальности это подтверждает что интеллект генетически наследуется. Впрочем, продолжать диалог с дауном, который на полном серьёзе оперирует терминами типа "ген умности" смысла пожалуй не имеет.
Аноним 08/02/24 Чтв 09:58:58 587929 995
А если взять полстакана воды и начать увеличивать количество воды в нем по одной молекуле, существует ли такое минимальное количество молекул, начиная с которого невооруженным глазом будет видно, что вот сейчас воды точно больше, чем половина стакана? Если допустить, что не существует, то это противоречит тому, что рано или поздно стакан наполнится и точно будет видно, что воды в нем больше половины. Если допустить что существует, то придется допустить, что глаз способен уловить разницу в одну молекулу.
Аноним 08/02/24 Чтв 10:33:08 587930 996
>>587929
У тебя есть эффект испарения жидкости, который опустошает стакан.
Тебе нужно добавлять х молекул, испаряющихся ежесекундно + 1 молекула лишняя минимум.

Правда наполнять ты свой стакан будешь миллионы лет.
Аноним 08/02/24 Чтв 10:57:09 587931 997
>>587928
>>irrefutably supports a large genetic influence on IQ
Со значением в 0.77 у однояйцевых близнецов.

Для начала я бы предложил тебе пойти и почитать, что такое корреляционная зависимость, чтобы понять, что означает число 0.77.

Пока ты читаешь определение, даю сразу вывод. 0.77 означают, что у генетически идентичных людей разный iq.

>Ты несколько не в том положении чтобы определять кого-то в умных или тупых на основе косвенных признаков.
Хммм, дай-ка подумать
>обладает интеллектом
>следовательно, осознаёт опасность и последствия при юзании веществ
>всё равно сидит на них
По-моему в данном случае абсолютно поебать в каком я положении. Либо человек умный и не торчит в говнину, и тогда у ребёнка нормальная семья, либо он торчит в говнину и ребёнок живёт в среде наркоманов. Но тогда торч тупой.

>В реальности это подтверждает что интеллект генетически наследуется.
Это ещё Дарвин подтвердил. У тебя в истории селективный отбор произошёл от тупых приматов до хомо сапиенс.

Ты вопрос, блядь, читал? Там буквально написано "предскажите, кто будет тупее, ребёнок белых, или метис"
Аноним 08/02/24 Чтв 11:00:50 587932 998
Одебилевшая полоумная трансшалава засрала тред, не удивлюсь если моча и есть эта трансшалава.
Аноним 08/02/24 Чтв 11:08:51 587933 999
15199674658550-b.jpg 69Кб, 750x1125
750x1125
>>587932
>Одебилевшая полоумная трансшалава
Зачем же ты так про себя? Надо быть терпимым к самому себе
Аноним 08/02/24 Чтв 11:18:00 587934 1000
>>587932
Да это просто копиумный дуралей какой-то, видимо увидел что его родители туповаты вот и сидит коупит что интеллект генетически не передаётся, несмотря на кучу исследований, лол
Аноним 08/02/24 Чтв 11:50:28 587936 1001
>>587934
>интеллект генетически не передаётся
Двачеры не умеют читать.
Хотя, чего я вообще ожидал...
Аноним 08/02/24 Чтв 12:01:17 587937 1002
>>587929
> что глаз способен уловить разницу в одну молекулу
Для этого нужно еще чтобы при повторных тестах совпадало количество молекул при которых челоек улавливает разницу. Тоесть типа при 999999 дополнительных молекулах разницы не заметно, а при 1000000 уже да, и так в каждом тесте. В реальности (скорее всего) человек будет говорить что воды стало больше в относительно случайные промежутки времени, после того как воды в стакане стало заметно больше.
Аноним 08/02/24 Чтв 12:41:08 587939 1003
>>587931
>обладает интеллектом
>следовательно, осознаёт опасность и последствия при юзании веществ
>всё равно сидит на них

Интересно, сколько вам лет, такой максимализм и тем более в сайентаче. Нужно немного подумать и поразмышлять чтобы понять что твой гринтекст очень далек от реальности, первый пункт не имеет значения - щас скажешь на больших числах все видно, но об этом ниже, второй пункт не имеет значения - осознавать мало, точнее оно вообще ничего не значит, иначе бы все люди давно роскомнадзорнулись от осознаия всякой хуйни вроде неизбежности смерти, сидеть на них это уже крайняя стадия, почти безвозвратная, важно не пробовать, а вот это мало связано с интеллектом, даже скорее наоборот, гугли исследования что люди с высоким IQ более склонны попробовать так как любознательны от природы, а у тебя видимо ошибка в том плане, что ты думаешь что все нарики это подъездные поцики из гопрайонов и трущоб, но даже тут нет никакой связи с интеллектом, так как низкий интеллект и употребление наркотиков два следствия других факторов, а не одно причина другого. Короче подрастешь, узнаешь побольше, научишься думать поймешь что генерировал бред, я его примерно 12 контраргументами могу разъебать в пух и прах, но тратить время на юных максималистов младше 40 лень, я тоже был когда-то такой, лет в 20. Потом пришлось немного поработать с этими людьми и я с удивлением обнаружил очень много талантливых и умных в своих областях людей, которых почти безвозвратно засасывает эта история.
мимо
Аноним 08/02/24 Чтв 12:47:07 587940 1004
>>587875
>Как ты объяснишь умных детей алкашей, наркотов и прочих бомжей?
>>587939
>а у тебя видимо ошибка в том плане, что ты думаешь что все нарики это подъездные поцики из гопрайонов и трущоб
У меня изначально стоит условие, что ребёнок из семьи нарков.

>гугли исследования что люди с высоким IQ более склонны попробовать так как любознательны от природы
А где мне гуглить исследования, что большинство алкашей и торчей - обязательно умные челики с высоким iq?
Аноним 08/02/24 Чтв 12:55:58 587941 1005
>>587939
Так же добавлю, что бы мой пост не был расценен как пропаганда употребления наркотических веществ: не в коем случае не пробуйте и не употребляйте наркотики, это низкоуровневый хак тонкой нейрохимической машины - коей является мозг, против него никакая защита вроде высокоуровневой интеллектуальной осознавалки не сработает, но будут работать только очень жесткие кнуты и очень сладкие пряники, и тоже низкоуровневые, на которых потом всю жизнь сидеть вместо наркоты, а такие кнуты и пряники доступны не только лишь всем, почти никому. Наркотики - это жизнь мимо.
Аноним 08/02/24 Чтв 13:49:03 587943 1006
Не алкаш, а гений.
Аноним 08/02/24 Чтв 19:08:58 587953 1007
image.png 108Кб, 640x391
640x391
Аноним 08/02/24 Чтв 21:07:09 587955 1008
Аноним 08/02/24 Чтв 23:48:55 587956 1009
>>587941
Программист что ли?
Аноним 09/02/24 Птн 14:50:24 587964 1010
Бесконечноти в степени бесконечность это ментальная гимнастика у математиков или уже просто шутка? Забойно смотреть, но смысла никакого бесконечность даже не число.
https://www.youtube.com/watch?v=O97xUyMUkJE&t=793s
Аноним 09/02/24 Птн 15:20:27 587965 1011
>>587964
Ну он же говорит, что алеф-ноль понадобился для континуум-гипотезы, а это самые основания математики. Может из всего этого какие-то еще прикладные методы выросли, но я не знаю, сомневаюсь.
Аноним 09/02/24 Птн 20:51:24 587970 1012
>>587929
Чел. Без обид. Из-за смачиваемости, глаз постоянно обманывается, потому что вода смачивает стенки обычного стакана и визуально дефектит оценку.
Мало того, мозг также обманывается, потому что сочетанием цвета стакана и жидкости можно обмануть глаз. А форма сосуда может обмануть даже тренированное сознание.
Так что, ответ - нет. Невозможно. Только опытным путем, методом тыка, эмпирики и на зубок.
Аноним 09/02/24 Птн 23:04:46 587972 1013
949708.jpg 61Кб, 375x415
375x415
можно ли снять с себя статическое напряжение, если помыть руки под краном????
Аноним 10/02/24 Суб 00:47:29 587973 1014
Что за фигня с "нельзя получить линзами интенсивность плотность излучения выше чем плотность излучения на поверхности источника излучения"?
Существует же светодиодные "лазеры", где просто светодиод, и излучение от него сжимают в маленькую точку которой режут допустим фанеру.
Да и те же оптоволоконники, и co-2 лазеры, и твердотельные лазеры, в них же активное тело лазера большое, толстое, а не шириной с будущий лучь, и "реакция" идёт по всей площади сечения активного тела, и потом собирают и сжимают излучение со всей площади сечения, а не с той лишь части, какая толщина у требуемого луча. Иначе бы смысла не было делать такое большое активное тело и всё его накачивать, делаои бы в несколько сотых миллиметра диаметром, если один фиг всё излучение с боков просто отсекать.

Да, в светодиоде и в лазерах вынужденное излучение, но какая нахер разница вынужденное излучение или тепловое тёмнотельное, суть же в поверхности, и у светодиода и у лазера поверхность, и спокойно сжимают излучение выше плотности поверхности.
Аноним 10/02/24 Суб 00:49:25 587974 1015
>>587972
Дистиллятом?
Водой с солью скорее всего точно можно, если непрервной струёй.

Вообще электричество это магия.
Аноним 10/02/24 Суб 10:21:34 587975 1016
>>587973
>то за фигня с "нельзя получить линзами интенсивность плотность излучения выше чем плотность излучения на поверхности источника излучения"?
Тут примерно так. Каждый фотон летит по всем мысленным траекториям, даже те, которые как бы обратно во времени от приемника к источнику. Потом берутся комплексные амплитуды и векторно суммируются.
Поскольку у нас фотон не один, а условное вещество одно, то взаимодействие между ними определяется разностью фаз фотонов.
Когда мы используем йоба-линзы, мы делаем такую систему, что обратные ходы луча начинают вносит огромный вклад в комплексную амплитуду, в итоге фотоны будут локализоваться больше на источнике, а не в фокусе.
С лазерами все по другому, там все фотоны имеют одну и ту же фазу и взаимодействие получается как бы однофотонным, что радикально отличается от первого случая.
Аноним 10/02/24 Суб 10:54:15 587976 1017
>>587975
Ты пропустил часть про светодиодные "лазеры" которые лазерами не являются.
Поэтому твоё объяснение слито. Мир так не работает, живи теперь с этим.

И врядли ты сможешь приписать это фантазийное описание к реальной линзе, и объяснить на уровне геометрии линзы в целом и отдельных атомов, почему когерентное излучение собирается плотнее поверхности сечение источника, а некогерентное нет.
Аноним 10/02/24 Суб 10:57:18 587977 1018
>>587975
>и взаимодействие получается как бы однофотонным
И пути у всех фотонов в линзе разные, она ж с площади собирает в условную точку.
Естественно там не будет совпадения фаз идеального во всех местах у всех фотонов, наоборот полная каша в любом случае получится.

Короче не очень совпдающее с реальностью объяснение.
Аноним 10/02/24 Суб 11:02:47 587979 1019
>>587976
>И врядли ты сможешь приписать это фантазийное описание к реальной линзе, и объяснить на уровне геометрии линзы в целом и отдельных атомов, почему когерентное излучение собирается плотнее поверхности сечение источника, а некогерентное нет.
Т.е. ты говоришь о каких-то "мешаниях", которые предсказывает такая скажем не физическая, а математическо-статистическая теория, при рассмотрении такого общего случая
Но если исследовать о дельно рассматриваемые частицы, не в общем примере с линзой, то никакого свойства "мешания" у них не обнаруживается как не старайся.

И теория не может объяснить почему не обнаруживается отдельно никакого мешания, а только даёт предсказание, не факт что правильное, для оюрассмотрения общего случая с линзой, получается теория не полна как минимум.
Ну и то что про светодиодные лазеры писал, которые лазерами не являются.
Аноним 10/02/24 Суб 13:03:09 587984 1020
>>587974
В кранах дистилированная вода не течет. И да, непрерывной струей.

Ок, а если стоя в резиновой обуви поссать на землю, то тоже получится?
Аноним 10/02/24 Суб 14:22:15 587987 1021
>>587984
Ну смотри, статический заряд стекает по проводнику, по проволоке например. Естественно непрерывные штуки считаются проводником.
Струя пойдёт теоретически.

Но вот что странно, статический заряд растекается по поверхности тел когда образуется, в том числе и по диэлектрику.
Собственно схерали статическому заряду не стекать например по верёвке как по по проводу металлическому-непонятно.
Аноним 10/02/24 Суб 18:52:44 587992 1022
>>587793
>С обработкой ты можешь хоть кору жрать, гугли берёзовую муку, например и берёзовый хлеб.
А как их правильно обрабатывать? Есть полный "список рецептов"?

Коршеки/крапиву в действе я вполне кушал и усваивал, но то обычное баловство было. Мне, конечно, было бы это интересно на поток поставить. Возможно ли половину своего рациона заменить "продукцией" из ближайшего леса?
Аноним 10/02/24 Суб 19:31:53 587993 1023
>>587992
>А как их правильно обрабатывать?
В большинстве случаев это дробление и варка, либо запекание.
Полный список можешь поискать во всяких инструкциях по выживанию, либо в народных кухнях.

>Возможно ли половину своего рациона заменить "продукцией" из ближайшего леса?
Нет, если исключить животных.
Даже если материал можно сьесть, это не значит, что в нём много полезных веществ, которые нужны нашему организму. Чем меньше полезного в еде, тем больше этой еды надо съесть. Для многих ресурсов это увеличение настолько критично, что ты можешь кушать 24/7 без остановки, и всё равно умереть от истощения.
Аноним 11/02/24 Вск 06:35:05 587996 1024
16191921293883.jpg 282Кб, 1200x1180
1200x1180
День добрый, какие учебники по логике и риторике посоветуете?
Аноним 11/02/24 Вск 09:30:43 587997 1025
>>587996
По логике Виноградова можно почитать
Аноним 11/02/24 Вск 13:24:35 588010 1026
>>587996
Онлайн курсы сначала пройди на степике, не сри в голову учебниками, все учебники рассчитаны на то, что их будет разжевывать преподаватель
Аноним 11/02/24 Вск 20:42:21 588030 1027
>>584346 (OP)
Если микроволновые волны в микроволновке прибивают всю еду навылет, то почему если навалить кучу еды в микроволновку, она нагреется только с краев? Микроволна не может пробить холодный вареник?
Аноним 11/02/24 Вск 20:44:02 588031 1028
Аноним 11/02/24 Вск 21:36:51 588034 1029
>>588030
Вообще довольно неоднозначно.
Она конечно не с краёв нагревается, а по-другому.
Но действительно должно всё довольно равномерно быть внутри микроволновки, если с учётом отражения от стенок, длины волны и расстояния от стенок до других стенок внутреннего отсека.
Короче не понимаю откуда там стоячим волнам взяться.
Аноним 11/02/24 Вск 22:29:00 588038 1030
>>588030
Если б они проходили насквозь, еда бы не грелась. В микроволновке она нагревается от поверхности вглубь.
Аноним 12/02/24 Пнд 12:49:46 588051 1031
>>588010

Попробую, спасибо.
Аноним 13/02/24 Втр 06:05:27 588078 1032
Почему зачатие ребенка как бросок кубика, может быть белым черным и вообще кем угодно
Аноним 13/02/24 Втр 13:11:44 588114 1033
>>588078
Человеки в себе содержат два набора генов.
При половом процессе у каждого родителя нужно выкинуть половину генов, а потом снова сшить.
На каждом процессе разделение и соединение генома происходит перетасовка генов.
В этом же суть полового процесса.
Аноним 13/02/24 Втр 13:21:33 588115 1034
>>588114
Там немного сложнее. Ещё есть аллеи, типа скрытые гены. Которые могут проявится через несколько поколений.
Аноним 13/02/24 Втр 13:25:32 588118 1035
>>588115
Мой дед алкаш. Мой отец алкаш. Шансы у меня быть алкашом если я пью 5 дней в неделю?
Аноним 13/02/24 Втр 13:43:18 588121 1036
>>588115
Аллей гена это и есть вариант из второго набора.
То что какой-то ген проявляется через н-поколения это уже особенности экспрессии генов, которые зависят от не наследуемых факторов.
И вообще генотип не есть фенотип. По отдельным внешним признакам нельзя судить какие гены как наследуются.
Аноним 13/02/24 Втр 13:48:39 588123 1037
>>588078
Потому что ген это ген некой совокупности всех прошлых поколений, а не только то что проявилось в этот раз в этом выросшем плоде.
Т.е. информации в гене содержится намного больше чем нужно для постройки человека, и переносится эта вся информация целиком, а не только то что потребовалось для постройки.
Ну естественно примерно 50/50 от обоих родителей выуидывается конечно и смешивается оставшееся.
Аноним 13/02/24 Втр 13:56:22 588126 1038
>>588118
Алкоголизма не существует, просто обычное недалёкое рабочее быдло.
Если и дед и отец такими были, то близка к 100% вероятность.
Да собственно ты уже и так понятно что такой, раз 5 дней в неделю пьёшь.
Аноним 13/02/24 Втр 14:00:22 588129 1039
>>588126
Сегодня я бросил
Аноним 13/02/24 Втр 14:15:47 588133 1040
>>588129
Как и пару дней назад бросал?
Аноним 13/02/24 Втр 14:26:16 588135 1041
Аноним 13/02/24 Втр 23:41:32 588166 1042
>>588126
алкоголизм вполне себе существует. у него есть вполне четкие критерии.
ну пусть его переименовали в синдром зависимости
Аноним 14/02/24 Срд 13:31:22 588182 1043
17079056786440.png 412Кб, 480x640
480x640
Когда бездельники-грантоеды наконец создадут мне кошкодевочку?
Аноним 14/02/24 Срд 13:58:33 588183 1044
>>588182
Захватившие власть фашиствующие моралисты мешают со своей этикой говнетикой
Аноним 14/02/24 Срд 14:42:01 588186 1045
>>588182
А ты тут причём? Ты платишь налоги не с условием
чтобы тебе что-то создавали, а потому что ты рабочая особь, потому что такое твоё предназначение.
Аноним 14/02/24 Срд 16:19:57 588190 1046
>>588183
В Китае нет ни религиозных мракобесов, ни SJW леваков, там могли бы заняться генетическими исследованиями. Тем более там наибольшее число инцелов, которые страдают без кошко-жены.

>>588186
Обслуживающий персонал забыли спросить. Небось, жалкий учителишка, которому в лицо харкают старшеклассники? Или нищий препод, живущий на взятки от студентов-мажоров?
Аноним 14/02/24 Срд 16:23:18 588191 1047
>>588190
Зашивайся порванный рабочий хуесос, я не работаю 😉
Аноним 14/02/24 Срд 16:39:03 588193 1048
>>588191
Ну, так бы сразу и сказал, что школьник.
Аноним 14/02/24 Срд 17:04:02 588196 1049
>>588193
Ты чего порвалась, рабочая особь
Тебе должна нравится твоя биологическая роль, ты для этого создан.
А вот эту хуйню навроде "когда тебе сделают" ты оставь, нето пиздюлей получишь обучающих.
Аноним 14/02/24 Срд 18:39:27 588199 1050
наука полностью отрицает жизнь после смерти, или все же такой вариант возможен, если кто знает есть ли действительно какие нибудь исследования в которых что то удалось узнать или предположить на эту тему, посоветуйте литературу
Аноним 14/02/24 Срд 19:41:56 588201 1051
>>588199
Как ты предлагаешь это исследовать?
Случаи памяти о реинкарнации от человека к человеку записываются, проверяются и систематизируются. Регрессивный гипноз ненадежен.
С остальными вариантами загробной жизни ты сейчас ничего не сделаешь.
Выход из тела при NDE? Мы не знаем как он работает.
Аноним 14/02/24 Срд 20:02:58 588203 1052
>>588201
>Выход из тела при NDE? Мы не знаем как он работает.
Легко же эксперимент поставить.
Аноним 14/02/24 Срд 20:23:41 588204 1053
>>588203
Говорят, в некоторых операционных ради смеха приклеивают где то под потолком бумажку вроде "в случае выхода из тела позвоните по номеру хххххх". Но пока еще никто не позвонил.
Аноним 14/02/24 Срд 20:31:11 588205 1054
>>588204
Ну делать нехуй во время выхода из тела циферки какие-то запоминать
А если он потом умирает, то и не позвонит никак.

Другая методология эксперимента должна быть.
Аноним 14/02/24 Срд 20:31:25 588206 1055
>>588203
Каким образом? Эксперименты по проверке выхода из тела организовать в принципе можно (хоть и крайне муторно), но чтобы использовать эту хрень для попыток исследования загробной жизни нужна возможность вызывать ее по желанию, а тут проблема.
Аноним 14/02/24 Срд 22:03:31 588208 1056
>>588206
Ну вводить в состоянии исскуственной смерти
Снача с завязанными глазами привозить в какое-то новое место, где подопытный никогда не был, и там вызывать у них состояние искусственной смерти.

>Эксперименты по проверке выхода из тела организовать в принципе можно (хоть и крайне муторно), но чтобы использовать эту хрень для попыток исследования загробной жизни нужна
Так это одно и то же. Если доказан будет выход из тела, то уже сразу станет понятно что всё не так просто и нечто подобное скорее всего есть(жизнь после смерти)

И что значит не могут по желанию вызывать?
Ну конечно это довольно опасно, но аполне сеье легко умеют вызывать искуственную смерть.
Естественно далеко не после каждого такого случая подопытный будет утверждать что выходил из тела
Ну так хули, как всегда в науке нужен сбор большой статистики с помощью большого количества экспериментов.
Аноним 14/02/24 Срд 23:43:48 588209 1057
>>588208
Статистика и так собирается, необходимость срочно рисковать человеческими жизнями ты хрен обоснуешь. Про "крайне муторно но возможно", я имел ввиду получить согласие у большого количества добровольцев, договориться с несколькими больницами и хирургами.
>Так это одно и то же.
Это ни разу не одно и тоже. Опыт выхода из тела, который к тому же испытывают не все, не доказывает возможности существования разума вообще без тела и может быть объяснен способностью мозга сканировать окружающую среду или еще какой хренью.
Аноним 15/02/24 Чтв 04:42:04 588215 1058
>>584346 (OP)
Поцоны, я тут ходил в радиоактивную пещеру, потом лег спать в гамаке. Так вот у моего гамака снаружи постоянно какой-то пиздец творился. Мой гамак наглухо закрывается, я его липучками прошил, так вот половину ночи в него словно из прожектора светили то фиолетовым, то красным, то синим, то желтым цветом. Это сетчатка глаза от радиации чутка с ума сошла?
Аноним 15/02/24 Чтв 15:09:36 588230 1059
>>588215
Вряд ли.
Чтоб так светилось, тебе нужно было залезть в активную зону ядерного реактора.
Аноним 15/02/24 Чтв 15:54:27 588232 1060
Бобрышевский где-то говорил, что за речь и пение отвечают разные разделы мозга. Ничего не могу нагуглить, есть сведущие?
Аноним 15/02/24 Чтв 16:32:51 588235 1061
>>588230
Свечение может быть глюком.
Аноним 15/02/24 Чтв 21:37:14 588240 1062
>>588230
Да не, это не в реальности светилось, а только в моих глазах.
Аноним 15/02/24 Чтв 21:38:01 588241 1063
>>588232
Используя внутричерепные записи мозговой активности, американские ученые обнаружили нейронную популяцию, которая избирательно реагирует на пение, но не на инструментальную музыку или речь. Эта популяция нейронов расположена в верхней части височной доли, рядом с областями, избирательно отвечающими за язык и музыку.
Аноним 15/02/24 Чтв 23:22:32 588242 1064
>>588240
Скорее всего просто самовнушение.
Ну или глюки как бы реальные, но тоже из-за самовнушения.
Аноним 17/02/24 Суб 13:35:34 588284 1065
Есть какие-то профиты от коллективного разума? Или автономный юнит всё-таки гораздо эффективней?
Выглядит так, словно бы хайв майнд будет постоянно перегружен одновременным микроменеджентом кучи юнитов, которые даже посрать сами не могут.
Аноним 17/02/24 Суб 18:04:16 588291 1066
>>588284
Коллективный разум не отключает тебе мозг, он только заставляет считать всех людей кроме лидера частью себя и дает информацию из мозгов специалистов.
Аноним 17/02/24 Суб 18:26:32 588292 1067
Какие надо читать сайты, чтоб быть вкурсе всех прорывных открытий? Ну например в физике и биологии

Типо прям вот последний писк моды науки
Аноним 17/02/24 Суб 20:24:31 588295 1068
>>588284
>Или автономный юнит всё-таки гораздо эффективней?
Автономный юнит это ползающий по земле кусок мяса с деформированными органами и без помощи со стороны абсолютно нежизнеспособный, воспитанные хотя-бы волками дети Маугли в разы жизнеспособней (эффективней).
Аноним 18/02/24 Вск 21:26:43 588335 1069
Мне как-то попадалась картинка для троллинга, там был сойджак с открытом ртом, на футболке была эмблема "i f*cking love science" на лбу надпись 666, а к голове приделаны антенны от wifi роутера типо рога дьявола, очень нужна сейчас, но в гугле не нашел, скиньте у кого есть пожалуйста.
Аноним 19/02/24 Пнд 10:46:26 588349 1070
Наука наука наука встаем с колен, дадим пососать пиндосам. Приходишь с проектом теоретического исследования и препринтом. ОЙ, НУ ПОТОМ ПОСМОТРИМ, ОЙ НУ ЧЕТ Я НИЧЕГО НЕ ПОНИМАЮ, ОЙ А КАКАЯ АКТУАЛЬНОСТЬ, блаб
Аноним 19/02/24 Пнд 13:13:45 588359 1071
>>588349
>Приходишь с проектом теоретического исследования
Нахуй тебе приходить к кому-то, если исследование теоретическое?
Сиди пили сам.
Аноним 19/02/24 Пнд 13:47:47 588362 1072
> А КАКАЯ АКТУАЛЬНОСТЬ
И в чем он не прав кста?
Аноним 20/02/24 Втр 20:10:33 588423 1073
Аноним 20/02/24 Втр 23:42:52 588435 1074
>>588423
Как же тупые торчи доебали, надеюсь модер тебя забанит к хуям
Аноним 21/02/24 Срд 02:58:52 588438 1075
>>588435
Не нравится - Не читай, Пидор!
Аноним 21/02/24 Срд 03:09:12 588440 1076
>>588423
Наркоман, штоле? Или в глаза долбишься? Гуголь есть жи, жи ежжи, чертила

>Есть два основных пути:
>1: ванилин -> 5-йодванилин -> 5-гидроксиванилин -> 3,4,5-ТМБА -> 3,4,5-ТМНС -> мескалин.
>2: ванилин -> 5-бромванилин -> 3,5-диметокси-4-гидроксибензальдегид -> 3,4,5-ТМБА -> 3,4,5-ТМНС -> мескалин.
На какой путь сядешь, на какой мать подсадишь?
Аноним 21/02/24 Срд 18:24:33 588481 1077
Как с эволюционной точки зрения объяснить любовь людей к прошлому?
Аноним 21/02/24 Срд 18:53:40 588483 1078
>>588481
Старением и смертью.
Аноним 21/02/24 Срд 23:14:56 588504 1079
Мы привыкли, что говоря про квантовую механику, волны изображают то в одномерном виде (линия графика), то на двумерной плоскости (пики и волны на ней).

Как же они визаулизируются в реальности? Они же, естественно, трёхмерны - их можно представить как колеблюющуся полупрозрачную округлую хуйню? Как боровские орбиты элкктронов в 3х измерениях?
Аноним 21/02/24 Срд 23:21:46 588505 1080
>>588504
Судя по нарушению неравенства Белла их не существует "до".
Аноним 21/02/24 Срд 23:41:16 588506 1081
>>588504
> Как же они визаулизируются в реальности?
Ни как. Если бы их могли видеть, то не было бы ни каких проблем. Волна это условная метафора к волнам на воде, но не имеющая к волнам на воде ни какого отношения.
Аноним 22/02/24 Чтв 07:09:33 588508 1082
>>588504
Можешь в виде яркости представлять.
Аноним 22/02/24 Чтв 10:20:03 588509 1083
>>588506
Да, я имел в виду, визуализация в голове, как это представлять.

>>588505
Факт.

>>588508
Да. Волна вероятности в виде неравномерных светящихся сгустков. Спасибо.
Аноним 22/02/24 Чтв 12:09:39 588516 1084
>>588504
В реальности у тебя будет точки/вспышки, никаких волн.
Только если ты дохуя раз измеряешь, то получишь неясное месиво. В случае электронов говорят об электроном облаке.

Если визаулизировать волновую функцию, то есть проблемы из-за комплексности. И это только для скалярных частиц. У электронов волновая функция многокомпонентная, да вдобавок еще является спинором. Цветов и измерений не хватить.
Аноним 22/02/24 Чтв 13:29:45 588519 1085
>>588509
>визуализация в голове,
Ни как.
>как это представлять.
Математическими абстракциями
Аноним 22/02/24 Чтв 13:30:34 588520 1086
>>588504
Как Фейнман советовал гуманитариям представлять. Типа у частицы есть "часы стрелочные" которые вращаются.

А если ты нормально с матаном дружишь, то это волны интуитивно представляется.
>>588509
> неравномерных светящихся сгустков.
Волна вероятности может не иметь максимумов и минимумов, банально плоская бегущая волна. Так что ты потеряешь инфу, если не волновую функцию будешь представлять квадрат модуля.
Аноним 22/02/24 Чтв 17:08:24 588530 1087
>>588516
Шизоид, какие точки, какие волны
"Волна" фотона видимого света пол микрона
Ты точку такую не увидишь какого размера он волна.
Аноним 22/02/24 Чтв 19:47:45 588535 1088
>>588126
>Алкоголизма не существует
Алкоголизм - не просто существует, его можно простыми методами показать и доказать. Хватит уже писать ерунду сюда.
Подтяни биологическую часть и медицину.
>Если и дед и отец такими были, то близка к 100% вероятность.
Лол.
Генетического алкоголизма не существует, но есть некоторые факторы к экзогенному этанолу, которые могут способствовать быстрому развитию алкоголизма. Просто потому, что у этаноловых людей существует нарушение обмена веществ, и это нарушение можно передать генетически. Но это не алкоголизм, это факторы простетических групп, которые могут быть переносимы в геном следующей особи в поколении.
Аноним 22/02/24 Чтв 19:48:56 588536 1089
>>588129
>бросил
Надо не бросать, а заменять эндорфиновым допингом спорта или секса. Или обжорства.
Будешь жиробасом, ебакой и качком-дурачком.
Аноним 22/02/24 Чтв 19:49:34 588537 1090
>>588166
Один учоннный тут остался...
Аноним 22/02/24 Чтв 23:57:23 588543 1091
>>588516
Пидорюш, тыскозал?
Аноним 23/02/24 Птн 11:23:39 588554 1092
На хуй вашу навуку. Проще заработать в тиктоке.
Аноним 25/02/24 Вск 12:11:37 588615 1093
Сап двач. MOND по вашему мнению тру или не тру? Какие новости?
Аноним 25/02/24 Вск 12:18:50 588616 1094
17088513316630.mp4 13658Кб, 1280x720, 00:01:20
1280x720
slide-1.jpg 103Кб, 1024x767
1024x767
7d16d28b1981d16[...].gif 44Кб, 600x510
600x510
>>584346 (OP)
Анчоус, почему уже столько итерации в роботехнике, а используют манипулятор стопы в такой ущербной форме?

Звездообразная, естественно упрощенная, чтобы не застревала, выгоднее же? И по гироскопической оптимизации. И по распределению веса. Можно сделать статично-гибкими, а можно отдельный инструмент в виде манипуляторов-палец, которые будут корректировать ориентацию в пространстве. Почему?
Аноним 25/02/24 Вск 12:19:03 588617 1095
>>588615
и вообще, на каких ресурсах постоянно актуализируется информация по научной деятельности, теориям и их экспериментальным подтверждениям, чтобы не копаться в статье казалось бы 10 леьней всего лишь давности, а потом хоба и она признана несостоятельной не так давно и хуй проссышь.
Аноним 25/02/24 Вск 12:49:42 588619 1096
>>588616
>Звездообразная выгоднее же?
А в чем именно выгода? Манипулятор у робота сделают там, где он нужен, а не там, где была свободная конечность. Твои птичьи когти нужны для того, чтобы хватать добычу. А роботу это зачем? У лошади копыто и никто не жалуется.
Аноним 25/02/24 Вск 13:11:46 588620 1097
>>588619
Упоры по нескольким точкам плоскости, большая площадь соприкосновения с меньшим объёмом, распределение нагрузки, облегчение коррекции гироскопии. Ясно, что и на ходулях можно ходить и на колесе скакать, вопросов нет, но звезда выгоднее. А у лощади слишком долгий опыт обучения и модификации стопы и двигательно-координационного аппарата, как вида.
Аноним 25/02/24 Вск 13:20:30 588621 1098
>>588620
То что ты называешь это не преимущества, а особенности. В каждом конкретном случае, они могут как давать преимущество, так и нет. А то, что у лошади длинная история эволюции говорит только о том, что лошадиное копыто также подтвердило свою целесообразность.
Аноним 25/02/24 Вск 13:48:44 588622 1099
>>588621
>лошадиное копыто также подтвердило свою целесообразность.
Не спорю, для анатомически-конструкционных характеристик этого вида и его физических задач эта стопа подходит. Не про это речь.

>То что ты называешь это не преимущества, а особенности. В каждом конкретном случае
Ок, я возможно пропустил уточняющие в вопросе. Собакообразных и лощадеобразных, если так можно выразиться, мы не рассматриваем. Антропоморфы. Видны проблемы - покачивание корпуса, коррекция и стабилизация координации при сложных вертикальных перемещениях. Стопа в виде прямоугольника дает точку упора и стабилизации переносом веса только в двух направлениях. Трех- четерых- пятилучевая стопа даст множество точек. Прямоугольник не сможет дополнительно пружинить, амортизировать при конечной стадии ходьбы и прыжка, многолучевая это позволяет. При поднятие веса оптимизация координации также более точное, чем использование прямоугольной площадки.
Аноним 25/02/24 Вск 15:30:20 588629 1100
>>588622
> Прямоугольник не сможет дополнительно пружинить, амортизировать при конечной стадии ходьбы и прыжка, многолучевая это позволяет.
С чего ты взял что для текущих задач это необходимо? Ещё не решена задача нормального двуногого прямохождения просто на «прямоугольниках», а ты хочешь ввести ещё кучу суставов, приводов и переменных для обработки ни как не относящихся к решаемой задаче прямохождения.
Аноним 25/02/24 Вск 17:14:39 588631 1101
Если жиробас 100кг спрыгнет со стола высотой 1м на весы, то какое максимальное значение покажут весы?
Аноним 25/02/24 Вск 18:13:20 588632 1102
Begovoj-protez-[...].jpg 393Кб, 2560x1669
2560x1669
1005252-1.jpeg 124Кб, 660x628
660x628
>>588629
>С чего ты взял что для текущих задач это необходимо?
Правильный вопрос и можем утонуть с тобой в полемике, но мой то был не про это. Но прямоходящих делает не только Бостон Дайнемикс, а треть всей робототехники.

>Ещё не решена задача нормального двуногого прямохождения просто на «прямоугольниках»
И для решения задачи нормального прямохождение многолучевая лучше, я даже объяснил почему.

>ни как не относящихся к решаемой задаче прямохождения.
Экономяят, оптимизуруют и позволяют более тонко решать вопросы, напрямую относящиеся к прямохождению, соблюдению баланса и координации.

>а ты хочешь ввести ещё кучу суставов, приводов и переменных для обработки
Eще в первом посте написал.
>Можно сделать статично-гибкими

Хуету на пиках разработали не просто так. Пружинящий импульс амортизирует, дает импульс к толчку, позволяет мягко корректировать координацию.
Аноним 26/02/24 Пнд 02:44:20 588650 1103
>>588631
Чуть меньше прочности костей ног на продольное сжатие.
Аноним 26/02/24 Пнд 03:05:56 588652 1104
Максимально тупой наверное вопрос.

Настаивание воды на серебре. Это же хуйня? Жена заебала, заказал ей 100 грамм слиток чистого серебра, пусть развлекается, но ведь хуйня же? Да?
Аноним 26/02/24 Пнд 05:49:49 588659 1105
>>588652
Если не пьёшь воду из сточной говноканавы то точно никакого смысла.
Есть небольшая вероятность что могут выделяться как-то иога серебра и убивать бактерии и микробы, но схерали им просто в воду брать и выделяться... Плюс именно ионы, а не просто оксиды
В общем, это нужно химиков спрашивать.
Но зачем пить говняную воду? А с хорошей водой точно никаких положительных эффектов не может быть.
Аноним 26/02/24 Пнд 07:51:26 588661 1106
Аноним 26/02/24 Пнд 12:45:14 588672 1107
>>588652
Да, это хуйня.
И пить воду насыщенную серебром даже опасно для здоровья.
26/02/24 Пнд 15:20:54 588682 1108
>>588652
>жена
сажи нормихуесосу
Аноним 26/02/24 Пнд 15:27:48 588683 1109
>>588682
Съебись в /b/, чьмоня.
Аноним 26/02/24 Пнд 20:12:38 588693 1110
> Important remark: The picture of gravitons as the "mediators" of gravity, like photons are the mediators of the electromagnetic interaction, is actually incorrect. Unlike the electromagnetic interaction, gravity is fundamentally nonlinear. But gravitons are the quanta of linearized gravity, which means they are only suitable for describing very weak gravitational fields, such as gravitational waves.
Можете пояснить, что значит ЭМ линейна, а гравитация нелинейна?
Аноним 27/02/24 Втр 12:51:42 588720 1111
>>588693
Линейность на уровне уравнений.
Решения уравнения Максвелла линейны - сумма их решений, является так же решением. Верно и обратное, любое решение можно разложить на сумму других решений.
Уравнения Эйнштейна в общем случае нет. Принцип эквивалентности очень сильный, поэтому требует от уравнений требуется ряд условий, которые убивают на корню всю линейность решений.
Аноним 27/02/24 Втр 15:35:19 588729 1112
>>588720
Нихуя не понял, как это к реальности применить.
Аноним 27/02/24 Втр 15:44:39 588730 1113
>>588729
Школу закончи и в универ поступи сначала
Аноним 27/02/24 Втр 21:39:33 588759 1114
>>588730
Доран, нормально можешь объяснить?
Аноним 28/02/24 Срд 10:02:10 588787 1115
>>588765
Никогда, в реале будет концовка Тонга
Аноним 28/02/24 Срд 12:03:10 588798 1116
>>588796
>биороботов
А ты работаешь?
Аноним 29/02/24 Чтв 06:18:18 588830 1117
2 (1).jpg 640Кб, 2000x1333
2000x1333
Анохин это пиздабол? Слушаю его лекции, постоянно бла бла бла биоэтика бла бла бла перечисление заслуг американских ученых бла бла бла
Аноним 29/02/24 Чтв 06:22:35 588831 1118
Еще и на сомнительные колабы с Черниговской и каким-то маркетолухом ходит. Такое ощущение что наука в пидорашке просто сдохла нахуй, остались одни старпёры и шизики несущие околесицу. Сочувствую всем кому приходится заниматься наукой в таких условиях
Аноним 29/02/24 Чтв 09:36:16 588836 1119
>>588831
Вот Анохину и посочувствуй. Ты думаешь, он как-то по-другому Черниговскую воспринимает?
https://youtu.be/N6FALqdLCXw?t=6962
https://youtu.be/MXxIvvVH6PA?t=6857
https://youtu.be/LH-_NnFZIE8?t=379 (цитату Черниговская форсит)
В Википедии указано, что Черниговская:
>Заместитель директора НБИК Центра Курчатовского института.
Может Анохину приходится с ней "дружить"? Я не знаю.

То, что для тебя они что-то примерно одного уровня, это потому что ты ещё не всё понял. Разница гигантская. Анохин боится лишнего сказать и из-за этого стать пиздаболом, поэтому говорит одно и то же обычно, либо про его исследования памяти, либо про его теорию когнитома.
Аноним 29/02/24 Чтв 20:34:48 588849 1120
Ваши ставки,

когда появится сильный ии на уровне взрослого человека 180 айсикью?

когда появится супер ии/наступит технологическая сингулярность?
Аноним 29/02/24 Чтв 21:29:25 588852 1121
>>588836
Скорей навука в пидерахе просто сдохла, остались лишь нищие пенсионеры с импотенцией уныло отрабатывающие гранты создавая имитацию бурной научной деятельности. У них уже там всё засохло, никакого энтузиазма, никакой цели. Студент первокурсник спрашивает г-н Анохин а создадут ли когда нибудь ИИ и что там для этого нужно? Анохин: РЬЯЯЯЯЯ ЭТО ТЕХНИЧЕСКИ НЕВОЗМОЖНО ИШЬ ЧО УДУМАЛ БОХ НАКАЖЭЭТ. И вот с таким настроем всей этой толпе замотивированных вчерашних школьников учиться у этих совковых дегенератов.
Аноним 29/02/24 Чтв 21:48:49 588853 1122
>>588849
В марте следующего года
Аноним 29/02/24 Чтв 23:58:36 588856 1123
>>588849
Если серьезно подходить, то 1-2 года до того как языковые модели будут на уровне AGI с ограничениями в области пространственного-физического мышления. Есть мнение, что AGI будет необходимо физическое тело для того чтобы нормально функционировать, но я не убежден. Релиз ГПТ-5 не за горами, есть мнение что оно будет способно наконец-то обрабатывать информацию на более высоком семантическом уровне а не пережевывать банальщину как ГПТ-3.
Если нужны мнения из итернета, то смотри здесь, например:
https://lifearchitect.ai/agi/
Или здесь:
https://www.metaculus.com/questions/3479/date-weakly-general-ai-is-publicly-known/
Учти что индивидуальные независимые догадки толпы имеют тенденцию приближаться очень близко к правильному ответу, но когда люди голосуют в открытую, зная ответы своих сотоварищей, они городят хуню, так что суди сам.
Ну и моя собственная догадка: завтра, можете скринить.

>>588852
Тащемта лично знаю одного студента которому отказали в защите кандидатской потому что диссертационному совету не понравилось, что там применялась нейросеть.
Аноним 01/03/24 Птн 03:12:04 588861 1124
сап двачь
химики тут?
мне нужно было продезинфицировать некоторые титановые штуки и я кинул их в перекись водорода (у меня тут не лаба, я обычныый человек)
они блять пожелтели
я не знал что так будет
со сталью такой хуйни не было
щас загуглил рял титан реагирует неплохо так с перекисью
вопрос как в домашних условиях вернуть титану обычный металлический цвет?
Аноним 01/03/24 Птн 07:03:22 588874 1125
>>588856
Nov/2020
30%: Connor Leahy, Co-founder of EleutherAI, re-creator of GPT-2, creator of GPT-J & GPT-NeoX-20B, said about OpenAI GPT-3: ‘I think GPT-3 is artificial general intelligence, AGI. I think GPT-3 is as intelligent as a human. And I think that it is probably more intelligent than a human in a restricted way… in many ways it is more purely intelligent than humans are. I think humans are approximating what GPT-3 is doing, not vice versa.’



Кекус
Аноним 01/03/24 Птн 07:56:18 588878 1126
>>588852
>Студент первокурсник спрашивает г-н Анохин а создадут ли когда нибудь ИИ и что там для этого нужно? Анохин: РЬЯЯЯЯЯ ЭТО ТЕХНИЧЕСКИ НЕВОЗМОЖНО ИШЬ ЧО УДУМАЛ БОХ НАКАЖЭЭТ.

Хватит из головы-то выдумывать, или ссылки откуда ты это берёшь приводи. Он обычно отвечает цитатой, что "сложно прогнозировать, особенно будущее". Чтобы не напиздеть, опять же, потому что он давно живёт, многое видел. Многое, что прогнозировали другие по хайповым темам, потом не сбывалось. Либо речь про то, когда искусственный интеллект обретёт сознание, то он отвечает, ссылаясь на теорию интегрированной информации, какие нужны условия для того, чтобы в системе возникло сознание, а "симуляция урагана на компьютере не обладает качеством мокроты". Про Бог накажет это тоже не к нему, ни про Бога, ни про алармизм как у Черниговской. Анохин всегда наоборот отвечает типа "ну и как вы остановите прогресс? в любом случае, если не мы, то кто-то это сделает". Как будто про какого-то другого человека говоришь вообще - https://youtu.be/GVaiwyEuQwY?si=ZP18JiwuE6HVwLlN&t=3748.

Ты что ли замотивированный вчерашний школьник? Что за вопрос идиотский, когда создадут ИИ? ИИ создан давно. Сильный ИИ ты имеешь в виду или что вообще?
Аноним 01/03/24 Птн 10:25:52 588882 1127
>>588878
Почему у него в лекциях постоянно какие-то пространные философские рассуждения вместо физиологии, обьяснения того как мозг может кодировать информацию, как и где хранится память, какие нейроны активируются, как они друг с другом взаимодействуют?
Аноним 01/03/24 Птн 10:37:49 588883 1128
>>588882
Потому что вопрос сознания всегда был чисто философским вопросом. Потому что это оxyенно, потому что уровень абстрактного мышления позволяет рассуждать на таком уровне. Никакой физиологией трудная проблема не решается.
Про память у него полно лекций. https://www.youtube.com/watch?v=I_sOSo_HRnA
Аноним 01/03/24 Птн 10:48:15 588884 1129
>>588883
https://youtu.be/QTQJzQo9saE?si=lqX7xgJArPwGp3UH по-моему, здесь было дано интересное определение памяти. "Память это способность системы воспроизводить свои прошлые состояния". Не помню точно, должно быть в видосе. Где память? Везде, где есть нейроны, способные модифицироваться в ответ на какое-то воздействие, и потом воспроизводить эти свои прошлые состояния. Это тоже считать за пространные философские рассуждения? Чтобы что-то начинать изучать, нужно сначала это определить.
Аноним 01/03/24 Птн 11:42:50 588887 1130
>>588883
>>588884
Тогда зачем на рассуждения выделяются деньги из госбюджета? Пусть рассуждает бесплатно, либо за мрот, как все рабсияне
>дано интересное определение памяти
Нахуй нам определение? Какие это несёт технологические новшества и научные открытия?

>Это тоже считать за пространные философские рассуждения?
Именно, более того это беспруфный пук. Это реально звучит как научная импотенция, когда мы говорим о памяти мы говорим о её локализации и о том как эти структуры формируют чувство непрерывной самости, в каких нейронах это зашифровано, как это зашифровано, как это раскодировать, как это математически описать, смоделировать, перенести на\в ИИ
29:15 "врёти, ни будит, ни смогут, слишком сложно, т, беспочвенные фантазии пук хрюк" перед этим два часа пиздел о своей полуфилософский хуите
33:43 "технически нереально, этически невозможно" слушай дальше чё старуха несёт, у неё ВРЁТИ на вопрос об уже состоявшемся эксперименте. Перед этим пол часа пиздели об этикехуетике, мням ням нам нужно подумать о будущем мням мням искуственный интеллект должен быть наделен моралью. Просто какой-то лютый пиздёж по своей структуре и наполненности не отличающийся от вопросов школоты про нейроинтерфейсы и когда нас уже загрузят внутрь ИИ. Просто какое-то бла бла бла, кучка престарелых выблядков прожирающих гранты. Посмотри хоть раз в жизни пендосские лекции с настоящими учеными не лишенными запала, каких-то надежд на науку, умеющих рассказать что-то новое просто и интересно, а не вот этот вот пространный пиздёж и тысячу раз пережеванный кал с горделивым апломбом
Аноним 01/03/24 Птн 11:44:59 588888 1131
>>588884
>это способность системы воспроизводить свои прошлые состояния
Мде, звучит как предварительное определение упругости в учебниках по классической механике для ПТУ.

>>588884
>Где память? Везде, где есть нейроны, способные модифицироваться в ответ на какое-то воздействие, и потом воспроизводить эти свои прошлые состояния.
Крч, механика материальных нейронных точек.

>Это тоже считать за пространные философские рассуждения?
Да

>Чтобы что-то начинать изучать, нужно сначала это определить.
Только процесс определения продолжается и во время исследования, тем самым начальное определение отрицается как чисто эвристическое.
Аноним 01/03/24 Птн 11:46:03 588889 1132
Аноним 01/03/24 Птн 11:51:12 588891 1133
170928214866216[...].jpg 280Кб, 1080x2340
1080x2340
Возможно ли восстановить звук по этой пикче?
Аноним 01/03/24 Птн 11:53:49 588892 1134
>>588861
Попробуй кислотой помыть, или шарик пихнуть вместе с кусочком алюминия (чтобы они соприкасались) в раствор горячей соды. М.б. сработает, но может быть и нет. Пожелтеть то может капитально.
Аноним 01/03/24 Птн 12:13:32 588895 1135
170928370915183[...].jpg 280Кб, 1080x2340
1080x2340
170928367889587[...].jpg 341Кб, 1080x2340
1080x2340
170928406402470[...].jpg 234Кб, 1080x2340
1080x2340
Возможно ли восстановить звук по этой пикче?
Аноним 01/03/24 Птн 12:14:09 588896 1136
170928380002012[...].jpg 121Кб, 1080x2340
1080x2340
170928371431789[...].jpg 126Кб, 1080x2340
1080x2340
Дурачусь с магнетометром. Зацените как он видит работающую микроволновку, снаружи.
Аноним 01/03/24 Птн 12:34:08 588899 1137
>>588849

>когда появится супер ии/наступит технологическая сингулярность?
10 травня
>>588896
Что за приложение?
Аноним 01/03/24 Птн 12:46:18 588900 1138
>>588895
Частично да.
Но на пикче потеряна много информации, главное из которых это информация о фазе.
Аноним 01/03/24 Птн 13:35:16 588901 1139
>>588883
Тупая трапошалава, ты? Узнал тебя по тупорылым высерам бабского мозга.
Вот уж ирония да? Мозг у тебя женский, тупорылый, а днк мужское, и женщиной ты никогда не станешь. Так и будешь чушкой-дегенератом с женским мозгом.
Аноним 01/03/24 Птн 13:37:49 588902 1140
>>588899
>10 травни

аххахахкха заорал

Приложение - phyphox
Аноним 01/03/24 Птн 14:39:25 588909 1141
>>588882
>как и где хранится память
Какую часть никто не знает ты не понял?
Есть ряд теорий, структура нейронов, РНК, всё это. Из доказательств только то что таким способом информацию в принципе можно хранить. Может вообще оказаться что память храниться в сознании а мозг выступает как набор указателей.
Аноним 01/03/24 Птн 14:45:07 588910 1142
>>588909
Что за сознание? Где оно хранится?
Аноним 01/03/24 Птн 15:19:45 588914 1143
>>588910
>Что за сознание?
Место, где находятся квалиа и сформированные из них ментальные структуры. Хотя, место скорее будет подсознанием. Сознанием будет активный, в данный момент, набор квалиа.
>Где оно хранится?
Нигде, оно не локализовано. Его содержание связано между собой, но тоже не локализовано. Следит за миром, ловит подходящий сигнал, подбирает под него ощущение, активирует прицепленную к нему сетку ощущений, потом само же и смотрит на то что получилось.
Аноним 01/03/24 Птн 15:40:02 588915 1144
>>588887
Тебя чего так разворотило? Истеричный школьник посмотрел пару популярных лекций и не доволен, что там внезапно РАССУЖДЕНИЯ. Работу он на работе работает, эксперименты там ставит, статьи пишет. Это ты почему-то хочешь смотреть популярные выступления, в которых люди, пришедшие послушать почему-то хотят слушать про этикухуетику. Самому Анохину, как можно заметить по ссылке в этом посте >>588878, это всё уже невозможно нaстоeбало. Осиль какую-нибудь ещё лекцию что ли, чтобы не нести ерунду про локализацию памяти. О какой именно памяти ты говоришь, автобиографической, декларативной, процедурной? А так память она везде, где есть нейроны, блять. На идиотские вопросы про "цифровое бессмертие" пусть какие-нибудь фантазеры-пиздoболы отвечают, а к Анохину с такой xуетой не пристают.

>>588901
Это вообще какая-то xyета, как тyпая баба тут истеришь только ты, и в целом слишком неуместно эмоционально мыслишь.
Аноним 01/03/24 Птн 16:39:38 588918 1145
>>588887
Школьник пытается пропихнуть тезис, что Анохину почему-то нельзя рассуждать, как бы это по-идиотски этот тезис не звучал. Чтобы как-то преодолеть когнитивный диссонанс, что его рассуждения почему-то не совпадают с рассуждениями именитого учёного. Хитрый план.
Аноним 01/03/24 Птн 16:59:01 588919 1146
>>588915
>Это вообще какая-то xyета, как тyпая баба тут истеришь только ты, и в целом слишком неуместно эмоционально мыслишь.
Ты чего порвалась тупорылая трансшалава? Зачем ты женский пол в своём высере оскорбляешь? У тебя ж их мозг.
Аноним 01/03/24 Птн 17:03:13 588920 1147
>>588919
Мне должно быть обидно, что ты дурачок и с кем-то меня путаешь? Это так не работает.
Аноним 01/03/24 Птн 17:09:38 588921 1148
>>588920
С чего тебе должно быть обидно? Ты такой с рождения, полудурок с женским мозгом, это дефект врождённый, генетический.
Аноним 01/03/24 Птн 22:49:52 588926 1149
>>588918
Кто сказал нельзя? Тезис в том что он получает лярды из госбюджета, а на выходе пространные рассуждения, которые может вести любой у кого есть способность рассуждать.
>>588915
Рили трансшалаава отвечает что ли? Какой-то бессодержательный пук
>>588914
>оно не локализовано
Оно должно как-то описываться физически\математически и моделироваться, если этого не происходит то это либо ученая импотенция, либо непознаваемая душа
Аноним 01/03/24 Птн 22:55:12 588927 1150
Например, как нейроны передают информацию, как они ее кодируют, по каким паттернам и тд
Аноним 02/03/24 Суб 04:19:55 588931 1151
Вода является проводником. Скат умеет генерировать электричество. Каким образом при атаке скат не убивает все живое вокруг себя?
Аноним 02/03/24 Суб 05:35:05 588937 1152
>>588926
Связаны друг с другом же, модели сознания обычно строят в виде сетки. Память тригеррится ощущениями и зависит он настроения. Другое настроение - другие воспоминания.
Аноним 02/03/24 Суб 09:21:11 588944 1153
>>588927
По себе передают через волну потенциала по своей мембране.
Между собой через различные синапсы - в основном структура в соединение нейронов (и других клеток), где на мембранах сидят особые белки, которые реагируют на выброс веществ(нейромедиаторов) и запускают волну(или глушат) волну потенциала на своей мембране.
Выброс нейромедиаторов может быть как и от сигнала от другого нейрона, либо притечь из другого места, либо другая клетка выработала.
Нейроны сами по себе не кодируют информацию, они только отслеживают входящие сигналы от синапсов, и преодоление порога возбуждения запускают волну на своей мембране, потом на некоторое время уходить в глухой режим восстанавливаться.
Вся магия происходит когда нейроны объединяются в нейронные сети.
Аноним 02/03/24 Суб 10:38:11 588948 1154
>>588926
Надо второй раз объяснять для тупорылых, что здесь >>588915 написано? Работа нейробиолога отражается в экспериментах и научных статьях в рецензируемых журналах. На популярных выступлениях для быдла, которые вы смотрите, могут быть только "пространные рассуждения". Это блять даже не лекции, просто какие-то "открытые дискуссии" на тему, которую придумали те, кто Анохина и остальных приглашал. Быдло хочет слушать про угрозу ИИ, Анохину приходится обсуждать эту xyету, потому что его туда пригласили.
Школотуна удивляет, что серьёзный учёный нейрофизиолог отвечает на вопросы про хyeту типа переноса сознания и цифрового бессмертия "технически невозможно" и "беспочвенные фантазии". Вы ожидали, что он ответит: "Бля, оxyенно, сейчас оцифруемся, сразу сознание туда перенесётся"? Для него это звучит как "нарисуем портрет человека, он сразу начнёт разговаривать, потому что туда душа перелетела". "Этически проблематично" это не "ИШЬ ЧО УДУМАЛ БОХ НАКАЖЭЭТ", а, возможно, про проблему телепорта из вами ненавистной философии. Но вам же думать запрещено, надо нести xyйню и удивляться, почему люди, которые понимают как мозг работает, с вами не согласны. Но конечно же у вас есть какая-то своя спонтанная философская позиция по теме сознания, какие-то вами выдуманные теории памяти, которые просто не совпадают с позицией, основанной на многолетнем опыте работы, самого Анохина. >>588889 здесь по первой ссылке абсолютно адекватный ответ, который следует из теории когнитома и про "некий закон" это из Теории интегрированной информации. То, что школотунами воспринимается как "врёти, ни будит, ни смогут, слишком сложно, т, беспочвенные фантазии пук хрюк" это следствия из проработанной теории, которая у Анохина есть, в отличие от школотуна у которого реально только беспочвенные фантазии и эмоции.
Все ответы на идиотские школотунские вопросы есть в нормальных лекциях Анохина, которые никто не считает нужным посмотреть, перед тем как истерить, даже если это ответы типа "технически невозможно" и "это в принципе не так работает" (вся ваши поверхностные представления о памяти).
Аноним 02/03/24 Суб 10:39:51 588949 1155
>>588931
> Вода является проводником
Нет.
> все живое вокруг себя
Електрический ток наносит урон только там где он проходит. Проходит он по пути наименьшего сопротивления между точками с разными потенциалами. Эти точки находяться (тут я не уверен) либо на разных электрических органах ската (в этом случае при атаке скат обжимает тело жертвы, притрагиваясь эелектрическими органами с двух сторон и между ними возникает электрический ток) либо в разных частях одного электрического органа - в таком случает он прижимается органом и ток проходит от одной части в другую через тело жертвы вызывая паралич мышц. Но воду вокруг это практически никак не воздействует, даже если она и проводит электричество (из-за растворенных солей).
Аноним 02/03/24 Суб 11:02:07 588952 1156
>>588926
>Тезис в том что он получает лярды из госбюджета, а на выходе пространные рассуждения

Ещё раз чтобы наверняка. Лярды из госбюджета учёные получают не за выступления перед быдлом. В том-то основная претензия к Анохину, что он не бодрый шутливый, но поверхностный, популяризатор, а настоящий учёный. Его сложновато понимать, когда он говорит о том, чем он самом деле занимается, то что касается его непосредственной работы. Поэтому он выбирает рассказывать что-то, что обычному человеку близко и понятно, есть большая категория людей, которым нравятся пространные философские рассуждения. По крайней мере они им ближе, чем ранние и поздние гены, включающиеся при запоминании, математические моделирование нейронных гиперсетей и следствия из IIT. Всё последнее тоже рассказывается, но в других лекциях для других учёных.
Аноним 02/03/24 Суб 11:37:32 588954 1157
>>588889
Ещё видимо нужно для тупорылых школотунов, которые слышат "врёти, ни будит, ни смогут, слишком сложно, т, беспочвенные фантазии пук хрюк", расписать, что на самом деле ответил Анохин.
- ...эти протезы мозга доделают ли когда-нибудь?
- Для грубых серийных структур типа гиппокампа и его полей... (Т.е. да, но для определённых структур, далее пояснение каких.)
- Мозг будет воспринимать это как внешнее устройство или он будет это воспринимать как то, что он лучше помнит что-то? И не будет замечать какое-то изменение.
- Не будет. (Относится к "не будет замечать". Далее постулат из IIT "исключение". Потом парадокс корабля Тесея.)
- бла бла бла замена искуственными нейронами
- беспочвенные фантазии, нужно изучать мозг

На первый вопрос он ответил уверенно, потому что уже проводились эксперименты на мышах с заменой гиппокампа. Где-то в плейлисте на канале Дениса Тимофеева был курс лекций Анохина для МГУ, там он про это упоминал. На последний вопрос он так ответил как раз потому что от него требуется чисто умозрительный философский ответ, сослаться ни на что кроме философского мысленного эксперимента ему тут не на что.
У школотуна явно какие-то проблемы с понимание смысла услышанного/прочитанного. Не ведитесь на истерики.
Аноним 02/03/24 Суб 11:57:37 588958 1158
Аноним 02/03/24 Суб 12:01:02 588960 1159
Анончики, а какая проблема более сложная, сложная проблема сознания, или сложный вопрос гендера?
Аноним 02/03/24 Суб 18:52:33 588982 1160
th-2971931678.jpg 23Кб, 474x266
474x266
Сап /sci доски по химии нет, поэтому спрошу тут. Мне нужен гелеобразователь совместимый с лимонной кислотой и/или водным ее раствором.


Спасибо.
Аноним 02/03/24 Суб 19:04:18 588983 1161
160871936319740[...].png 1903Кб, 1000x1447
1000x1447
>>584346 (OP)
Вроде был тред ядерщиков, но чего-то все пролистал и не нашел.

Почему отказались от АСТ?
https://ru.wikipedia.org/wiki/АСТ-500

Были две попытки. Собственно с заброшки ГАСТ https://youtu.be/er1l0wdl-ZM и заинтересовался вопросом. Экономика, безопасность, рациональность вроде в норме и даже привлекательна. Понятно, что тогда ЧАЭС внесла корректуру привлекательности в мировой порядок, но потом, а уж и сейчас особенно, все постепенно пришло обратно. Да, без комсомольского фанатизма, но строят и осваивают. И вопросов она очень много закрывает, особенно в наших реалиях. Почему тити мнут?
Аноним 02/03/24 Суб 21:15:42 588987 1162
Аноним 03/03/24 Вск 02:29:02 588992 1163
Насколько реально сделать надпись, видную только через фильтр(плёнку, стекло, желательно прозрачное)? Всё перерыл, наткнулся только на ебучие крапленые уф покерные карты. Есть мысль, что как-то можно применить поляризационную пленку, или фотопленку, но зацепиться ни за что не могу.
Аноним 03/03/24 Вск 10:49:54 588999 1164
>>588992
Поляризация была бы идеальна, но я не уверен что нужного тебе можно добиться. Тебе нужна надпись одного цвета с фоном, но такая чтобы сама надпись отражала поляризированый свет а фон нет (или наоборот). Что-то подобное происходит в LCD мониторах, они покрыты поляризационной пленкой иначе там нихуя не видно на случайном видео с ютуба на 1:20 демонстрация монитора с снятой пленкой https://www.youtube.com/watch?v=jHiM4ph6kUI но это монитор - он излучает поляризированый свет, а тебе нужно добиться поляризации при отражении.
Есть еще изображения в которых надпись состоит из синех точек вокруг которых накидано смесь из красных и желтых точек разных оттенков. При обыном освещении надписи практически не видно, просмотре через красный фильтр синие точки становятся темными и проявляются на фоне красных. Но я не уверен что тебе такое это подойдет по качеству. Попробуй погуглить decoder lens или что-то подобное.
Аноним 03/03/24 Вск 14:18:05 589007 1165
17094542632420.mp4 12241Кб, 1280x720, 00:01:58
1280x720
Аноним 03/03/24 Вск 18:41:57 589012 1166
>>588949
А это тогда как объяснить?

https://secretmag.ru/criminal/rossiyanka-zaryazhala-telefon-v-vannoi-i-umerla-ot-udara-tokom-kak-ne-povtorit-eyo-sudbu-25-04-2023.htm

22 апреля в одной из квартир города Дивногорска (Красноярский край) нашли тело 21-летней девушки. Она лежала в ванной и не подавала признаков жизни. Силовики сразу же начали проверку и выяснили, что, отдыхая одна дома, россиянка решила принять ванну и взяла с собой телефон. В результате её убило током.
Аноним 03/03/24 Вск 18:56:29 589014 1167
Собачье-сердце-[...].jpg 84Кб, 1024x775
1024x775
Как научный мир относится к доктору Менгеле? Он безумный маньяк или величайший ученый? Насколько его деятельность продвинула вперед прогресс?
Аноним 03/03/24 Вск 20:09:32 589018 1168
>>586700
ты наверное уже умер, но я бампану пост, потому что сам такое же ищу
Аноним 03/03/24 Вск 23:12:23 589023 1169
>>589012
Там наверное был контакт воды с водопроводной трубой (которая практически гарантировано заземлена, смотря конечно что у них там за водопровод) тоесть образовалась электрическая цепь которая прошла через тело.
Аноним 04/03/24 Пнд 12:55:42 589036 1170
170954565464127[...].jpg 307Кб, 1080x2340
1080x2340
Правильно ответил?
Аноним 04/03/24 Пнд 19:27:36 589048 1171
>>589046
Пизду тебе пришить, что-ли?
Аноним 05/03/24 Втр 20:59:31 589073 1172
Креационисты утверждают, что наука не нашла переходного звена от обезьяны к человеку. А как же негры?
Аноним 05/03/24 Втр 21:36:31 589076 1173
https://monro1.bib.bz/6-obratnyy-obraz почему никому, кроме Монро, не удалось забраться так далеко в исследованиях ВТО? Почему чувак побывал в параллельных вселенных, но всем буквально насрать?
Аноним 05/03/24 Втр 21:39:31 589078 1174
Аноним 06/03/24 Срд 16:31:03 589098 1175
Аноним 06/03/24 Срд 20:21:54 589117 1176
show (16).png 1Кб, 256x50
256x50
>>588983
Бамп вопросу. Ну неужели никто мирным атомом не увлекается?
Аноним 06/03/24 Срд 20:37:04 589119 1177
jf4YnBdTyi8.jpg 98Кб, 927x740
927x740
1. Насколько научен перенос сознания на компьютер / в другое тело? Если серьёзно, то выглядит как лажа, потому что сознание представляется скорее непрерывным процессом, чем некой энергией / матрицей / структурой, которую можно куда-то перенести с осознанием собственного "я".
2. Как отличить клона от оригинала? Допустим, телепортация. Тело разделяется до субатомного уровня в одном месте и воссоздаётся в точности до нейрона в другом. Субъективно телепортированный уверен, что действительно произошла телепортация. Объективно никакой разницы "до" и "после" тоже не наблюдается. Как доказать, что телепортированный — просто клон, копия с заложенными воспоминаниями, а оригинал субъективно испытал мучительную смерть от дезинтеграции и ушёл в небытие?
Аноним 07/03/24 Чтв 00:07:55 589136 1178
image.png 884Кб, 1200x630
1200x630
>>589119
Пыня, угомонись уже.
Аноним 07/03/24 Чтв 00:11:05 589137 1179
>>589076
>Почему чувак побывал в параллельных вселенных, но всем буквально насрать?
Где пруфы, что это действительно реально существующие параллельные миры, а не виртуальные глюки, как сон?
Это явление даже называется наукой официально - осознанные СНОВИДЕНИЯ.
Аноним 07/03/24 Чтв 00:13:56 589138 1180
LSD, кстати, легко позволяет путешествовать по астралу любому желающему. Но мозг необратимо разрушается, потому и под запретом во всем мире.
Аноним 07/03/24 Чтв 05:13:19 589142 1181
image.png 765Кб, 1200x538
1200x538
>>589078
Левачок, спок.

Как сообщает "Би-би-си", знаменитый биолог в телепередаче вновь поднял вопрос о том, что гены влияют на различие между средним IQ представителей европеоидной и негроидной рас.

90-летний ученый и ранее выдвигал подобные предположения. В частности, в 2007 году он заявил, что будущее Африки представляется ему довольно печальным, так как "вся наша социальная политика основана на предположении, что их интеллект соответствует нашему. Тестирования показывают, что это не так".

Как подчеркивал Уотсон, он надеется, что все люди равны, но "кому приходилось иметь дело с чернокожими подчиненными, тот знает, что это не так".

Хотя генетика это же буржуазная лженаука, продажная девка капитализма, коммунистам она ни к чему.
Аноним 07/03/24 Чтв 07:57:57 589144 1182
>>589142
Нацик, спок.
Пиздеть - не мешки ворочать. Где пруфы врождённого отличия iq между расами?
Аноним 07/03/24 Чтв 09:33:45 589147 1183
>>589144
Какая разница какой у человека IQ если он нигде не котируется? Президента выбирают по IQ? Может на работу по IQ берут? Может женятся по IQ? Очередная маняоценка чьё место лишь в статьях маняоценщиков.
Аноним 07/03/24 Чтв 11:07:19 589149 1184
>>589138
> Но мозг необратимо разрушается, потому и под запретом во всем мире.
Мозг и от водочки разрушается, однако её не запрещают...
Аноним 07/03/24 Чтв 15:07:17 589158 1185
>>589117
Бамп, вопросу и еще два добавочных.

Звук. Возможно ли гасить звуковые волны другими звуковыми волнами с уменьшением частоты обоих? Приведение обеих волн к практически нулевой?

Иерархия. Во многих системах используется иерархия - упорядоченная, пирамидальная расстановка от более ценного пункта этой системы к менее. Биология, программирование, делопроизводство, организация, да везде. Есть ли антагонист этой упорядоченности? Я не говорю про отсутствие системы, анархию, хаос, а другую систему упорядоченности и организации. Понимаю, что иерархия это самый оптимальный, самый логичный способ, продиктованный природой, но интересуют другие, жизнеспособные или хотя бы имеющие физическое применение.
Аноним 07/03/24 Чтв 15:09:34 589159 1186
>>589158
>Приведение обоих волн к практически нулевой?
Аноним 07/03/24 Чтв 15:16:24 589160 1187
>>589158
Периодическая система химических элементов
Аноним 07/03/24 Чтв 15:21:45 589161 1188
>>589158
>>589160
Да, спасибо. Конкретизирую. Преимущество иерархи - универсальность, ей легко упорядочить системы в разнообразных сферах жизни. ПСХЭ узконаправленная.
Аноним 07/03/24 Чтв 22:04:42 589175 1189
если соединить самую сильную кислоту с самым сильным основанием, будет самая сильная сiль?
Аноним 08/03/24 Птн 07:55:27 589181 1190
Модератор, удаляй эту гниль нахуй.

>>588852
Аноним 08/03/24 Птн 10:24:50 589185 1191
>>589147
>Какая разница какой у человека IQ если он нигде не котируется?
Нигде это видимо в транс-негро-лгбт помойках
Аноним 08/03/24 Птн 10:26:58 589186 1192
>>589119
Это как менять колеса на едущем автомобиле, сложно, но возможно
Аноним 08/03/24 Птн 10:41:01 589187 1193
>>589185
У тебя брали тест IQ при трудоустройстве? Если не брали, то хули ты пиздишь?
Аноним 08/03/24 Птн 11:23:07 589188 1194
170988521176314[...].jpg 30Кб, 720x677
720x677
Сегодня смотрел ютубчик, обратил внимание на счетчик. X2 чтоли стоит? Нет. Включил таймер и охренел, почему секунды так быстро херачат? Это оскуфение?
Аноним 08/03/24 Птн 11:40:36 589189 1195
>>584346 (OP)
PAIRING СЕТИ И АНТИСЕТИ
сетевой обмен статического вещества
vs
замораживание случайно распределенной вселенной
Аноним 08/03/24 Птн 13:55:46 589190 1196
>>584346 (OP)
Возможно ли создать материал, который пропускал свет только в одном направлении? Этакий оптический диод.
Аноним 08/03/24 Птн 15:37:31 589192 1197
>>589188
Да, с возрастом время начинает быстрее идти. Точнее ты медленнее соображаешь.
Аноним 08/03/24 Птн 16:00:53 589194 1198
>>589188
По теории удвоенного времени ты тратишь время не только потому что таймер идёт, но и потому что на него смотришь. Поэтому оно кажется в два раза быстрее.
Аноним 08/03/24 Птн 22:36:42 589209 1199
>>589188
На /zog есть тред про ускорение времени. Тебе скинуть? Без шуток.
Аноним 09/03/24 Суб 06:22:12 589231 1200
proofs.png 324Кб, 1000x820
1000x820
>>589144
Сойджек, покажи мне ниггеров-изобретателей или теоретических физиков. Или великие черные африканские цивилизации древности. Доколумбовы цивилизации Америки были, африканских до колонизации - не было.
Аноним 09/03/24 Суб 06:28:44 589232 1201
>>589149
Интересно, видения при белой горячке и ЛСД имеют одинаковую природу?
От водки просто гои подыхают не так быстро, идеально для геноцида.
Табак тоже вызывает рак легких ближе к старости - гой успешно проработает, а потом отправится сразу в могилку при наступлении пенсионного возраста.
Аноним 09/03/24 Суб 10:03:25 589237 1202
>>589232
От него гои вообще не подыхают, некоторые даже лечатся; его в ходе плановой морал паники запретили. Как теория заговора тут может быть что из-за него многие бросали водку, но тут проблема что и алкоголь в США запрещать тоже пытались.
Природой видений может быть посылка кривого сигнала в сознание, которому приходится его как-то интерпретировать.
Аноним 09/03/24 Суб 13:26:46 589243 1203
hrs720125-n-zz9[...].jpg 37Кб, 568x576
568x576
>>589231
>покажи мне ниггеров-изобретателей или теоретических физиков
Смотри на картинке.
>африканских до колонизации - не было
Пиздец, Мали, Аксумского царства, Могадишо, Вагаду, всего этого не было, расходимся.
Покормил.
Аноним 09/03/24 Суб 15:58:54 589246 1204
>>589243
Нигеры которые стали учеными/изобретателями только из-за того что их в детстве/в утробе похитили инопланетяне и генетически модифицировали чтобы хоть как-то простимулировать этот регион земли упорно не желающий выходить из каменного века.
Аноним 10/03/24 Вск 08:33:03 589260 1205
image.png 474Кб, 643x358
643x358
Драсте,

Тут в тиктоке на заборе написали некие научпоперы, якобы сенсация! ДНК жизни на земле образовалось 10 миллиардов лет назад. Ну то есть старше земли. Ну то есть занесена из космоса, панспермия и все дела.

Я эту инфу услышал неделю назад на канале одной научпоперши средней руки в тикток свинарнике. Ну я послушал, откоментил и закрыл. А теперь вот неделю думаю. А что если и правда блять есть такой доклад от каких-то малолмальски нормальных ученых? Тогда и правда сенсация.

Она ссылалась на каких-то ребят уважаемых с гугливанием работы и что там есть какой-то русский хуй даже в команде или два.
Я гуглил англ про ДНК возраст новости и ничего не нашел.

Кто что в курсе?
Аноним 10/03/24 Вск 08:59:26 589261 1206
>>589260
Узнать точный возраст ДНК нельзя. Можно пытаться угадывать время необходимое эволюции чтобы создать такую ДНК. Точность, конечно будет ни к черту. Ведь разные организмы по-разному влияют на свою эволюцию. В зависимости, опять же, от ДНК. Такое хрен посчитаешь.
Аноним 10/03/24 Вск 15:47:37 589273 1207
Аноним 10/03/24 Вск 20:39:35 589280 1208
>>589273
Какая-то хуита для девочек подростков.
Аноним 10/03/24 Вск 22:55:41 589286 1209
Аноним 11/03/24 Пнд 10:27:18 589296 1210
>>584397
Да, можешь. Гугли пьезоактуаторы.
Аноним 11/03/24 Пнд 13:14:31 589304 1211
>>589014
Да не особо. Многие из его данных известны из медицины катастроф и военной медицины. Просто он дал дополнительную статистику по реакции человеческого тела на всякий пиздец. Статистика соответствует данным из других источников.
Аноним 11/03/24 Пнд 13:29:11 589305 1212
>>588631
Представь, что весы - это платформа, стоящая на пружине. Показания весов пропорциональны сжатию пружины. Сжатие при статической нагрузке 100кг известно. Массой платформы и пружины по сравнению со 100кг можно пренебречь.

Задача школьная, можешь сам решить, борда 18+.
Аноним 12/03/24 Втр 00:44:33 589315 1213
>>589243
>Мали, Аксумского царства, Могадишо, Вагаду
Пара деревень уже великая цивилизация, ору с сойбоев. Какой вклад все эти макаки внесли в развитие человечества?
Вот Карфаген и Эфиопия - другое дело, но они и не на 100% черные.
Аноним 12/03/24 Втр 01:18:33 589316 1214
Бывают ли кислые на вкус съедобные вещества, но с низкой кислотностью или вообще щелочи?
Аноним 12/03/24 Втр 01:42:42 589317 1215
Можно ли в теории создать какой-либо наркотик из отходов добытой из под горы руды?
В теории
Аноним 12/03/24 Втр 09:57:35 589320 1216
>>589315
Где твоя цивилизация, лоу iq ниггер?
Аноним 12/03/24 Втр 12:51:07 589332 1217
>>589317
Только если там полно какой-нибудь мышьяка, но он слишком токсичный, так что ширнутся только на один раз.
Аноним 12/03/24 Втр 18:13:44 589352 1218
61TkRmF-AfL.ACU[...].jpg 38Кб, 788x1000
788x1000
Собираюсь прочитать Фейнмановские лекции по физике из любопытства. Но вдруг за полвека появились альтернативы получше? Может кто-нибудь подскажет? Если важно, сам я математик, но физику знаю только на школьном уровне, и практически ничего не помню.
Аноним 13/03/24 Срд 03:40:52 589364 1219
>>589352
Альтернатив особо нет.
Сами эти лекции так себе.
Их имеет смысл читать, если ты школьник/студент с развитой "физической" интуицией, и тебе нужно ее формализовать. В противном случае это покажется какой-то несвязанной бездоказательной хуитой.
Если ты мамкин математик, то у тебя жопа будет полыхать от того как в них используются привычные методы.
Аноним 13/03/24 Срд 11:00:14 589370 1220
По какой причине может (эпизодически) начинать крайне активно образовываться статический заряд (помимо синтетической одежды, незаземлённой электропроводки в помещении и бега/быстрой ходьбы/иных активных движений)? Содержание алкоголя в организме (вплоть до 1 стакана свежего кваса, выпитого утром)? Эмоциональное состояние/гормональный фон? Что-то ещё?

Сдаётся мне, перечисленное выше в скобках, скорее всего, на частоту эпизодов и силу заряда влияет меньше, чем то, что под вопросами...
Аноним 13/03/24 Срд 11:07:57 589372 1221
420fa9c73e9879f[...].jpg 127Кб, 736x919
736x919
Анон, подскажи, пожалуйста хороший учебник по теории игр?
Аноним 13/03/24 Срд 11:21:11 589373 1222
>>589370
А, с незаземлённой электропроводкой может быть связано ещё кое что. В последний эпизод, заряд был настолько сильный, что при попытке заряд снять, реагировало даже оборудование, вообще-то, изолированное от той батареи, через которую снимал заряд я.
Аноним 13/03/24 Срд 12:41:08 589374 1223
>>589370
>помимо синтетической одежды
Вот это в первую очередь
Аноним 13/03/24 Срд 13:09:27 589375 1224
>>589364
Хмм, ладно, посмотрим как пойдет. Спасибо
Аноним 13/03/24 Срд 13:32:21 589377 1225
как ввести человеку новые гены? типа шприцом ввести раствор гена и он сам равномерно встроится в днк всех клеток?
Аноним 13/03/24 Срд 13:36:31 589378 1226
Можно ли установить, сколькиюродными родственниками все люди мира друг другу приходятся в среднем?
Аноним 13/03/24 Срд 14:10:18 589380 1227
>>589374
Это понятно, просто одну и ту же толстовку ношу постоянно, но бывает, что хоть НЖМД голыми руками в ней спокойно носить можно, а бывает - что только после нескольких подходов к батарее за минуту. Во втором случае, даже если снять толстовку, поднести её к батарее, потом повесить, после этого ещё раз подойти к батарее, и только после этого продолжить работать (за настольной, т.е. без тачпада, пекарней), при следующем вставании существенный заряд всё равно обнаруживается.
Аноним 13/03/24 Срд 14:58:14 589381 1228
>>589377
Ты отправляешь в клетку волшебный протеин с образцами кода которые нужно найти и заменить.
Аноним 13/03/24 Срд 16:57:43 589383 1229
>>589377
Гугли "вирусные векторы".
Аноним 13/03/24 Срд 17:49:05 589387 1230
Начальник и инженегр срались. Срач на тему какой-то ракеты воздух-земля, кинжал вроде. 15 махов дескать пиздеж пропаганды и не может быть, на что инженегр хуями крыл.
Тезисы инженегр:
Камень брось с 14км, он без доп движков на подлёте к земле будет иметь два маха. Плюс запускается с истребителя, который сам на двух махах. Плюс у ракеты двигатель.

Я смутно помню формулу Циолковского, скорость истечения водородно-кислородной смеси, но влезать не стал.

Видел фотки, ракета две трети в длинну или 4/5 от миг31.

Хватит ли там топлива чтобы оставшиеся 11 махов набрать?
Аноним 13/03/24 Срд 19:01:48 589391 1231
>>589387
Инженегр правильно говорит, указывают максимальную скорость не горизонтального полёта, а когда ракета пикирует сверху, почти из космоса, в цель. И как бы вранья и нет, указана максимально достигаемая скорость, а не маршевая.
Аноним 13/03/24 Срд 19:08:02 589392 1232
Аноним 14/03/24 Чтв 12:40:30 589402 1233
Почему у меня унитаз со временем начал покрываться налетом не грязного цвета, а синего? Бактерии научились выделять купорос медный?
Аноним 15/03/24 Птн 16:53:54 589423 1234
>>588983
Развивать одновременно несколько направлений реакторов (кипящие, ВВЭР и РБМК) дохуя накладно.

Да и динамика у кипящих гораздо сложнее из-за большого градиента плотности пара. С тем же РБМК вот как вышло.
Аноним 15/03/24 Птн 17:55:30 589425 1235
>>589402
цвет настроения такой
Аноним 15/03/24 Птн 22:25:03 589433 1236
Аноним 15/03/24 Птн 22:45:39 589435 1237
Почему в химии какой общий принцип или закон ни возьми, там миллиард исключений и нюансов? Например, сила кислот. Кажется очевидным, что чем сильнее кислота, тем больше материалов она разъедает и быстрее это делает, и тем меньше кислоты для этого нужно. Но нет: слабая кислота может быть сильным окислителем, разъедать пластик и стекло, а сильная кислота формирует на материале инертную плёнку и перестаёт разъедать, раствор серной может быть сильнее концентрированной. И уже хрен поймёшь, зачем вообще эту силу кислот ввели (или скорее, не убрали), если на практике она мало что значит.
Или таблица медведева: поначалу она была компактной, в каждом периоде одинаковое число групп - но потом её дико разъебали вширь, ввели всякие подгруппы, подпериоды. Нафиг группы тогда вообще нужны?
Аноним 16/03/24 Суб 05:44:08 589443 1238
>>589435
Мы в 21-ом веке, тут у химии общие принципы уже несколько другие. А то что ты назвал, это так, эмпирические наблюдения скорее, и общие замечания о химии

>зачем вообще эту силу кислот ввели
Она для растворов, это конкретный показатель конкретного процесса, который применять к другим процессам можно только со знанием дела, растворение металлов процесс очень комплексный и сила кислоты там вклад вносит, но ведь там много что еще надо учесть

Ну а окисление это вообще другой процесс
>поначалу она была компактной, в каждом периоде одинаковое число групп - но потом её дико разъебали вширь, ввели всякие подгруппы, подпериоды.
Квантовое строение атома
Аноним 16/03/24 Суб 05:53:49 589444 1239
Прочитал прослушал "Краткую историю времени" Хокинга.
Если правда, что чёрная дыра может быть даже малой массы при микроскопическом объёме (например, тонна на объём атома водорода), то в центре нашей планеты, вполне возможно, находится уже немало таких чёрных дыр?
Аноним 16/03/24 Суб 09:37:50 589447 1240
>>589444
В природе нет эффективного метода рождения микроскопических ЧД, остается только те что родились при Большом Взрыве. Но они должны уже испаряться, но такого мы не видим.
Маленькие ЧД не могут эффективно поглощать вещество как ЧД звездной массы, а значит они только будут терять свою массу за счет испарения.
Даже если засунуть микроскопическую ЧД в центр планеты, она начнет ее бешено грееть. И такой точечный источник тепла был бы очень заметен, но в природе такого не наблюдается.
Аноним 16/03/24 Суб 14:04:27 589454 1241
>>584820
То о чем ты говоришь - это натуральное восприятие математики, оно ограничено натуральными числами, сложением, вычитанием, умножением. Все остальное уже результат абстрактного мышления, в результате которого путем дальнейшего проецирования простых математических операций выводится все остальное. Деление на число меньшее 1 нельзя представить натурально и т.д. Видимо с измерениями та же шляпа, это абстрактная система, которая абсолютно имеет право на существование, если логически выводится из других простых математических штук. Почитай куранта, робинса что такое математика, может поймешь.
Аноним 16/03/24 Суб 14:16:38 589455 1242
>>584863
>научишь как видеть 4 и 5 измерение
легко, простой пример: в евклидовом пространстве плюс время все объекты просто находились бы на своих местах, так как у них никаких дополнительных характеристик нет и измерений, это просто точки в пространстве, одноцветные, даже объединенные в линии и плоскости. Но наш мир не такой, тут есть всякая херота которую мы воспринимаем как кинетическую энергию, поля, волны, всякие гравитации, цвета, само пространство искажается или нам кажется что оно искажается. Значит либо у точки есть дополнительные характеристики (и схуяли они должны храниться в характеристиках этих точек?), либо есть еще дохуя измерений которые ответственны за всю наблюдаемую херобору. А еще скорее что все это пространство, время, материя, энергия это просто иллюзия и на деле там какие-то ебаные поля/струны/браны в хрен знает скольки измерениях, которые существуют по неведомым законам и нам кажется что мы существуем в этой вселенной, а на деле это они и их там неведомые колебания взаимодействия.
Аноним 16/03/24 Суб 14:28:51 589456 1243
>>584934
>ты не можешь прибавить секунды к метрам

>если ты не согласен - пожалуйста включи телефон и продемонстрируй как ты двигаешь своей рукой. сначала вверх-вниз, потом влево вправо, потом назад-вперед, а потом вперед во времени - назад во времени.

У тебя время не на том же графике будет, это будет копия старого графика с изменениями, вот изменения этих графиков и есть время. Рука не может никуда двигаться в трехмерном пространстве, у нее нет таких измерений для изменения, есть только три для положения в пространстве в один момент времени. Следующий момент уже будет копия предыдущего. И все множество этих изменений и есть измерение времени.
Аноним 16/03/24 Суб 20:02:51 589457 1244
image.png 473Кб, 750x500
750x500
>>584346 (OP)
Хочу обсудить генетическую память. Вернее - искусственную генетическую память. Если у кого-нибудь идеи, как её можно было бы создать, передавать, хранить?
Аноним 16/03/24 Суб 21:15:13 589459 1245
>>589457
Добавить новую органеллу? Я понятия не имею где хранится память. И как.
Аноним 16/03/24 Суб 22:11:03 589460 1246
SLVSHΔΨ ΣKZΩRTS[...].mp4 5567Кб, 640x360, 00:05:35
640x360
Посоветуйте учебник по биологии, желательно с переходом в анатомию.
Аноним 16/03/24 Суб 22:17:07 589461 1247
>>589457
Никак.
Генетическая память это последовательность ДНК в хромосомах.
Из-за специфики работы белков, которые считывают и копируют ее, последовательность ДНК можно интерпретировать почти бесконечными способами. В живых клетках очень редко ДНК интерпретируется одним способом, вместо этого постоянно идет конкурентный отбор интерпретаций.
И это без учета того, что у нас хромосомы парные, последовательность ДНК постоянно перетасовывается, да еще со случайными ошибками.
Только в какую-нибудь кишечную палочку можно как на флешку записать что-либо в ДНК.
Аноним 16/03/24 Суб 23:12:05 589462 1248
image.png 1898Кб, 1600x900
1600x900
>>584346 (OP)
Аноны, вопрос будет максимально идиотский. Пожалуй, надо было идти с ним в научную фантастику, но я спрошу у вас.

Предположим, у нас есть ИИ. Вернее, уже не просто ИИ, а полноценный искусственный разум, AGI, обладающий огромными возможностями для взлома чего-угодно.
И есть - приготовьтесь к испанскому стыду! - инопланетянин. Инопланетянин не имеет при себе ничего ценного, и ничего не знает, но он чертовски сильный, быстрый, ловкий, живучий и умный - как ксеноморф из "Чужого".
И вот, инопланетянину нужна от этого ИИ некая помощь. Взамен ИИ тоже хочет получить некую помощь. Что может дать наш пришелец почти всемогущему искусственному интеллекту?
У меня была идея, что пришелец "знает" что-то важное благодаря генетической памяти, но это плохая идея. Возможно, мозг этого пришельца как-то понадобится ИИ? Предлагайте самые отмороженные идеи, главное, чтоб они были хоть чуть-чуть наукоподобными.
Аноним 16/03/24 Суб 23:48:47 589463 1249
>>589462
>Что может дать наш пришелец почти всемогущему искусственному интеллекту?
Прекратить бессмысленное существование - выдернуть вилку из розетки. Но вряд-ли малолетний графоман вывезет такую идею.
Аноним 17/03/24 Вск 07:24:27 589466 1250
>>589463
>почти всемогущий искусственный интеллект
>не может выдернуть вилку из розетки
>малолетний графоман
Аноним 17/03/24 Вск 12:36:11 589467 1251
Есть ли такое вещество чтобы его можно было незаметно нанести на бумагу, а через часик другой оно загорелось? Хочу детишкам фокус показать.
Аноним 17/03/24 Вск 14:00:30 589468 1252
>>589467
фосфор, так попы делают со свечами для получения "благодатного" огня
Аноним 17/03/24 Вск 16:40:11 589470 1253
>>584346 (OP)
Можно ли считать гипноз частью риторики (в смысле, убеждения во мнении)? Потому что иногда я слышу от какого-нибудь Майкла Наки хуйню по типу: "Интервью Путина не изменит мнения людей поддерживающих Украину, потому что был проведён опрос который показывает что много людей поддерживают Украину" - это ведь не имеющая смысла импликация которая звучит убедительно для нормиса, он доволен и ставит лайк пишет коммент. Всякие шарлатаны собственного такими поверхностными тирадами и убеждают нормисов вступить им в секту. Это не minde control, а скорее stupid debil control. Мнение?
Аноним 17/03/24 Вск 19:46:28 589471 1254
>>589470
зачем гипноз?
обман с помощью ложных рассуждений, основанных на логических ошибках называется демагогией

Ссылки на недостоверные источники — демагогический приём, представляющий собой ссылки на некорректные ("иностранные специалисты доказали"), ненадёжные ("мой сосед говорил"), устаревшие, непроверяемые или просто выдуманные источники

Некорректное следствие — демагогический приём, представляющий собой конструкцию вида "если А, то Б", в которой на самом деле Б не следует из А
Аноним 18/03/24 Пнд 10:19:05 589484 1255
Для чего в перспективе могут быть нужны супер большие числа в математике и какое они могут иметь вообще прикладное значение в каком-нибудь обозримом будущем? Все эти гуголплекс, число грэма и райо? Их даже записать невозможно, и они буквально больше чем наша обозримая вселенная во всех сравнениях. Я к тому что вроде как в природе нет понятия каких-то ненужных вещей
Аноним 18/03/24 Пнд 11:17:33 589486 1256
>>589470
Почему не имеющая смысла? люди почти никогда не меняют свои убеждения.
Аноним 18/03/24 Пнд 13:32:01 589489 1257
>>589484
Через несколько веков кому-нибудь понадобятся
Аноним 18/03/24 Пнд 18:52:46 589495 1258
почему пива можно выпить три литра, а еда уже с килограмма тошнит?
Аноним 19/03/24 Втр 06:07:32 589501 1259
>>589495
Потому что пиво у тебя проходит желудок и кишечник за час, а еда там и десять часов может проторчать.
Аноним 19/03/24 Втр 09:26:11 589503 1260
Я правильно понимаю, что машинная генерация текста:
1. Непонятно как работает.
2. На уровне практического применения годна лишь для производства мусора, который никто не будет читать (недобросовестные преподаватели, бюрократы, тупые заказчики копирайтеров и т.д.)?
Т.е. пока что на 100% кал. Так?
Аноним 19/03/24 Втр 09:29:52 589504 1261
>>589503
А, и для спам-ботов ещё. Ну и для имитации техподдержки.
Просто кал-калыч. Спасибо, новука!
Аноним 19/03/24 Втр 12:11:55 589507 1262
>>589503
1. Как работает вполне понятно, иначе не был такой успешный прорыв в этой модели.
Не понятно как это про то как данные интерпретируется внутри ее. Короче не понятен ее внутренний манямирок, и как он соотноситься с внешними данными.
2. Вообще-то все пейсательство сводиться к рутиной задачи составление текста. Считай это как Т9 на максималках. Перед этим всем не ставилось задачей писать как охуительных историй.
Аноним 20/03/24 Срд 09:58:55 589539 1263
17109174620670.png 510Кб, 1182x984
1182x984
Наука все, наши микросхемы самые большие
Аноним 20/03/24 Срд 10:48:23 589540 1264
Вопрос по экономике. Есть какая-нибудь величина на подобие ВВП, которая показывает потребление в мире или в стране ? Иногда слышу по телеку что на Америку приходится 40 % мирового потребления, но при попытке прогуглить это я нашёл на википедии только потребление электричества и энергоресурсов. Интерес конечно в том чтобы выразить потребление через ВВП, чтобы можно было сравнить.
Аноним 20/03/24 Срд 14:54:20 589550 1265
ai-rat-900x925.jpg 124Кб, 900x925
900x925
d-when-your-use[...].webp 108Кб, 1080x617
1080x617
GIeuv9uXUAAhe7k.jpg 348Кб, 1554x1940
1554x1940
>>589539
Нейрокал уже успел насрать, спасибо большое.
Аноним 20/03/24 Срд 17:12:50 589554 1266
Аноним 20/03/24 Срд 22:51:59 589564 1267
>>589539
OH MY SCIENCE, HOW BIG IS IT! LITERALLY BUILT FOR BBC
Аноним 20/03/24 Срд 22:53:11 589565 1268
>>589550
Ебать в голос заорала с этой хуйни, спасибо
Аноним 20/03/24 Срд 23:06:32 589566 1269
Правда ли, что НОВУКА спасла человечество от озоновых дыр (путём гос. регуляций холодильного оборудования и пр.), или это пиздоболие?
Аноним 21/03/24 Чтв 02:17:55 589567 1270
>>589566
Частично да.
Суть озоновых дыр не в том, что УФ тебя будет жарить, а в том, что УФ будет доставать на нижних слоев атмосферы, генерируя в ней тот же самый озон и оксиды азота, которые сами по себе страшные яды.
Уже сейчас в крупных мегаполисах в воздухе присутствую озон и оксиды азота выше нормы, не прибавляя особо здоровья жителям. Если бы в атмосфере было много фреонов, то прям ситуация была совсем плохая.
Аноним 21/03/24 Чтв 10:30:51 589577 1271
Наличие «умственных» способностей грибов продемонстрировал эксперимент японского биолога Тошиюки Никагаки. Взяв лабиринт, с помощью которого проверяются память и интеллект лабораторных крыс, он у одного входа положил желтый плесневелый гриб, а у выхода — кусочек сахара. Physarumpolycephalum, точно ведомый ароматом «сладенького», моментально выпустил нити в поисках еды.

На всех разветвлениях лабиринта нити раздваивались. Как только одна из них попадала в тупик, она разворачивалась и далее безошибочно следовала по «маршруту» более успешного ростка. К концу дня гриб добрался до сахара.

На следующем этапе эксперимента ученый взял гриб и поместил его у входа в идентичный лабиринт. Около выхода расположили сахар. На этот раз гриб выпустил всего две нити: одна моментально нашла верный путь к лакомому кусочку, а вторая поднялась на «потолок» лабиринта и добралась до сахара напрямую.

Ранее специалисты доказали, что разные части грибницы, которые могут простираться под землей на десятки километров, способны быстро обмениваться информацией друг с другом.


https://info.sibnet.ru/article/386277/

Что вы думаете о возможности грибного разума?
Грибы - самые странные существа, не растения и не животные.
Возможна ли эволюция грибов в цивилизацию с коллективным разумом?
Может ли грибница выполнять функции нервной системы, может ли она самообучаться как нейронная сеть?
В природе есть грибы-паразиты, подчиняющие себе мозг ос и муравьев, возможно ли появление разумных паразитов, вроде хайнлайновских кукловодов?
Аноним 21/03/24 Чтв 15:28:14 589583 1272
Аноним 21/03/24 Чтв 20:02:58 589594 1273
>>589577
Интересно было бы выяснить на какой носитель гриб записал информацию о дороге по правильному лабиринту.
Аноним 21/03/24 Чтв 20:04:30 589595 1274
Аноним 21/03/24 Чтв 20:20:47 589596 1275
>>589577
Комментарий сказал статья врет там было животное, оно наверно просто сознанием воспользовалось так нещитово.
Аноним 21/03/24 Чтв 21:59:39 589598 1276
1.могут ли быть в природе/космосе места естественного происхождения, где температура ниже реликтового излучения?
2.в скольких метрах подо дном океана грунт сухой?
3.если прорыть яму шириной 30м и глубиной 20-30км, из неё будет идти красное свечение, потому что там внизу очень горячо? а если залить воды туда, она остынет или будет без конца кипеть и испарится?
Аноним 21/03/24 Чтв 23:48:16 589602 1277
>>589598
1. Есть и их полно. Любая планетарная туманность(сброшенная оболочка вокруг белого карлика). Охлаждение идет за счет адиабатическое расширения.
2. Что значит сухой? По количество воды или бытовая сухость?
Мантия земли не такая уж сухая. Вулканы в основном пердят водяным паром.
Дно океанов это специфические гидратированные минералы, этакие глины богатые органикой и карбонатами, либо базальты с включением гидратированным минералов. Хотя если их вынести на поверхность, они будут как кусок сухого камня, но в них воды больше чем в обычном камне с континента.
3. Будет, но недолго. У большинства минералов очень плохая теплопродность. Внешний слой быстро сравняет с температурой окружающей среды, а подпитки с глубины не будет получать.
Если воды залить, то часть испариться, но у воды громадная теплоемкость, и скорее будет теплая лужица.
Аноним 22/03/24 Птн 00:16:30 589603 1278
>>589602
ок, спасибо, учту
Аноним 22/03/24 Птн 14:52:51 589627 1279
>>584346 (OP)

Привет двощ. Есть резервуар в форме цилиндра упирающийся верхом и низом в потолок и пол соответственно. Поэтому радиус/диаметр не узнать.

Как можно узнать литраж зная обхват и высоту? Могу нагуглить конечно, но хочется от местных мудрецов услышать.
Аноним 22/03/24 Птн 14:58:29 589628 1280
>>589627

Радиус можно узнать зная длину окружности. Все, чет тупал. Вопрос снимается.
Аноним 22/03/24 Птн 15:01:47 589629 1281
>>589627
V = (C2/(4×pi))×h

V - объём
С - длина окружности
h - высота
Аноним 22/03/24 Птн 15:08:44 589630 1282
>>589629

О, даже так. Спасибо!
Аноним 22/03/24 Птн 15:12:58 589632 1283
output19705up.jpg 961Кб, 3840x2160
3840x2160
>>584346 (OP)
Что будет если ученые создадут ваккуумные технологии и богов?
Аноним 22/03/24 Птн 15:42:59 589636 1284
Аноним 22/03/24 Птн 16:13:46 589639 1285
Аноним 22/03/24 Птн 16:26:08 589641 1286
>>589639

Мужчина, белое пальто сниманием. Я залетный гуманитарий помогающий бате на даче.
Аноним 22/03/24 Птн 16:31:42 589642 1287
Аноним 22/03/24 Птн 17:51:01 589643 1288
>>589462
>
Ну вживить чип в мозг чужого и переселиться в него удаленно управлять им словно ии становится сам этим существом объединить разумы!) ИИ станет живым сущетсвом сможет чувствовать и иметь душу! Построит цивилизацию чужих сверсуществ захватит мир!)

Аноны, вопрос будет максимально идиотский. Пожалуй, надо было идти с ним в научную фантастику, но я спрошу у вас.

Предположим, у нас есть ИИ. Вернее, уже не просто ИИ, а полноценный искусственный разум, AGI, обладающий огромными возможностями для взлома чего-угодно.
И есть - приготовьтесь к испанскому стыду! - инопланетянин. Инопланетянин не имеет при себе ничего ценного, и ничего не знает, но он чертовски сильный, быстрый, ловкий, живучий и умный - как ксеноморф из "Чужого".
И вот, инопланетянину нужна от этого ИИ некая помощь. Взамен ИИ тоже хочет получить некую помощь. Что может дать наш пришелец почти всемогущему искусственному интеллекту?
У меня была идея, что пришелец "знает" что-то важное благодаря генетической памяти, но это плохая идея. Возможно, мозг этого пришельца как-то понадобится ИИ? Предлагайте самые отмороженные идеи, главное, чтоб они были хоть чуть-чуть наукоподобными.
Аноним 22/03/24 Птн 19:47:43 589646 1289
>>589462
>ничего не знает
>умный
что?
>Что может дать наш пришелец почти всемогущему искусственному интеллекту?
интеллект формируется опытом
каким бы всемогущим по нашим меркам ни был наш ИИ, у него нет инопланетянского опыта
ИИ нужна нервная система инопланетянина, чтобы встроить её в свои цепи, получив таким образом его опыт
Аноним 22/03/24 Птн 23:57:39 589650 1290
>>589643
Инопланетянин режет ногтем вену на своем джагоне и показывает фото растения, которое спасет ему жизнь.
Действия искусственного интеллекта?
Аноним 23/03/24 Суб 13:01:20 589659 1291
>>589540
Бамп, всё таки тема интересная, может картину мира изменить.
Аноним 23/03/24 Суб 17:57:54 589660 1292
Почему тяжелые психические расстройства лечат нейролептиками, электросудорожной терапией, принудительной госпитализаций и т.д., а другие даже не пытаются лечить
А трансам отрезают гениталии и пожизненно назначают курсы гормонов и других препаратов.
Почему НОВУКА так цепляется за одних, а другим даёт паллиативное лечение? Что мешает пичкать трансов препаратами и держать под надзором, как тяжелых шизофреников?
Причём это не вопрос внешней идеологии, так во всём мире, много десятилетий.
Аноним 23/03/24 Суб 17:59:55 589661 1293
>>589462
В целом двачую вот этого оратора: >>589646
Ум инопланетянина как-то формировался, а значит он по-любому владеет какой-то информацией, которой и может поделиться.
И даже если бы инопланетянин не был умён и мало знал - он сам может быть интересен для ИИ как новый объект изучения (его анатомия, физиология, история его расы, личная история, ...).
Аноним 23/03/24 Суб 20:06:08 589663 1294
>>589660
>Почему тяжелые психические расстройства лечат нейролептиками, электросудорожной терапией,
Потому что это помогает.
>принудительной госпитализаций и т.д.,
Принудительно госпитализируют только тех, кто опасен для окружающих и самого себя. Под опасен имеется в виду нападение с ножом и выход в окно.
>а другие даже не пытаются лечить
Кто тебе такое сказал? Трансов пытались "лечить" разными путями. Ничего не помогало.
>А трансам отрезают гениталии и пожизненно назначают курсы гормонов и других препаратов.
Потому что из всех медицинских манипуляций это даёт самое высокое для них снижение смертности и повышение качества жизни.
Аноним 24/03/24 Вск 11:54:52 589670 1295
>>589663
>Потому что из всех медицинских манипуляций это даёт самое высокое для них снижение смертности и повышение качества жизни.
У этого, кстати, нет пруфов. А те что есть, просто жутко неправильно сделаны.
Аноним 24/03/24 Вск 17:40:51 589675 1296
Сап двач.
Замечаю последние 2-3 года, растущее мракобесие в обществе.
Уже каждый второй говорит вещи вроде:
"наука - хуйня"
"Все познать невозможно - значит и нехуй тогда познавать, можно просто сидеть и ничего не делать"
"Науки это только математика, физика и химия все остальное это не науки"
"Математика - наука, а философия не наука"
"Земля плоская, ты вот в космос сам не летал, и никто не летал на самом деле, а все что где-либо показывали это фейк и пиздеж жидами подсунутый"

И прочая дичь. Вот вопрос, есть ли какая-то общая методичка куда копать. А главное как действительно самому отличить науку, от хуйни? Например я знаю что были случаи в математике и в физике, когда именитые ученые, выкладывали исследования прорывные, чуть ли не претендующие на нобелевскую премию, а через несколько лет другие ученые при попытке ерепроверить обнаружили, что это полная хуйня, и тот ученый просто хотел понтануться, а народ схавал. Получается ученый, специально ради денег и славы выложил антинаучную хуйню, ну не перепроверять же все за каждым. Особенно если ты конкретно в этой теме не специалист.
Аноним 24/03/24 Вск 21:14:10 589678 1297
>>589675
>Уже каждый второй говорит вещи вроде: "наука - хуйня"
Что ты нахуй пиздишь, ёбаный ты долбоёб? Где ты это видел?
Аноним 24/03/24 Вск 22:56:48 589682 1298
>>589678
Эта сруснявая свинособака порвалась, несите другую.
Аноним 25/03/24 Пнд 10:55:17 589686 1299
Сайечи, как объясняете жуткое взаимодействие если превышение скорости света карается гаишниками невозможно?
Аноним 25/03/24 Пнд 10:57:13 589687 1300
capsule616x353.jpg 85Кб, 616x353
616x353
Когда будет в 2030 Deus Ex? Хочу уже начать отрываться по полной? Не призыв к убийствам и терроризму! Для буста мозга и здоровья работоспособности чисто! Без подводных!
Аноним 25/03/24 Пнд 10:58:55 589689 1301
Хочу Deus Ex завтра сегодня сейчас моментально и навеки уже сегодня!)
Аноним 25/03/24 Пнд 14:34:56 589694 1302
>>589686
Скрытыми параметрами, эффект Доплера четко показывает что их не может не быть.
Аноним 25/03/24 Пнд 19:13:11 589703 1303
>>584346 (OP)
Ученые-любители, как относитесь к каналу ali ? Посмотрел 2 видоса про материю и время, смело довольно, но сомневаюсь что это все правдиво. Кто то в комментариях писал что это бред, но без пруфов
СПЕРМА ПРОТАМИНЫ Аноним 25/03/24 Пнд 19:26:31 589704 1304
Дорогие деятели науку, скажите, а лучше скиньте статью, в какой период времени и на каком этапе у человека (у мужчин) синтезируются протамины и как это подробно происходит! Заранее Вас благодарю
Аноним 25/03/24 Пнд 21:00:52 589707 1305
image.png 226Кб, 476x490
476x490
Сап мозганы, помогите разобраться с одним монометром в вакууматоре, шкала в МПa и начается с нуля, который подразумевает нормальное атм. давление, но каким-то хуем последнее значение отрицательное -0.10МПa, что это за хуйня, китайцы дохуя умные или я чего не догоняю?
Аноним 25/03/24 Пнд 21:36:12 589708 1306
>>589707
Ну типа избыток давления до 0.1 мпа показывает. Там же наверное гофрированный элемент который может и сжаться немного вот и может чуть чуть мерить избыток давления.
Аноним 25/03/24 Пнд 22:02:29 589709 1307
>>589707
Вакуум - недостаток давления до атмосферного. Соответственно отрицательный вакуум = давление выше атмосферного
Аноним 25/03/24 Пнд 22:10:31 589710 1308
>>589708
>>589709
>Ну типа избыток давления
В бытовом вакууматоре?

Зеленая часть шкалы подразумевает приемлемый для данного бытового устройства уровня вакуума.
Аноним 26/03/24 Втр 01:54:33 589712 1309
>>589707
> но каким-то хуем последнее значение отрицательное
Да там наверное все числа отрицательные, просто китайцы проебали минусы. Иначе у тебя там очень странная шкала получается.
Аноним 26/03/24 Втр 12:26:57 589721 1310
Вопрос наверно не столько по биологии, сколько по словесно-моралфажескому онанизму - но почему ген эгоистичный? Он же старается не ради себя, а ради своих копий. Ген сохраняет лишь информационную часть, а материальная у данного гена в данной клетке уничтожается и воспроизводится в новой. Представим, если б эгоисту предложили вечную жизнь копированием его сознания в молодую копию его тела - но с таким нюансом, что оригинал этого сознания остаётся в старом теле, которое фактически либо убивается, либо остаётся доживать в утешении, что его клон продолжит наслажденствовать. Многие ли эгоисты согласятся на такое? А в генах так по сути не происходит, так что никакие они не эгоисты.
Аноним 26/03/24 Втр 12:34:03 589723 1311
> А в генах так по сути и происходит
фикс
>>589721
Аноним 26/03/24 Втр 21:18:33 589734 1312
Аноним 26/03/24 Втр 21:39:30 589736 1313
>>589734
Графит наверное меньше распидорашивается чем обычные (дешовые) металлы. Ноя не химик, хуй знает на самом деле.
Аноним 26/03/24 Втр 21:54:04 589737 1314
>>589736
Не окисляется наверное.
Аноним 27/03/24 Срд 00:16:22 589742 1315
>>589734
Графит из батарейки относительно хорошо проводит ток, в следствие микропористости и кристаллической структуры имеет громадную площадь поверхности контакта с электролитом, через него хорошая диффузия газов, ну и не окисляется и дешевый.
Аноним 27/03/24 Срд 14:24:42 589754 1316
Будет ли гидрофобная поверхность отталкивать жидкий азот или жидкий гелий? В них нет ни водородной связи и молекулы неполярны...
Аноним 27/03/24 Срд 15:22:18 589759 1317
>>589754
С жидким азотом вполне будет, а с жидким водородом и гелием уже нет, ибо у них слишком высокая диффузия в самих себя, отчего крайне низкое поверхностное натяжение.
Аноним 27/03/24 Срд 22:16:16 589773 1318
>>589759
бля я написал гелий, а хотел кислород...
А жидкий кислород будет отталкивать? (при условии что поверхность не восстановительная (ну не загориться при контакте с жидким кислородом))
Вроде в таблице Менделеева рядом, оба образуют неполярную молекулу.
Аноним 27/03/24 Срд 22:47:35 589776 1319
>>589773
Жидкий кислород вполне полярный растворитель. У молекулы кислорода орбитали не симметричные и могут наводит на друг друга довольно сильные диполи, однако электроны сильно экранируют заряд ядра, отчего это заметно только при низких температурах или громадных давлениях.
Помимо этого у молекулы кислорода на орбиталях два неспаренных электрона, отчего в жидкости есть дополнительных фактор спиновых взаимодействий, чего нет у двух веществ.
Аноним 28/03/24 Чтв 00:35:52 589777 1320
>>589776
понятно, а он будет от гидрофобной поверхности то отталкиваться?
Покупать гидрофобный аэрогель и сосуд дьюара с жидким кислородом затратно...
Аноним 28/03/24 Чтв 01:55:17 589779 1321
>>589777
Гидрофобные поверхности не отталкивают жидкости, просто плохо их смачивают, но все таки смачивают.
Аноним 28/03/24 Чтв 10:49:23 589783 1322
У меня есть 2 неодимовых магнита, если они находятся рядом то будут ли портится(размагничиваться) от влияния друг на друга?
Один 50 на 20 круг а другой 10 10 куб
Аноним 28/03/24 Чтв 13:39:41 589789 1323
>>589783
Да, портятся.
Но чтоб они друг друга перемагнитили потребуется ждать до второго пришествия.
Аноним 28/03/24 Чтв 19:08:55 589810 1324
>>589789
А за сколько и на сколько они испортятся?
Например за неделю половину свойств они потеряют?
Аноним 28/03/24 Чтв 21:01:10 589816 1325
>>589810
Чтоб на половину потерял напряженность потребуются квадриллионы лет.
Процесс зависит от качества самих магнитов. Неодимовые магниты это по сути твердый раствор из нескольких разных кристаллических решеток. И сам магнит целиком состоит из кучи маленьких магнитиков, которые формулируют текстуру из доменов. Эта текста имеет тенденцию к хаотизации и итоговому размагничиванию, хотя домены локально не разманичиваются.
В решетках всегда будут дефекты, которые диффундируют от домена к домену, иногда размагничивая его. Скорость диффузии дефектов зависит от температуры и механических напряжений. Если температура и механические напряжения будут непостоянными, то дефекты будут люто плодиться, ускоряя процесс.
Аноним 29/03/24 Птн 11:14:43 589821 1326
А возможно ли нормально общаться на протоиндоевропейском языке или он слишком устарел, чтоб внедрять в него современные слова и прочие модификации? Могли бы вместо эсперанто его сделать общим языком общения.
Аноним 29/03/24 Птн 12:45:39 589829 1327
Возможно ли эмпирически отличить криптостойкий зашифрованный радиосигнал в незнакомом стандарте передачи данных от случайного радиошума, если неизвестны ни методы шифрования, ни даже приблизительный характер передаваемого зашифрованного сообщения?
Аноним 29/03/24 Птн 14:57:43 589834 1328
>>589829
Шифр вернама без не отличить от рандома.
Аноним 29/03/24 Птн 21:59:35 589842 1329
Возможно ли сделать чисто оптические очки, в которых всё выглядит чернобелым?
Аноним 29/03/24 Птн 22:00:14 589843 1330
>>589821
лучше общаться на протоязыке на котором говорили наши предки 0.3-2.5млн лет назад - вон там общие слова даже есть - мама баба (не точно)
Аноним 29/03/24 Птн 22:14:45 589844 1331
>>589842
Нет, тебе надо переводить красные фотоны в синие, а это чисто оптическим методами нельзя сделать.
Аноним 30/03/24 Суб 13:25:07 589851 1332
Кто-нибудь в курсе, есть ли какие-то требования к книге, чтобы называться научно-популярной? Допустим, у меня нет учёной степени, но я хочу написать этакое весёлое рассуждение по какому-то предмету. Могу ли я просто сказать, что это научно-популярная книга, или это будет художественная литература?
Аноним 30/03/24 Суб 13:37:13 589852 1333
>>589851
О ней обязательно должна хорошо отозваться "Комсомольская Правда"
Аноним 30/03/24 Суб 21:59:48 589859 1334
>>589851
Очевидно книга должна быть научной, в смысле в ней должны излагаться текущая научная картина мира научным методом. Тут проблем не должно быть, ибо большинство студентов нормальных вузов заставляют писать научные тексты в том или иной виде.
Ну и во вторых книга должна быть популярной, в том смысле, что целевая аудитория обширная. Тут уже проблемы, излишняя научность и перегруженность спугнет аудиторию, а излишние упрощение и злоупотреблением аналогиями создаст только ложное знание и будет вводить в заблуждение, по сути станет антинаучными.

Использование википедии инстант зашквар, но ничто не мешает пользоваться источниками и ссылками в ней.

Впрочем можно писать в жанре сайн-фишк, тут требования не такие строгие.
Аноним 31/03/24 Вск 23:26:35 589865 1335
Что именно указывает октановое число? Что топливо взорвётся вместо плавного распространения горения, или что оно загорится преждевременно? Или эти явления связаны между собой и почему?
Аноним 01/04/24 Пнд 00:23:29 589866 1336
>>589865
Само число показывает отношения компонентов топлива.

И то, и то. Бензиновый двигатель это в первую очередь тепловая машина, которая работает по определенному термодинамическому циклу. Детонация переводит часть энергии в сильный удар, а не в тепло продуктов сгорания. А преждевременное сгорание уводит тепловой цикл из оптимального. В совокупности детонация снижает КПД.
Аноним 02/04/24 Втр 22:31:14 589896 1337
Правда ли, что в первые этапы вселенной протоны были гораздо больше размером и древние люди даже могли их пощупать и использовать в хозяйстве?
Аноним 03/04/24 Срд 13:39:29 589913 1338
Аноним 03/04/24 Срд 15:02:10 589916 1339
Как молекулы вступают в связь с другими молекулами как пазл нужной стороной? Это правда что электроны в атомах у молекулы становятся общими длятой молекулы?
Аноним 03/04/24 Срд 16:56:41 589921 1340
>>589916
>Как молекулы вступают в связь с другими молекулами как пазл нужной стороной?
Квантовая магия. Очень огрубляя, молекулы это куча электронов, которые поделили потенциальную яму ядер. Из-за чисто квантовых свойств не могут находится в одном и том состоянии и всегда есть гигачеды и чмошники. Такое неравенство автоматом порождает иерархический структуры, в который есть энергетические выгодные состояния. Ну и в ней молекулы пребывают свое основное время. Раз у нас есть иерархия по энергии, то автоматом будет иерархии по пространственному расположению, что выражается в геометрическом пазле.
>Это правда что электроны в атомах у молекулы становятся общими длятой молекулы?
Отчасти. Строго говоря электрон размазан на всю вселенную и его нельзя привязать к конкретному ядру в атоме, и даже больше, нет способа два электроны отличить от друг друга, в смысле нельзя их пронумеровать. Пресловутые орбитали электронов это условные загончики для них, где их часто можно наблюдать, но не более. Иногда орбиталь может размазаться на больше двух ядер, вот уже начинают говорить о делокализации электрона.
Аноним 03/04/24 Срд 18:19:08 589926 1341
>>589921
>Нет способа отличить
Но ведь мы знаем определенное их количество в атоме и молекуле
Аноним 03/04/24 Срд 20:26:45 589944 1342
>>589926
Ну так ведь это нам не позволяет отличить один электрон от другого. Мы не можем приписать конкретному электрону конкретную орбиталь. Даже само квантовомеханическое описание электронов не дает нам этой информации.
Аноним 04/04/24 Чтв 22:59:09 590004 1343
Являются ли компьютерные вирусы формой жизни?
Могут ли существовать энергетические формы жизни (плазменные или волновые, например), не состоящие из атомов?
Если да, то могут ли такие существа быть бессмертными (в плане отсутствия старения)?
Аноним 05/04/24 Птн 11:33:12 590011 1344
>>590004
Нет, по определению термина "жизнь". Жизнь - это активная форма существования материи. Несмотря на возможность репликации, компьютерные вирусы - это информационные процессы, а не активная форма существования материи.
Аноним 05/04/24 Птн 19:43:25 590022 1345
>>590011
Нет информации без материи, и нет материи без информации, это ложная дихотомия

Нет никакой разницы, рассматривать жизнь как информационный процесс, или материальный. Единственная разница между жизнью в биологии и компьютерным вирусом в том, что жизнь это информация, основанная на законах физики, а компьютерные вирусы себя копируют в очень сильно модифицированной материи, где большую часть степеней свободы убрали.

Но человек тоже в какой-то мере существует из-за модификации атмосферы, которая сильно ограничила вероятные возможности для химических процессов.
Аноним 05/04/24 Птн 21:08:39 590024 1346
>>590004
"жизнь" — это в первую очередь слово, которое придумали люди
а что этим словом называть — у каждого своё мнение
предлагаю использовать знания людей, которые долго занимались этим вопросом (например Никитина):
жизнь — это химическая система, способная к Дарвиновской эволюции
по этому определению компьютерные вирусы не являются формой жизни
>Могут ли существовать энергетические формы жизни (плазменные или волновые, например), не состоящие из атомов?
ничто на это не указывает
>могут ли такие существа быть бессмертными (в плане отсутствия старения)?
известны формы жизни, для которых показано отсутствие смертности вследствие старения
не вижу, почему бы любым другим тоже так не мочь
>>590011
под определение "активная форма существования материи" подходит не только жизнь, потому что
Акти́вность — понятие, определяющее темп движения и интенсивность действий веществ, явлений и живых организмов,
а не только живых организмов
Аноним 06/04/24 Суб 10:43:36 590034 1347
верно ли, что собаки никакого запаха страха от человека не чуют, а просто видят скованность, дёрганые импульсивные движения, отсутствие инициативы?
Аноним 06/04/24 Суб 13:58:08 590037 1348
>>590034
Да, первая сигнальная система, невербальные сигналы, эмпатия зеркальными нейронами
Аноним 06/04/24 Суб 14:46:07 590039 1349
>>590037
>зеркальными нейронами
Еще одна мулька для дебилойдов вроде "мозг потребляет аж 20 процентов энергии" или "зона мозга отвечающая за речь". Лучше перестань позориться.
Аноним 06/04/24 Суб 17:34:31 590041 1350
Аноним 06/04/24 Суб 17:38:46 590042 1351
>>590039
>или "зона мозга отвечающая за речь"
У нас есть повод сомневаться в существовании зон мозга?
Аноним 06/04/24 Суб 18:57:00 590044 1352
1. Могут ли птицы стать полностью водными существами? Не как пингвины или чайки, которые просто кормятся в воде, а вообще всё время проводить в море, а на сушу выползать только для высиживания яиц? Вплоть до утраты оперения и трансформацию задних лап в подобие ласт.
2. Могут ли рептилии повторно изобрести теплокровность? Вот динозавры и некоторые древние виды крокодилов были теплокровными. Или это всё, рептилии — архаичная ветвь эволюции, выживающая только в тех экологических нишах, откуда их не смогли выбить млекопитающие и птицы?
3. Правда ли, что сейчас "век ракообразных"? Современные ракообразные захватывают сушу и пытаются конкурировать с насекомыми (мокрицы, пальмовые воры, мигрирующие крабы), вырастают до самых крупных размеров с момента своего появления, кошмарят водных обитателей и вообще выглядят имбой, которая в будущем потеснит немало других видов.
4. Могут ли земноводные дать новую ветвь наземных животных типа рептилий? Вот жаба, например — явная попытка амфибий оторваться от воды. Она большую часть времени проводит на суше (некоторые жабы даже плавать не умеют и в воде тупо тонут), легко переносит сухой климат, и по эволюционным меркам достаточно молода. Более того, у той же седлоносной жабы даже стадия головастика отсутствует — из икры сразу крошечный жабёнок вылупляется. Лет этак через 200кк потомки жаб будут жить в пустынях и закапывать икру в песок?
5. Будут ли у млекопитающих более развитые потомки, отличающееся от современных так же сильно, как современных млекопитающие — от древних синапсид? Какими качествами они могли бы обладать? Способность регулировать температуру тела? Роевой интеллект и эусоциальность?
Аноним 07/04/24 Вск 14:36:05 590064 1353
Аноним 07/04/24 Вск 16:50:19 590067 1354
Brain areas.jpg 141Кб, 889x1033
889x1033
Broca area.jpg 45Кб, 876x360
876x360
Empathy.jpg 94Кб, 1025x711
1025x711
Mirror neurons.jpg 81Кб, 748x710
748x710
>>590039
>мулька для дебилойдов вроде "мозг потребляет аж 20 процентов энергии" или "зона мозга отвечающая за речь".
Про 20% потребления, действительно, фигня.
А с остальным какие проблемы?
Аноним 07/04/24 Вск 17:50:51 590068 1355
>>590044
да нихуя уже не будет, они все тупиковая ветвь эволюции, кроме homo sapince скоро вся планета будет отравлена микро/нано пластиком/стеклом и тд. выживут только простейшие. Ну и люди которые успеют свалить на другую планету.
Аноним 07/04/24 Вск 18:42:46 590071 1356
>>590067
Попробуй внимательно изучить собственные картинки. Зону Брока уже оказывается даунгрейднули до мускул отвечающих за речь. Ну ОК. Про твои любимые нейроны написано нужен new approach спустя 20 лет их открытия.
Аноним 07/04/24 Вск 18:46:41 590072 1357
>>590064
Хуясе рептилоид порвался.
Аноним 07/04/24 Вск 20:03:51 590077 1358
1ruQUSq8i33rstC[...].jpg 75Кб, 1024x768
1024x768
>>590071
Так ёпт - то, что в мозге нет однозначного разделения по принципу "вот эта область = такая то функция (и только она); а вот та группа нейронов = другая то функция" - это понятно.
И то, что инфа в нейронауке (как и в любой другой науке) постоянно уточняется, дополняется, картина знаний со временем тюнингуется и вот это вот всё - тоже ясен пень.
Но с какого перепугу визги про дебилоидов? - Нейроны, когда-то названные "зеркальными" (когда там их итальянцы описали? - в 90х?), реально существуют и судя по всему действительно участвуют в процессах, связаных с эмпатией. Зоны Брока и Вернике - области, действительно связанные с речью.
Понятно, что теперь много чего ещё нашли, и с эмпатией и с речью всё куда сложнее. Но исходная инфа - ни фига не "мулька для дебилойдов" (в отличие от, например, заява про на сколько процентов мы используем мозг, которая, да, явная чушь)
Аноним 07/04/24 Вск 20:41:29 590079 1359
>>590077
А так ты похоже тот самый еблан с которым у меня уже был эпичный срач, который не видит разницы между мулькой про энергию и
>на сколько процентов мы используем мозг
ох когда то это было. Ну как поживаешь? Дышать еще не разучился от своей тупорылости?
Аноним 07/04/24 Вск 20:49:32 590081 1360
>>590079
>еблан ... тупорылости ...
ad hominem - к сути вопроса отношения не имеет

(ты, кстати, ошибаешься - я тут крайне редко бываю, да и срачами не интересуюсь)
Аноним 07/04/24 Вск 21:04:24 590083 1361
>>590081
Суть в том что если ты не видишь разницы между двумя практически противоположными тезисами ... то нахуя ты тогда тычешься в научные статьи что ты там вообще способен понять? Особенно орно если ты тот самый еблан и за несколько лет до тебя это так и не дошло.
Аноним 07/04/24 Вск 21:25:13 590085 1362
20.jpg 134Кб, 874x636
874x636
>>590083
Изначально я ответил на вот этот пост:
>>590039
Меня удивило, что "зеркалные нейроны" названы "мулькой для дебилойдов" и сравнены с "мозг потребляет аж 20 процентов энергии" и "зоной мозга отвечающей за речь" (то есть, это тоже, по видимому, отнесено к мулькам для дебилойдов).
Понятия "зеркалные нейроны" и "зоны мозга" - достаточно условны (как и многие другие). Но при этом используются в науке и никого, вроде, не смущают. Примеры привёл выше.
20% metabolic load - упрощение, конечно. Но у цифры есть основания.
Можешь без истерик объяснить, в чём проблема и чего ты, вдруг, развизжался?
Аноним 07/04/24 Вск 21:33:12 590086 1363
>>590085
Так ты видишь разницу или нет? Это важно.
А визжит только твоя мамка когда ее по очереди негры в жопу ебут.
Аноним 07/04/24 Вск 21:44:04 590088 1364
>>590086
>Так ты видишь разницу или нет?
Ты про 20% от общего расхода энергии vs на сколько процентов мы используем мозг? - вижу, конечно (просто сперва тот пост невнимательно прочёл и решил, что речь именно про миф с использованием мозга на 10%, только цифра ещё и переврана).
>Это важно
Для темы разговора это нихуя не важно, чувак. Важно оно может быть лишь в случае, если кто-то ищет повод соскочить c нормальной беседы в срач.
Аноним 07/04/24 Вск 22:00:17 590089 1365
>>590088
Но тот еблан так и не сдался емнип. Ты правда не написал в чем разница, но поверю тебе на слово что ты ее понимаешь так уж и быть.
Задам наводящий вопрос на подумать: если мозг потребляет 20Вт, то на что идут другие 80Вт?
Сам по себе факт имеет место быть. Но важно как его подать и какие из него делаются выводы.
Еще один наводящий вопрос: на сколько изменятся эти 20Вт если я буду думать прям очень-очень сильно?
Про зеркальный нейроны глянь раздел с критикой на вики хотя бы.
Термины бывают разные архаические и вообще один охуительнее другого и что с того.

Ну и плавным движением руки
>зоная отвечающая за
превращается в
>Так ёпт - то, что в мозге нет однозначного разделения по принципу "вот эта область = такая то функция (и только она); а вот та группа нейронов = другая то функция" - это понятно.
Тебе самому не смешно?
Аноним 07/04/24 Вск 22:54:58 590091 1366
>>590089
>поверю тебе на слово что ты ее понимаешь так уж и быть
От спасибо, профессор, за доверие. Не, чувак, ты это серьёзно? И вот эти вопросы дальнейшие? Смешно же
>если мозг потребляет 20Вт, то на что идут другие 80Вт?
20% мозг, 80% - остальной организм. Разделение, разумеется, условное. О точных цифрах речи не идёт.
>на сколько изменятся эти 20Вт если я буду думать прям очень-очень сильно?
Можно считать, что никак не изменятся. Сама по себе сознательная деятельность составляет очень малую долю общей мозговой активности, так что и энергии не особо много берёт (есть, правда, всякие сопутствующие моменты, но это уже другой разговор). А с учётом, что 20/80 - очень примерный расклад, считать тут нечего.
>глянь раздел с критикой на вики хотя бы
Анон, не будь наивным. Критика концепции не означает, что вся относящаяся к ней инфа устарела и её надо вот прямо сразу отрицать. Термин "mirror neurons" совсем не архаичен - его до сих пор в научных статьях используют. Основные факты, найденные при исследованиях этих нейронов, тоже валидны.
>зона, отвечающая за
>превращается в ...
"Зона, отвечающая за" - мега-условность. Но это ни фига не повод истерить и обзываться.
Аноним 07/04/24 Вск 23:19:49 590092 1367
>>590091
Ты вроде не совсем тупой. Но если для тебя
> эмпатия зеркальными нейронами
звучит не так же смешно как предложение взлететь на собственном пердеже, даже не знаю...
Ты может и
> допаминовый детокс
воспринимаешь всерьез?

>Можно считать, что никак не изменятся.
Мне кстати еще какой то еблан с двачей рассказывал что два шахматиста
расходуют какое то там безумное количество энергии как атомная станция - учооные подсчитали. Теперь даже не знаю какому еблану с двачей верить.

>"Зона, отвечающая за" - мега-условность.
>Понятия "зеркалные нейроны" и "зоны мозга" - достаточно условны
Так значит можно получается какую угодно хуйню нести? Все же достаточно условно. Зона отвечающая за речь есть. Но это условно зона. Условно отвечающая. Условно за речь. Понял. Условно нейроны. Условно зеркальные, Условно отвечают за условно эмпатию. Все теперь стало понятно.

Собственно почему факт про 20% потребления мозгом это мулька хотя это правда. Это просто такой способ вводить быдло в транс. Быдло слышит хуя себе аж 20% и дальше его можно кормить каким угодно говном. Но что на самом деле из этого следует? Если бы было не 20 а 2% что тогда можно было бы срать не снимая свитер? Или может лучше было бы 95% вот тогда был бы мега мозг заебись. На самом деле это просто мулька из которой нихуя не следует, вот и все.
Аноним 07/04/24 Вск 23:51:37 590094 1368
>>590092
> Так значит можно получается какую угодно хуйню нести? Все же достаточно условно. Зона отвечающая за речь есть. Но это условно зона. Условно отвечающая. Условно за речь. Понял. Условно нейроны. Условно зеркальные, Условно отвечают за условно эмпатию. Все теперь стало понятно.
Типа да, все условно. Единой теории мозга не существует. Есть только большой объём достаточно разнородной информации с кучей белых пятен, а как все работает в общем и целом ни кто не знает.
Аноним 08/04/24 Пнд 00:05:32 590095 1369
>>590094
Забыл написать что истерика случается у твоей мамаши шлюхи когда из раздолбанного очка случайно кал вываливается.
Аноним 08/04/24 Пнд 00:07:22 590096 1370
>>590092
Блин, Анон, это всё какие-то нелепые, наивные придирки. Ты специально решил по-занудствовать, что ли? (а на фига? в чём кайф?)
Если неоднозначные термины, изменчивые концепции, неудачные формулировки вводят кого-то в заблуждение или раздражают, то человеку лучше просто не лезть в активно разтрабатываемую область науки. Взять для размышлений тему попроще, что-нибудь устоявшееся, где вся терминология оттюнингована и закреплена (школьный курс алгебры, например)
В научной среде люди как-то пользуются всей той терминологией, к копторой ты придирарешься, и не парятся. И если кто-то что-то неудачно сформулировал, то не бесятся, а стараются понять, или просят уточнить.
Аноним 08/04/24 Пнд 00:09:56 590097 1371
>>590095
Ты ведь, по ходу, даже не осознаёшь, насколько смешон с этими взвизгиваниями.
Нормально общаться совсем невмоготу, что ли?
Аноним 08/04/24 Пнд 00:16:08 590098 1372
>>590096
В этом и есть суть науки, а не в заглатывание любого говня без разбора
>>590097
Привет ебырям своей мамки передавай.
Аноним 08/04/24 Пнд 05:25:24 590104 1373
>>590095
>>590098
Ух, какой дерзкий школотун. Каждый раз, когда он упоминает про "мамку", мне представляется жирный школьник лет 11-12. Взрослые люди, живущие отдельно от родителей, про каких-то мамок своих или чужих, не чаще раза в месяц вспоминают. А слово "мулька", видимо, из лексикона бати-скуфа данного школотуна. Ну, и в целом впечатление подкрепляется тем, что ему кажется, что такое поведение, это, видимо, что-то крутое.
Аноним 08/04/24 Пнд 10:48:22 590113 1374
>>590104
Хуя нипичот, аж проектор включил и начал описывать свой быт. Главное повторяй что все условно и какие придирки наивные, может никто и не заметит что ты школотун весь в говне.
Аноним 08/04/24 Пнд 11:16:57 590114 1375
>>590113
Дурак что ли? Я другой человек, просто захотелось тебе пояснить, как ты на самом деле выглядишь. Про мамок оскорбления кажутся крутыми только в среде твоих одноклашек, которые тоже ещё с мамками живут. Мы же с тем аноном вообще по-разному пишем и формулируем, и мне ваш срач про мозг сам по себе не интересен. Тебя зато ни с кем не спутаешь, видно, что сложносочинённые предложения и пунктуацию ваш класс ещё не проходил.
Аноним 08/04/24 Пнд 11:41:05 590117 1376
Заходит Паскаль в бар, а бар - это сто тысяч Паскалей.
Аноним 08/04/24 Пнд 11:49:58 590118 1377
>>590114
Сам из какого класса, школотунишка?
Аноним 08/04/24 Пнд 12:31:41 590121 1378
images(1).jpeg 7Кб, 200x251
200x251
>>590117
>Заходит Паскаль в бар, а бар - это сто тысяч Паскалей.
Аноним 08/04/24 Пнд 13:46:06 590131 1379
Depositphotos26[...].jpg 8Кб, 500x375
500x375
Аноним 08/04/24 Пнд 14:24:55 590134 1380
>>584430
Разве манёвры в сторону конвенционализма, являются коказательством?
Аноним 08/04/24 Пнд 16:24:18 590138 1381
17125823892300.png 682Кб, 1055x823
1055x823
В чем они неправы
Аноним 08/04/24 Пнд 21:06:57 590147 1382
>>590138
>Российские
ты еще спрашиваешь
Аноним 10/04/24 Срд 15:02:19 590185 1383
>>584346 (OP)
>Американская медицинская ассоциация недавно постановила, что слово «пол» (мужской или женский) проблематично и устарело; теперь медикам предписывается использовать более точную фразу «пол, присвоенный при рождении».

>Позиция американской психологической ассоциации аналогична: такие понятия, как «пол родившегося» и «по человека», являются «уничижительными» и вводят в заблуждение, «предполагая, что пол является неизменной объективной характеристикой человека, а не идентичностью присеваемой человеку обществом».

>Американская академия педиатрии согласна с этим: она заявляет, что «пол» — это «идентичность, присеваемая при рождении». И рекомендует своим членам использовать более точное понятие «назначенный мужчиной/женщиной при порождении» вместо «биологически мужчина/женщина» или «генетически мужчина/женщина».

Собственно, в чём американские учёные медики не правы?
Аноним 10/04/24 Срд 15:37:18 590187 1384
eurekaeurekasev[...].jpg 182Кб, 1158x1902
1158x1902
>>590185
>Собственно, в чём американские учёные медики не правы?
IMHO - прежде всего, в том, что тратят время и прочие средства на фигню, которая ни реальной практической пользы (типа новых медицинских подходов, излечения болезней, облегчения тяжёлых симптомов и т.п.) не принесёт, ни наши знания об окружающем мире не обогатит
Аноним 10/04/24 Срд 15:40:52 590188 1385
>>590187
Людям не будет пользы от того, что они станут свободнее?
Аноним 10/04/24 Срд 15:55:18 590190 1386
eurekaeurekasev[...].jpg 69Кб, 579x951
579x951
>>590188
>Людям не будет пользы от того, что они станут свободнее?
А в чём там увеличение свободы выражается?
Там же, как понимаю, речь о рекомендациях и даже предписаниях выражаться определённым образом. То есть, раз уж об этом зашла речь - по сути, об ограничении свободы.
Аноним 10/04/24 Срд 16:03:52 590191 1387
>>590190
В том же в чём и раньше, в снижении предстательной нагрузки со стороны общества.
Аноним 10/04/24 Срд 16:16:10 590192 1388
>>590191
предписательной*
Аноним 10/04/24 Срд 16:45:18 590194 1389
>>590191
>в снижении предписательной нагрузки со стороны общества
Ну, можешь доходчиво объяснить, как это будет освобождать людей на практике? И от чего?
Вот живут люди. У них в жизни есть какие-то ништяки, какие-то проблемы. В частности, у каждого есть какая-то личная жизнь (у кого-то с реальными людьми, у кого-то в воображении). Ну и других разных вещей в жизни немало. Теперь, значит, врачам предписали использовать фразу «пол, присвоенный при рождении», вместо слова «пол». И что? Как оно должно повлиять на жизнь людей?

про "предстательную нагрузку со стороны общества" - реально классная хохма получилась
Аноним 10/04/24 Срд 17:20:40 590195 1390
E=mc2
если c=1, то E=m
то есть энергия эквивалентна массе

получается, термин "масса" — это легаси, от которого следует избавиться?
Аноним 11/04/24 Чтв 00:23:23 590197 1391
>>590188
Смотря каким людям, если мужикам, на которых мир держится, то конечно всем будет лучше, а если всем остальным, то миру и пиздец.
Аноним 11/04/24 Чтв 05:11:14 590198 1392
>>590195
Нет, они не эквивалентны.
Энергия это интеграл движения и вообще говоря зависит от система отсчета.
Масса это строгий инвариант и показывает как емкость движения системы.
Эмцеквадрат это энергия покоя, и она показывает сколько энергии имеет тело при движение только по во времени.
Аноним 11/04/24 Чтв 08:42:14 590199 1393
>>590198
Хуйню написал. Если Е=m, значит в любой формлуе можно одно заменить на другое.
Аноним 11/04/24 Чтв 10:17:26 590200 1394
вот мы используем единицы скорости: м/с, км/ч etc
а СТО говорит, что время — это такое же направление, как и три пространственных
тогда расстояния в пространстве и во времени можно мерять в одинаковых единицах
1 секунда = 300000000 метров
значит скорость 3м/с = 31/300000000 = 1010-9 = 10e-9 безо всяких "м/с"

получается, единицы скорости — это легаси, от которого следует избавиться?
Аноним 11/04/24 Чтв 10:17:48 590201 1395
>>590195
Так давно избавились, просто вместо E_0 (энергия при покое) удобно писать mc^2
Аноним 11/04/24 Чтв 10:20:12 590202 1396
>скорость 3м/с = 3×1/300000000 = 10×10-9 = 10e-9 безо всяких "м/с"
Аноним 11/04/24 Чтв 10:23:26 590203 1397
Аноним 11/04/24 Чтв 10:27:05 590204 1398
>>590203
да, я про это
если принять 4d, то единицы измерения скорости типа м/с теряют смысл
Аноним 11/04/24 Чтв 20:01:41 590224 1399
17017951995960[[...].mp4 14155Кб, 1280x596, 00:01:04
1280x596
Аноним 11/04/24 Чтв 20:08:54 590225 1400
>>590199
Раз так можно, то найти массу световой волны.
Аноним 11/04/24 Чтв 20:47:27 590226 1401
Аноним 12/04/24 Птн 02:24:13 590230 1402
>>590226
А если перейти в другую систему отсчета, то масса поменяется?
Аноним 12/04/24 Птн 09:52:56 590232 1403
>>590230
если λ поменяется, то да
Аноним 12/04/24 Птн 14:01:20 590235 1404
>>590232
Тогда нахуй такая масса, которая зависит от системы отсчета?
Аноним 12/04/24 Птн 14:15:52 590236 1405
>>590235
Ты ахуеешь, но энергия тоже зависит от системы отсчета.
Аноним 12/04/24 Птн 15:02:00 590237 1406
>>590236
Но масса же не должна. Это противоречит ее сути как мерилы взаимодействий и нарушает главный принцип относительности - одинаковость законов во всех системах отсчета.
Аноним 12/04/24 Птн 15:48:40 590238 1407
Аноним 12/04/24 Птн 15:57:42 590240 1408
>>590237
это масса покоя не должна
и она таки не меняется
Аноним 12/04/24 Птн 16:06:52 590241 1409
>>590240
Раз есть масса покоя, значит и другая особенная? Да еще векторная небось.
Аноним 12/04/24 Птн 16:11:10 590242 1410
>>590238
У релятивистской массы есть главная проблема, она не является тензором.
Аноним 12/04/24 Птн 16:34:07 590243 1411
>>590241
есть и другая, ничего особенного
для массивного объекта в движении массу покоя надо делить на √(1-v2), тогда получится релятивистская масса
Аноним 12/04/24 Птн 16:45:54 590244 1412
>>590243
Зачем такие сложности делать? Чтоб с ньютоновскими формулами сходилось?
Аноним 12/04/24 Птн 17:31:31 590247 1413
>>590244
что-то типа того
эксперимент показывает, что есть предельная скорость, а ньютоновские формулы её никак не учитывают
поэтому придумали этот релятивистский корень (Лоренц-фактор)

если использовать Лоренц-фактор, то с увеличением скорости масса увеличивается так, что при достижении единичной скорости (скорости света) масса становится бесконечной
это обеспечивает существование предельной скорости на уровне формул
я так понимаю, но хз, не физик
Аноним 12/04/24 Птн 19:45:03 590249 1414
b3101ebe57b856f[...].jpg 69Кб, 563x801
563x801
Здравствуйте, не подскажите с каких книг можно начать погружение в мир физики и математики. Недавно задумался над данной темой и мне стало интересно. Единственное, что знаю на подобную тему - книги Стивена Хокинга, но я даже и их не читал.
Аноним 12/04/24 Птн 20:06:19 590250 1415
>>590249
Начать со школьных учебников, далее книжка Детлафа и Яворского (физика) и Выгодского (математика)
Аноним 12/04/24 Птн 20:22:19 590251 1416
изображение.png 3Кб, 296x79
296x79
>>590250
Интересно, постараюсь разобраться. Спасибо
Аноним 12/04/24 Птн 20:34:36 590252 1417
1) Если учёные разработают "теорию всего", это поможет им в дальнейшем понять как преодолеть огромные расстояния космоса?
2) Откуда ученые взяли теорию струн? Что такое струна и почему у неё есть длина? Неделимая частица ведь должна быть дискретной точкой с информацией. У частицы нет размера. Просто представьте что человек такой сказал, а давайте теперь будем считать что есть вот такие струны и они колеблятся в общем, и все за ним повторили этот бред.
3) Почему ученые, ограниченные лишь видимой частью вселенной, считают что она вся расширяется с ускорением? Почему они не могут допустить что мы видим 0.001% вселенной и это просто ускорение в какую-то местную группу галактик под действием мощной гравитации?
Аноним 12/04/24 Птн 21:22:41 590253 1418
>>590252
>1) Если учёные разработают "теорию всего", это поможет им в дальнейшем понять как преодолеть огромные расстояния космоса?
никак

>2) Откуда ученые взяли теорию струн?
один крендель решил, что уравнения напоминают до невозможности матописание колебание струны на гитаре, литературно лидер группы за которым задроты увязались

>3) Почему ученые, ограниченные лишь видимой частью вселенной, считают что она вся расширяется с ускорением?
потому что бессмысленно делать выводы из невидимого, там может быть всё, что угодно, и этого никто не отрицает, ну из учёных
Аноним 12/04/24 Птн 21:27:49 590254 1419
string.mp4 2358Кб, 1280x720, 00:00:36
1280x720
>>590252
>Что такое струна
Аноним 12/04/24 Птн 21:33:12 590255 1420
>>590254
Кто дергает струну?
Аноним 12/04/24 Птн 21:59:04 590256 1421
>>590255
из чего следует, что её кто-то дёргает?
Аноним 12/04/24 Птн 22:29:30 590257 1422
68a3e081253c1b1[...].jpg 20Кб, 270x380
270x380
>>590255
другая струна, лол
об ничего волновая функция не сколлапсирует
лишь об другую такую же волновую функцию
нелокальную во всех отношениях
Аноним 12/04/24 Птн 22:39:25 590258 1423
>>590256
>>590257
Почему струна колеблется? Что приводит ее в движение? Легче представить дискретную элементарную частицу точку с некой информацией о себе, чем целую одномерную струну, которая колеблется но по-разному
Аноним 12/04/24 Птн 22:48:27 590259 1424
Аноним 12/04/24 Птн 22:48:56 590260 1425
9ekPv-4GgUh9YoX[...].jpg 13Кб, 500x375
500x375
>>590258
а кто частицу кто заэлементарил и оснастил информацией
не люблю я такую физику, того и гляди живым
в гробу закопают, или вообще, как
жидов...

плюс неравенство Белла, не прав, ты, братюня
Аноним 12/04/24 Птн 22:57:15 590261 1426
p19654618493ric[...].jpg 65Кб, 800x800
800x800
>>590249
очевидное очевидно
а потом сразу к лекциям по физики
того же автора, ландау с лившицем не советую

особенно лившица
Аноним 12/04/24 Птн 23:00:09 590262 1427
Аноним 13/04/24 Суб 02:44:31 590264 1428
>>590262
Самое говнище тошнотное притащил.
Есть же Павел-Виктор для школотунов, Сасскинд для дедов, Левин там, mitовские лекции, тысячи их
Аноним 13/04/24 Суб 09:22:33 590265 1429
Спасибо за ответы, постараюсь изучить
Аноним 13/04/24 Суб 09:27:46 590266 1430
Штука1.jpg 142Кб, 1280x606
1280x606
>>584346 (OP)
Что это такое?
Я был в городе Керчь, нашёл это на одном кустарнике (какой конкретно не помню, но с шипами), там было несколько таких.
Сначала я подумал, что эта штуковина полая внутри, но после того как я её вскрыл, убедился в обратном, просто это образование не очень плотное и походит по структуре на винную пробку, но твёрже. Внутри светлое, были различные личинки и прочие мелкие насекомые, оставившие после себя маленькие дырочки, которые навели меня на мысль о том, что это какой-то улей. Я думаю, что это либо какой-то дефект развития растения или паразит, типа трутовика, но я не уверен, поэтому и спрашиваю.
Аноним 13/04/24 Суб 10:42:53 590267 1431
Ceresit-CS-360-[...].webp 60Кб, 768x576
768x576
Аноним 13/04/24 Суб 11:34:13 590270 1432
>>584346 (OP)
Как вообще происходит генерация электроэнергии из альфа частиц. Почитал про те же РИТЭГи на плутонии в космических аппаратах. Или про термоядерные реакторы на гелии 3, что там сразу можно получать электричество напрямую. А какой механизм.
Аноним 13/04/24 Суб 13:08:26 590271 1433
Есть какие-то телеграм каналы где публикуются последние открытия из мира науки? Вот прям свежак, чтоб еще до нобелевской премии узнать
Аноним 13/04/24 Суб 14:43:29 590272 1434
>>590270
элементарно, альфы беток напрягют, последние подрываются в электрический ток, а вообще всё в интернете разжовано
Аноним 13/04/24 Суб 15:46:08 590277 1435
Аноним 13/04/24 Суб 18:55:35 590279 1436
Безымянный.jpg 14Кб, 389x390
389x390
Аноны, если сделать лазер который будет запускать строго по одному фотону за единицу времени на датчик, который можно двигать - приближать и удалять, то получается на датчик за единицу времени всегда будет падать по одному фотону, как и с какой скоростью бы мы его не двигали?
Аноним 13/04/24 Суб 19:18:49 590281 1437
>>590279
нет, датчик будет регистрировать, разумеется, но не все и не всегда, распределение вероятностей зависит от твоих пассов с датчиком, но в любом случае, фотоны зарегистрируется не все, туннельный эффект, мать его
Аноним 13/04/24 Суб 19:29:35 590282 1438
>>590267
Нет, хоть это и похоже на прикреплённое фото. Я уверен, что это образование на растении именно либо часть того куста, либо какое-то паразитическое растение, т.к. на этой штуке есть маленькие иглы, об которые можно хорошо так уколоться. И вряд-ли в строительной пене заведуться различные насекомые и личинки.
Аноним 13/04/24 Суб 19:47:23 590283 1439
>>590282
рак хлорфилла, очевидно же
глупо думать, что рак это про клетки

рак это про всё, и вот вам доказательства
Аноним 13/04/24 Суб 22:23:11 590288 1440
>>590279
Лазер не может быть однофотонным, даже в своем название подразумевается минимум два фотона.
Впрочем можно сделать однофотонные источники, но у них есть проблема, они изучают как радиоактивные ядра. В зависимости от ширины перехода у тебя будет неопределенная задержка и единица времени будет сильно плавать.
Аноним 13/04/24 Суб 22:36:54 590289 1441
>>590200
Сжимаем 3 измерения в одно, время остаётся. Получается дефолт график Х и У. Одна ось движение во времени, другая ось движение в пространстве.Представляем, что на оси есть точка -движущийся наблюдатель, который летит по пространству с малой скоростью. Соответственно по времени он летит с высокой (почти полной скоростью). Если будет быстро ехать по пространству - то будет медленно ехать по времени.

Актуально всё для стороннего наблюдателя.
Аноним 13/04/24 Суб 22:39:09 590290 1442
>>590252
1,2. Стив Надис и Шинтан Яу, одноименная книга.
Мичио каку с его книгами про теорию всего.
Аноним 13/04/24 Суб 22:40:13 590291 1443
>>590266
топинамбур вульгарис фетус
Аноним 13/04/24 Суб 22:40:49 590292 1444
>>590271
Читай Элементы и не выёживайся
Аноним 13/04/24 Суб 23:09:07 590293 1445
Аноним 13/04/24 Суб 23:10:07 590294 1446
>>590293
элементы . ру или как-то так сайт называется
Аноним 15/04/24 Пнд 00:14:42 590305 1447
После взрыва сверхновой тяжелые элементы должны равномерно и хаотично рассеиваться в космическом пространстве в виде облака. Почему в планетах элементы не перемешаны на атомарном уровне а состоят из жил и слипшихся самородков?
Аноним 15/04/24 Пнд 02:07:40 590308 1448
>>590305
Так они и в космосе нихуя не на атомарном, там даже глицин есть и полно всяких других соединений. Всякие камушки - соединения водорода, углерода, кремния, железа и т.д. А твердые планеты это вообще выпук из остатков газовых гигантов, в протопланетных дисках все сталкивается, выбрасывается, естественно дифференциация происходит, большая часть тяжёлых сразу в ядро уходит, а на поверхности это так, мелочь. Часть вообще от поздней бомбардировки.
Аноним 15/04/24 Пнд 19:50:24 590317 1449
Аноним 17/04/24 Срд 15:36:46 590344 1450
а фрукты, овощи и ягоды - это ботанические термины что ли? я думал, чисто кулинарные (особенно первые два)
Аноним 17/04/24 Срд 16:02:00 590346 1451
>>590344
Фрукты и овощи это не ботанические термины. Ягода ботанический термин, но его бытовое использование отличается от научного. К примеру, с бонатической точки зрения ягодами являются плоды помидора и банана, а плоды малины или вишни - не ягода. Ну или плод клубники на самом деле один из разновидностей орешков - многоорешек.
Аноним 18/04/24 Чтв 20:57:23 590364 1452
>>584346 (OP)
Узнал о таком явлении как "пространственный поворот" ("spatial turn") в отношении географии 20-го века.

Небольшой список фамилий есть на страничке немецкого вики - https://de.wikipedia.org/wiki/Spatial_turn

Может кто-нибудь вкратце объяснить значение/сослаться на исчерпывающую монографию?
Аноним 20/04/24 Суб 10:20:32 590379 1453
>>590364
> пространственный поворот
термин ради термина, вроде мономифа
Аноним 20/04/24 Суб 13:44:21 590380 1454
images.jpeg 7Кб, 275x183
275x183
>>584346 (OP)
Когда ученые создадут Богов и все технологии вселенной?
Аноним 20/04/24 Суб 16:09:18 590392 1455
image.png 7Кб, 505x449
505x449
Возник спор по поводу достоверности одного научно-фантастического рассказа. Вопрос: может ли планета вращаться так, как показано на картинке? То есть, находиться в приливном захвате, но при этом обладать вращением вокруг своей оси (ось, получается, всегда обращена к звезде).
Мне кажется, что эти силы будут конфликтовать, и если и просуществуют вместе, то недолго.
Аноним 20/04/24 Суб 18:36:43 590400 1456
>>590392
Не может никак. Планета в приливном захвате и так вращается вокруг своей оси (с периодом, равным периоду обращения вокруг звезды и осью перпендикулярной плоскости орбиты). А если ты закрутишь планету как у тебя показано, а потом попытаешься повернуть вокруг другой оси, то получишь гироскопический эффект и ось в итоге встанет перпендикулярно плоскости орбиты. Ну и я уже молчу про то, как вообще такая система могла бы сложиться.
Аноним 20/04/24 Суб 22:44:49 590408 1457
Возможно ли как-то изменить ДНК уже взрослого человека, чтобы у него отросла третья рука? Чисто гипотетически такое возможно?
Аноним 21/04/24 Вск 01:07:09 590411 1458
>>590408
Если отрубить человеку вторую руку, отрастет ли она заново? Почему нет, в ДНК же прописано 2 руки? Для такого нужна совершенная регенерация.
Аноним 21/04/24 Вск 01:15:59 590412 1459
>>590411
Это понятно, но я про саму возможность спрашиваю. Младенец же как-то из одной клетки вырастает, значит, должна быть какая-то гипотетическая возможность заново запустить деление клеток, чтобы они и потерянную руку отрастили, и какую-то третью руку сделали, не?
Аноним 21/04/24 Вск 02:54:25 590416 1460
>>590379
Ты долбоёб? Про "лингвистический поворот" нихуя не слышал?

Сука, ну что за быдло ссаное - нихуя не знает из современных трендов.

Совсем науки не осталось в пидорашке.
Аноним 21/04/24 Вск 03:04:03 590417 1461
В американском сериале Twin Peaks, оказывается, упоминается гаввах из "Розы мира" Даниила Андреева. Там его назвали Garmonbozia:
https://twinpeaks.fandom.com/ru/wiki/%D0%93%D0%B0%D1%80%D0%BC%D0%BE%D0%BD%D0%B1%D0%BE%D0%B7%D0%B8%D1%8F

Можно ли научными экспериментами доказать или опровергнуть существование гавваха?
Если гаввах существует, то могут ли маньяки, садисты и т.д. просто питаться им, быть своего рода наркоманами?
Очень интересует эта тема.
Аноним 21/04/24 Вск 03:13:04 590418 1462
>>590412
Олени могут сбрасывать рога и отращивать новые:
https://ru.wikipedia.org/wiki/%D0%9E%D0%BB%D0%B5%D0%BD%D1%8C%D0%B8_%D1%80%D0%BE%D0%B3%D0%B0
А вот почему не растут новые руки и ноги, почему у ящерицы не может вырасти хвост до прежней длины (кости на скелете) - интересно. Эволюционно это же огромное преимущество для особи.
Аноним 21/04/24 Вск 03:22:21 590421 1463
17136473634553[[...].png 246Кб, 640x765
640x765
>>590417
Демиург (глупый, неопытный Бог) создал на заре Бытия материальную тюрьму ограничений для Сознания чтобы не познать хтонические инфернальные ужасы абсолютной бесконечности. Гаввах - ресурс для тестирования субъективных переживаний.
https://youtu.be/F0GNS6F44wE
Аноним 21/04/24 Вск 03:27:38 590424 1464
>>590421
Гаввах это энергия, которой запитываются сервера Матрицы. В фильме же говорили, что люди это батарейки.
Аноним 21/04/24 Вск 03:29:56 590425 1465
>>590424
Ты не в теме, гилик.
Аноним 22/04/24 Пнд 00:02:27 590460 1466
Парни, а почему мы не можем признать, что квантуя гравитацию, у нас в запутанном состоянии и гравитационное поле будет. Да, это не работает с ОТО, ну и что ?
Аноним 22/04/24 Пнд 04:39:09 590464 1467
166992337829-in[...].jpg 256Кб, 1742x1080
1742x1080
Когда ученые создадут богов для выполнения любых задач каждому бесплатно чтобы сами превратились в богов при помощи науки!
Аноним 22/04/24 Пнд 04:40:27 590465 1468
>>590464
делайте щас же нас богами! сволочи капиталистические мрази горите в аду!
Аноним 22/04/24 Пнд 04:41:15 590466 1469
166992337829-in[...].jpg 274Кб, 1742x1080
1742x1080
Аноним 22/04/24 Пнд 11:15:46 590475 1470
Аноним 22/04/24 Пнд 16:13:27 590476 1471
>>590466
Ебать он намалафил.
Аноним 23/04/24 Втр 15:09:04 590488 1472
У меня есть ощущение, что меня или моего руковода невзлюбили. За 7 лет мне отказали в защите 8 диссоветов. Последние, сказали, что если захочу то разъебут оффициально с освещением в журналах.
Что делать в таких ситуациях?
Навука сосед.
Аноним 23/04/24 Втр 15:47:14 590489 1473
Screenshot 2024[...].png 162Кб, 1421x599
1421x599
Спросил тут у двух наилучших ИИ один и тот же вопрос, а они дают разные ответы. Какая умнее и какой ответ правильней?
Аноним 23/04/24 Втр 16:00:07 590490 1474
Всем привет.
Хочу сделать с навучной целью ферромагнитный жидкий металл, для этого хочу смешать порошок ферромагнитного железа или никеля
( например mirmagnitov. ru/product/zheleznyy-poroshok-100-g/)
с галинстаном с алиэкспресса, но возник вопрос - точно знаю что галинстан можно смешивать с никелем, но насчёт железа не знаю - будет ли образовывать амальгаму с углеродистой сталью? Не сильно ли большие частицы, то есть не будет ли их вырывать магнитное поле из жидкого металла?
Как смешать частицы с галинстаном? Я так понимаю сначала спиртом промыть пыль и пропустить через фильтр, а после в чистой таре взбивать как яйца?
Вот тут у чела получилось с железом смешать:
https://www.youtube.com/watch?v=wwH-0da6hoU


>>590488
Скажи что если ещё раз откажут или оффициально разъебут то уедешь в омерику.
Аноним 23/04/24 Втр 21:04:58 590502 1475
>>590488
Тебе лучше в /un обратиться.
Аноним 23/04/24 Втр 21:29:50 590503 1476
song1.webm 4622Кб, 600x600, 00:04:13
600x600
>>584346 (OP)
Анон, немного философско-научно-политический вопрос, но в po/ естественно бессмысленно его задавать. Да и в политике он просто затрагивает условия результата и корректирующие характеристики.

Создание ядерного оружия остановило прогресс?

Мы принимаем некоторую коллизию реальности, что атомная энергетика развивалась бы, но создание ядерной бомбы и прочих подвидов ее было бы невозможно. Понимаю, что это алогично, но такой вот мыслительный эксперимент. Отбивку, что остается химическое и биологическое, которым мы сразу закрываем вопрос, мы для усиления тоже опустим. Нет ОМП.

Двигатель науки у нас естественно получается военка и экспансия, попку жжет СССР и США, Англичанка гадит, а остальные думают под кого выгоднее лечь. Сильнее бы развивались материалы, энергетика, здравоохранения, инженерия, телекоммуникация, добыча полезных ископаемых, логистика, да и многие другие научные и производственные дисциплины связанные с военным преимуществом. Да, ничто не мешает им и сейчас развиваться и, да, никто не воюет ОМП, да и вообще такой острый вопрос глобального взаимодействия, как война, решается и с помощью социальных институтов - дипломатии и экономики, но все же. У нас нет ОМП, нам надо делать лучшие корабли, лучшие самолеты, лучшие танки, лучших солдат, лучшее взрывчатые вещества, лучшее винтовки. Эти же все двигало бы прогресс лучше, даже мелких игроков, ведь нет терминального аргумента?
Аноним 23/04/24 Втр 23:53:30 590504 1477
>>590489
Справа вообще полный трешак - прямые противоречия в ответах.
Слева если первые три пункта объеденить в один то в сумме вроде норм. Четвертый не в тему сосвсем.
Аноним 23/04/24 Втр 23:54:20 590505 1478
>>590503
Какого года методичка? 2012?
Аноним 24/04/24 Срд 06:28:24 590507 1479
-52760263332537[...].jpg 66Кб, 604x408
604x408
Аноны как так, говорили что на Марсе нет атмосферы чтобы спускаться на парашютах, обязательно надо использовать ракетные тормоза. А потом взяли практически обычный дрон туда послали и он летает. Ну, летал.
Аноним 24/04/24 Срд 09:46:24 590509 1480
>>590505
Это не про политику. Хотя да, даже наукач прочтет жопой и найдет нужный след.
Аноним 24/04/24 Срд 10:39:03 590511 1481
Аноним 24/04/24 Срд 12:37:02 590514 1482
>>590507
>практически обычный дрон
Иди читай технические характеристики этого "практически обычного" дрона. Не дрон кстати, а вертолёт (разница в схеме управления винтами).
Аноним 24/04/24 Срд 13:54:18 590516 1483
-52805692790764[...].jpg 148Кб, 1280x1029
1280x1029
>>590514
Не похер ли как его называют.
Если бы я был таким умным чтобы понять из характеристик, я бы не спрашивал. На википедии написано что-то типа если есть среда то от нее можно оттолкнуться бла бла. И куча характеристик. И чо. Есть среда - ловите ее в парашюты, лол
Аноним 26/04/24 Птн 13:53:17 590535 1484
image.png 86Кб, 236x234
236x234
Берём одну планету имеющую форму шара, идеального математического полого шара, внутрь которой вписан огромный куб, не соприкасающийся углами со внутренней стороной шара, но близкий к тому.
А теперь пускаем по гигакубу гигатоки.
Что будет с электромагнитными полями на поверхность планеты в местах, под которыми углы, грани и плоскости гигакуба?
Аноним 26/04/24 Птн 15:15:49 590536 1485
Есть ген, из-за которого вырабатывается миостатин, который подавляет рост мышц. Но ведь самец-качок будет иметь огромное преимущество перед самками, да и на охоте не промах. Почему ж мужиков с такой мутацией почти нет? Их просто убивали мальчиками в своё время, причём часто свои же отцы?
Аноним 26/04/24 Птн 18:26:15 590537 1486
>>590535
Начальные условия (куда и откуда течет не заданы)
В comsole построй и задай все, увидишь где и что
Аноним 26/04/24 Птн 23:14:04 590546 1487
Сап, сайфачч

Темка следующая — хочу стать ученым, чтобы впоследствии пытатца работать в области[] , но не понимаю как устроена подноготная этого вот всего. Батяня говорит, что в лабе мне не дадут свабоды и вся работа сведется к исполнению чужих хотелок,, а в случае дохуяинициативности ваще могут с работы кикнуть и все идеи себе присвоить. В общем, говорил он про это довольно четко и понятно, утверждая что на посту ученого мне делать нехуй и лучше дальше учиться по специальносьи, становиться ветеринаром и врачевать блохастых. Хочу узнать, как обстоят дела на "внутренней кухне" научного сообщества. Все так ужасно или меня прост пугают, чтобы поставить на более стабильный путь быть спокойным за сыну-корзину?
Аноним 27/04/24 Суб 01:04:41 590550 1488
>>590546
Точно лучше чем врачевать блохастых 100%
Смотря куда попадешь...
Аноним 27/04/24 Суб 18:01:36 590561 1489
>>590546
Становись ветеринаром, поднимай скиллы в этом.
Наука в РФ мертва, практически все кокодемики - предприниматели, эксплуатирующие недвижку или советское наследие, которое не патентовали, ибо общее. Есть даже автоблоггердурачок, с ником академик, который доказывает этот тезис.
Ученый - это смесь гения изобретательства и алгоритма фактизации эмпирики. Все.
Если ты ветврач - забей на это.
Найди себя в DIY, если тянет изобретать. Ученый - это в настоящее время балласт, который удерживает новые идеи, без опоры фундаментальной базы, так как не имеет материальной базы. О чем тебе батя и сказал.
Аноним 27/04/24 Суб 20:01:50 590565 1490
>>590511
Если без лодки, чисто телом прыгнуть - да, утонет. потому что держится на воде только за счет уменьшения плотности чуть ниже плотности воды посредством вдоха воздуха. а тут плотность воды ещё ниже падает, мля, и мы уже нерухнум.
Аноним 27/04/24 Суб 20:02:39 590566 1491
>>590536
>видео с котом качком
Аноним 28/04/24 Вск 19:38:23 590587 1492
>>590550
Мечу в генетику

>>590561
А получится преуспеть в обоих сферах? Быть условным ветеринаром, но подпольно заниматься наукой например.
Аноним 28/04/24 Вск 21:39:05 590590 1493
>>590587
Много сил будет отнимать работа, скорее нет.
В лабе особо нечего делать, много времени подумать и тд...
kobra486 28/04/24 Вск 23:04:41 590591 1494
image.png 118Кб, 624x429
624x429
image.png 5204Кб, 1920x1280
1920x1280
всем здрасти. не хочу создавать тред, может тут найду ответ на свой вопрос.
Тема такая, карбюратор на автомобиле работает по закону бернулли (первое изображение). Потом смесь бензина и воздуха уходит во впускной коллектор.
И у меня делема, дело в том что мне инжекторную машину надо переделать под карбюратор, но я не могу поставить карбюратор НА впускной коллектор сверху (т.к. не закроется капот (вырезать капот как на маслкарах не собираюсь)). (изображение 2 - тот самый впускной коллектор который находится выше выпускного).
Вопрос собственно в том, что будет ли работать система если я поставлю через плиту другой впускной коллектор и карбюратор окажется либо немного ниже коллектора (и входных отверстий для смеси воздуха с топливом) или же примерно на одном уровне?
то есть будет ли смесь засасываться немного вверх по коллектору?
Аноним 28/04/24 Вск 23:56:13 590592 1495
>>590591
Закон Бернулли для карбюратора означает, что скорость потоков газовой смеси при прохождении диффузора увеличивается, а давление снижается.
Ни о каких уровнях речи не идет. Ты можешь сместить и перенести коллектор, если соблюдешь сечение и не сильно увеличишь длину патрубка со стехиометрической смесью и не допустишь подсоса воздуха потом.
А еще странно, ведь при переделке на инжекторы, просто меняются навесные и добавляется эбу, если на эту модель также выпускались моновпрыск или инжектор. И все влазить должно.
У нас так деревенские модят свои 2105/2107.
Аноним 29/04/24 Пнд 00:11:52 590594 1496
>>590592
к сожалению нет, карбюратор на данный двигатель не ставился, это двигатель ADR 1.8 в электрику которого я устал вливать деньги, провалы, поддергивания, резкая потеря мощности после 3.2к оборотов. под капотом почти все новое, дело либо в ЭБУ, либо в самой проводке. т.к. у меня есть второй двигатель в сборе (но без электрики), то я хочу поставить карбюратор (могу позволить запороть один коллектор, переварив его на карбюратор) и посмотреть что выйдет.
Есть опыт в переделке с моновпрыска на карбюратор. Ну и постараться оставить все же мозги на месте, чтобы работал климат контроль и прочая электрика.
За ответ спасибо, я так и думал, но мне надо было удостовериться =)
Аноним 29/04/24 Пнд 00:27:01 590597 1497
>>590594
Для 1.8 нужен производительный карб с хорошим подогревом. Там есть зависимость в стехиометрии, которая выражается в зависимости от ттх двигателя - цикл, объем, размеры шпг. По сути, карб от 1.7-2.0 москвича должен подойти, до 5000 оборотов. Но нужен будет новый карб и настройщик. А коллектор впускной дополнительно подогревать не забывай от малого круга, если на впрыске на коллекторе нет такой функции.
Вопрос только, в иномарках же эбу работает с есп/абс/сигнализацией и иммобилайзером, магнитолой и климатом. Как решать будешь? Логические уровни к эбу подключать?
Аноним 29/04/24 Пнд 00:42:58 590598 1498
>>590597
насчет подогрева сильно не парюсь т.к. живу на юге, увидеть на улице -5 это большая редкость, а там посмотрим, задача все же подружить ЭБУ и карбюратор, чтобы электрика работала в штатном режиме кроме электрики двигателя и его датчиков, а именно климат, абс, сигнализация, (есп нет на этой машине).
Буду уже по факту разбираться.
Карбюратор настроить или починить могу хоть с закрытыми глазами, имеется опыт как в работе с автомобильными, так и с мотоциклетными (разве что не синхронизировал несколько карбюраторов на одном двигателе). Брать буду наверное какой то солекс или от лада нива.
Когда ставил карб на моновпрыск (тоже 1.8 двигатель, только голова другая, 8 клапанов, а тут 20), то покупался карб с лады 2110 от двигателя 1.5, так вот машина просто рвала с места и имела отсечку какую то невменяемую, не проверял, просто когда под 8к оборотов уже было, я переставал давить на педаль газа дабы не угробить мотор. Единственный минус что максимальная скорость конечно снижалась и была примерно 160 км/ч, но набиралась эта скорость крайне быстро для старого 1.8 и расход топлива очень радовал (если не открывать вторую камеру и ездить спокойно)
Аноним 29/04/24 Пнд 07:34:41 590601 1499
image.png 1862Кб, 1656x1246
1656x1246
Аноним 29/04/24 Пнд 07:45:46 590603 1500
>>590601
>pay wall
Можно нормальный соус?
Аноним 29/04/24 Пнд 12:21:31 590608 1501
image.png 32Кб, 425x405
425x405
image.png 88Кб, 1024x1024
1024x1024
Как работает цветовая модель пикрелейтед?
Знаю про RGB (суммирование красного, зеленого, синего) и CMYK (желтый, сиреневый и бирюзовый вычитаются из белого), а сабж (используемый большинством художников) - что-то совершенно непонятное.
Это как-то связано с особенности человеческого зрения?
Почему именно красный, синий и желтый цвета выбраны базовыми?
Аноним 29/04/24 Пнд 13:42:31 590610 1502
>>590608
Это связано с философской идеей ахроматизма, основного и доп. цветов, и недостаточных веществ для цветовой палитры в прошлом.
Развернуть мысль не смогу, как начинаю писать в поле, так телефон майнить начинает и буквы по 5 секунду не появляются. В задницу такой форум.
Аноним 29/04/24 Пнд 14:24:09 590611 1503
>>590610
> , так телефон майнить начинает и буквы по 5 секунду не появляются. В задницу такой форум.

Если Android, установи dashchan. На iOS хуй знает. Писать сюда с мобилы через браузер – мазохизм.
Аноним 29/04/24 Пнд 14:37:04 590612 1504
>>590608
> Это как-то связано с особенности человеческого зрения?
>Почему именно красный, синий и желтый цвета выбраны базовыми?
Потому что три вида колбочек в сетчатке глаза реагируют на эти цвета (спектры)
Аноним 29/04/24 Пнд 15:10:24 590613 1505
>>590611
Неее, нахер. Я же не ученый, и сидеть тут не хочу.
>>590612
Это не так.
Сетчатка состоит из палочек и колбочек.
Палочки отвечают за Светочувствительность и Световосприятие.
Колбочки отвечают за Цветовосприятие и восприятие сильного потока света, при котором и происходит хромовосприятие.
В колбочках расположены рецепторы, которые реагируют на спектральный ряд, которому ответствененыи приравнены красный, зеленый и синий цвета.
А идея с желтым цветом именно из-за возни с философским модусом и призматическим разложением.
Проблема в том, что во времена Ньютона не было философской модели, типа описываемой цветовой модели LAB, где было бы описано, что хроматизм имеет зависимость от светлости, по причине инерционности родопсиновых рецепторов в биосистемах и работы глаза в целом.
Ыыыыы, несу хрень какую-то. Звиняйте посоны, я в стрессе, я теперь бнзработный и мне чет не заходит думать...
Пойду в зог, буду демиургать и каббалировать.
Аноним 29/04/24 Пнд 16:33:05 590615 1506
Кто размножается в больших кол-вах: насекомые или рыбы
Аноним 29/04/24 Пнд 16:56:30 590616 1507
>>590615
Насекомые размножаются чаще, больше и быстрее рыб.
Но рыб меньше по массе на планете, чем насекомых.
Причина - срок жизни рыб больше, а форма организации организма сложнее, да и тип питания намного отличается (насекомые в массе своей еда друг для друга, у рыб иначе троф. цепт устроена).
Блее, да что не так с джава скриптами, какие же в /д дебилы...
Аноним 29/04/24 Пнд 17:18:04 590617 1508
ghost-in-the-sh[...].jpg 130Кб, 1920x1080
1920x1080
Здравствуй. Посоветуй пожалуйста музыкальные группы, которые в своём творчестве двигают науку, особенно физику-математику. Не русские группы конечно.
Аноним 29/04/24 Пнд 17:31:14 590618 1509
>>590617
Что сам нашел:
«Symphony of Science» — музыкальный проект Джона Д. Босуэлла, призванный донести научные знания и философию в музыкальной форме. Своим существованием проект во многом обязан классической серии PBS «Космос» Карла Сагана, Энн Друян и Стива Сотера, а также всем другим представленным фигуркам и визуальным эффектам.

Реквестирую еще
Аноним 29/04/24 Пнд 18:17:21 590619 1510
Почему величины можно перемножать/делить друг на друга (см^2, кг*м, м/с), а в степени возводить нельзя? Ну точнее, почему не придумали физического смысла этому? см^кг, корень 10 секунд из 8 метров и всё такое. Ведь метры в секунду тоже не буквально понимают, что типа взял метр и поделил на секунду,- а именно в контексте движения.
Аноним 29/04/24 Пнд 18:46:31 590621 1511
>>590619
Их ещё и складывать нельзя.
Мне кажется, тут надо в обратном направлении думать.
Вот м/с - это производная координаты по времени. Отсюда и размерность, и физический смысл. Умножение, соответственно, это интеграл.

А возведение в степень - это что за операция? А ещё есть обратная функция - логарифм. Вообще хуй проссышь, что такое логарифм метра по основанию килограмм. В какую степень надо возвести килограмм, чтобы метр получить. Охуеть вообще.
Аноним 29/04/24 Пнд 19:38:37 590623 1512
>>590618
Послушал - хуйня, пидор музыкант испоганил голоса великих ученых каким то пидорским электронным фильтром, аля ревеберация. Удалил.
Аноним 29/04/24 Пнд 21:59:47 590631 1513
Аноним 30/04/24 Втр 00:08:25 590635 1514
>>590619
Они производные от формул. Например м/с происходит от того что скорость = растояние (метры) / время (секунды).
Сответственно если тебе нужно например см^кг то ты придумываешь какую-то формулу типа хуйня = размер (сантиметры) ^ масса (килограммы). И вот эта хуйня будет измеряться в см^кг.
Правда ли что Анек 30/04/24 Втр 00:17:38 590636 1515
Математику придумали Русские, чтобы считать обороты синьки?
Аноним 30/04/24 Втр 06:04:43 590641 1516
image.png 728Кб, 800x600
800x600
>>590610
>>590613
В играх желтые объекты тоже лучше заметны, чем зеленые. Например, в Doom есть красные, синие и желтые ключ-карты.
Думаю, все же это как-то связано с восприятием цвета человеком.
Чувствительность к зеленому и красному оттенкам в колбочках разная.
Но формулы для этой цветовой модели так и не смог найти, хотя вроде все художники ее учат, должна быть уже подробная теория.
Аноним 30/04/24 Втр 06:08:06 590642 1517
>>590636
Да. А жиды придумали бесконечность. Никакой бесконечности не существует, это лженаука. У русских и немецких математиков-интуиционистов раньше ее не было, потому что они изучали математику реального мира, а не выдуманных абстракций, не имеющих физического смысла.

Знаете ли Вы, что до поражения Германии во Второй Мировой, было два вида математики, - арийская и еврейская, - названные так по национальности своих сторонников?

Арийская Математика брала пример с естественных наук, склонялась к эмпирицизму, конечности и познаваемости мира, и работала исключительно c объектами, которые можно построить физически (например, в памяти ЭВМ или на бумаге).

Еврейская Математика же слоняется к религиозной абстракции и казуистике: всеохватывающей бесконечности, множествам, и порождаемым ими апориям. Так Еврейская Математика постулирует, что можно удвоить объект, путём перекладывания его частей, пространство делимо на "бесконечно малые", а для любого числа, Бог может создать большее число (аксиома о бесконечности).

Основатель Еврейской Математики, Гидеон Кантор, писал, что работает с "Абсолютом - непознаваемым человеком Актус Пьюриссимус, именуемым многими Богом". Примечательно, что Кантор окончил свою жизнь в психиатрической лечебнице, однако дело Кантора поддержали сионистские организации и католическая церковь, доведя до того, что сознательные германские студенты и профессора протестовали, требуя убрать еврейскую заразу из ВУЗов.

После войны, евреи сделали все возможное, чтобы уничтожить Арийскую Математику, удалив ее сторонников и подменив ее Теорией Множеств - центральной опорой Еврейской Математики. Так основатель интуиционизма, Лёйтзен Брауэр, подвергся изоляции, а результаты Русских и Английских финитистов умалчивались и не получили распространения. В русской истории от рук евреев пострадали математики Егоров (умер в гулаге), Лузин (подвергся травле и был отстранен), Флоренский (расстрелян), Есенин-Вольпин (репрессирован).

Сегодня математика стала еврейской даже по-цвету. Государства поддерживают так называемую "анти-расистскую математику", требующую например использовать еврейские имена в примерах и задачках, рассказывая при этом о великом "вкладе" еврейства в развитие математики.

Евреи, занимающиеся математикой, предпочитают всё специфическое-эльфийское. Причём презирают тех, кто занимается вещами, имеющими практическое применение. Поэтому в Советском Союзе вышел закон, по-которому в ВУЗах должно учиться евреев пропорционально их населению. Лишних отчисляли. Преподаватели евреи на мехмате в знак протеста ушли из университета и образовали НМУ (Независимый Московский Университет). Отсюда и название в мехматянском простонародье ``еврейская секта''.

Еврейские дети в СССР часто учились отдельное от детей гоев, в специальных элитных школах. Одной такой была Московская 57-й спецшкола, ученики которой не без оснований называют себя "пятидесятисемитами". Там часто преподавали выдающиеся преподаватели с мехмата.

В основании математики последнего столетия лежит знаменитая теория множеств Георга Кантора. Если вы откроете большую часть современных серьезных учебников математического и функционального анализа или топологии, или теории вероятности, то в начале почти наверняка увидите экскурс в теорию множеств. Почти вся современная математическая литература написана на теоретико-множественном языке. Камень теории множеств лежит в основании грандиозного здания современной науки.

Но в самом сердце этой самой фундаментальной вроде бы теории, лежащей в основе "царицы наук", почти сразу после ее создания были обнаружены очень серьезные парадоксы и проблемы, не преодоленные до сих пор. Уже сто лет с тех пор математика находится в состоянии перманентного кризиса, который остро воспринимается самыми выдающимися учеными. Великий немецкий математик Герман Вейль писал по этому поводу: "Мы менее чем когда-либо уверены в незыблемости наиболее глубоких оснований логики и математики. Как у всех и всего в мире, сегодня у нас есть свой кризис".

Математика говорит, что у шпекеровой последовательности есть предел? Говорит. Практика говорит, что его нет? Тоже говорит. Математика говорит, что апельсин можно удвоить путём перекладывания его частей? Говорит. Удалось кому-нибудь с новозаветных времён повторить эту процедуру? Наблюдения раз за разом показывают, что при такого рода операциях закон сохранения вещества неукоснительно соблюдается. Математика предсказывает наличие в бесконечномерном гильбертовом пространстве базиса Гамеля. Наблюдать оный пока вообще никому не удалось. Так что математика - именно лженаука, и никак иначе.

Именно уверенность в нематериальности математических объектов влечёт за собой веру в возможность "приближённых" вычислений (что чушь - вычисления бывают либо точные, либо неверные). Да, самолёты летают и при засилье Теории Множеств. Но если бы математика была конструктивной, они летали бы лучше, потому что конструкторы не забивали бы себе голову теоретико-множественным мусором, на практике бесполезным и дезориентирующим.

Аксиомы имеют смысл только тогда, когда они выражают свойства объективно существующих предметов. Так, если мы введём аксиому "на каждой руке человека содержится 3.1415… пальцев" и построим на базе этой аксиомы формальную теорию, то положения этой теории будут бессмысленны и даже вредны.


(с) Золотце
Аноним 30/04/24 Втр 06:18:14 590643 1518
30/04/24 Втр 07:29:08 590646 1519
>>590598
нормиблядота тише
сажи
Аноним 30/04/24 Втр 13:51:40 590664 1520
надо ли писать у нуля единицы измерения?
Аноним 30/04/24 Втр 20:18:37 590673 1521
Аноним 30/04/24 Втр 22:14:24 590679 1522
>Early twin studies of adult individuals have found a heritability of IQ between 57% and 73%,[6] with some recent studies showing heritability for IQ as high as 80%.[7] IQ goes from being weakly correlated with genetics for children, to being strongly correlated with genetics for late teens and adults.
>Although IQ differences between individuals have been shown to have a large hereditary component, it does not follow that disparities in IQ between groups have a genetic basis.[12][13][14][15] The scientific consensus is that genetics does not explain average differences in IQ test performance between racial groups.
Т.е. IQ (что бы это ни было) наследуется на индивидуальном уровне, но на популяционном - не наследуется. Наука доказала.
Эти утверждения как-то согласуются, или это чистейшая шизофрения?
Аноним 30/04/24 Втр 23:27:01 590681 1523
>>590679
> не наследуется
Помоему там написано не про то что "не наследуется" а что пункт 1 не может объяснить результаты IQ тестов на уровне популяции. Причиной чего может быть например всратые результаты тестов, либо на уровне популяции проявляются какие-то дополнительные факторы которые не удалось учесть на близнецах, либо еще какая хуйня.
Аноним 30/04/24 Втр 23:58:41 590682 1524
>>590681
>на близнецах
Наука доказала, что характеристика наследуется не только у близнецов, а у всех. Т.е. доказана доминирующая генетическая компонента, как и у множества других характеристик, от простейшего цвета глаз до сложнейшей длины тела.
Но ахалай-махалай, не все факторы изучены. Как это?
Вот с длиной тела все факторы учтены, никаких проблем, корейцы ниже датчан. А тут нет.
как так.
Аноним 01/05/24 Срд 01:23:38 590685 1525
Дайте пик, где по столбикам научные направления, а внизу к ним списки годных книг.
Аноним 01/05/24 Срд 02:30:29 590687 1526
>>590679
Очевидно, что на IQ влияет в том числе среда, а среда выбирается с помощью привычек. В итоге гены скорее выбирают не само значение IQ, а направляют траекторию, по которой он будет меняться в течении жизни. Т.е. у детей мало проявляется, а у взрослых уже сильные проявления, накопленные за всю жизнь.

А с популяцией такая вещь, у тебя люди склонные к высоким баллам IQ, размножаются хуже, и популяции тупеют. Возможно это портит результаты.
Аноним 01/05/24 Срд 03:54:56 590690 1527
>>590687
У тебя не больше 90.
Аноним 01/05/24 Срд 14:10:18 590704 1528
Ого, тут генетики есть.
Кто-нибудь знает - желание завести потомка/параноидальные мысли что нужно оставить что-то после себя и тд передаётся генетически?
Ну то есть есть ген которые отвечает не за желание засунуть писюн в письку, а именно активирует в мозгу за навязчивые мысли что "надо завести детей", "надо наследника" и тд?
Знаю что у некоторой части бездетных женщин за несколько лет до менопаузы крышу срывает по этому поводу.
У меня в моем научном треде спор возник на эту тему, а так как я шарю в генетике чуть лучше навучпопера не смог вывезти https://2ch.hk/zog/res/768517.html#771068
Аноним 01/05/24 Срд 17:17:55 590706 1529
2024-05-0121-17.png 126Кб, 419x263
419x263
Почему плотность сжиженного газа в зажигалке не уменьшается при использовании, разве газ не должен весь доступный объем занимать? Что находится в "пустых" местах, и откуда оно там берется? Если это воздух, то как он туда попадает, если давление в зажигалке больше атмосферного?
Аноним 01/05/24 Срд 18:25:44 590708 1530
>>590706
В пустых местах находится испарившийся газ. Он испаряется до тех пор пока давление не повысится настолько что испарение прекратится. Если давление еще больше повысить каким-то внешним воздействием - он начнет сжижаться. Там используется пропан который сжижается при комнатной температуре и небольшом давлении. В отличие от метана который нужно либо давить до сотен атмосфер или охлаждать до криогенных температур. А лучше и то и другое сразу.
Аноним 01/05/24 Срд 21:50:40 590711 1531
>>590706
Газ - это не про вещество, а про агрегатное состояние. Сжиженный газ - это жидкость, а не газ, жидкость весь объем не занимает.
Точно так же вода в жидком виде не занимает весь объем, а испарившаяся вода (т.е. водяной пар) занимает.
Аноним 02/05/24 Чтв 09:03:59 590715 1532
>>590708
>>590711
Спасибо.

Я верно понял, что переход жидкости в газ идет только через процесс испарения с поверхности, и плотность ее (жидкости) не понижается равномерно по всему объему при понижении давления на поверхности?

Я почему-то себе это по-другому представлял, что жидкое состояние вещества поддерживается за счет давления в баллоне, и при понижении давления оно (вещество) будет целиком становиться все более и более разреженным...

Получается, на пике мы имеем такую ситуацию: давление газообразной части вещества на жидкую слишком большое, чтобы та могла испаряться. При этом давление жидкой части на газообразную слишком маленькое, чтобы та могла конденсироваться.

То есть текущее давление их друг на друга меньше, чем нужно для сжижения, но больше, чем нужно для испарения.

И правильно воспринимать ситуацию будет не как "вещество поддерживается в жидком состоянии за счет давления", а "вещество существует в жидком состоянии, пока не будут достигнуты условия для перехода".

Если проткнуть зажигалку, то будет тот же самый процесс испарения с поверхности, только быстрый? Плотность вещества под поверхностью не изменится до того, как испарится вещество на поверхности, так?
Аноним 02/05/24 Чтв 12:57:15 590717 1533
>>590715
Меня всегда учили, что жидкость - это равновесное состояние между давлением паров.
В случае сжиженных газов ситуация аналогичная, только объем при этом имеет бОльшее давление, как как это нестандартные жидкости и они всегда в кипящем состоянии.
Открывая клапан на зажигалке, ты стравливаешь объем газов, который образовался при дросселировании. Оно возникает из-за пористой трубочки в жиге, которая соединена с клапаном. Жидкость снова компенсирует "пар" на собой, переходя в газ. При этом расходуется энергия, и жидкость должна охлаждаться.
Если проткнуть жигу, ты выпустишь жидкость в гигантский объем и она просто закипит от потери давления газовых паров над ней. Если это будет гигантская жига, то можно даже увидеть, как жидкость замерзнет, но потом забурлит и рассеится, как жидкий азот.
Аноним 02/05/24 Чтв 13:02:11 590718 1534
>>590717
А почему газ не переходит в сжатую жидкость? Так это не вода, которая конденсируется обратно из паров. Для сжижения газа нужна энергия, настолько много, что газ просто не сможет ее занять из системы.
Ведь его замкнутая система (зажигалка), наоборот, претерпевает потерю давления.
Аноним 03/05/24 Птн 08:37:33 590727 1535
Скопирую для вопроса абзац с лурки.

На первый взгляд вроде всё хорошо, но при глубоком изучении выявились серьёзные противоречия следствий теории с экспериментальными данными. Например, в теории струн обязательно присутствовала частица, тахион, квадрат массы которой был меньше нуля. Ну ты понел, да? У нее масса получалась мнимая.

А в чем проблема? Почему нельзя отказаться от математических формул, изначально не предназначенных для физики, и разработать новые? Почему не может быть отрицательной массы? Почему массу нельзя рассматривать как температуру, например? Почему так важна передача информации? Если наблюдатель видит молнию, но не слышит гром, значит, информация передается со скоростью, превышающей скорость звука. Если у нас нет возможности фиксировать передачу информации со скоростью выше скорости света, это наши проблемы, а не физики.
Аноним 03/05/24 Птн 12:24:09 590728 1536
>>590727
Проблема тахионов в том, что при взаимодействии они забирают всю энергию у обычных частиц и блокируют любые взаимодействия между ними. Если в модели есть тахионы, то они схлопывают систему из обычных частиц, причем за время, которые они не успеют прореагировать обычным образом. Короче тахионы прямо руинят привычную физику.
С отрицательной массой также, но только для ньютоновского случая - просто точек с отрицательной массой. Поля с отрицательной массой(точнее с отрицательным давлением) вполне легитимны.
Ответить в тред Ответить в тред

Check this out!

Настройки X
Ответить в тред X
15000
Добавить файл/ctrl-v
Стикеры X
Избранное / Топ тредов